eBook Dirichle

July 16, 2017 | Author: Nguyễn Hữu Điển | Category: N/A
Share Embed Donate


Short Description

Download eBook Dirichle...

Description

, ˜ HU ˜,U ÐIÊN NGUYÊN

, , ´ P ÐIRICHLÊ PHUONG PHA , ´ ` VA UNG DUNG .

, ´ BAN KHOA HOC VA ` XUÂT ` KY ˜ THUÂT NHA . . ` NÔI HA 1999 .

, ` ` I NOI ´ ÐÂU LO

, , ,, , ˜d Nguyên l´y nh˜ung c´ ai lô`ng v` a c´ ac ch´ u tho d ¯a ¯uo. c biê´t d ¯ê´n t`u râ´t , , ,, , , ˜ l` lâu. Ngay trong chuong tr`ınh phô thông co so ch´ ung ta c˜ ung d am ¯a , , , , , ´i phuong ph´ quen vo ap giai to´ an n` ay. Thu. c ra nguyên l´y n` ay mang ,`, ´, tên nh` a b´ ac hoc Legien Dirichlet (1805. nguoi Ð,uc Pête Gutxtap , , , 1859). Nguyên l´y ph´ at biêu râ´t d ung ta nhô´t tho v` ao ¯on gian: Nê´u ch´ , , , c´ ac lô`ng m` a sô´ lô`ng ´ıt hon sô´ tho, th`ı thê n` ao c˜ ung c´ o môt . lô`ng nhô´t ´ıt , nhâ´t hai con tho. , , , , `˘ng nguyên l´y d ˜ Chı ba ang loat ac b` ai to´ an d ¯on gian nhu vây ¯a . h` . c´ , ,, d ¯uo. c giai. , ,, , Cuô´n s´ ach d lai theo t`ung chu d o liên quan d ¯uo. c biên soan ¯ê` c´ ¯ê´n . . , , , , , ˜ c´ ´p dung nguyên l´y, môi ach giai trong v´ı du. cua t`ung chuong l` aa . , , ,´, ´ d ai tâp o liên quan d¯ên b` ai ¯iên h`ınh nguyên l´y Ðirichlê. B` . giai truoc c´ , , , ` ´ ´ giai sau nên cân luu ´y khia d ach. Voi mong muôn c` ung ban ¯oc ¯oc . s´ . d . , ,, , ´ thao luân ap chung minh to´ an hoc a hy vong cung . môt . phu,ong ph´ . v` . ´ ` câp môt ai liêu ac thây cô gi´ ao v` a c´ ac em hoc . t` . bô ´ıch cho c´ . sinh ham , mê t`ım t` oi trong to´ an hoc, ac gia manh dan ach . t´ . . biên soan . cuô´n s´ , , ´˘n ´ ´ ´ ` ˘c cha n` ay. Do kha n˘ ang v` a thoi gian c` on han ach cha . chê, cuôn s´ , , , , ´ng g´ không tr´ anh khoi thiê´u s´ ot. Ch´ ung tôi mong d op ´y ¯uo. c su. d ¯o , , , ,, , ´ ` ´ kiên cua d op ´y xin gui vê Nh` a xuât ban Khoa hoc a K˜y ¯oc . gia. Thu g´ . v` , ` thuât a Nôi. . - 70 Trân Hung Ðao, . H` . 3

4

, L`oi n´ oi d¯â`u

, , , ˜ Hô`ng Tân d ˜d T´ ac gia xin chân th` anh cam on PGS-TSKH Ðô ¯a ¯oc , ,. ` ´ ´ng g´ v` ad op nhiêu ´y kiên qu´ı b´ au trong qu´ a tr`ınh ho` an chınh ban ¯o , thao. , T´ ac gia

, , CHUONG

1

´ ÐIRICHLÊ VA ` V´I DU. NGUYÊN LY

1.1. Nguyên l´ y Ðirichlê ,, , Nguyên l´y Ðirichlê nhiê`u khi ngu`oi ta goi a ¨Nguyên l´y nh˜ung . ,l` , ˘. c biêt ng˘ an k´eo¨. Ðây l` a môt o nhiê`u ¯on gian, d ¯a . nguyên l´y râ´t d . c´ , , , ´ ng dung u trong c´ ac l˜ınh vu. c kh´ ac nhau cua to´ an hoc. ung nguyên . . D` , , ,`, , ˜ ´ ng minh tô`n tai ´,i t´ınh ang chu l´y n` ay nguoi ta dê d` ¯ô´i tuo. ng vo . môt . d , , , , , châ´t x´ ac d Dang o thê ph´ at biêu nhu sau: ¯inh. ¯on gian nhâ´t c´ . . d Nê´u c´ o m vât ˘. t v` ao n c´ ai ng˘ an k´eo v` a m > n th`ı c´ o ´ıt nhâ´t môt . d¯a . , ´ ng˘ an k´eo chu´a ´ıt nhât hai vât. . , ,, ,, , , `˘ng vo ´,ng minh d ´i nguyên l´y n` Tuy ra ay ngu`oi ta chı chu ¯uo. c su. tô`n , , ,, ,, ,, , tai a không d¯ua ra d ap t`ım d cu. thê, nhung ¯uo. c phuong ph´ ¯uo. c vât . m` . , , , , , trong thu. c tê´ nhiê`u b` ai to´ an ta chı câ`n chı ra su. tô`n tai ad ¯u rô`i. . l` , , , Nguyên l´y Ðirichlê l` a môt at biêu ¯inh . l´y vê` tâp . d . ho. p h˜uu han. . Ph´ , ch´ınh x´ ac nguyên l´y n` ay nhu sau: ,, , , ˜ Cho A v` a B l` a hai tâp o sô´ phâ`n tu h˜uu . , h,o. p không rông c´ , , ,, , ´,n ho,n sô´ lu,o.,ng phâ`n tu, cua han, a sô´ luo. ng phâ`n tu cua A lo . m` , , ˜ phâ`n tu, cua A cho tu,o,ng ´ ´,i môt ˘c n` B. Nê´u vo ao d ¯â´y, môi . qui ,ta , ,, , ´,ng vo ´,i môt u kh´ ac nhau cua A . phâ`n tu B, th`ı tô`n tai . hai phâ`,n tu , , ´, , , ´,i c` m` a ch´ ung tuong u ng vo ung môt . phâ`n tu cua B. , , ,, , , `˘ng c´ ´, cho ra Ðê d˜ê hiêu ta cu ac phâ`n tu cua tâp a ¨Nh˜ung ng˘ an . B l`

6

,, Chuong 1. Nguyên l´y Ðirichlê v` a v´ı du.

, ,, , ,, ˘. t v` k´eo¨ v` a c´ ac phâ`n tu cua A d ao c´ ac ng˘ an k´eo cua n´ o. Trong ¯uo. c d¯a , , ,, , , , `˘ng ` ph´ at biêu cua nguyên l´y trên c´ ac phân tu h˜uu han ¯uo. c t´ınh ba . d , , ´i tâp sô´ tu. nhiên, v`ı vây c´ o liên quan mât . Nguyên l´y Ðirichlê . thiê´t to . , , ´ , , ´ ´ ho. p sô tu. nhiên v` a c´ ac t´ınh chât cua tâp ay. . ho. p sô n`

1.2. V´ı du.

, , , . 1.1. Ðê ky niêm am ng` ay giai ph´ ong Miê`n Nam, tai anh ., 20 n˘ . ,môt . th` , , , , , , , ˜ ˜ ng ngu`oi 20 tuôi. Ng` ay 30 ˘p ma ˘. t nhu phô´ ngu`oi ta tô chu´c buôi lê ga , . , ´ trong buôi ga th´ ang 4 n˘ am d¯o ˘. p ma ˘. t c´ o 400 thanh niên. Chu´ng minh , ,, , , , ` ra ˘ng c´ o ´ıt nhâ´t hai ngu`oi trong sô´ ngu`oi t´ oi du. c` ung chung môt ay . ng` sinh. , , ˜ ng` `,i giai. N˘ Lo am 1995 c´ o 365 ng` ay. Ch´ ung ta coi môi ay nhu môt . , ´ ´ ´ ` ´ ng˘ an k´eo v` ad a nh sô t u 1 d ê n 365 (ng˘ a n k´ e o cuô i c` u ng l` a ng` a y 31 ¯ ¯ , ˘. t nh˜ung thanh niên c´ th´ ang 12 n˘ am 1995). Ch´ ung ta d o ng` ay sinh ¯a , , ´, , , ´,n ´. Nhung sô´ thanh niên d tuong ung v` ao c´ ac ng˘ an k´eo d ¯o ¯ê´n du. l˜ê lo ,, ,, , hon sô´ ng˘ an k´eo, theo nguyên l´y Ðirichlê c´ o ´ıt nhâ´t hai ngu`oi d ¯uo. c ˘. t v` ´ c´ d ao c` ung môt an k´eo. Ðiê`u d o ngh˜ıa l` a ho. sinh c` ung môt ¯a ¯o . ng˘ . ng` ay. , ,, ˜ ngu,`o,i . 1.2. Trong sinh hoc ˘`ng sô´ t´ oc trên d¯â`u cua môi . ngu`oi ta biê´t ra , , ,, không qu´ a 200.000 c´ ai. Chu´ng minh ra ˘`ng trong sô´ ngu`oi cua th` anh , , , , phô´ H` a nôi, oi sô´ dân hon 2.000.000, c´ o ´ıt nhâ´t 11 ngu`oi c´ o c` ung sô´ . v´

J

t´ oc. , ,, , `,i giai. Ch´ ´nh sô´ t`u 0 d Lo ung ta x´et 200.000 ng˘ an k´eo d ¯uo. c d ¯a ¯ê´n , , ˜ ngu`oi dân H` ˘. t¨ môi 199.999. Ch´ ung ta ¨d a nôi v` ao môt an k´eo ¯a . ng˘ , , ,., , , ,, `˘ng sô´ thu ´ tu. cua ng˘ m` a sô´ t´ oc ba an k´eo. Gia su không c´ o 11 ngu`oi , ,, ˜ ng˘ c´ o c` ung sô´ t´ oc, nhu vây an c´ o nhiê`u nhâ´t l` a 10 ngu`oi c´ o c` ung . môi ´ sô´ dân H` sô´ t´ oc, do d a nôi a 200.000×10=2.000.000, ¯o . nhiê`u nhâ´t l`

1.2. V´ı du.

7

, ´,i gia thiê´t l` ´,n ho,n 2 triêu. ´ ng vo d ay không d a sô´ dân H` a nôi ¯iê`u n` ¯u . lo .

J

, , . 1.3. Ba muo,i hoc am b` ai viê´t ch´ınh ta. Môt . sinh l` . trong sô´ hoc . sinh , , ˜ ˜ ´ ´ bi. 14 lôi, c` d¯o on c´ ac hoc ac ma ˘c lôi ´ıt hon. Chu´ng minh ra ˘`ng . sinh kh´ , , ˜ ba ˘´c sô´ lôi ˘`ng nhau. c´ o ´ıt nhâ´t ba ngu`oi ma , ,, , `,i giai. Ch´ ´nh sô´ t`u 0 d Lo ung ta x´et 15 ng˘ an k´eo d ung ¯uo. c d ¯a ¯ê´n 14. Ch´ ˜ ˜ ` ´ ´ ˘t¨ môi hoc ˘ng sô lôi ´ ng ba ta ¨d ao môt an k´eo mang sô d ¯a ¯u . sinh v` . ng˘ , . ˜ ba `˘ng nhau, cua hoc ay. Nê´u không c´ o ba hoc ao c´ o sô´ lôi . sinh n` . sinh n` , ˜ ng˘ th`ı trong môi an mang sô´ t`u 0,1,2,. . . ,13 s˜e c´ o nhiê`u nhâ´t hai hoc . , , , , ´ ` ´ ´ ´ sô luo. ng cua nh˜ung hoc sinh. Khi d ay nhiêu nhât l` a 28. Nêu ¯o . sinh n` ˜ ´ ´ ˘c 14 lôi (trong ng˘ ´ hoc thêm v` ao d an k´eo sô 14) ch´ ung ta s˜e ¯o . sinh ma , ,, ˜ d ` ´ ´ ` nhân ay dân ¯uo. c nhiêu nhât 29 hoc ¯iêu n` ¯ê´n . d . sinh viêt ch´ınh ta, d , , ´i d ˜ cho. su. vô l´y vo ¯iê`u kiên ¯a . d

J

, ,, ˜ nh´ o ´ıt nhâ´t hai . 1.4. Chu´ng minh ra˘`ng trong môi om ban . 5 ngu`oi c´ , , , , , ,`, , , , ˜ a nhu ˜ ng ngu`oi trong nh´ o c` ung sô´ luo. ng ngu`oi quen giu om d¯os. nguoi c´ , ,, , , ` ´ Chu´ng minh ra ˘ng c` ung kêt luân oi nh´ om ban o sô´ luo. ng . nhu vây . v´ . c´ th` anh viên bâ´t k`y. , , ˜ ngu,`o,i `,i giai. Ch´ ´nh sô´ t`u 0 d¯ê´n 4. Môi Lo ung ta x´et n˘ am ng˘ an k´eo, d ¯a , ,, ´,i sô´ ngu,`o,i trong ˘. t v` tham du. d ao ng˘ an k´eo mang sô´ tr` ung vo ¯uo. c d ¯a ,, ´ quen. nh´ om m` a ngu`oi d ¯o , ,, , ,, a) Nê´u c´ o môt . ngu`oi không quen ai ca trong, sô´ nh˜ung ngu`oi ,, c` on lai, an sô´ 4 l` a trô´ng (v`ı nguo. c lai an 0 v` a4 . th`ı ng˘ . th`ı ca hai ng˘ , , , ˜ d ˜ ngu`oi trong sô´ 5 d ¯ê`u không trô´ng, dân ¯ê´n vô l´y). Nhu vây, . môi ,`, , , ´,i sô´ lu,o.,ng 4 ng˘ ˘. t v` nguoi d ao c´ ac ng˘ an mang sô´ 0,1,2,3 vo an. ¯uo. c d ¯a ,, , , , T`u nguyên l´y Ðirichlê suy ra ´ıt nhâ´t c´ o hai ngu`oi o trong môt an, . ng˘ , , , , hay l` a, ho. c´ o chung sô´ luo. ng ngu`oi quen.

8

,, Chuong 1. Nguyên l´y Ðirichlê v` a v´ı du.

,, ,, ,, ˜ ngu,`o,i s˜e d b) Nê´u moi o ´ıt nhâ´t môt ¯uo. c . ngu`oi c´ . ngu`oi quen, môi ´,i sô´ lu,o.,ng 4 ng˘ ˘. t v` d ao c´ ac ng˘ an mang sô´ 1,2,3,4, vo an. Phâ`n c` on lai ¯a . ´p dung a nguyên l´y Ðirichlê. .

J

, , , ˜ ca ´ tham du. 16 d¯ôi. . 1.5. Trong môt ong d¯a ˘. p hai d¯ôi phai . giai b´ . ,Môi . , , , , ˜ th`o,i d d¯â´u v´ oi nhau. Chu´ng minh ra ˘`ng tai o ´ıt nhâ´t ¯iêm cua giai c´ . môi , ˜ d¯â´u nhu nhau. 2d o sô´ trân ¯ôi . c´ . d¯a , , `˘ng `,i giai. Ch´ ´nh sô´ t`u 0 d Lo ung ta x´et 16 ng˘ an k´eo d u ´y ra ¯a ¯ê´n 15. Ch´ , , , , , ˜ d ´n nhâ´t c´ 15 l` a sô´ luo. ng lo ac trân b´ ong m` a môi c´ o thê d th`oi ¯ôi ¯â´u tai . . . , ˜ d `˘ng ˘. t môi d ay d ong v` ao ng˘ an k´eo mang sô´ ba ¯iêm d ¯ang x´et. H˜ ¯a ¯ôi . b´ , , `˘ng ˜d ´. Ch´ sô´ c´ ac trân ad ung ta nhân ¯ôi ¯a ¯â´u d ¯ê´n th`oi d ¯iêm d ¯o . m` . d . ra ra , , , , , c´ ac ng˘ an 0 v` a 15 không thê d a nhu vây ¯ô`ng th`oi không trô´ng d ¯uo. c v` . , ´p dung c´ o thê a nguyên l´y Ðirichlê. .

J

, , . 1.6. Trên tr´ai d¯â´t sô´ng ho,n 5 ty ngu`o,i, biê´t ra˘`ng không qu´a 1% sô´ng , , ,, ung trên môt am tuôi. Chu´ng minh ra ˘`ng ´ıt nhâ´t c´ o hai ngu`oi sinh c` . tr˘ môt . giây d¯ô`ng hô`. , , `,i giai. Theo du,o,ng lich Lo anh 100 n˘ am c´ o ´ıt hon 37000 ng` ay. . hiên . h` , , , ˜ ˜ ´ 100 n˘ Môi ng` ay c´ o 24 gi`o, môi gi`o c´ o 3600 giây. Khi d am c´ o ´ıt hon ¯o , , ` ,, , ,, 3,33 ty giây. T`u d ch´ ung ta t`ım d ai ¯iêu kiên ¯uo. c nh˜ung ngu`oi trên tr´ . , , , , , ´ ´ d a 100 tuôi ´ıt nhât l` a 99% t`u 5 ty ngu`oi ngh˜ıa l` a ´ıt nhâ´t ¯ât không qu´ , , ,, ˘. t 4,9 ty ngu`oi ´p dung c´ o 4,9 ty. Viêc c` on lai a nguyên l´y Ðirichlê: d ¯a . . . , an k´eo. v` ao 3,33 ty ng˘

J

, . 1.7. Trong th`o,i gian k´eo d`ai môt n˘ am hoc môt hoc sinh giai ´ıt nhâ´t . . . . , , , ˜ ng` môt ai tâp ay. Ðê tr´ anh c˘ ang tha ˘ng hoc ang tuâ`n . b` . môi . sinh giai h` , , không qu´ a 12 b` ai tâp. ra ˘`ng trong th`oi gian k´eo d` ai liên . Chu´ng minh , , ˜ ng` ´ng 20 b` tuc ay hoc ay phai giai d¯u ai tâp ay. . môt . sô´ ng` . sinh n` . môi

1.2. V´ı du.

9

, , ,, `,i giai. Ch´ ˜ giai ung ta k´y hiêu a sô´ luo. ng b` Lo ai tâp ¯a . a1 l` . , hoc . sinh d ,, ˜ giai trong hai ng` trong ng` ay d a sô´ luo. ng b` ai tâp ay ¯â`u tiên, a2 l` ¯a . d , ,, ` ´ ` ˜ giai trong ba ng` d a sô luo. ng b` ai tâp d ay d a v.v. a77 ¯âu, a3 l` ¯a ¯âu, v` ,. , ,, ´ ` ` ˜ giai trong 77 ng` l` a sô luo. ng b` ai tâp ay d ¯a ¯âu (11 tuân). Theo gia . d , , thiê´t a77 ≤ 11.12 = 132. Ch´ ung ta x´et tâp ac sô´ tu. nhiên . ho. p c´ M = { a1 , a2 , a3 , . . . , a77 , a1 + 20, a2 + 20, a3 + 20, . . . , a77 + 20}. N´ o ,, , , ´ ` ´ ´ ´ chua 154 phân tu v` a sô lon nhât trong ch´ ung l` a a77 + 20 ≤ 152. `˘ng nhau. Nhu,ng ´ ´ Theo nguyên l´y Ðirichlê trong M c´ o ´ıt nhât hai sô ba c´ ac sô´ a1 , a2 , a3 , . . . , a77 l` a ho` an to` an kh´ ac nhau. suy ra tô`n tai a . ak v` , ` al m` a ak = al + 20, l < k ≤ 77. Nhu vây ay ¯iêu n` . ak − al = 20, d , , , , , ´ ` ´ ´ c´ o ngh˜ıa l` a tu ng` ay thu l + 1 d ay thu k hoc ay phai giai ¯ên ng` . sinh n` ´ ng 20 b` d ai. ¯u , , , , ,, . 1.8. Trong môt . khu tâp . thê sô´,ng 123 ngu`oi. Tông sô´ tuôi cua ho. , ,, , ,, l` a 3813. Chu´ng minh ra ˘`ng c´ o thê chon 100 ngu`oi sô´ng o khu tâp . thê . , , , , , ´ n` ay, m` a tông sô tuôi cua ho. không nho hon 3100.

J

, , , ,, ,, `,i giai. Ch´ Lo ung ta h˜ ay chon 100 ngu`oi nhiê`u tuôi nhâ´t v` a gia su . , , , , , ,, , ´ ngu`oi tre nhâ´t trong sô´ tông sô´ tuôi cua ho. nho hon 3100. Khi d ¯o , ,, ,, ,, ˘. t kh´ ngu`oi d l` a 3100:100=31 tuôi. Ma ac ngu`oi n` ay không ¯uo. c chon . , , , , , ,`, ´ tông sô´ tuôi cua 23 tre hon 23 nguoi c` on lai ach chon. ¯o . theo c´ . Khi d , , , , ,, ´,n ho,n 23.31=713. Suy ra tông sô´ tuôi cua tâ´t ca ngu`oi n` ay không lo , , , ,, ˜ d moi ¯ê´n . ngu`oi sô´ng trong tâp . thê nho hon 3100+713=3813 dân vô l´y. , , , . 1.9. N˘am ca˘. p vo., chô`ng tô chu´c môt ˘. p ma ˘. t. Khi ga ˘. p nhau ho. . buôi ga , , , , ba ˘´t tay nhau, nhung không ai tu. ba ˘´t tay ngu`oi trong gia d¯`ınh m`ınh ,`, , ˜ ba v` a nguoi m` a chô`ng m`ınh (hoa ˘. c vo. m`ınh) d¯a ˘´t tay rô`i. C˜ ung không , , , ` ng ban d¯â`u, ai ba ˘´t tay c` ung môt ˘. p ch´ uc mu . ngu`oi hai lâ`n., Sau cu,ôc . ga , , ,`, , , , ˜ ng ngu`oi c´ `n ông tên l` môt a H` ung hoi tâ´t ca nhu o ma ˘. t, kê ca . nguoi d¯a

J

10

,, Chuong 1. Nguyên l´y Ðirichlê v` a v´ı du.

,, , ˜ ba a ho. d¯a ˘´t tay d¯uo. c bao nhiêu lâ`n. Ho. nhân vo. m`ınh, l` ˘`ng ch´ın . thâ´y ra , , , , ,`, , , , ` , nguoi d¯uo. c hoi d ac con sô´ kh´ ac nhau. Nhu vây ung ¯êu tra l`oi c´ . vo. cua H` ´ ` ˜ ba d¯a ˘t tay bao nhiêu lân? , , , ,, ˜ môt ´˘t tay không qu´ `,i giai. Môi Lo ngu`oi kh´ ach ba a 8 lâ`n. V`ı câu tra l`oi . , , ,, ´ phai l` cua 9 ngu`oi l` a c´ ac sô´ kh´ ac nhau nên c´ ac sô´ d a 0,1,2,3,4,5,6,7 ¯o , , ,`, ´ ,, , ˘t tay 8 lâ`n phai l` ˘. c chô`ng) cua ngu`oi không v` a 8. Nguoi ba a vo. (hoa ,, ,, ´˘t tay 8 lâ`n m` ´˘t tay lâ`n n` ´ không ba a ba ao (nê´u nguo. c lai ¯o . th`ı ngu`oi d , , , , , , , ´˘t tay 7 lâ`n nhiê`u nhâ´t chı l` a 7 lâ`n thôi). Tuong tu. nhu vây . ngu`oi ba ,`, , , , ´˘t tay môt ´˘t tay 6 lâ`n c´ ˘. c chô`ng) ba c´ o nguoi vo. (hoa o . lâ`n, ngu`oi ba ,`, , , , , , , ´˘t tay 2 lâ`n, ngu`oi ba ´˘t tay 5 lâ`n c´ ˘. c chô`ng) ba nguoi vo. (hoa o ngu`oi vo. , ,, ´˘t tay 3 lâ`n. Chı c` ´˘t tay ˘. c chô`ng) ba (hoa on lai . môt . ngu`oi duy nhâ´t ba , , , , ´ ch´ınh l` 4 lâ`n, d¯o a ngu`oi vo. cua H` ung. , , ,, . 1.10. Môt ˘`ng: Môt ung vuong . câu chuyên . cô t´ıch kê lai . ra . lâ`n vua H` , 18 c´ o m`oi c´ ac quan trong triê`u hop môt ai b` an tr` on. Theo . ngô`i quanh . c´ , , ,, ˜ `n d ˜ viê´t tên cua môi quan trên b` lênh cua vua, môt an truo´c ¯a . . cân . thâ , ,, ac quan trong triê`u không d¯uo. c b´ ao chiê´c ghê´ m` a ông ta phai ngô`i. C´ ,´, ˜ ´ ˜ ngô`i không theo sa ˜ d¯inh truoc nên ho. d¯a ˘p xê´p d¯a a chiê´m chô môt . ,m` . , ` ´ ` c´ ach bât k`y. Chu´ng minh ra ˘ng ông cân o thê quay chiê´c b` an sao . thân c´ , ´ng vi. tr´ı tên cua m`ınh ? cho ´ıt nhâ´t c´ o hai ông quan ngô`i d¯u

J

, ,, `,i giai. Ða ˘. t sô´ luo. ng c´ ˘. t b` ´ ma Lo ac quan l` a n. Khi d an c´ o n trang ai, ¯o . th´ , , , ´i c´ ´i c´ ´. vo ac trang th´ ai n` ay d ac quan l` a biên d ao d ¯ô´i diên ¯ê` tên n` ¯o . . vo , , ˜ môt ´i môi Ngo` ai ra vo o môt trang th´ ai, m` a khi ngô`i . ông quan chı c´ . ,. , , ˜˘n. ´i ch´ınh tên cua m`ınh trên biên d ´ ng th`ı ông â´y d d ¯u ¯ô´i diên ¯ê` sa . vo , , , , ˜ trang Ngh˜ıa l` a, nê´u môi th´ ai cua b` an (v`ı b` an c´ o thê xoay d ¯uo. c) ta . , , ´, , , , `˘ng sô´ luo. ng c´ ´i môt ´ ng vi. tr´ı cho tuong ung vo sô´ ba ac quan ngô`i d ¯u , ,. , , , , tên m`ınh, th`ı tông cua tâ´t ca nh˜ung sô´ nhân ai ¯uo. c (moi . d . trang . , th´ , , , ´˘p b` an) s˜e không nho hon n. Nhung môt th´ ai d¯â`u tiên cua sa . trang .

1.3. B` ai tâp .

11

, , ´, ˜ ). Nê´u gia, su,, ´,i 0 (không ai ngô`i d ´ ng chô ng vo xê´p b` an cho tuong u ¯u , , , ´, ´,i sô´ nho ho,n 2 ˘. t b` trong n − 1 trang th´ ai ma an c` on lai tuong u ng vo . . , , , , , ,, ´,c l` ˘. c 0), th`ı tông cua n sô´ nhân (tu a chı c´ o sô´ 1 hoa d ¯uo. c s˜e nho hon . , ,, , ˘. t b` ´ không thê d n, d th´ ai ma an c` on ¯iê`u d ¯o ¯uo. c. Suy ra t`u n − 1 trang . ,`, , ´ ´ ´i ch´ınh tên lai o ´ıt nhât môt th´ ai m` a hai nguoi s˜e d ¯ôi diên . vo ., c´ . trang . cua m`ınh.

J

` tâp 1.3. Bai . . 1.11. Trong sân cung d¯iên a vua hôi . nh` . hop . 2n(n ≥ 2) ông quan, , ˜ ´,ng ˘. t tai ˜ quen biê´t không ´ıt hon n ông c´ ´. Chu môi ông quan d o ma ¯a ¯o . d , ,, ,, ˜ b` `˘ng ngu,`o,i xê´p b` minh ra an tr` on c´ o thê xê´p d an 4 ngu`oi sao ¯uo. c môi , ˜ ngu,`o,i d ´,ng gi˜u,a hai ngu,`o,i quen cua m`ınh. cho môi ¯u , ˜ chiê`u 1km. . 1.12. Môt o dang h`ınh vuông môi . khu r`ung thông c´ . , , , Trong r`ung c´ o 4500 cây thông, cây to nhâ´t c´ o d ¯u`ong k´ınh 0,5m. , , , ` ´ ´ ` ˘ng trong khu rung c´ Chung minh ra o ´ıt nhât 60 manh d ¯â´t, diên . t´ıch , 2 ˜ môi manh 200m , không c´ o môt ao. . cây thông n` ´,a . 1.13. Trong môt a s´ ach c´ o 25 ng˘ an. Ta thâ´y c´ o môt an chu . gi´ . ng˘ , `˘ng ´,a sô´ s´ ´,ng minh ra 10 cuô´n, c` on c´ ac ng˘ an kh´ ac chu ach ´ıt hon. Chu , , , , , ´ a c` c´ o ´ıt nhâ´t ba ng˘ an s´ ach chu ung sô´ s´ ach nhu nhau (kê ca nh˜ung ng˘ an không c´ o s´ ach). , , ,, `˘ng tô ho.,p ´nh sô´ ba . 1.14. Tai anh phô´ biên xe ôtô d ¯uo. c d ¯a . môt . th` , ,, `˘ng trên môt ´,ng minh ra ´, ch˜u c´ ai rô`i d ay sô´. Chu ¯ê´n d˜ ¯oan ¯u`ong cu . d . d , , c´ o 11 chiê´c ôtô d ung c´ o hai chiê´c ôtô c´ o c` ung ch˜u ¯i qua th`ı bao gi`o c˜ sô´ tân ung. . c` , ,, ,, ´,n d . 1.15. Môt ong ¯uo. c boc . chuyên tiê´p s´ . chiê´c hô` lo . boi 4 tram , , , , thông tin. Gi˜ua hai tram dung c´ ac trung tâm ph´ at s´ ong . ngu`oi ta xây , , . ,, ,`, , , ´n nhâ´t l` v` a nhân ong, d ong bao phu lo ad on c´ o tâm o ¯uong s´ ¯u`ong tr` . s´

12

,, Chuong 1. Nguyên l´y Ðirichlê v` a v´ı du.

`˘ng vo ´,ng minh ra ´,i bô´n trung tâm trung tâm v` a d¯i qua hai tram. Chu . , ,, , , , ,, ˘. p tram ˘. t hô` s˜e d o c´ ac d an bô. ma ¯oan ¯uo. c phu . gi˜ua cua t`ung ca . th`ı to` s´ ong thông tin.

, , CHUONG

2 ´ HOC SÔ .

, ´ chia sô´ tu. nhiên 2.1. Phep , Trong c´ ac ph´ep t´ınh trên sô´ nguyên: công, tr`u, nhân, chia, th`ı . , ˘. c biêt. ph´ep chia l` a râ´t d o h` ang loat a tâ´t ca ¯a . Ph´ep chia c´ . t´ınh châ´t m` , c´ ac ph´ep t´ınh c` on lai o. V´ı du. c´ ac ph´ep to´ an d ¯ê`u thu. c hiên . không c´ . , , , , , , ´ ´ ´i sô 0 d vo ¯uo. c, nhung riêng ph´ep chia cho sô 0 th`ı không d ¯uo. c. Ph´ep , , , ´i ph´ep chia cho 0. Vo ´i c´ ˘. c biêt chia không chı d ac ph´ep t´ınh công, ¯a . vo . , , , ´ ´ ´i ph´ep chia th`ı tr`u, nhân trên sô nguyên cho ta sô nguyên, nhung vo , ´ ´ không c` ´ ng v`ı không phai l´ t´ınh chât d on d uc n` ao ta c˜ ung nhân ¯o ¯u . , ,, ´ , , d a ¯uo. c sô nguyên sau ph´ep chia. Nh`o nh˜ung di. biêt . cua ph´ep chia m` , , , ´ ` ˘n môt trong to´ an hoc . xây du. ng ha . l´y thuyêt vê ph´ep chia nh˜ung sô´ , ,, nguyên. Nh˜ung v´ı du. v` a b` ai tâp ay c´ o liên quan mât . chuong n` . thiê´t , ´˘c lai gi˜ua ph´ep chia v` a nguyên l´y Ðirichlê, nên ch´ ung ta nha ¯inh . . d ngh˜ıa ph´ep chia: ˜,ng sô´ nguyên, vo´,i b > 0. Ch´ ˘`ng a Cho a v` a b l` a nhu ung ta n´ oi ra chia hê´t cho b, k´y hiêu a b| a, khi tô`n tai . l` . môt . sô´ nguyên q sao cho , , ´c sau d ˘ng thu ´ ng a = bq. d ¯a ¯u , ,, ,, , ˘. c b l` Ch´ ung ta thu`ong goi sô´ a l` a bôi cua b, hoa a uo´c cua a. Sô´ q . . , , ,, goi a thuong sô´ cua ph´ep chia a cho b. Trong ph´ at biêu d ¯inh . ngh˜ıa . l` , `˘ng a trên, nê´u không tô`n tai ao ca, th`ı ch´ ung ta n´ oi ra . môt . sô´ q n`

14

,, Chuong 2. Sô´ hoc .

không chia hê´t cho b v` a k´y hiêu a b 6 | a. . l` , ,, ´,ng minh d T`u d¯inh ung ta d˜ê d` ang chu ac t´ınh châ´t ¯uo. c c´ . ngh˜ıa ch´ sau ´,i moi 1) Vo ung ta c´ o a| a, Ph´ep chia hê´t c´ o . sô´ nguyên a > 0 ch´ , t´ınh phan xa. . ´˘c câ`u. ´ 2) Nêu b| a v` a a|c th`ı b|c- ph´ep chia hê´t c´ o t´ı nh ba 3) Nê´u b| a v` a b|c, th`ı b|( ac). , 4) Nê´u a, b, m, n l` a nh˜ung sô´ nguyên v` a nê´u c| a v` a c|b, th`ı c|(ma + nb). , Ðinh l´y sau d o quan trong cho ph´ep chia môt ¯ây gi˜u vai tr` . . . sô´ nguyên cho môt . sô´ nguyên. , ´ i hai sô´ nguyên bâ´t k` Vo y a v` a b sao cho b > 0, tô`n tai . duy , , ´ ´ ˜ ng sô nguyên q v` nhât nhu a r thoa m˜ an a = bq + r v` a 0 ≤ r < b. , , , C` on râ´t nhiê`u t´ınh châ´t kh´ ac cua sô´ nguyên c˜ ung nhu sô´ thu. c , , ,´, nhung ch´ ung ta không d ng n` ay, m` a chı d` ung c´ ac t´ınh châ´t ¯i theo huo , , , cua sô´ hoc a Nguyên l´y Ðirichlê d ac b` ai to´ an. ¯ê giai c´ . v`

2.2. V´ıdu. , , , . 2.1. Cho k l`a môt a tâp . sô´ tu. nhiên, A l` . ho. p gô`m k + 1 sô´ tu. nhiên. ,, , Chu´ng minh ra ˘`ng c´ o ´ıt nhâ´t môt . hiêu . hai phâ`n tu trong A chia hê´t cho k. , ,, , `,i giai. Goi Lo a c´ ac phâ`n tu cua A, c` on b1 , b2 , . . . , bk+1 . a1 , a2 , ,. . . , ak+1 l` , , ´ a1 = l` a nh˜ung sô´ du cua ph´ep chia c´ ac sô´ trên cho k. Khi d ¯o , ´i c´ kc1 + b1 , a2 = kc2 + b2 , . . . , ak+1 = kck+1 + bk+1 ,vo ac sô´ nguyên c1 , c2 , . . . , ck+1 sao cho 0 ≤ b1 ≤ k − 1, 0 ≤ b2 ≤ k − 1, . . . , 0 ≤ ,, , , ´, ´,i sô´ bk+1 ≤ k − 1. Môt ng vo . phâ`n tu bâ´t k`y as thuôc . A cho tuong u

2.2. V´ıdu.

15

, , , , , , ˜ phâ`n tu,, cua a B. Nhu vây, môi du bs cua n´ o.Goi ac sô´ du l` A . . tâp . ho. p c´ , ,, , ,, , , ´, , , ` ` ´ ´i môt ˘. t tuong ung vo d m tât ca c´ ac ¯uo. c d ¯a . phân tu cua tâp . ho. p B, gô , ,, , , , , , ´ ´ ´ ` sô nguyên t`u 0 d ¯ên k − 1. Nhung sô luo. ng phân tu cua A theo gia ,, thiê´t l` a k + 1, c` on B c´ o sô´ luo. ng k. Theo nguyên l´y Ðirichlê suy ra , ,, , ´ ngh˜ıa l` tô`n tai ac nhau cua A c´ o c` ung sô´ du. Ðiê`u d a, ¯o . hai phâ,`n tu kh´ , ` ´ ´ tôn tai ac nhau s v` a t voi as = kcs + bs v` a at = kct + bs . hai chı sô kh´ , ,, ` sau khi tru d ¯i cho nhau ta d ¯uo. c at − as = k (ct − cs ).

J

, , ˜ sô´ không . 2.2. Cho A môt . tâp . ho. p bâ´t k`y gô`m 101 sô´ tu. nhiên, môi , , , l´ on hon 200. Chu´ng minh ra ˘`ng trong A c´ o ´ıt nhâ´t hai sô´ m` a môt . sô´ n` ay chia hê´t cho sô´ kia. , , , , ,´, ˜ sô´ a cua `,i giai. Môi ´,i k l` i dang a = 2 k b vo a Lo A c´ o thê biêu di˜ên duo . , , ˜ ´i môi sô´ a thuôc sô´ nguyên không âm, c` on b l` a môt sô´ le. Vo . A cho , . ˜ ,, , , ´, , , ˜ ` ´i sô´ b trong su. biêu diên o trên. Ba ˘ng c´ tuong ung vo ach n` ay, môi , , ,, , ,, , , ´, , , ´,i môt ˘. t tuong u phâ`n tu a cua A d ng vo ¯uo. c d ¯a . phâ`,n tu cua tâp . ho. p B , ,, , , , gô`m c´ ac sô´ le gi˜ua 1 v` a 200. Nhung tâp ho. p B chı c´ o 100 phâ`n tu v`ı . , ,, , , , ´,n ho,n sô´ phâ`n tu, cua B. Ta c´ ´p dung vây o thê a . sô´ phâ`n tu cua A lo . ,, ` u kh´ a c nhau a v` a a2 nguyên l´y Ðirichlê, suy ra tô`n tai hai phâ n t 1 . , , , ´, , , ´i c` thuôc a ch´ ung tuong ung vo ung môt . A m` . sô´ cua tâp . ho. p B. Ngh˜ıa k k 2 1 l` a, a1 = 2 b, a2 = 2 b v` a nê´u k1 < k2 , th`ı sô´ a2 chia hê´t cho a1 .

J

, , ˜ sô´ không . 2.3. Cho M l`a tâp a môi . ho. p bâ´t k`y gô`m 75 sô´ tu. nhiên . m` , , , , , , ˜ sô´ tu. nhiên l nho ho,n hoa l´ on hon 100. Chu´ng minh ra ˘`ng v´ oi môi ˘. c ,, , ba ˘`ng 49 tô`n tai o hiêu a l. . hai phâ`n tu cua M c´ . l` , ,, , `,i giai. Goi Lo ac phâ`n tu cua M l` a x1 , x2 , . . . , x75 . K´y hiêu a tâp . c´ . A l` . , , ,, , , ˜ ´ ´ ´ ho. p c´ ac sô´ tu. nhiên t`u 1 d ê n 150. V o i m ôi sô 1, 2, 3 . . . , 75 cho tu o ng ¯ ,, ´,ng vo ´,i c´ u ac sô´ x1 , x2 , . . . , x75 , c` on c´ ac sô´ 76, 77, 78, . . . 150 lâ`n luo. t ´,ng vo ´,i x1 + l, x2 + l, . . . , x75 + l. V`ı xm ≤ 100(m = 1, 2, . . . , 75) v` u a

16

,, Chuong 2. Sô´ hoc .

, , , ´, ˜ phâ`n tu,, cua ´,i ng vo l ≤ 49 th`ı xm + l < 150. Suy ra môi A tuong u ,, , , , , môt ¯ê´n 149. V`ı sô´ phâ`n . ,phâ`n tu cua B gô`m nh˜ung ,sô´ tu. nhiên t`u 1 d ,, ,, , , ´n hon sô´ phâ`n tu cua B, theo nguyên l´y Ðirichlê tô`n tai tu cua A lo . , ,, , , ´, , ` ´i c` hai phân tu kh´ ac nhau cua A, m` a ch´ ung tuong ung vo ung môt . , ,, , , , , ` ´ ` ´i c´ phân tu cua B. Nhung vo ac gi´ a tri. kh´ ac nhau cua m tu 1 d ¯ên 75 , ,, , , ´, , ´ ´ d ac gi´ a tri. kh´ ac nhau cua x1 d ¯uo. c cho tuong ung voi c´ ¯ên x75 trong , , ,, ,, , , , ´, ´ ´,i B. Tuong tu. c´ ac gi´ a tri. cua m o khoang 76 d ng vo ¯ên 150 tuong u , , ´ suy ra tô`n tai c´ ac gi´ a tri. kh´ ac nhau trong khoang c` on lai. ¯o . T`u d . xm v` a xn m` a xm = xn + l, ngh˜ıa l` a xm − xn = l.

J

, ˜,ng sô´ tu., nhiên v` . 2.4. Cho k ≥ 1 v`a n ≥ 1 l`a nhu a A l` a tâp . ho. p gô`m , , , ˜ sô´ n` (k − 1)n + 1 sô´ nguyên duo,ng, môi ay d¯ê`u nho hon hoa ˘. c ba ˘`ng kn. , , , , , , ˜ nhu, Chu´ng minh ra ˘`ng ´ıt nhâ´t c´ o môt o thê biêu diên . phâ`n tu cua A c´ , , ,, tông cua k phâ`n tu trong A. , , , `,i giai. Vo ´,i k = 1 b` ´ ng, ch´ Lo ai to´ an hiên nhiên l` a d¯u ung ta gia thiê´t , `˘ng m ≤ n v` k ≥ 2. K´y hiêu a sô´ nho nhâ´t thuôc a . m l` . A. D˜ê thâ´y ra , , , ´n hon m nhung không ´ ng n − m sô´ thuôc tô`n tai a ch´ ung lo ¯u . d . A m` ,, vuo. t qu´ a kn. , , ´ ng minh b` Ðê chu ai to´ an ch´ ung ta t`ım hai sô´ x v` a y thuôc . A sao , ˜ ´ ´ thuôc cho x = y + (k − 1)m; ngh˜ıa l` a biêu diên môt ao d ¯o . sô n` . A, , ` ´ ´ ˘ng m. Chı ´ c´ th` anh tông k sô hang o k − 1 sô hang ¯o . thuôc . A trong d . ba ` ´ cân t`ım sô x thuôc a x > (k − 1)m v` a x − (k − 1)m thuôc . A m` . A. , Thât o kn − (k − 1)m = . vây, . trong khoang ∆ = ((k − 1)m, kn] c´ k (n − m) + m sô´ nguyên. V`ı k ≥ 2, nên (k − 1)m ≥ m, theo nhân . x´et ban d¯â`u suy ra c´ o nhiê`u nhâ´t n − m sô´ trong ∆ không thuôc A. . ´,a ´ıt nhâ´t s = k(n − m) + m − (n − m) = Ðiê`u n` ay ngh˜ıa l` a A chu , (k − 1)(n − m) + m sô´. Nhung s ≥ n, v`ı (k − 2)(n − m) ≥ 0. Goi . , ´ ´ a1 , a2 , . . . , as thuôc A, v o i ( k − 1 ) m < a ≤ kn, i = 1, 2, . . . , s. Khi d ¯o i .

2.2. V´ıdu.

17

, , a nh˜ung nh˜ung hiêu . a1 − (k − 1)m, a2 − (,k − 1)m, . . . , as − (k − 1)m l` ´ trong sô´ nguyên kh´ ac nhau trong khoang [1, kn]. Nê´u môt ao d ¯o . sô´ n` ch´ ung không thuôc ung ta nhân . A, th`ı theo nguyên l´y Ðirichlê ch´ . , ,, , ´ ´ ng n − 1 sô trong khoang n` d ai A c´ od ay. Nhu ¯uo. c s ≤ n − 1, v`ı ngo` ¯u , ´,c d ´,ng minh s ≥ n. Suy ra tô`n tai ´,i bâ´t d ˘ng thu ˜ chu vây ai vo ¯a ¯a . tr´ . môt . hiêu . ai − (k − 1)m thuôc . A.

J

, , , `, n + 1 sô´ du,o,ng kh´ . 2.5. Chu´ng minh ra˘`ng tu ac nhau nho hon 2n, c´ o , , , ,, thê chon ung ba ˘`ng sô´ thu´ ba. . d¯uo. c ba sô´ sao cho tông hai sô´ trong ch´ , , `,i giai. K´y hiêu ˜ cho. Lo a nh˜ung sô´ d ¯a . 0 < a1 < a2 < . . . < an+1 l` Ch´ ung ta x´et c´ ac hiêu a c´ ac sô´ . , sô´ a2 − a1 , a3 − a1 ,,. . . , an+1 − a1 v` , a2 , a3 . . . , an+1 . V`ı tâ´t ca c´ ac sô´ n` ay d ac sô´ trên ¯ê`u nho hon 2n nên c´ , , , ,, `˘m trong khoang 1, 2, . . . , 2n − 1. Nhu vây ung ta s˜e t`ım d chı na ¯uo. c . ch´ ,, , , `˘ng môt ´, nhâ´t ba ´, hai: ak − a1 = al , môt om thu om thu . sô´ o nh´ . sô´ o nh´ suy ra ak = a1 + al .

J

, , oi môt o dang . 2.6. Chu´ng minh ra˘`ng v´ . sô´ bâ´t k`y n tô`n tai . môt . sô´ c´ . ´ 111 . . 000} m` a chia hêt cho n. | .{z , ˜ sô´ n chu , , , `,i giai. Ch´ a nh˜ung Lo ung ta x´et nh˜ung sô´ 1, 11, 111, . . . , 111 . . 111} v` | .{z ˜, sô´ n chu , ˜ cho gô`m n phâ`n sô´ du khi chia d˜ ay sô´ trên cho n. V`ı d˜ ay sô´ d ¯a ,, , , , , ac nhau khi chia ch´ ung cho n c´ o sô´ tu, nên nh˜ung sô´ du duong kh´ , , ,, luo. ng n − 1. C´ o thê gia thiê´t không c´ o môt ao trong d˜ ay trên . sô´ n` , , ,, , ´ ´ ˜d ´ chia hêt cho n v`ı nêu nguo. c lai ai to´ an d ¯a ¯uo. c giai. Khi d ¯o . th`ı b` ´ s˜e c´ o hai sô trong ch´ ung, v´ı du. 111 . . 111} v` a 111 . . 111}, l > k, | .{z | .{z , ˜ sô´ ˜, sô´ k chu l chu , ´ l−k = m` a khi chia ch´ ung cho n s˜e cho c` ung môt ¯o . sô´ du. Do d

18

,, Chuong 2. Sô´ hoc .

J

111 . . 000} s˜e chia hê´t cho n. | .{z ˜, sô´ 1, k chu ˜, sô´ 0) (l-k chu , , , . 2.7. Cho p l`a sô´ nguyên tô´ l´ on hon 5. Chu´ng minh ra ˘`ng tô`n tai . môt . ´ ´ sô c´ o dang a chia hêt cho p. . 111 . . . 111 m` , `,i giai. Ta x´et d˜ Lo ay sô´ 1, 11, 111, . . . , 111 . . . 1} . Nê´u trong d˜ ay trên | {z ˜, sô´ ) (p chu , , ´, ˜ sô´ vo ´,i sô´ không c´ o sô´ n` ao chia hê´t cho p, th`ı ta cho tuong u ng môi , , , , , du cua ph´ep chia. Tâp ac sô´ du chı c´ o 1, 2, . . . , p − 1 gô`m p − 1 . h,o. p c´ ,, , phâ`n tu (v`ı 0 không thê c´ o trong tâp ay). Nhung v`ı ch´ ung ta c´ op . n` ,, ´ ` ´ sô o dang o c` ung . , trên, nên theo nguyên l´y Ðirichlê tôn tai . hai sô c´ ,, , , ´ ´ ´i m > n. Khi ´ l` sô du. Gia su c´ ac sô d a 111 . . . 1} v` a 111 . . . 1} vo ¯o | {z | {z , , ˜ sô´ ) ˜ sô´ ) (m chu ( n chu ´ d o 1 ≤ n < m ≤ p. V ây ¯ . . . . 1} = 111 . . 000} 111 . . . 1} − 111 | {z | {z | .{z , , , ˜ sô´ ) (n chu ˜ sô´ ) ˜ sô´ 1, n chu ˜, sô´ 0) (m chu (m-n chu = 111 . . . 1} .10n | {z ˜, sô´ ) (m-n chu T´ıch n` ay chia hê´t cho p v`ı ( p, 10) = 1, suy ra

111 . . . 1} | {z ˜, sô´ 1) (m-n chu , , `˘m trong d˜ chia hê´t cho p v` a n´ o c˜ ung na ay o trên. M` a1 ≤ m−n ≤ p , , ˜ ´ ´ ´ mâu thuân voi gia thiêt không c´ o sô n` ao trong d˜ ay chia hê´t cho p.

J

, , , . 2.8. (Ðê` thi Olympic to´an thê´ gi´ oi lâ`n thu´ 14) Cho M l` a tâp . ho. p bâ´t , , , , ˜ sô´ không l´ k`y gô`m 10 sô´ tu. nhiên, môi on hon 100. Chu´ng minh ra ˘`ng , , , ,, , tô`n tai a tông cua c´ ac phâ`n tu trong ch´ ung . hai tâp . ho. p con cua M m` ba ˘`ng nhau.

2.2. V´ıdu.

19

, , , `,i giai. C´ ´,ng minh nê´u tô`n tai o thê chu hai tâp thoa m˜ Lo an kê´t luân . . . , , ,, cua b` ai to´ an, th`ı ta c´ o thê chon o c` ung t´ınh châ´t . d¯uo. c hai tâp . con c´ , , â´y nhung không giao nhau. Thât vây, Cho X, Y l` a hai tâp con cua M . . . , ,, `˘ng nhau. Ch´ c´ o tông c´ ac phâ`n tu ba ung ta k´y hiêu ac phâ`n . X1 gô`m c´ ,, , ,, , , tu cua X m` a không thuôc ac phâ`n . Y. Tuong tu. nhu vây . Y1 gô,`m c´ ,, , tu cua Y m` a không thuôc o r` ang X1 v` a Y1 c´ o tông c´ ac phâ`n . X. R˜ ,, , , `˘ng nhau m` tu ba a không giao nhau. Goi A l` a tâp ho. p moi tâp ho. p . . . . , ,, , ,, ˜ cua con không rông M. Sô´ luo. ng phâ`n tu cua A l` a 210 − 1 = 1023. , ,, , , , Ch´ ung ta x´et tông S c´ ac phâ`n tu cua môt o . tâp . ho. p con nhu vây, . r˜ , ` r` ang S ≤ 91 + 92 + · · · + 100 < 10.100 = 1000. Nhu vây . tôn tai . , , ´ , không qu´ a 1000 tông kh´ ac nhau. K´y hiêu a tâp ac . B l` . ho. p tât ca c´ , , ,, , ,, , , ,, ´ ` ´ sô luo. ng phân tu cua B nho hon 1000 v` tông nhu vây. a nho ¯o . Do d , ,, , ,, , , ´, , , ˜ phâ`n tu,, cua ˘. t tuong u hon sô´ luo. ng phâ`n tu cua A. Ða ng môi tâp . ho. p , , , ,, `˘ng c´ ´,i tông c´ ´p dung A vo ac phâ`n tu cua n´ o. Ta thâ´y ra o thê a nguyên . ,, , ` ´ ´ l´y Ðirichlê o d ac nhau c´ o ¯ây. Suy ra tôn tai . ıt nhât hai tâp . ho. p con kh´ , ,, ` c` ung môt tô ng c´ a c phâ n t u . . , , . 2.9. (Ðê` thi hoc an Câ´p II to` an quô´c 1983) Chu´ng minh . sinh gioi to´ , ra ˘`ng trong c´ ac sô´ tu. nhiên thê´ n` ao c˜ ung c´ o sô´ k sao cho 1983k − 1 chia hê´t cho 105 . , , , `,i giai. Cho k lâ´y gi´ ´,c Lo a tri. t`u 1 d ao biêu thu ¯ê´n 105 + 1 rô`i thay v` ,, 1983k − 1 s˜e nhân a tri. kh´ ac nhau. Chia 105 + 1 sô´ ¯uo. c 105 + 1 gi´ . d , , ,, , , 5 ´ v`ua nhân ed a 105 sô´ du. Do d ¯uo. c nhiê`u nhâ´t l` ¯o . o trên cho 10 , s˜ , , theo nguyên l´y Ðirichlê phai c´ o ´ıt nhâ´t hai sô´ cho c` ung môt . sô´ du. , ,, ´ l` Gia su d a sô´ 1983m − 1 v` a 1983n − 1(m > n). Thê´ th`ı (1983m − ¯o 1) − (1983n − 1) chia hê´t cho 105 m` a (1983m − 1) − (1983n − 1) = , (1983m − 1983n ) = 1983n (1983m−n − 1). Nhung 1983 v`a 105 nguyên , tô´ c` ung nhau, do vây o (1983m−n − 1) chia hê´t cho 105 . Sô´ . phai c´ , k = m − n thoa m˜ an d ai. ¯iê`u kiên . d¯â`u b`

J

J

20

,, Chuong 2. Sô´ hoc .

, ˜,ng sô´ nguyên a, b v` . 2.10. Chu´ng minh ra˘`ng tô`n tai a c, không . nhu , , ` ˜ ` ´ ´ d¯ông th`oi ba ˘ng 0 v` a gi´ a tri. tuyêt a 1000000, . d¯ôi cua môi sô không qu´ √ √ , − 11 thoa m˜ an | a + b 2 + c 3| < 10 .

√ √ , , , , 18 `,i giai. Ða ˘. t S l` a tâp Lo + t 3 vo´,i moi . ho. p cua 10 sô´ thu. c r + s 2√ . √ 6 6 ˘. t d = (1 + 2 + 3d)10 . r, s, t thuôc a d¯a . {0, 1, 2, . . . , 10 − 1} v` , ˜ ` ` ˘m trong khoang 0 ≤ x < d. Chia ´ môi x trong S d Khi d ¯o ¯êu na , ˜ d `˘ng nhau, môi d ay th` anh 1018 − 1 phâ`n ba od ai ¯oan ¯oan ¯ô. d` . n` . nho c´ d 18 e = . Theo nguyên l´y Ðirichlê tô`n tai . hai sô´ trong 10 sô´ 18 − 1 10 , , , `˘m trong c` cua S na ung môt ay k´y hiêu a ¯oan . d . nho. Hiêu . cua hai sô´ n` . l` √ √ 107 ´ ch´ınh l` a+b 2+c 3 d a c´ ac sô´ a, b, c v`ı e < 18 = 10−11 . ¯o 10

J

` tâp 2.3. Bai . , , ˜ sô´ không . 2.11. Cho A l`a tâp . ho. p bâ´t k`y gô`m 201 sô´ tu. nhiên, môi , , ,, , , `˘ng A chu ´ ng minh ra ´ a ´ıt nhâ´t hai sô´, m` vuo. t qu´ a 300. Chu a ty sô´ cua , ch´ ung l` a l˜ uy th`ua bâc . ba. , , . 2.12. Cho k l`a sô´ tu. nhiên bâ´t k`y, c`on a v`a b l`a nh˜ung sô´ nguyên , `˘ng nê´u M l` ´,ng minh ra sao cho a ≤ b v` a b − a < 2k − 2. Chu a tâp ho. p . , , , , `˘m trong khoang [ a, b], v` k sô´ tu. nhiên na a l l` a sô´ tu. nhiên thoa m˜ an ,, , , ´ ` 1 ≤ l ≤ 2k + a − b − 2, th`ı c´ o ´ıt nhât môt . hiêu . nh˜ung phân tu cua , ´i l. M tr` ung vo ,, ˜ sô´ a1 , a2 , a3 , . . . , a41 , m` ˜ phâ`n tu,, chı, d . 2.13. Cho dây a môi ¯uo. c tao . , ,, ,, ,, ´ ´ c´ boi sô´ 1 v` a, sô´ 2, trong d o ´ıt nhâ´t 21 sô´ chı d b o i c´ a c sô 1. ¯o ¯uo. c tao ., , ,, , ` ´ ng minh ra ˘ng tô`n tai Chu ay c´ o tông . môt . sô´ phâ`n tu liên tiê´p cua d˜ ` ˘ng d ´ ng 20. ba ¯u , `˘ng tô`n tai ´,ng minh ra . 2.14. Chu . môt . sô´ tu. nhiên n, sao cho sô´

2.3. B` ai tâp .

21

´ ng nê´u ta thay 139 111 . . . 1} chia hê´t cho 139. (B` ai to´ an c` on d ¯u | {z ˜, sô´ ) (n chu `˘ng môt ´,i 10). ba ung nhau vo . sô´ nguyên tô´ c` ,, , `˘ng trong moi ´,ng minh ra . 2.15. Chu . sô´ tao . boi 100 ch˜u sô´ N tô`n tai . ´ ´ môt sô chia hê t cho 1967. . ,, `˘ng bao gi`o, c˜ ´,ng minh ra . 2.16. Chu ung t`ım d ¯uo. c sô´ 19971997. . . 19970. . . 0 chia hê´t cho 1998. , `˘ng c´ ´,ng minh ra . 2.17. Chu o môt sô´ tu. nhiên chia hê´t cho 1997, m` a . , , ´ ´ ´ ˜ bôn chu sô cuôi c` ung cua n´ o l` a 1998. `˘ng nê´u c´ ´,ng minh ra . 2.18. Chu ac sô´ nguyên m v` a n nguyên tô´ c` ung ,, ´ , k ´ nhau th`ı t`ım d ¯uo. c sô tu. nhiên k sao cho m − 1 chia hêt cho n.

22

,, Chuong 2. Sô´ hoc .

, , CHUONG

3 ´ ˜ SÔ DAY

˜ y sô´ vô han 3.1. Nguyên l´ y Ðirichlê cho da . ,´, Trong phâ`n n` ay ch´ ung ta x´et nguyên l´y Ðirichlê duo i dang: . , , ˘. t vô han ung ng˘ an k´ eo m` a ch´ ung ta d Nê´u c´ o h˜uu han ¯a . nh˜ . , , ´ ´ ˜ ´ , th`ı ´ıt nhât c´ ao d o môt an k´ eo chua vô han nhung vât ¯o . v` . ng˘ . , ˜ ˜ c´ o. nhung vât ¯a . d , , , ,, `˘ng nguyên l´y n` Ch´ ung ta d˜ê c´ o cam tuong ra ay l` a hiên nhiên nên , , , `˘ng phan chu ´,ng c´ ´ ng minh nguyên l´y n` ´ıt ch´ o. Ba o thê chu ay u ´y d ¯ê´n n´ ,, , ´ ng. Trong sô´ hoc, l` ad o liên quan d a ¯u ¯ê´n vô han . tâp . ho. p c´ . phâ`n tu l` , ˜ dây sô´. Ch´ ung ta biê´t râ´t nhiê`u d˜ ay sô´ d ay câ´p sô´ công, d˜ ay ¯ep . nhu d˜ . , , ˘. c d˜ câ´p sô´ nhân, d˜ ay c´ ac sô´ nguyên tô´, hoa ay Fibonaxi,. . . Chuong , , , , ´p dung n` ay ch´ ung ta chı quan tâm d d at biêu trên d ¯ê´n a ¯iê`u ph´ ¯ê giai . ˜ sô´. Nh˜u,ng tâp c´ ac b` ai to´ an liên quan d ac b` ai ¯ê´n dây . vô han . trong c´ ,´, , to´ an duoi d ac d˜ ay sô´. ¯ây ta x´et nhu c´

3.2. V´ı du. ˜, sô´ cuô´i c` . 3.1. X´et d˜ay sô´ 6, 62 , 63 , 64 , 65 , . . . , 6n , . . . v`a viê´t 4 chu ung , , ` ´ ´ cua c´ ac sô n` ay 0006, 0036, 0216, 1296, 7776, . . .. Chung minh ra ˘ng ba ˘´t , , ˜ vu ˜ tuâ`n ho` ` môt ` a lâp ´ dây d¯â`u tu ao d¯o a dây an. . sô´ n0 n` . l` , , ,, 4 `,i giai. V`ı tô`n tai Lo ach chon ac nhau . h˜uu han . sô´ luo. ng (10 ) c´ . kh´

24

,, Chuong 3. D˜ ay sô´

, ,, ´˘c cha ´˘n t`ım d ˜ cho cha ay d c´ ac sô´ c´ o 4 ch˜u sô´, nên trong d˜ ¯a ¯uo. c hai , ,, c´ ach chon o c` ung 4 ch˜u sô´ cuô´i. C´ o ngh˜ıa l` a t`ım d ¯uo. c hai sô´ n0 . c´ , ´,i ch´ v` a n0 + t m` a vo ung th`ı 6n0 v` a 6n0 +t+1 c´ o c` ung 4 ch˜u sô´ cuô´i , (6n0 +t+1 − 6n0 = 104 .6k). N´oi chung, ch˜u sô´ 6n v`a 6n+t vo´,i bâ´t k`y , n > n0 s˜e c´ o c` ung 4 ch˜u sô´ cuô´i (6n+t − 6n = 104 .6n−n0 ).

J

. 3.2. (Ðê` thi To´an Olympic quô´c tê´ lâ`n 17 n˘am 1975) Cho , , a1 , a2 , . . . , an , . . . l` a d˜ ay t˘ ang nga ˘. t c´ ac sô´ tu. nhiên. Chu´ng minh ra ˘`ng , , , ,, , , ˜ trên c´ ˜ duo´i dang vô han ac phâ`n tu an cua dây o thê biêu diên . . c´ ,, , , , ˜ ng sô´ nguyên duong v` a y l` a nhu a p 6= q. an = xa p + yaq ,o d¯ây x v` , , , `,i giai. Nê´u a1 = 1 kê´t luân Lo ai to´ an l` a hiên nhiên. Thât . cua ,b` . vây, . , ˜ ´ ´i moi vo n ≥ 3 sô h ang a c´ o biê u di ên d ang a = a + ( a + n n n n −1 . . . an−1 ) = 1.an−1 + ( an − an−1 ).a1 c´ o t´ınh châ´t mong muô´n. Ch´ ung ta , , , , ´ ng minh tô`n tai ´n hon 1 sao cho vô han s˜e chu ac sô´ hang . ,chı sô´ p lo . c´ . , , , , ´ ´i dang ´ ˜ cho c´ cua d˜ ay d o thê viê´t duo xa + ya v o i c´ a c sô nguyên ¯a p 1 . , ,, , , ´, , ˜ sô´ hang ´,i ˘. t tuong u duong th´ıch ho. p x v` a y. Môi cua d˜ ay ta d ng vo ¯a . , , , , sô´ du cua n´ o khi chia ch´ınh n´ o cho a1 . Tâp ac sô´ hang . ,ho. p tâ´t ca c´ . , , , , ´ ´ cua d˜ ay l` a vô han, c` o n tâ t c a c´ a c kh a n˘ a ng c ua sô du khi chia c´ a c sô´ . , ,, , , ` ´ ng to ra ˘ng vô han ´ chu hang a h˜uu han. ¯o . cho a1 l` . Ðiê`u d . phâ`n tu ´,i n1 < n2 < . . . < nk < . . . an1 , an2 , . . . , ank , . . . , vo , , cho c` ung môt sô´ du r khi chia cho a1 . Không mâ´t t´ınh tông qu´ at . , , , , , , ta gia thiê´t n1 > 1, v`ı trong tru`ong ho. p nguo. c lai ac sô´ . ta x´et c´ , an2 , an3 , . . . , ank , . . . c˜ ung l` a d˜ ay vô han a cho c` ung sô´ du r khi chia . v` , , , ´i moi cho a1 . Vo . k = 1, 2, . . . tô`n tai . sô´ nguyên duong xk sao cho ank = ´ a n k − a n1 = ( x k a 1 + r ) − ( x 1 a 1 + r ) = ( x k − x 1 ) a 1 xk a1 + r. Khi d ¯o , , ´,c ank = an1 + ( xk − x1 ) a1 = ´i moi ˘ng thu suy ra vo od ¯a . k ≥ 2 ta c´ , , 1.an1 + ( xk − x1 ) a1 . Ngh˜ıa l` a, nh˜ung sô´ an2 , an3 , . . . , ank , . . . ., c´ o biêu , , , `i hoi. Thât di˜ên v` a c´ ac t´ınh châ´t nhu b` ai to´ an d a n1 ¯o . vây, . chı sô´ 1 v`

3.2. V´ı du.

25

, , , , ´, , kh´ ac nhau v`ı theo c´ ach chon n hon 1. Chı c` on phai . trên n1 thu. c su. lo , ,, `˘ng sô´ xk − x1 l` ´,i k ≥ 2, d ˘ng d¯inh ´ kha a sô´ nguyên duong vo ¯iê`u d ¯o . ra , ´ ng v`ı t`u n1 < nk suy ra x1 < xk . d ¯u , , . 3.3. Cho sô´ tu. nhiên bâ´t k`y k. Chu´ng minh ra˘`ng tô`n tai tô´ . sô´ nguyên , , p v` a môt ay sô´ tu. nhiên t˘ ang nga ˘. t a1 , a2 , . . . , an , . . . sao cho tâ´t ca c´ ac . d˜ ,, , , ` ´ ˜ ng sô nguyên phân tu cua d˜ ay p + ka1 , p + ka2 , . . . , p + kan , . . . l` a nhu ´ tô. , , , `,i giai. K´y hiêu ´,i moi Lo a tâp ac sô´ nguyên tô´. Vo . P l` . ho. p tâ´t ca c´ . , ´ ´ i = 0, 1, . . . , k − 1 k´y hiêu P l` a t âp h o p c´ a c sô nguyên tô m` a khi i . . . , `˘ng moi `˘m trong môt chia cho k c´ o sô´ du i. D˜ê thâ´y ra . sô´ nguyên tô´ na . ,, , ´ ´ trong c´ ac tâp h o p P , P , P , . . . , P . B o i v` ı sô nguyên tô l` a vô h an, 0 1 2 k −1 . ,. . , ´ ´ ´,a ´ vây ıt nhâ t ph ai c´ o m ôt trong sô c´ a c t âp h o p P , P , P , . . . , P ch u k −1 . . , . . 0 1 2 ,, ´,a vô han vô han tô´. Gia su Pi chu a k´y hiêu a phâ`n . sô´ nguyên . sô´ v` . p l` , ,, , ´,i ´ moi tu nho nhâ´t cua n´ o. Khi d o dang ¯o . sô´ x thuôc . Pi c´ . ,x = p + ka vo , , , môt a c´ ac phâ`n tu cua Pi xê´p theo . sô´ tu. nhiên a. Lâ´y x1 , x2 , x3 , . . . l` xn − p ˜ , `˘ng ´, tu., lo ´,n dâ`n. Vo ´,i moi ˘. t an = thu . Dê thâ´y ra ¯a . sô´ tu. nhiên n d k sô´ nguyên tô´ p v` a d˜ ay a1 , a2 , . . . , an , . . . c´ o t´ınh châ´t mong muô´n. , , . 3.4. Cho f l`a d¯a thu´c k d¯ô´i sô´ v´ oi hê. sô´ nguyên v` a a1 , a2 , . . . , an , . . . l` a , , , ´ ˜ ng sô nguyên thoa m˜ d˜ ay nhu an hê. thu´c an+1 = f ( a, a2 , . . . , an−k+1 ), ,, , ´ v´ oi moi a n ≥ k. Ch´ ung ta x´et sô´ duong bâ´t k`y m . sô nguyên n, k m` , , , , v` a v´ oi moi a sô´ du không âm nho nhâ´t cua . n = 1, 2, . . .. K´y hiêu . an l` , an theo mô d¯un m. Chu´ng minh ra ˘`ng d˜ ay a1 , a2 , a3 , . . . , an , . . . . l` a d˜ ay ` tuân ho` an. , , ,, `,i giai. Ch´ ´,c ˘ng d Lo ung ta s˜e su dung kha sau: Nê´u g l` a d ¯inh ¯a thu . . ´,i hê. sô´ nguyên v` k d a x1 , x2 , . . . , xk , y1 , y2 , . . . , yk l` a c´ ac sô´ ¯ô´i sô´ vo nguyên sao cho x1 ≡ y1 (mod m), x2 ≡ y2 (mod m), . . . , xk ≡ yk (mod m), th`ı g( x1 , x2 , . . . , xk ) ≡ g(y1 , y2 , . . . , yk ) (mod m).

J

J

,, Chuong 3. D˜ ay sô´

26

, `˘ng môt Moi cua d˜ ay ba ac sô´ a1 , a1 , a3 , . . . , an , . . . . sô´ hang . . trong c´ , , ´˘p thu ´ tu., gô`m k phâ`n tu, 0, 1, . . . , m − 1. Ch´ ung ta x´et c´ ac bô. sa

( a1 , a2 , . . . , ak ), ( a2 , a3 , . . . , ak+1 ), . . . , ( an , an+1 . . . , an+k−1 ).. , , ,, ´˘p nhu, vây, C´ o tâ´t ca vô han ac bô. k sô´ . bô. sa . nhung sô´ luo. ng c´ , , (α1 , α2 , α3 , . . . , αk ), vo´i 0 ≤ αi ≤ m − 1, i = 1, 2 . . . , k l`a h˜uu han . , `˘ng mk theo l´y thuyê´t tô ho.,p). Theo nguyên l´y Ðirichlê tô`n tai (ba hai . , chı sô´ i v` a j, i < j sao cho a i = a j , a i +1 = a j +1 , . . . , a i + k −1 = a j + k −1 ˘. c l` hoa a x1 ≡ y1 (mod m), x2 ≡ y2 (mod m) , . . . , xk ≡ yk (mod m). , , T`u d ay a1 , a2 , a3 , . . . , an . . . l` a tuâ`n ho` an (chu k`y cua n´ o ¯ây suy ra d˜ ,´, ´ , , , ´ c vo ´i hê. sô´ nguyên nên l` a uoc sô cua j − i). Thât ad ¯a thu . vây, . v`ı f l` , ´ ng minh trên ch´ theo c´ ach chu ung ta c´ o f ( ai+k−1 , ai+k−2 , . . . , ai ) ≡ f ( a j+k−1 , a j+k−2 , . . . , a j ) (mod m) , =⇒ ai+k ≡ a j+k (mod m) hoa˘. c l`a ai+k ≡ a j+k . Biê´n d¯ôi môt ut d˜ê . ch´ , , , `˘ng vo ´ c sau an+(i− j) ≡ an . ´i moi ˘ng thu thâ´y ra od ¯a . n ≥ i ta c´

J

, , . 3.5. Cho d˜ay x1 , x2 , . . . , xn , . . . . d¯uo.,c x´ac d¯inh . theo công thu´c sau x1 = 1, x2 = 0, x3 = 2, xn+1 = 2xn−1 + xn−2 , n ≥ 3. ,, , , , Chu´ng minh ra ˘`ng v´ oi moi . sô´ tu. nhiên m tô`n tai . hai phâ`n tu liên tiê´p , cua d˜ ay m` a ch´ ung d¯ê`u chia hê´t cho m. , , `,i giai. Công thu ´,c hô`i quy trên c´ Lo o thê viê´t lai . xn−2 = xn+1 − 2xn−1 (3.1) , , , , `˘ng d˜ ´,c ´ chı ra ra ay c´ o kha n˘ ang ph´ at triên vê` ph´ıa tr´ ai, tu T`u d¯o ´,i n ≤ 0. V´ı du. vo ´,i n = 2, 1, 0 ch´ l` a x´ ac d xn vo ung ta nhân ¯inh . . , ,, , ` ˘ d u o c x = 0, x = 0, x = 1. Nhu m uc 3.4 ch ı ra r a ng d˜ a y ¯ . 0 −2 −1 .

3.2. V´ı du.

27

, , , , ´, ng x1 , x2 , . . . , xn , . . . theo x1 , x2 , . . . , xn , . . . gô`m nh˜ung sô´ du tuong u , ˜ ` ´,c (3.1) suy ra môi môdd un m, l` a d˜ ay tuân ho` an. T`u công thu , ,, , , phâ`n tu cua d˜ ay { xn } v` a suy ra ca { xn } x´ ac d duy nhâ´t t`u 3 ¯inh . , ,, , , ´c n´ ´ nê´u (r1 , r2 , . . . , rk ) l` phâ`n tu truo o. Khi d a phâ`n chu k`y cua d˜ ay ¯o , ` ` ` x1 , x2 , . . . , xn , . . . th`ı phân n` ay s˜e chuyên d an vê ph´ıa tr´ ai ¯ông . tuân ho` , cua d˜ ay . . . , x−3 , x−2 , x−1 , x0 , x1 , x2 , . . . v` a s˜e c´ o dang . . . . , r1 , r2 , . . . , r k , r1 , r2 , . . . , r k , r1 , r2 , . . . , r k . . .

(3.2)

, `˘ng x−1 = x0 = 0, suy ra x−1 = x0 = 0. T`u, (3.2) Bây gi`o ta ch´ u ´y ra ,, , `˘ng d˜ ´,a vô sô´ ca ˘. p phâ`n tu suy ra ra ay c´ ac sô´ du theo môdd un m chu `˘ng không. N´ ˘. p sô´ liên tiê´p liên tiê´p ba oi c´ ach kh´ ac tô`n tai ac ca . vô sô´ c´ , ,, ˜ ˘. p d cua d˜ ay x1 , x2 , . . . , xn , . . . m` a môi phâ`n tu trong ca ¯ê`u chia hê´t cho m. , , , ngh˜ıa ba ˘`ng c´ ac d¯a ˘ng thu´c F1 = . 3.6. D˜ay sô´ Fibonaxi d¯uo.,c d¯inh . , F2 = 1, Fn+2 = Fn+1 + Fn , n ≥ 1. Chu´ng minh ra ˘`ng ´ıt nhâ´t môt . trong , ,, 1.000.000.000 phâ`n tu d¯â`u tiên cua d˜ ay chia hê´t cho 10.000.

J

, , , `,i giai. Tu,o,ng tu., nhu, 3.5 ch´ Lo ung ta x´et c´ ac sô´ du cua c´ ac sô´ , , , ´ ng o, vi. ˜ cho khi chia cho 10.000. K´y hiêu trong d˜ ay d ¯a ¯u . sô´ du d ´, k khi chia cho 10 000 l` ´ th`ı r1 = 1, r2 = tr´ı thu a rk . Khi d ¯o 1, r3 = 2, r4 = 3, . . . . . . .rk = rk−1 + rk−2 . R˜ o r` ang c´ o 10.000 sô´ , 2 ˘. p sô´ ´ c´ du kh´ ac nhau do d o 10000 = 100000000 (tr˘ am triêu) ca ¯o . , , ˘. p sô´ du (r1 , r2 ), (r2 , r3 ), (r3 , r4 ) . . . du kh´ ac nhau. X´et 100000001 ca . . . (r100000001 , r100000002 ). Theo nguyên l´y Ðirichlê trong sô´ n` ay c´ o ´ıt , , , , ´ c l` ´i n, p ˘p sô´ tr` nhâ´t 2 ca ung nhau, tu a t`ım d a p vo ¯uo. c hai sô´ n v` , . , , , d ¯ê`u nho hon 100000002,n nho hon p sao cho rn = r p , rn+1 = r p+1 . , , , , , , Nhung nê´u biê´t sô´ du cua tông hai sô´ v` a sô´ du cua môt . sô´ th`ı sô´ , ,, du kia c˜ ung t´ınh d o r n −1 = r p −1 , r n −2 = r p −2 , . . . . ¯uo. c. V`ı vây . ta c´ ´,c cho d công thu ¯ê´n khi r2 = 1 = r p−n+2 , r1 = 1 = r p−n+1 , Áp dung .

,, Chuong 3. D˜ ay sô´

28

,, , ´,i p − n ≤ 100000001 − 1 = sô´ du hô`i qui o trên ta c´ o r p−n = 0 vo , , ´,ng o, vi. tr´ı p − n s˜e thoa m˜ 100000000. Ngh˜ıa l` a sô´ d an d ai ¯u ¯iê`u kiên . b` ´ ra, chia hêt cho 10 000.

J

. 3.7. (Ðinh l´y Fecma) Nê´u môt . . sô´ nguyên tô´ p không chia hê´t sô´ , , ´ng a p−1 ≡ 1( mod p). nguyên a, th`ı d¯a ˘ng thu´c sau d¯u , , , `,i giai. Ch´ ´,ng minh mênh Lo ung ta chu d at hon. Cho m > 1 ¯ê` tông qu´ . , ´,i m. Ch´ l` a sô´ tu. nhiên bâ´t k`y v` a a l` a sô´ nguyên tô´ c` ung nhau vo ung , , , ta x´et d˜ ay nh˜ung l˜ uy th`ua liên tiê´p cua a a1 , a2 , a3 , . . .

(3.3)

v` a k´y hiêu . r ,r ,r ,... (3.4) ,1 2 3 , , , , ´, l` a nh˜ung sô´ du tuong u ng cua (3.3) khi chia cho m, ngh˜ıa l` a ak ≡ rk

(mod m), 1 ≤ rk ≤ m − 1.

,, ,, , ´ sô´ luo. ng c´ Khi d ac sô´ trong (3.3) l` a vô han, c` on nh˜ung sô´ o (3.4) ¯o . , , , ,, chı c´ o thê nhân a tri. trong 1, 2, 3, . . . , m − 1 nên sô´ luo. ng l` a . nh˜ung gi´ , , , , ´ h˜uu han. du rk s˜e c´ o . Suy ra theo nguyên l´y Ðirichlê, gi˜ua nh˜ung sô , ´ ´ ` ´ ´,i ´ıt nhât hai sô tr` ung nhau; n´ oi c´ ach kh´ ac tôn tai a j vo . hai chı sô i v` , ´ ch´ i 6= j sao cho ri = r j . Khi d ung ta c´ o ai ≡ a j ( mod m). Theo gia ¯o ,, ´,i i 6= j ch´ thiê´t ( a, m) = 1, vo ung ta nhân ¯uo. c ai− j ≡ 1( mod m). . d , , ´,c sau ˘ng thu Ch´ ung ta c´ o kê´t luân ¯a . tô`n tai . sô´ tu. nhiên l sao cho d ´ ng: d ¯ây d ¯u al ≡ 1 (mod m) (3.5) - Sô´ l trong (3.5) không x´ ac d on tô`n tai ¯inh . duy nhâ´t, thâm . ch´ı c` . , , ´ ´ vô sô sô tu. nhiên l thoa m˜ an (3.5). , ,`, , - Trong truong ho. p m = p l` a sô´ nguyên tô´, Fecma t`ım ra l c´ o thê chon a sô´ p − 1. . l`

3.2. V´ı du.

29

, , ,, , `˘ng l c´ ´,ng minh ra o thê chon -Tru`ong ho. p m bâ´t k`y th`ı Ole chu a . l` , , ,, ´ ´ ` h` am chı sô cua m (ch´ ung ta không xem x´et vân d ay o d ¯ê n` ¯ây, d ¯ôc . , , , ´ ´ ´ ´ cuôn s´ gia c´ o thê t`ım trong bât cu ach sô hoc ao). . n`

J

˜ vô han . 3.8. Cho x1 , x2 , x3 , . . . l`a dây ac sô´ nguyên v` a k l` a môt . c´ . sô´ , , , , ˜ sô´ gô`m nhu ˜ ng phâ`n tu, tu. nhiên bâ´t k`y. Chu´ng minh ra ˘`ng tô`n tai dây . , , , ˜ m` liên tiê´p cua dây, a tông cua ch´ ung chia hê´t cho k. , , , ,, , `,i giai. Ch´ ´i han Lo ung ta c´ o thê gio lai, gi˜ua moi bô. k phâ`n tu liên . . . , , ,, ,, ˜ c´ tiê´p cua dây o thê chon d sô´ phâ`n tu c´ o t´ınh châ´t mong ¯uo. c môt . . , , , ,, muô´n. Ðê d ¯on gian ta xem x´et k phâ`n tu d ¯â`u tiên x1 , x2 , x3 , . . . , xk . , Ch´ ung ta x´et tông S1 = x 1 , S2 = x 1 + x 2 , S3 = x 1 + x 2 + x 3 , . . . , S k = x 1 + x 2 + · · · + x k , ,, ´ trong sô´ trên chia hê´t cho k, th`ı b` Nê´u môt tông n` ao d ai to´ an d ¯o ¯uo. c . , ,, ,, giai. Nguo. c lai, ac sô´ S1 , S2 , . . . , Sk (c´ o sô´ luo. ng k) khi chia cho k . c´ ,, , , d ac sô´ du 1, 2, 3, . . . , k − 1. T`u nguyên l´y Ðirichlê suy ra c´ o môt ¯uo. c c´ . , , ˘. p chı sô´ i v` a j, 1 ≤ i < j ≤ k, m` a c´ ac tông Si v` a S j cho c` ung môt sô ca . ´ , ,, , ´ tông c´ du khi chia cho k. Khi d ac phâ`n tu liên tiê´p xi+1 , xn+2 , . . . , x j ¯o , ˜ cho chia hê´t cho k, v`ı xi+1 + xn+2 + · · · + x j = S j − Si . cua d˜ ay d ¯a

J

, , ˜, sô´. Chu´,ng minh ra . 3.9. Cho d˜ay vô han ac chu ˘`ng v´ oi moi . c´ . sô´ tu. , nhiên n, nguyên tô´ c` ung nhau v´ oi 10, trong d˜ ay vô han . trên tô`n tai . ,, , , ´ ´ ´ ´ ´t ˜ ˜ môt nh´ o m ch u sô liên tiê p, m` a sô t ao b o i c´ a c ch u sô trong nh´ o m (viê . . , , , , , , , ´ ´ theo thu´ tu. chı sô l´ on d¯u´ng truo´c) chia hêt cho n. , , `,i giai. Cho d˜ ung ta x´et c´ ac sô´ Lo ay c´ ac ch˜u sô´ a1 , a2 , . . . , an , . . .. Ch´ A 1 = a 1 , A 2 = a 2 a 1 , . . . , A n = a n a n −1 . . . a 1 , . . . , A n +1 = a n +1 . . . a 1 . , , ,, , ,, , V`ı sô´ luo. ng nh˜ung sô´ n` ay l` a n + 1, c` on sô´ luo. ng kha n˘ ang cua sô´ du khi chia ch´ ung cho n l` a n, nên theo nguyên l´y Ðirichlê tô`n tai . ´ıt nhâ´t

,, Chuong 3. D˜ ay sô´

30

, ung l` a Ai v` a A j , (i < j). Khi hai sô´ cho c` ung môt . ch´ . sô´ du ta k´y hiêu ´ ´ hiêu d oi c´ ach kh´ ac ¯o . A j − Ai chia hêt cho n. Hay n´ A j − Ai = a j . . . a1 − ai . . . a1 = a j . . . ai−1 .10 j−i+1 v`ı (n, 10) = 1, nên a j . . . ai−1 chia hê´t cho n.

J

. 3.10. Cho k l`a sô´ nguyên du,o,ng bâ´t k`y v`a x1 , x2 , . . . , x n , . . . y1 , y2 , . . . , y n , . . . , ˜ ´ ˜ ng chuôi sô nguyên bâ´t k`y. Chu´,ng minh ra l` a nhu ˘`ng tô`n tai ˘. p . vô sô´ ca , , , ˜ ´ chı sô (i, j), v´ oi i < j sao cho môi tông x i +1 + x n +2 + · · · + x j ; y i +1 + y n +2 + · · · + y j d¯ê`u chia hê´t cho k. , , , `˘ng trong bô. sô´ k2 phâ`n tu, liên tiê´p `,i giai. Chı câ`n chu ´,ng minh ra Lo , , , ,, , ˜ trên c´ ´,i t´ınh châ´t d ˜ chı ra. V`ı vây cua 2 dây o thê chon d ¯uo. c tông vo ¯a . . , , ,, ˜ d ˜ cho. ac chuôi ch´ ung ta chı quan tâm d ¯â`u tiên cua c´ ¯a ¯ê´n k2 phâ`n tu d , , , `˘ng c´ Ba ach tông qu´ at h´ oa c´ ach giai b` ai to´ an 3.8, lâ´y tông S1 = x 1 , S2 = x 1 + x 2 , S3 = x 1 + x 2 + x 3 , . . . , S k 2 = x 1 + x 2 + · · · + x k 2 T1 = y1 , T2 = y1 + y2 , T3 = y1 + y2 + y3 , . . . , Tk2 = y1 + y2 + · · · + yk2 ,, ˜ m = 1, 2, 3, . . . , k2 d ´,ng ca ´,i môi ´,i ca ˘. t tuong u ˘. p (Sm , Tm ) vo ˘. p Vo ¯a , , , ( RSm , RTm ) cua nh˜ung sô´ du, khi chia Sm v`a Tm cho k. V`ı RSm , , , v` a RTm l` a môt ac sô´ 0, 1, 2, . . . , k − 1, nên tô ho. p tâ´t ca . trong c´ c´ ac dang ac nhau ( RSm , RTm ) l` a không qu´ a k2 . Nê´u tô`n tai . , kh´ ,. , ´ ´ môt ch ı sô m, sao cho ( RS , RT ) tr` u ng v o i ( 0, 0 ) , th` ı m oi tô ng m m . . Sm = x1 + x2 + · · · + xm v` a Tm = y1 + y2 + · · · + ym d ¯ê`u chia hê´t cho , ˘. p sô´ ( RSm , RTm ), m = 1, 2, . . . .., k2 k. V`ı nê´u không nhu vây, ac ca . th`ı c´ , , ,, c´ o nhiê`u nhâ´t l` a k2 − 1 kha n˘ ang kh´ ac nhau. Nhung sô´ luo. ng

3.3. B` ai tâp .

31

, `˘ng ˘. p sô´ n` ay l` a k2 suy ra c´ o ´ıt nhâ´t hai trong ch´ ung ba nh˜ung ca , nhau. N´ oi c´ ach kh´ ac, tô`n tai a j, sao cho 1 ≤ i < . hai chı sô´ i v` ,, , 2 ˜ sô´ j ≤ k v` a ( RSi , RTi ) = ( RS j , RTj ). Trong tru`ong ho. p n` ay môi xi+1 + xn+2 + · · · + x j = S j − Si ; yi+1 + yn+2 + · · · + y j = Tj − Ti d ¯ê`u chia hê´t cho k. , ,, ´: Ðây l` Ch´ uy a b` ai to´ an tông qu´ at h´ oa b` ai to´ an 3.8. Mo rông kê´t . , qua n` ay c´ ac ban ay xem v` a l` am b` ai tâp 3.15. . h˜

J

` tâp 3.3. Bai .

, , , , . 3.11. C´o tô`n tai a c´ ac ch˜u sô´ cuô´i c` ung cua n´ o . lu˜y th`ua cua sô´ 3 m` l` a 0001 không ? , , ,, . 3.12. Cho F l`a tâp a x1 , x2 , . . . . h˜uu han . nh˜ung sô´ nguyên duong v` ,, , ˜ , xn , . . . v` a y1 , y2 , . . . , yn , . . . l` a hai dây vô han nh˜ung phâ`n tu thuôc . . , , `˘ng tô`n tai ´ ´,ng minh ra ˜ F. Chu nh u ng ch ı sô i v` a j, i < j sao cho t´ ıch . , , cua xi+1 , xi+2 , . . . , x j v` a yi+1 , yi+2 , . . . , y j l` a môt o l˜ uy th`ua bâc . sô´ c´ . k. , `˘ng . 3.13. Cho u1 , u2 , . . . , un , . . . l`a d˜ay nh˜ung sô´ nguyên x´ac d¯inh . ba , , 2 ´ c u1 = 39, u2 = 45, un+2 = un+1 − un (n ≥ 1). Chu ´ ng minh công thu ,, , `˘ng 1986 chia hê´t cho vô sô´ nh˜ung phâ`n tu trong d˜ ra ay n` ay. , , . 3.14. Cho k l`a môt ay x1 , x2 , . . . , xn , . . . thoa m˜ an c´ ac . sô´ tu. nhiên. D˜ , 1 ´,c x0 = 0, x1 = 1 v` ´,i moi ˘ng thu d a xn = ( xn+1 − xn−1 ) vo ¯a . n ≥ 1. k `˘ng gi˜u,a nh˜u,ng sô´ x1 , x2 , . . . , x1986 tô`n tai ´,ng minh ra Chu a . hai sô´ m` , t´ıch cua ch´ ung chia hê´t cho tich 19.86. , . 3.15. Cho k l`a sô´ nguyên duo,ng v`a x11 , x21 , . . . , xn1 , . . . x12 , x22 , . . . , xn2 , . . .

,, Chuong 3. D˜ ay sô´

32

............... x1s , x2s , . . . , xns , . . . , ´,i ˘. p chı sô´ (i, j), vo ´ tô`n tai l` a s d˜ ay sô´ nguyên. Khi d vô han c´ ac ca ¯o . . , i < j sao cho c´ ac tông sau d ¯ây xi1+1 + xi1+2 + · · · + x1j

d ¯ê`u chia hê´t cho k.

xi2+1 + xi2+2 + · · · + x2j ..................... xis+1 + xis+2 + · · · + x sj

, , CHUONG

4 H`INH HOC .

4.1. V´ı du.

, , ´,p Trong sô´ c´ ac b` ai to´ an h`ınh hoc trong to´ an hoc tô ho. p c´ o môt . . . lo , `˘ng phu,o,ng ph´ b` ai to´ an giai ba ap Ðirichlê râ´t thuân a r˜ o r` ang. . tiên . v` , , , , Ban o thê t`ım thâ´y nh˜ung c´ ach giai kh´ ac, nhung v`ı muc ¯oc ¯´ıch . d . c´ ., d , , , chuyên d ap ch´ ung ta d ung ta chı khao ¯ê` phuong ph´ ¯ang x´et nên ch´ s´ at c´ ac v´ı du. sau. , ,, , , , . 4.1. Trong h`ınh vuông v´ oi canh 1 d¯on vi. d 101 d¯iêm. Chu´ng ¯uo. c chon . . , , , , ,, , , ˜ chon ac d¯iêm d¯a o thê phu boi d¯u`ong minh ra ˘`ng c´ o n˘ am d¯iêm trong c´ . c´ 1 tr` on b´ an k´ınh . 7 , `,i giai. Chia h`ınh vuông ra ra 25 h`ınh vuông con c´ Lo o canh 0,2. , , . , , , ˜ chon Nh˜ung h`ınh vuông n` ay c´ o sô´ luo. ng 25 v` a v`ı tâ´t ca sô´ d ¯iêm d ¯a . , , , ´ ´ ´ l` a 101, th`ı ´ıt nhâ´t c´ o môt h` ınh vuông nh o ch u a ıt nhâ t 5 d iê m. M` a ¯ . , 1 1 ,, `˘ng √ < . b´ an k´ınh d on ngoai ¯u`ong tr` . tiê´p h`ınh vuông nho ba 7 5 5 , . 4.2. Chu´ng minh ra˘`ng trong moi . khô´i d¯a diên . lô`i tô`n tai . ´ıt nhâ´t hai ma ˘. t c´ o c` ung sô´ canh. .

J

, , `,i giai. K´y hiêu ´,n nhâ´t cua khô´i d ˘. t c´ Lo F l` a ma o sô´ canh lo ¯a diên. . . . Nê´u , ˘. t (v`ı c´ ˘. t sô´ canh cua F l` a k, th`ı khô´i d o ´ıt nhâ´t k + 1 ma o k ma ¯a diên . . c´

34

,, Chuong 4. H`ınh hoc .

, ,, ˜ ma ´,i F), c` ˘. t l` c´ o canh chung vo on sô´ luo. ng c´ ac canh cua môi a môt . . . ´ ´ ˘. t trong c´ ac sô 3, 4, . . . , k. Theo nguyên l´y Ðirichlê c´ o ´ıt nhât hai ma ´ c´ o c` ung sô canh. . , , ,, , . 4.3. Trong phâ`n trong cua môt h`ınh tr` on v´ oi d¯u`ong k´ınh 5 d¯on vi, . , ,. ,`, , ` ´ ´ nguoi ta chon ˘ng ´ıt nhât c´ o hai d¯iêm . , bât k`y 10 d¯iêm. ,Chu´ng minh, ra , ˜ chon trong c´ ac d¯iêm d¯a o khoang c´ ach nho hon 2. . c´ , ,, `,i giai. Chia d Lo on ¯u`ong tr` , ` ˘ng nhau th` anh 8 re quat . ba C9 C , ,, , C2 ˜ ´ voi g´ oc o tâm môi re quat a . l` A1 , ,, B F on d 450 v` a du. ng d ¯ô`ng ¯u`ong tr` C8 C3 A ´,i b´ tâm C1 vo an k´ınh 1. K´y E D , hiêu a nh˜ung . C2 , C3 , . . . ,, C9 l` O B1 , , C7 C1 h`ınh t`u t´ am re quat ¯i . tr`u d , , ˜ phâ`n m` a d¯u`ong tr` on C1 d ¯a C4 , , ´ ´ chiêm. C´ o thê chung minh C6 , C5 ,, ´, hai d d ao ¯uo. c bâ´t cu ¯iêm n` thuôc ch´ın h`ınh . môt . trong , , H`ınh 4.1: ` trên d o khoang c´ ach nho ¯êu c´ , ,, , , hon 2. Thât ao d on d ¯iêm roi v` ¯u`ong tr` ¯ô`ng tâm th`ı . vây, . nê´u hai d , , , , ,, , , , khoang c´ ach gi˜ua ch´ ung nho hon 2. Gia su hai d a B roi ¯iêm A v` , v` ao môt sô´ t´ am re quat. an k´ınh OC v` a OD lâ´y . CDEF trong . Trên b´ , , , ´, tuong ung hai d a B1 sao cho OA1 = OA; OB1 = OB, ngh˜ıa ¯iêm A1 v` ,, [ ≤ A\ l` a AB ≤ A1 B1 (theo d l´y h` am cosin, boi v`ı AOB ¯inh 1 OB1 ). . , , `˘ng A1 B1 ≤ max{ A1 D, A1 E}. Thât `˘m trong Ðê ´y ra ¯iêm B1 na . vây . d , , ,, ˘ng tao d boi h`ınh chiê´u H cua A1 trên OD v` a ´ıt nhâ´t môt ¯oan . tha . , ,. , ,, ´ ˘ trong hai d iê m D, E, ch a ng h an d iê m D. B o i v ây h` ınh chiê u HD ¯ . ¯ . , , ,`, , ´ cua d u o ng xiên A D không b´ e ho n h` ınh chiê u HB ¯ 1 1 cua A1 B1 trên ´,ng minh nhu, trên ta c´ OD. Ngh˜ıa l` a A1 B1 ≤ A1 D. C˜ ung chu o

J

4.1. V´ı du.

35

, , ´nh gi´ a DA1 ≤ max{ DF, DC }, EA1 ≤ max{ EF, EC }. T`u su. d ¯a EF2 < CD2 = OC2 + OD2 − 2.OC.OD. cos 450 √ 25 25 2 =2 − < 3, 75 < 4 4 4 v` a EC2 = FD2 = OF2 + OD2 − 2OF.OD. cos 450 √ 25 5 2 5.1, 4 = 1+ − < 7, 25 − = 3, 75 < 4, 4 2 2 ,, ta d ¯uo. c AB ≤ A1 B1 ≤ max{ DF, DC, EF, EC } < 2. , , ,, , ,, , ` ˜ d . 4.4. Gia su môi ma ˘. t pha ˘ng d¯uo. c son ba ˘ng môt ¯iêm trong môt . . trong , , , ` ˜ nhât ´ hai mâ`u d¯o v` a xanh. Chu´ng minh ra ˘ng c´ o môt ao d ¯o . h`ınh chu . n` , , , trong ma ˘. t pha ˘ng m` a bô´n d¯ınh cua n´ o c` ung mâ`u. , `,i giai. D˜ê thâ´y theo nguyên l´y Lo , P1 P4 P3 P2 Ðirichlê, môt ¯iêm . tâp . bâ´t k`y 7 d , m` a son môt hai mâ`u th`ı ´ıt . trong Q2 Q1 , Q4 Q3 nhâ´t c´ o4d ung mâ`u. Trên ¯iêm c` , , ,, ˘ng c´ môt o7d R2 ¯u`ong tha ¯iêm th`ı R1 R3 R4 . d , , , ˘ ch´ ung ta phai c´ o4d iê m th a ng ¯ , ,, ´ l` h` ang c` ung mâ`u, gia su d a ¯o , H`ınh 4.2: P1 , P2 , P3 , P4 c´ o c` ung mâ`u d ¯o. Ta , , , ,, ,, ´,i d ˘ng song song vo chiê´u nh˜ung d ay xuô´ng hai d ¯iêm n` ¯u`ong tha ¯u`ong , , ´, ´,a ch´ ´,i chu ung tao a ( R1 , R2 , R3 , R4 ) tuong u ng vo . ra ( Q1 , Q2, , Q3 , Q4 ) v` , , ( P1 , P2 , P3 , P4 ). Nh˜ung d¯iêm n`ay tao môt ung . ra . sô´ h`ınh ch˜u nhât, . ch´ , , , ta ch´ u ´y d ac h`ınh ch˜u nhât od a Pi , i = 1, 2, 3, 4. Nhu vây ¯ê´n c´ ¯ınh l` . , c´ . , , , ´ ´ nê´u 2 d iê m bâ t k` y c ua Q l` a d o th` ı ta c´ o kê t qu a m ôt h` ınh vuông ¯ ¯ , , . ,, , ` Pi Pj Q j Qi c´ od ınh c` u ng mâ u. Tu o ng t u cho c´ a c d iê ¯ ¯ m R. Nê´u d ¯ô`ng . , , , , , , ˘. c th`oi không c´ od a R thoa m˜ an tru`ong ho. p trên th`ı c´ o 3 (hoa ¯iêm Q v`

J

36

,, Chuong 4. H`ınh hoc .

, , , , ´ v` ´ c´ hon) d ao d a3d ao d o c` ung mâ`u xanh. Nhung ¯iêm Q n` ¯o ¯iêm R n` ¯o , , , ´,i c´ ˘. p d trong bô. ba nhu vây phai c´ o ca ra h`ınh ch˜u nhât vo ac ¯ôi tao . . . , , ` ´ d ac d a R. ¯ınh mâu xanh trong sô c´ ¯iêm Q v` , ,, ,, , , ˜, nhât . 4.5. Gia su môt an c`o h`ınh chu o 4x7 ô vuông d¯uo. c son d¯en . b` . c´ , , , , hoa ˘. c tra ˘´ng. Chu´ng minh ra ˘`ng v´ oi c´ ach son mâ`u bâ´t k`y, trong b` an c`o ,, ˜, nhât luôn tô`n tai ac ô vuông, m` a bô´n ô o g´ oc l` a c´ ac ô . h`ınh chu . gô`m c´ ` c` ung mâu.

J

, `,i giai. Ch´ ´,ng Lo ung ta chu , minh cho b` ai to´ an b` an c`o ˜ so,n mâ`u c´ 3 × 7. Mâu o , , , , ´ ` thê xây ra voi b` an co n` ay , , ´ ` c´ o dang t u 1 d ê n 8. Gi a ¯ . ,, su môt ac côt . trong sô´ c´ . thuôc d ang 1. B` a i to´ a n s˜ e . . ,, , ´ ng minh nê´u tâ´t d ¯uo. c chu , , ca nh˜ung côt on lai . c` . trong 6 côt ac dang 1, 2, . thuôc . c´ , . ,, , , 3, 4. Nhu vây . gia su tâ´t ca H`ınh 4.3: c´ ac côt on lai . c` . thuôc . dang . ˘. c 8. Khi d ´ theo nguyên l´y Ðirichlê hai trong sô´ s´ 5, 6, 7, hoa au côt ¯o . , ,, , ´ c´ o hai côt c` u ng m ôt d ang v` a nhu v ây b` a i to´ a n c˜ u ng d u o c ch u ng ¯ . . . . . , ,, , ´,ng minh ho` minh. Chu an to` an tuong tu. nê´u môt c ôt c´ o dang 8. Gia . . . ,, , ˘. c 8. Nhu vây su không c´ o côt ao trong 7 côt o dang 1 hoa o7 . n` . c´ . . ta c´ , ´i 6 dang. côt Theo nguyên l´y Ðirichlê c´ o hai côt ung dang a b` ai . vo . . c` . v` , ,, , ´ ng minh d to´ an d ¯uo. c chu ¯â`y d ¯u. , , . 4.6. N˘am d¯iêm A, B, C, D, E na˘`m trong môt ma ˘. t pha ˘ng v` a toa . . d¯ô. , , , ˜ ng tam cua ch´ ung l` a c´ ac sô´ nguyên. Chu´ng minh ra ˘`ng trong sô´ nhu

J

4.1. V´ı du.

37

, , , , ´ trong c´ gi´ ac m` a d¯ınh cua n´ o l` a ba d¯iêm n` ao d¯o ac d¯iêm n` ay, c´ o ´ıt nhâ´t , ba tam gi´ ac v´ oi diên a c´ ac sô´ nguyên. . t´ıch l` , , , `˘ng nê´u môt `,i giai. Ta c´ ´,ng minh ra Lo o thê chu trong c´ ac toa d ¯ô. cua . . , , , ˜˘n, th`ı diên ˜ cho thay d c´ ac d ac d sô´ cha t´ıch cua tam ¯ınh tam gi´ ¯a ¯ôi môt . . , , , , gi´ ac c˜ ung thay d sô´ nguyên. Môt c´ ach tông qu´ at hon ta c´ o thê ¯ôi môt . . , , , , ˘ng d ac toa d tam gi´ ac thay d kha ¯inh ¯ô. d ¯ınh cua môt ¯ôi môt . nê´u c´ . . . sô´ , , ˜ ˘n, th`ı diên cha t´ıch cua n´ o c˜ ung thay d ¯ôi môt . sô´ nguyên. V`ı vây, . nê´u ,. , , , ´i nhân a sô´ nguyên, th`ı diên diên ac mo ¯uo. c l` . t´ıch cua tam gi´ . d . t´ıch , ´ ng minh). tam gi´ ac ban d ung l` a sô´ nguyên (h˜ ay v˜e h`ınh v` a chu ¯â`u c˜ , , , , ˜ cho A, B, C, D, E l` V`ı nh˜ung toa ac d a nh˜ung sô´ ¯ô. cua c´ ¯iêm d ¯a . d , ˜˘n th´ıch ho.,p, nguyên, sau khi thêm v` ao c´ ac toa d ay nh˜ung sô´ cha ¯ô. n` . , , ˜ toa ´ moi ˜ cho th`ı môi c´ ac gi´ a tri. 0 v` a 1. Do d 5d ¯ô. s˜e chı nhân ¯o ¯iêm d ¯a . d . . , ,, tao ac d nguyên l´y ¯iêm (0, 0), (0, 1), (1, 0), (1, 1). Áp dung . nên boi c´ . , , , ´ Ðirichlê suy ra ´ıt nhât hai d ac d ¯iêm trong c´ ¯iêm A, B, C, D, E biê´n d ¯ô i , , ,, ´ l` th` anh c` ung môt a ¯iêm trong {(0, 0), (0, 1), (1, 0), (1, 1)}. Gia su d ¯o . d , ˘ng d A, B. Ch´ ung ta s˜e kha ac tam gi´ ac ABC, ABD v` a ¯inh . diên . t´ıch c´ , ´ ´ ˜ ABE l` a nhung sô nguyên. Thât ac tam gi´ ac n` ay bi. biên th` anh . vây, . c´ , , ´ ˘ng (do A v` d tha a B biên th` anh c` ung môt t´ıch ¯oan ¯iêm) nên diên ., d , . , , . ,´, ` ´ ˘ng 0. Vây anh cua ch´ ung ba ung . truoc khi biên d¯ôi, diên . t´ıch cua ch´ , ´ phai l` a sô nguyên. , , , . 4.7. Trong môt ma ˘. t pha ˘ng cho môt tâp ho. p A c´ o n d¯iêm (n ≥ 2), . . . , , ,, , , môt ˘. p d¯iêm d¯uo. c nô´i v´ oi nhau ba ˘`ng d¯oan tha ˘ng. Chu´ng minh . sô´ ca . , , ,, ,, , ra ˘`ng trong A c´ o ´ıt nhâ´t hai d oi c` ung sô´ luo. ng c´ ac d¯iêm ¯iêm d¯uo. c nô´i v´ kh´ ac thuôc . A. , , `,i giai. Vo ´,i moi Lo ung ta k´y hiêu S( a) l` a ¯iêm bâ´t k`y a thuôc . d . A ch´ , . , , , , , ˘ng. B` sô´ luo. ng nh˜ung d a a nô´i th` anh d ai to´ an ¯iêm cua A m` ¯oan . tha , , , `˘ng tô`n tai ˘ng d muô´n kha ac nhau a1 v` a a2 cua A ¯inh ¯iêm kh´ . ra . hai d

J

38

,, Chuong 4. H`ınh hoc .

`˘ng 0 ≤ S( a) ≤ n − 1 vo ´,i m` a S( a1 ) = S( a2 ). Ch´ ung ta thâ´y ngay ra , ,, , ˘. t kh´ moi ac, c˜ ung không tô`n tai ¯iêm . phâ`n tu a thuôc . A. Ma . nh˜ung d ,, x1 v` a x2 thuôc a S( x1 ) = n − 1 v` a S( x2 ) = 0. Boi v`ı d ay ¯iê`u n` . ,A, m` , , , , ´ ´ ´i tât ca c´ c´ o ngh˜ıa l` ad ac d on lai on x2 ¯iêm x1 nôi vo ¯iêm c` . cua A, c` , , , , ˜ ´ ´ ´ ´i d không nôi vo ao cua A, d ac sô nguyên t`u 0 ¯iêm n` ¯ân d ¯ên vô l´y. C´ ,, , d o sô´ luo. ng l` a n. V`ı 0 v` a n − 1 không d a gi´ a tri. ¯ê´n n − 1 c´ ¯ô`ng th`oi l` , , ,, ` ´ vua S nên S nhân a n − 1 gi´ a tri. ai to´ an d ¯uo. c giai suy . B` . nhiêu nhât l` , ` ra tu nguyên l´y Ðirichlê.

J

,, ˜ . 4.8. Cho d¯a gi´ac d¯ê`u 100 canh nôi tiê´p trong d¯u`ong tr` on k. Môi . . , ,, , d¯ınh d¯uo. c g´ an môt ac sô´ 1, 2, . . . , 49. Chu´ng minh ra ˘`ng trên k . trong c´ , tô`n tai a CD v´ oi c´ ac t´ınh châ´t sau: . hai cung AB v` , , , ˜ cho. a) C´ ac d¯iêm A, B, C v` a D l` a d¯ınh cua d¯a gi´ ac d¯ê`u d¯a , b) C´ ac dây cung AB v` a CD song song v´ oi nhau ,, , c) Nê´u A, B, C v` a D c´ o nh˜ an tuong u´ng l` a c´ ac sô´ a, b, c, d th`ı a + b = c + d. , ,, `,i giai. Trong d Lo on k c´ o ¯u`ong tr` ,`, ´ d ac nhau m` a ¯ung 50 d ¯uong k´ınh kh´ D C , , ,`, ´ d iê m cuô i c ua d u o ng k´ ınh l` a m ôt ¯ ¯ . , , ˜ d ac d d ¯ınh cua d ¯a gi´ ¯ê`u 100 canh ¯a . , , cho. Nê´u PQ l` a môt ¯u`ong . trong sô´ d ,, ´ v` k´ınh d ad an nh˜ an ¯o ¯ir nh P d ¯uo. c g´ , , , ´i nh˜ p, c` on Q vo an q th`ı d ¯u`ong , , ´, , ´i sô´ nguyên k´ınh PQ tuong ung vo B A | p − q|. R˜o r`ang, 0 ≤ | p − q| ≤ 48. ,, ˜ d `˘ng c´ ˜ Ba ach n` ay môi ¯u`ong k´ınh d ¯a H`ınh 4.4: , , , , x´et (tông sô´ c´ o 50 tâ´t ca) d ¯uo. c cho , , ´, ´,i môt tuong u ng vo ac sô´ 0, 1, 2, . . . , 48. Suy ra c´ o ´ıt nhâ´t hai . trong c´

4.1. V´ı du.

39

,, ,, , , ´, ´,i c` ˘. t tuong u ng vo d ung môt ¯u`ong k´ınh d ¯uo. c d ¯a . sô´. , ,, , , ´, ´ l` Ta k´y hiêu a AB v` a CD, c` on c´ ac d an tuong u ng ¯o ¯ınh d ¯uo. c g´ . d , , , c´ ac nh˜ an a, b, c, d. Không mâ´t t´ınh tông qu´ at ch´ ung ta c´ o thê gia ´ dây cung AC v` thiê´t c ≤ a v` a b ≤ d. Khi d¯o a BD c´ o c´ ac t´ınh châ´t ,, , , , ´i d ˘. t tuong ˜d ung ta d mong muô´n. Thât ¯a ¯a ¯u`ong k´ınh AC ch´ . vây, . vo , `˘ng nhau, ´,ng vo ´,ng vo ´,i a − c, c` ´,i d − b v` u on BD u a nh˜ung sô´ n` ay ba ,, ch´ ung ta nhân a a + b = c + d. Ngo` ai ra, ¯uo. c a − c = d − b hay l` . d , , ´ gi´ ´i CD (thâm tu ac ABCD l` a h`ınh vuông, nên AB song song vo . ch´ı AB = CD).

J

, , , ,, ,, . 4.9. Tâ´t ca c´ac d¯iêm trong môt ˘. t pha ˘ng d¯uo. c bôi boi n mâ`u . ma ,, ,, kh´ ac nhau. Cho truo´c 2n − 1 d¯u`ong tr` on d¯ô`ng tâm kh´ ac nhau , , , , , , k1 , k2 , . . . , k2n −1 . Trong c´ ac d¯u`ong tr` on du. ng b´ an k´ınh tuong u´ng , ˜ ca OA1 , OA2 , . . . , OA2n −1 sao cho môi ˘. p b´ an k´ınh không c´ o d¯iêm , ,, on chung n` ao kh´ ac ngo` ai O. Chu´ng minh ra ˘`ng tô`n tai . môt . d¯u`o,ng tr` n k i , i = 1, 2, . . . , 2 − 1 sao cho trên n´ o v` a trên b´ an k´ınh OAi cua n´ o c´ o , , , , , , , ˜ ng d¯iêm tuong u´ng Xi v` nhu a Yi , m` a ch´ ung d¯uo. c bôi c` ung môt a . mâ`u v` , Yi không tr` ung v´ oi O v` a Ai . , `,i giai. K´y hiêu ˜ cho l` Lo ac mâ`u d a c1 , c2 , . . . , cn . Ch´ ung ta s˜e t´ınh ¯a ., c´ , , , , , , ´i ch´ sô´ luo. ng c´ ac tô ho. p kh´ ac nhau cua c´ ac mâ`u m` a vo ung c´ o thê , , ˘ng. Moi bôi mâ`u c´ ac d ac trô´ng ¯iêm trên môt ., h`ınh pha . tâp . con kh´ , , ´, , {i1 , i2 , . . . , ik }, 1 ≤ k ≤ n cua tâp . ho. p N = {1, 2, . . . , n} tuong ung , , , , ´i tô ho. p mâ`u {ci1 , ci2 , . . . , cik }. R˜ vo o r` ang nh˜ung tâp ac trô´ng . con kh´ , , , , , , , , `˘ng c´ ´ ng vo ´i nh˜ung tô ho. p mâ`u kh´ ac nhau cua N ba ach n` ay tuong u , , , kh´ ac nhau. Ngo` ai ra, moi o thê nhân . tô ho. p mâ`u {ci1 , ci2 , . . . , cik } c´ , ., ,, , , ´ ` d u o c nh o xây d u ng t âp con kh´ a c trô ng { i , i , . . . , i } . Suy ra, tô ho. p ¯ . 1 2 k . . , , , , ´i ch´ nh˜ung mâ`u c1 , c2 , . . . , cn m` a vo ung c´ o thê bôi c´ ac d ¯iêm môt . h`ınh , , , ´, , , , ´i nh˜ung tâp ˘ng l` pha a tuong ung môt ac trô´ng . - môt . vo . ho. p con kh´

40

,, Chuong 4. H`ınh hoc .

, , , , cua N = {1, 2, . . . , n}. Ngh˜ıa l` a sô´ c´ ac tô ho. p kh´ ac nhau cua mâ`u , , , , `˘ng sô´ c´ ba ac tâp ac trô´ng cua N, c´ o tâ´t ca 2n − 1 tâp . ho. p con kh´ . ho. p , nhu vây. . , ,´, ,, `˘ng tô`n tai Truo c tiên ch´ ung ta gia thiê´t ra on k i , i = ¯u`ong tr` . d , , n 1, 2, . . . , 2 − 1 m` a muô´n bôi k´ın n´ o phai d` ung tâ´t ca c´ ac mâ`u , , ´ ch´ c1 , c2 , . . . , cn . Khi d ung ta chon a d ¯o ¯iêm bâ´t k`y Yi l` ¯iêm trong . d , , ,, ,, , `˘ng mâ`u c1 . Nhung trên k i c´ cua b´ an k´ınh OAi . Gia su Yi d o ¯uo. c bôi ba , , , , ´ıt nhâ´t môt ˜ cho ung mâ`u nhu vây, ¯iêm Xi , d ¯uo. c bôi c` ¯a . d . v`ı moi . mâ`u d , , , , , , , ´˘t ga ˘. p trên k i ; vây d ai to´ an d ¯ê`u ba ¯uo. c giai trong tru`ong ho. p gia thiê´t . b` , ,, ,`, ,, , n` ay. Ta x´et truong ho. p c` on lai: Gia su không c´ od on n` ao m` a ¯u`ong tr` . , , , , , , , , ´i to` nh˜ung d od an bô. tô ho. p c1 , c2 , . . . , cn . ¯iêm cua n´ ¯uo. c bôi mâ`u vo , n , , , , ,, ´,i n´ ´ c´ Khi d o tâ´t ca 2 − 2 kha n˘ ang tô ho. p mâ`u, vo o c´ ac d ¯o ¯iêm d ¯uo. c , ,, , bôi cho c´ ac d on k1 , k2 , . . . , k2n −1 . Boi v`ı 2n − 1 > 2n − 2 nên ¯u`ong tr` , , ,, c´ o hai d on k i v` a k j , i < j trên n´ o s˜e c´ o c` ung môt tô ho. p mâ`u. ¯u`ong tr` . , ,, , , ´˘t cua OAi v` ´,n ho,n k j . K´y hiêu Gia su k i c´ o b´ an k´ınh lo ad a ¯iêm ca . Yi l` , , ,`, , , `˘m trên k j v` ´ Yi na k j . Trong truong ho. p d a v`ı nh˜ung d a ¯o ¯iêm cua k i v` , , , ,, , , , ´i c` kj d ung môt o d¯iêm Xi d ¯uo. c bôi vo ¯uo. c bôi . tô ho. p mâ`u, trên k i c´ , ´i Yi . c` ung mâ`u vo

J

, , , , , . 4.10. Tông cua d¯ô. d`ai môt ˘. t pha ˘ng l` a 4. Chu´ng . sô´ vecto, trong ma , , `, nhu ˜,ng vecto, n` minh ra ˘`ng tu ay c´ o thê chon a tông d¯ô. . môt . sô´ vecto m` , , , d` ai cua ch´ ung l´ on hon 1. , , , , `,i giai. Ch´ Lo ung ta d ao hê. toa a x´et vecto d ¯ua v` ¯ô. v` ¯ai . d . diên . nh˜ung , , , , ˜ cho tai vecto d ung ta chiê´u nh˜ung vecto n` ay xuô´ng ¯a ¯iêm gô´c. Ch´ . d , , , , , ˜ vecto c´ truc a Oy. V`ı môi o d¯ô. d` ai nho hon tông cua c´ ac ¯ô. Ox v` . toa . d , , , , d ai h`ınh chiê´u cua n´ o xuô´ng hai truc;nên tông d ai cua tâ´t ca ¯ô. d` ¯ô. d` . , , ´, , ´ trên ´ıt nhâ´t môt h`ınh chiê´u cua c´ ac vecto lo n hon 4. Khi d ¯o . trong , , , ,, , ´n ho,n 1, 4 nua truc ai cua h`ınh chiê´u s˜e lo ¯ô. tông d ¯ô. d` . cua hê. toa . d

4.2. B` ai tâp .

41

, , , , , , , ´, ´ c´ ng s˜e d o ngh˜ıa l` a tông cua d ai cua nh˜ung vecto tuong u ¯iê`u d ¯o ¯ô. d` , , , , , ´ ´ ´n hon 1. (d ´n hon th`ı tât nhiên d ˜ lo lo ai h`ınh chiêu d ai vecto ¯ô. d` ¯a ¯ô. d` ´,n ho,n.) c˜ ung lo

J

` tâp 4.2. Bai .

, , ´,ng . 4.11. Trong h`ınh vuông vo´,i canh 1d ¯on vi. cho 112 d ¯iêm. Chu . , , , 2 `˘ng ´ıt nhâ´t hai trong sô´ d ´ c´ minh ra o khoang c´ ach nho hon . ¯o 15 ´, 25 n˘ . 4.12. (Ðê` thi To´an Olympic quô´c tê´ lâ`n thu am 1984) Trong , , , , ˜ d ´i môi ˘. t pha ˘ng cho hai d ma ac nhau O v` a A. Vo ¯iêm kh´ ¯iêm X thuôc . , ` ˘. t pha ˘ng kh´ ˘ ma ac O ch´ ung ta k´y hiêu a ( X ) l` a d ô d o b a ng radian ¯ ¯ . . , ,, , cua g´ oc AOX, d ¯o theo chiê`u nguo. c kim d ¯ô`ng hô` t`u OA d ¯ê´n OX ,`, (0 ≤ a( X ) ≤ 2). C`on c( X ) l`a d¯uong tr`on tâm O v`a b´an k´ınh c´o d¯ô. , a( X ) ,´, , ˜ d ˘. t d` ai OX + . Cho truo c bô. h˜uu han mâ`u v` a môi ¯iêm trong ma . OX , , ,, `˘ng 1 trong sô´ d `˘ng tô`n tai ´,ng minh ra ˘ng d ´. Chu pha d ¯uo. c bôi ba ¯o ¯iêm ,. , ,, , , ´i a( X1 ) > 0 v` X1 vo a trên d on c( X1 ) c´ o ´ıt nhâ´t 1 d ¯u`ong tr` ¯iêm d ¯uo. c ´,i X . bôi c` ung mâ`u vo 1

, , , . 4.13. Trong ma˘. t pha˘ng cho n d¯iêm, n ≥ 7, sao cho khoang c´ach , , , ,, ˜ ca ˜ môt ´,i ˘. p d môi ung l` a kh´ ac nhau. Môi d ¯iêm gi˜ua ch´ ¯iêm d¯uo. c nô´i vo . , , ,, `˘ng không c´ ´,ng minh ra d o nhâ´t. Chu od ao d ¯iêm gâ`n n´ ¯iêm n` ¯uo. c nô´i , ´,i nhiê`u ho,n 5 d vo ac. ¯iêm kh´ `˘ng trong môt ´,ng minh ra . 4.14. Chu ınh tr` on b´ an k´ınh 1, không . h` , , , , ,, , thê chon a n˘ am d a khoang c´ ach gi˜ua hai d ¯uo. c qu´ ¯iêm m` ¯iêm môt . d . , , ´n hon 1. lo , , . 4.15. Ngu`o,i ta qu˘ang 120 h`ınh vuông c´o k´ıch thuo´,c 1 × 1 v`ao môt . , ,´, , , ` ´ ´ ´ ˘ ˘ h`ınh ch˜u nhât k´ ıch thu o c20 × 25. Ch u ng minh r a ng v o i m oi c´ a ch s a p . .

42

,, Chuong 4. H`ınh hoc .

, ,, , ˜ c` ˜ trô´ng d on chô xê´p c´ ac h`ınh vuông th`ı o trong h`ınh ch˜u nhât ¯ê . vân ,, ˘. t môt d on d ¯a ¯u`ong k´ınh 1. . h`ınh tr`

, , CHUONG

5

,, ´ ÐIRICHLÊ MO RÔNG NGUYÊN LY .

,, 5.1. Nguyên l´ y Ðirichlê mo rông .

,, , , ,, Cho A l` a tâp a sô´ luo. ng . h˜uu han . nh˜ung phâ`n tu, k´y hi, êu . s( A) l` ,, ,, , c´ ac phâ`n tu thuôc o thê mo rông . A. Nguyên l´y Ðirichlê c´ . nhu sau: ,, , , , Nê´u A v` a B l` a nh˜ ung tâp uu han a s( A) > k.s( B), o . ho. p h˜ . v` , , , ˜ phâ`n tu, cua A cho ´ v` d a môt ao d a nê´u môi ¯ây k l` ¯o . sô´ tu. nhiên n` , ,, , , ´, ´,i môt ´ cua B, th`ı tô`n tai ng vo phâ`n tu n` ao d tuong u ¯o . . ´ıt nhâ´t,k +, 1 ,, , , , , , , ´ ng vo ´ i c` a ch´ ung tuong u ung môt phâ`n tu cua A m` . phâ`n tu cua B ,, , ˜ x´et Thât a nguyên l´y Ðirichlê m` a ta d ¯a . vây, . tru`ong ho. p k = 1 l` , , , , , ´ ng minh mênh ´i gi`o. Ðê chu trong c´ ac b` ai tâp t`u d ung ¯â`u to ¯ê` trên ch´ . d , ,, ˜ . , ,, ,, , , , , ´ ng vo ´i nhiê`u nhâ´t k phâ`n tu, ta gia su môi phâ`n tu cua B chı tuong u , , ´,i gia thiê´t s( A) > k.s( B). ´ s( A) ≤ k.s( B) tr´ cua A. Khi d ai vo ¯o , , ,, , , ,, ˜d ´. Ngay t`u nh˜ung chuong d Ch´ u y ay d ¯â`u nguyên l´y n` ¯a ¯uo. c su , , ´,ng minh tu,o,ng tu., nhu, trên. Ðê tiê´p tuc ´,i c´ dung vo ach chu . . hiêu sâu , ,, ´,i ch´ thêm nguyên l´y o dang mo ung ta x´et môt ai to´ an d ¯iên . . loat . b` h`ınh.

44

,, ,, Chuong 5. Mo rông nguyên l´y Ðirichlê .

5.2. V´ı du.

, . 5.1. Trong môt o canh ˘`ng 1 ta chon . h`ınh vuông c´ ., ba . bâ´t k`y 51 d¯iêm. , ´ na Chu´ng minh ra ˘`ng ´ıt nhâ´t c´ o ba d¯iêm trong sô´ d¯o ˘`m trong môt . h`ınh vuông c´ o canh 0,2. . , `,i giai. Ch´ Lo ung ta chia h`ınh vuông th` anh 25 h`ınh vuông con c´ o , , ,`, , ` ´i c´ ˘ng c´ ˘ng song song vo canh 0,2 ba ac d ac canh cua h`ınh ¯uong tha . . , , , ,, , ˜ ` ´ ` ´ ˘ng môi h`ınh vuông vua nhân vuông.Nêu gia su ra d ¯uo. c chua không . , , , , ´,n s˜e c´ qu´ a2d ac d o sô´ d ¯iêm th`ı tâ´t ca c´ ¯iêm trong h`ınh vuông lo ¯iêm ˜ d nhiê`u nhâ´t l` a 2 × 25 = 50 dân ¯ê´n vô l´y.

J

, , ,, ´y d¯a gi´ . 5.2. Tâ´t ca 9 canh v` a 27 d¯u`ong ch´eo cua môt op d¯a ac . . h`ınh ch´ , , , ,, , , , 9 d¯ınh d¯uo. c bôi son: môt son d¯o, c` on lai ., sô´ bôi . bôi son xanh. Chu´,ng , ˜,ng d¯ınh minh ra ˘`ng tô`n tai op, m` a ch´ ung l` a nhu . ba d¯ınh cua h`ınh ch´ , ,, , , ung môt cua môt ac v´ oi c´ ac canh d¯uo. c son c` . h`ınh tam gi´ . . mâ`u. , , ,, , ` `,i giai. 9 canh ˘ng hai mâ`u. V`ı 9 > Lo bên cua h`ınh ch´ op d ¯uo. c son ba . ,, , ,, 4.2, t`u nguyên l´y mo rông trên suy ra ´ıt nhâ´t c´ o 5 canh bên d ¯uo. c bôi . , ,,. ,, ´ l` c` ung môt mâ`u. Gia su d a SA1 , SA2 , SA3 , SA4 , SA5 d ung ¯o ¯uo. c bôi c` , . ,, , , , ,, , ´ ´ mâ`u d o, o d ây S l` a d ınh h` ınh ch´ o p. Không anh hu o ng t o i kê t lu ân ¯ ¯ ¯ . , ,, , ,, ´ ´p ˘ b` ai to´ an ch´ ung ta gia su c´ ac d iê m A , A , A , A , A d u o c s a p xê ¯ 2 3 5 ¯ . 1 4 , , , theo chiê`u nguo. c kim d trong c´ ac canh cua ng˜ u ¯ô`ng hô`. Ít nhâ´t môt . , , . ,, ,`, ´y; gia su d ´ l` a A1 A2 . Ch´ ung ta gi´ ac l` ad ac d ¯o ¯uong ch´eo cua d ¯a gi´ ¯a , , , , , x´et tam gi´ ac A1 A2 A4 . Nh˜ung canh cua n´ o l` a c´ ac d ¯u`ong ch´eo cua d ¯a . , , , , ´y (do c´ gi´ ac d ach xê´p trên) v` a suy ra ch´ ung d ¯a ¯uo. c bôi son.Nê´u tâ´t ca , ˜ giai xong. c´ ac canh A1 A2 , A2 A4 , A4 A1 l` a mâ`u xanh th`ı b` ai to´ an d ¯a . ,, ,, , Tru`ong ho. p nguo. c lai ac canh a mâ`u . môt . trong c´ . ,A1 A2 , A2 A4 , A4 A1 l` , , ,, ´ l` ´ c´ d a A1 A2 . Khi d ac canh cua tam gi´ ac SA1 A2 l` a mâ`u ¯o, gia su d ¯o ¯o . , d ¯o.

J

5.2. V´ı du.

45

, , . 5.3. Chu´ng minh ra˘`ng trong moi ac lô`i v´ oi sô´ canh cha ˘˜n tô`n ¯a gi´ . d . , ,`, , tai oi môt n` ao cua d¯a gi´ ac. . d¯uong ch´eo không song song v´ . canh . , , `˘ng qui nap `,i giai. Ba Lo o thê d˜ê . ta c´ ´,ng minh d¯u,o.,c moi ´,i n chu ac vo ¯a gi´ . d n ( n − 3) , , d canh c´ o ¯u`ong ch´eo. . 2 , Bây gi`o ch´ ung ta x´et d ac lô`i bâ´t ¯a gi´ , ´i k (≥ 2) l` k`y P c´ o 2k canh vo a môt . . sô´

d1 d2 d k −1 a

nguyên. H`ınh 5.1: , ,, ˜ , ,`, , ´i môt ´ Gia su môi d n` ao d ¯uong ch´eo cua P song song vo ¯o . canh . , , ,`, , , ´, , ˜ ´i canh ´ môi d cua P. Khi d o thê cho tuong ung vo song ¯o ¯uong ch´eo d c´ . , , , ´i d. K´y hiêu song vo a sô´ c´ ac d ung ta c´ o ¯u`ong ch´eo ch´ . s l` 2k (2k − 3) s= = k(2k − 3) 2 = 2k(k − 2) + k > (k − 2).2k. ,, , Nhu vây mo rông suy ra tô`n tai . theo nguyên l´y Ðirichlê . . k−1 , ,`, , , ´, ´,i d ac P m` a ch´ ung tuong ung vo ¯uong ch´eo d1 , d2 , . . . , dk−1 cua d ¯a gi´ , c` ung môt a cua d ac, ngh˜ıa l` a a, d1 , d2 , . . . , dk−1 song song ¯a gi´ . canh . , , , `˘m trong ´i nhau. Suy ra c´ vo ac d ung na ¯u`ong ch´eo d1 , d2 , . . . , dk−1 c` , ,, ,, ˘. t pha ˘ng x´ môt ac d a v`ı P l` ad ac lô`i (h`ınh v˜e). ¯inh ¯a gi´ . boi canh . n ua m a . , ,, , ˘ng kh´ Ngo` ai ra d1 , d2 , . . . , dk−1 v` a a l` a nh˜ung d¯oan tha ac nhau v` a boi . , , , , , ,, ˜ d v`ı sô´ luo. ng cua ch´ ung l` a k, môi ac P l` ad ¯ınh cua d ¯a gi´ ¯iêm d ¯â`u cua , , , ,, ´,ng ´ trong ch´ ao d ung. Không anh huong d môt ¯oan ¯o ¯ê´n kê´t qua chu . d . n` , ,, ´,i a trong c´ minh ta gia thiê´t d1 l` ad ac d ¯oan ¯ô´i vo ¯u`ong ch´eo . xa nhâ´t d , `˘ng d `˘m ´ t`u l´y luân d1 , d2 , . . . , dk−1 . Khi d trên suy ra ra ac P na ¯o ¯a gi´ ,. ,, ,, ´,i ˘. t pha ˘ng x´ to` an bô. trong môt ac d ay tr´ ai vo ¯inh . boi d1 . Ðiê`u n` . nua ma , t´ınh lô`i cua d ac P. ¯a gi´

J

, , , ˜ d¯oan `,ng ca . 5.4. Trong ma˘. t pha˘ng cho 6 d¯iêm. Môi ˘ng nô´i tu ˘. p . tha

46

,, ,, Chuong 5. Mo rông nguyên l´y Ðirichlê .

, , , ,, , d¯iêm d¯uo. c bôi mâ`u d ˘. c xanh. Chu´ng minh ra ˘`ng ba d¯iêm trong sô´ ¯o hoa , , , , ,, c´ ac d¯iêm l` a d¯ınh cua môt ac, m` a c´ ac canh cua n´ o d¯uo. c bôi c` ung . tam gi´ . ` môt . mâu. , , `,i giai. K´y hiêu ˜ cho v` Lo A l` a môt trong c´ ac d¯iêm d a x´et n˘ am d ¯a ¯oan . . . , , , , , , , ˘ng, c´ ˘ tha o d a A. Nh˜ung d th a ng n` a y d u o c so n hai ¯ınh chung l` ¯oan ¯ . . ,, , mâ`u v` a v`ı 5=2.2+1, suy ra t`u nguyên l´y Ðirichlê mo rông ch´ ung . , ,, ´ l` ta c´ o ´ıt nhâ´t ba d¯oan ung mâ`u. Gia su d a AB1 , AB2 , AB3 mâ`u ¯o . c` , ˘ng B1 B2 , B2 B3 , B3 B1 l` xanh. Nê´u môt ai d a xanh th`ı tô`n tai ¯oan . v` . tha , . , , ´ ´ ˘ môt tam gi´ a c v o i ba c anh xanh v` a c´ o d ınh l` a A. Nê u ba d o an th a ng ¯ ¯ . . ,. , ˜ ra l` B1 B2 , B2 B3 , B3 B1 l` ad tam gi´ ac thoa m˜ an d an d a ¯o, th`ı môt ¯ê` to´ ¯a ,. B1 B2 B3 (c´ ac canh d ¯ê`u mâ`u d ¯o). .

J

, ,, . 5.5. Cho d˜ay vô han ac sô´ tu. nhiên u1 , u2 , . . . , un , . . . d¯uo. c x´ ac d¯inh . c´ . , theo công thu´c sau u1 = 3, un+1 = (n + 1)un − n + 1, n = 1, 2, . . .. , , , Cho n sô´ tu. nhiên bâ´t k`y v` a tâp ho. p M gô`m un d¯iêm sao cho không . , , , , ˜ d¯oan c´ o ba d ao tha ˘ng h` ang. Môi ˘ng nô´i hai d¯iêm kh´ ac nhau ¯iêm n` . tha ,, , ` ˜ cho. Khi d¯o ´ tô`n tai trong M d ˘ng môt ¯uo. c bôi son ba . trong n mâ`u d¯a . 3 , , , d¯iêm trong M l` a d¯ınh cua môt ac d¯ô`ng mâ`u. . tam gi´ , `˘ng qui nap `,i giai. Chu ´,ng minh ba ´,i n = 1 ch´ Lo ung ta c´ o . theo n. Vo , , , , , , ´ c t´ınh d ´i n = 2 t`u công thu u1 = 3 v` a kê´t luân a hiên nhiên. Vo ¯uoc . l` , ,, . ,, ,`, , , ´ ng minh o b` ˜ chu u2 = 6, d a truong ho. p ta d ai trên. Gia su b` ai ¯ây l` ¯a , , ,, , , , ´i un d ˘ng nô´i d to´ an vo a c´ ac d o môt ¯iêm v` ¯oan ¯uo. c bôi boi n mâ`u, c´ . tha . , , , ` ´ ´i c´ ` tam gi´ ac vo ac canh c` u ng mâ u. Bây gi o ta x´ e t u d iê m bâ t k` y , m` a n +1 ¯ . ,`, , , , `˘ng n + 1 mâ`u: c1 , c2 , . . . , cn+1 . Lâ´y d ung d ¯uong nô´i gi˜ua ch´ ¯uoc bôi ba , . , , , ´,i c´ A l` a môt ac d ay c´ o thê nô´i vo ac d on ¯iêm trên. Ðiêm n` ¯iêm c` . trong c´ , ,, , ´ c hô`i ˘ng bôi mâ`u. Ma ˘. t kh´ lai ac do công thu ¯uo. c un+1 − 1 d ¯oan . d . tha , qui un+1 − 1 = (n + 1)(un − 1) + 1, nhu vây theo nguyên l´y Ðirichlê , ,. ,, ,, ˘ng c´ mo rông o chung d ung mâ`u. ¯oan ¯ınh A d ¯uo. c bôi c` . ´ıt nhâ´t un d . tha

5.2. V´ı du.

47

, , ,, ,, ˘ng han Gia su AB1 , AB2 , . . . , ABun d ung mâ`u, cha ¯uo. c bôi c` . mâ`u c1 . , , , , , Nh˜ung kha n˘ ang c´ o thê xây ra nhu sau: , , , ˘ng nô´i gi˜ua c´ a) Nê´u môt ac d ac d ¯oan ¯iêm B1 , B2 , . . . . trong c´ . tha , ,, `˘ng mâ`u c1 th`ı tô`n tai ˘. p d , Bun theo t`ung ca ac d ¯uo. c bôi ba ¯ô`ng . tam gi´ , , , mâ`u c´ od ac canh cua tam gi´ ac n` ay son mâ`u c1 . ¯ınh A. C´ . , , ˘ng nô´i gi˜ua c´ b) Không c´ o môt d n` ao trong c´ ac d tha ac ¯oan ¯oan . . . , ,, , ´a ´p dung d an ¯iêm B1 , B2 , . . . , Bun d ¯uo. c son mâ`u c1 . Khi d ¯o . , qui nap . to´ , , , , , hoc d ay c´ o thê d ¯iêm B1 , B2 , . . . , Bun . Ðiê`u n` ¯uo. c v`ı tâ´t ca . cho nh˜ung , ,, , ˘ng d ˜ nô´i d c´ ac d ¯oan ¯a ¯uo. c bôi son n mâ`u c2 , c3 , . . . cn+1 . Trong . tha ,, , ˜ d tru`ong ho. p n` ay ta c˜ ung dân ac d ¯ê´n kê´t luân ¯ô`ng . tô`n tai . môt . tam gi´ mâ`u.

J

, ,, , ,, . 5.6. Cho sô´ hang cua d˜ ay un d¯uo. c d¯inh ai truo´c. Tâp . . . ngh˜ıa nhu b` ,, ,, , ` ´ ˜,ng sô´ tu., nhiên tu `, 1 d ho. p nhu ê n u − 1 d u o c phân chia b a ˘ ng phu o ng ¯ ¯ . n ,, , , ` ph´ ap bâ´t k`y v` ao n tâp h o p con, m` a t u ng d¯ôi môt không c´ o phâ`n tu . . . , , , chung. Chu´ng minh ra ˘`ng tô`n tai hai sô´ cua c` ung môt a . . tâp . ho. p con, m` , , , ung c˜ ung na ˘`m trong tâp ay ; hoa ˘. c l` a tô`n tai tông cua ch´ . ho. p con n` . môt . , ˜ ´ sô´ trong môt t âp h o p con n` a o d o , m` a sau khi nhân d ôi v ân thu ôc t âp ¯ ¯ . . . . . , ho. p con n` ay. , , , `,i giai. K´y hiêu ˜ cho l` Lo a l` a A1 , A2 , . . . , A n . ¯a . nh˜ung tâp . ho. p con d , ,, , ´nh Ch´ ung ta x´et tâp a ch´ ung d¯uo. c d ¯iêm, m` ¯a . ho. p bâ´t k`y M gô`m un d , , `˘ng phuong ph´ ´ trong c´ sô´ ba ap n` ao d ac sô´ 0, 1, 2, . . . , un − 1. Ch´ ung ¯o , , , `˘ng n mâ`u ˘ng nô´i c´ ta bôi mâ`u nh˜ung d ac d ¯oan ¯iêm trong M ba . tha , , ´, i v` ´,i ˘ng nô´i c´ c1 , c2 , . . . , cn theo c´ ach sau: Ðoan ac d a j, vo ¯iêm thu . tha , ,, , ` , ˘ng mâ`u ck khi v` i>jd a chı khi hiêu ôc ¯uo. c son ba ., i − j thu . tâp . ho. p , , , ,´, ´ cua M l` Ak . Theo b` ai to´ an truo c 5.5 ba d ao d ad ¯iêm n` ¯o ¯ınh cua môt . , ,, , tam gi´ ac c` ung mâ`u. Gia su c´ ac d iê m n` a y l` a i, j, k v` a i > j > k. Khi ¯ ´ ´ ung môt d o c´ a c sô i − j, j − k v` a i − k = ( i − j) + ( j − k) na`˘m trong c` ¯ .

48

,, ,, Chuong 5. Mo rông nguyên l´y Ðirichlê .

, tâp a ch´ ung c´ o c´ ac t´ınh châ´t ta câ`n t`ım. . ho. p As v`

J

, , . 5.7. Trong ma˘. t pha˘ng cô´ d¯inh hê. toa d¯ô. Oxy. Ch´ ung ta x´et tâp ho. p . . . , ,, , ˜,ng d¯iêm v´ ˜,ng sô´ nguyên R gô`m nhu oi toa a nhu ¯ô. ( x, y), o d¯ây x, y l` . d , ,, , ` ˜ d v` a 1 ≤ x ≤ 12, 1 ≤ y ≤ 10. Môi ay d¯uo. c son ba ˘ng môt ¯iêm n` . mâ`u , , , ˜ nhât tra ˘´ng, xanh hoa ˘. c d¯o. Chu´ng minh ra ˘`ng tô`n tai h`ınh chu o c´ ac . ,c´ , ,. , , , ˜ ng d¯iêm cua canh oi c´ ac truc a d¯ınh cua n´ o l` a nhu . song song v´ . toa . d¯ô, . m` ,, , R d¯uo. c son c` ung môt . mâ`u. , , `,i giai. Trong chu ´,ng minh ta d` ˘ng d Lo ung hai kha ¯inh . sau: , , , ´,i n phâ`n tu,, th`ı sô´ lu,o.,ng tâp a) Nê´u X l` a tâp . h˜uu han . vo . ho. p con , ,, n ( n − 1) ´,i x ∈ X, y ∈ X l` ˘. p phâ`n tu { x, y}, vo cua X gô`m c´ ac ca a . 2 , , ,, ˘ng b) Moi an bâ´t d ¯ê`u thoa m˜ ¯a . sô´ duong bâ´t k`y x1 , x2 , . . . , xn d ´,c thu x12 + x22 + · · · + xn2 x + x2 + · · · + x n 2 ≥( 1 ) 2 2 , , `˘ng chı xây ra khi x1 = x2 = . . . = xn . Dâ´u ba , , ,, Tro lai ai to´ an d ac d a 120, nên ¯ang x´et, v`ı tông sô´ c´ ¯iêm ta x´et l` . b` , , , ,, , , ˜ ´,i môi ´ıt nhâ´t c´ o 40 d ung mâ`u, gia su l` a mâ`u d ¯iêm d ¯uo. c bôi c` ¯o. Vo , , ,`, ˘ng d i = 1, 2, . . . , 12 ch´ ung ta k´y hiêu ad ¯uong tha ¯i qua d ¯iêm (i, 0) . li l` , , , ´i truc v` a song song vo o ni d ¯iêm d ¯o, i = 1, 2, . . . , 12, . Oy. Nê´u trên li c´ , , , ˘. p n1 + n2 + · · · + n12 ≥ 40. Ngo` ai ra t`u ni d o thê tao ac ca ¯iêm c´ . ra c´ ,, ´ , , n i ( n i − 1) ˜ i = 1, 2, . . . , 12 ´,i môi ˘p c´ d¯ôi d o sô´ luo. ng . Ngh˜ıa l` a vo ¯uo. c sa 2 , n ( n − 1) ˘. p sô´ nguyên { j1 , j2 } thoa m˜ ´ ng i i an 1 ≤ j1 ≤ tô`n tai ca ¯u . d 2 , , ,, ˘. t kh´ 10, 1 ≤ j2 ≤ 10 v` a c´ ac d a {i, j2 } l` ad ac sô´ luo. ng ¯iêm {i, j1 } v` ¯o. Ma , , , ´,i 1 ≤ j1 ≤ 10, 1 ≤ j2 ≤ 10 ˘. p nh˜ung sô´ nguyên { j1 , j2 } vo cua tâ´t ca ca

5.2. V´ı du.

49

, , 10(10 − 1) `˘ng tông c´ ´,ng minh ra = 45. Ch´ ung ta phai chu ac sô´ 2 n1 ( n1 − 1) n2 ( n2 − 1) n12 (n12 − 1) , ,..., 2 2 2 , , , , , ´n hon 45. T`u d ´,i ´ suy ra tô`n tai lo ac nhau i1 v` a i2 vo ¯o . nh˜ung chı sô´ kh´ ´,i 1 ≤ j1 ≤ 10, 1 ≤ j2 ≤ ˘. p sô´ nguyên { j1 , j2 } vo 1 ≤ i1 < i2 ≤ 12 v` a ca , , ,, , 10 sao cho 4 d ad ¯iêm {i1 , j1 }, {i1 , j2 }, {i2 , j1 }, {i2 , j2 } l` ¯o. Boi nh˜ung , , , , ´,i c´ ´,i truc d ay l` ad ac canh song song vo ¯iêm n` ¯ınh cua h`ınh ch˜u nhât . vo . . , ,, , ´ toa d ô, nhu v ây b` a i to´ a n d u o c ch u ng minh. ¯ ¯ . . . . , , , ´,ng minh Nhu vây, on phai chu . ta chı c` `˘ng ba

n1 ( n1 − 1) n2 ( n2 − 1) n ( n − 1) + + · · · + 12 12 > 45 2 2 2 , ,, ´,c b) ˘ng thu Su dung bâ´t d ¯a . S=

1 ((n1 − 2 1 = ((n1 − 2

1 2 1 1 1 1 1 1 1 ) − ) + ((n2 − )2 − ) + · · · + ((n12 − )2 − ) 2 4 2 2 4 2 2 4 1 2 1 2 1 2 1 ) + (n2 − ) + · · · + (n12 − ) ) − 12 2 2 2 8 1 1 1 (n1 − ) + (n2 − ) + · · · + (n12 − ) 1 2 2 2 )2 − 3 ≥ .12( 2 12 2 1 1 2 3 = ((n1 + n2 + · · · + n12 ) − 12 ) − 24 2 2 2 3 34 3 1 − > 45 ≥ (40 − 6)2 − = 24 2 24 2

S=

v`ı n1 + n2 + · · · + n12 ≥ 40.

J

, . 5.8. (B`ai thi Olympic to´an quô´c tê´ lâ`n thu´ 20, 1978) Môt an . hôi . to´ ,, , , hoc ac th` anh viên o 6 nuo´c. Danh s´ ach c´ ac hôi . bao gô`m c´ . viên gô`m ,`, , , , , ` 1 d¯ê´n 1978. Chu´ng minh ra ´nh sô´ b´ 1978 nguoi d¯uo. c d¯a ao danh tu ˘`ng , tô`n tai o sô´ b´ ao danh gâ´p d¯ôi sô´ b´ ao danh cua . ´ıt nhâ´t môt . hôi . viên c´

50

,, ,, Chuong 5. Mo rông nguyên l´y Ðirichlê .

, , ,, môt ac c` ung nuo´c, hoa ˘. c ba ˘`ng tông hai sô´ b´ ao danh cua hai . hôi . viên kh´ ,, , hôi ung môt oi m`ınh. . viên c` . nuo´c v´ , ,´, `,i giai. T`u, 329.6 < 1978 suy ra môt Lo ac nuo c (k´y hiêu a . trong c´ ., l` , , A) c´ o không ´ıt hon 330 d¯ai a ch´ ung ta c´ o thê viê´t . biêu trong hôi . v` , sô´ b´ ao danh a1 < a2 < . . . < a330 < . . . Ch´ ung ta x´et nh˜ung hiêu . , ´ ´ ´ xi = a330 − ai , i = 1, 2, . . . 329. Nê´u c´ o môt sô x n` a o d o tr` u ng v o i aj ¯ i . , , , (sô´ b´ ao danh cua môt ung ta c´ o a330 = ai + a j , ¯aj i biêu cua A) th`ı ch´ . d , , ´ ng minh xong. Nê´u xi 6= a j vo ´i moi ˜ chu b` ai to´ an d a ¯a . i, j, th`ı sô´ xi l` , , , , , ´c c` sô´ b´ ao danh cua môt on lai. ¯a. i biêu thuôc . sô´ d . 5 nuo . Bây gi`o, ,´, v`ı 65.5 < 329 , th`ı ´ıt nhâ´t c´ o môt am nuo c n` ay (k´y hiêu l` a . trong n˘ , . , B) s˜e c´ o không ´ıt hon 66 th` anh viên, m` a sô´ b´ ao danh cua ho. l` a ´ l` môt ac sô´ x1 , x2 , . . . , x329 . Cho c´ ac sô´ d a b1 < b2 < b3 < ¯o . trong c´ , ´i bi = xni , i = 1, 2, . . . , 66. Ch´ . . . < b66 < . . . vo ung ta lai . x´et hiêu . , ´ ´ ´ yi = b66 − bi , i = 1, 2, . . . , 65 Nê´u môt hi êu n` a o d o tr` u ng v o i sô b´ a o ¯ . . , , , , ´i hai sô´ i danh b j cua môt ¯ai . d . biêu cua B th`ı b66 = bi + b j . Nê´u vo ´ ch´ ung ta c´ o yi = ak , th`ı ak = b66 − bi = xn66 − xni = v` a k n` ao d ¯o ˘. c l` a330 − an66 − ( a330 − ani ) = ani − an66 hoa a ani = an66 + ak . Nê´u hai , ,`, , , truong ho. p trên không xây ra, th`ı nh˜ung sô´ n` ay s˜e l` a sô´ b´ ao danh , , ,´, ,´, cua d on lai a suy ra ´ıt nhâ´t môt ac nuo c n` ay ¯ai . biêu 4 nuoc c` . v` . trong c´ , ´i sô´ b´ c´ o sô´ hôi a 17 vo ao danh yi . Tiê´p tuc qu´ a tr`ınh . viên ´ıt nhâ´t l` , . , ˘. p lai nhu vây a la ung ta c´ o kê´t luân ai to´ an. . v` . l´y luân . trên ch´ . cua b`

J

,, ,, . 5.9. (Ðê` thi To´an vô d¯ich o . nuo´, c Anh, 1978) Môt . h`ınh lâp . phuong c´ , , ` ` ` canh ba ˘ng 15 chu´a 11.000 d¯iêm. Chu´ng minh ra ˘ng c´ o môt . . h`ınh, câu , , , ` ´ ´ ˜ b´ an k´ınh ba ˘ng d¯on vi. chu´a ´ıt nhât 6 d¯iêm trong sô 11.000 d¯iêm d¯a cho. , , ,, ˜ canh `˘ng `,i giai. Chia môi Lo cua h`ınh lâp anh 13 phâ`n ba . . phuong th` , , , , 3 nhau, thê´ th`ı h`ınh lâp anh 13 = 2187 ¯â`u d ¯uo. c chia th` . phuong ban d

5.2. V´ı du.

51

, ,, ,, ´,a 11.000 h`ınh lâp phuong nho. V`ı h`ınh lâp phuong ban d ¯â`u chu . . , , , ,, ´,a ´ıt nhâ´t 6 d d ¯iêm, nên tô`n tai ¯iêm. . môt . h`ınh lâp . phuong nho chu , , 15 ,, ´ b´ D˜ê thâ´y canh cua h`ınh lâp a , do d an k´ınh h`ınh ¯o . . phuong nho l` 13r r , 1 15 2 1 675 , , câ`u ngoai ar= 3( ) = < . tiê´p h`ınh lâp . phuong nho l` 2 13 2 169 r 1 676 1√ ´,a ´ıt = 4 = 1. Vây: an k´ınh 1 chu . Tô`n tai . môt . h`ınh câ`u b´ 2 169 , 2 , ˜ cho. nhâ´t 6 d ¯iêm trong 11.000 d ¯iêm d ¯a

J

, , , . 5.10. Trong ma˘. t pha˘ng cho môt ¯iêm, môt . , sô´ h˜uu han . d . sô´ trong ,, `˘ng d `˘ng môt ˘ng. Ch´ ch´ ung d tha ung ta n´ oi ra ay ¯uo. c nô´i lai ¯oan . ba . d˜ ,. , , , ˘ng d a môt e1 , e2 , . . . , em t`u c´ ac d tha ¯oan ¯uo. c nô´i l` . . dây chuyê`n, nê´u , ˘. p d ˘ng liê`n nhau ei v` moi a ei+1 c´ o chung môt ut, ¯oan ¯â`u m´ . ca . tha . d , , , , ˘ng trong dây chuyê`n i = 1, 2, . . . , m − 1. Sô´ luo. ng m nh˜ung d¯oan . tha , goi ad ai cua n´ o. Ch´ ung ta x´et b` ai to´ an: ¯ô. d` . l` , , , , ,, `, c´ ˜,ng d¯oan Cho n d¯iêm, tu ac d¯iêm n` ay du. ng nhu tha ˘ng, sô´ luo. ng d¯oan . . , , , , , , ˜, ,, ´ ˜ du. ng l` ´ tha ˘ng d¯a a q. Gia su nhu ng d¯oan th a ˘ ng d u o c d a nh sô m ôt c´ a ch ¯ ¯ . . . , ` ´ trong c´ bâ´t k`y ba ˘ng c´ ac sô´ 1, 2, . . . , q. Khi d¯o ach tao . h`ınh nhu vây . tô`n , 2q , tai oi d ai nho nhâ´t l` a , trên dây chuyê`n n` ay ¯ô. d` . môt . dây chuyê`n v´ n , , , , , ˜ ng d¯oan c´ ac sô´ u´ng v´ oi nhu ˘ng th` anh phâ`n lâp ay sô´ giam . tha . nên môt . d˜ nga ˘. t. , , , , , `,i giai. Goi ˘ng, Lo a câ´u h`ınh bâ´t k`y cua nh˜ung d ad ¯iêm v` ¯oan . G l` . tha ,, `˘ng sô´ tu., nhiên nhu, d ´,i moi ´nh sô´ ba ˜ ra. Vo m` a ch´ ung d ai d ¯uo. c d ¯a ¯ê` b` ¯a . , ` v thuôc ung ta k´y hiêu ad ai cua dây chuyên d` ai nhâ´t ¯ô d` . G ch´ . LG (v) l` , . , , , ´˘t d `˘ng phuong ph´ trong G, m` a n´ o ba a giam dâ`n. Ba ap qui ¯â`u t`u v v` , , , , ,, ´ ´ ´ ˘ng thuc sau d nap ung ta s˜e chung minh bât d an ¯a ¯uo. c thoa m˜ . ch´ LG (v1 ) + LG (v2 ) + · · · + LG (vn ) ≥ 2q

(5.1)

52

,, ,, Chuong 5. Mo rông nguyên l´y Ðirichlê .

, , , ,, ´,ng od a tâ´t ca c´ ac d ¯ây v1 , v2 , . . . , vn l` ¯iêm thuôc . G. Kê´t luân . phai chu ,, , minh suy ra t`u (5.1) theo nguyên l´y Ðirichlê mo rông: Môt . . trong 2q , , , `˘ng ´n hon hoa ˘. c ba c´ ac sô´ LG (v1 ), LG (v2 ), . . . , LG (vn ) lo , nhu vây . n , , , , , ´ ng ´i t´ınh châ´t d ˜ chı ra. Do d ´ chı c` tô`n tai on phai chu ¯a ¯o . dây chuyê`n v,o , , , ,, ,, , ´i n d ˘ng d an vo aqd minh (5.1) d ¯iêm cua G v` ¯oan ¯uo. c ¯uo. c thoa m˜ . tha , , , , , ` `˘ng qui nap ´,i q = 1 bâ´t d ´nh sô´ nhu mô ta o d d ai. Ba ¯a ¯âu b` ¯ang . vo , , , , , , ´ c (5.1) l` thu a hiên nhiên. Gia su (5.1) thoa m˜ an cho moi . câ´u, h`ınh , , , ´i q − 1 d ´i n d ˘ng v` vo tha a ch´ ung ta xem x´et câ´u h`ınh vo a ¯oan ¯iêm v` , . , , , ´i hai d ˘ng. Nê´u d ˘ng k´y hiêu qd a q nô´i vo a ¯oan ¯oan ¯iêm vi v` . tha . th, a . l` , , ,, ´ 0 ˘ng n` v j th`ı ch´ ung ta bo d ay v` a nhân ¯oan ¯uo. c câu h`ınh G cua n . , tha . d , , ´i câ´u h`ınh n` ˘ng. Vo d a q−1 d ay theo qui nap ung ta ¯iêm v` ¯oan . tha . ch´ , c´ o LG0 (v1 ) + LG0 (v2 ) + · · · + LG0 (vn ) ≥ 2(q − 1). Bây gi`o ch´ ung ta , , , x´et d at t`u x j c´ od ai LG0 (v j ), ¯iêm v j . Dây chuyê`n giam dâ`n xuâ´t ph´ ¯ô. d` , ,, , , , , ˘ng trong dây chuyê`n n` v` a nh˜ung d ay d ¯oan ¯uo. c k´y hiêu . tha . boi nh˜ung ˜ 1, 2, . . . , q − 1. Nê´u nô´i thêm v` ´ trong dây sô´ n` ao d ao dây chuyê`n ¯o , ,, ˘ng c´ môt d o k´y hiêu ung ta s˜e nhân ¯oan ¯uo. c dây chuyê`n . th,a . q ch´ . d ,. , , ´i d ´i d giam vo a vo ai LG0 (v j ) + 1. Theo d ngh˜ıa ¯iêm d ¯â`u vi v` ¯ô. d` ¯inh . , `˘ng môt ´,ng cua LG (vi ) ch´ ung ta c´ o LG (vi ) ≥ LG0 (v j ) + 1. Ba ach chu . c´ ,, , ,, ´,ng minh d minh ho` an to` an tuong tu. ch´ ung ta c˜ ung chu ¯uo. c LG (v j ) ≥ , ´,i moi ˘. t kh´ L0G (vi ) + 1. Ma ac, LG (vk ) = LG0 (vk ) vo ¯iêm vk thuôc . d . G m` a n´ o kh´ ac vi v` a v j . Suy ra, LG (v1 ) + LG (v2 ) + · · · + LG (vn ) ≥ LG0 (v1 ) + LG0 (v2 ) + · · · + LG0 (vn ) + 2 ≥ 2(q − 1) + 2 = 2q.

J

` tâp 5.3. Bai . , ˜ . 5.11. (Ðê` thi vô d¯ich . Áo - Balan, 1978) Cho 1978 tâp . ho. p, môi ,, , , `˘ng hai tâp ´ ng môt tâp o 40 phâ`n tu. Biê´t ra o d¯u . ho. p c´ . ho. p bâ´t k`y c´ ., ,, ,, , ` ` ` ` ´ ´ ng minh ra ˘ng, tôn tai phân tu chung. Chu . môt . phân tu thuôc . tât ca , 1978 tâp . ho. p trên.

5.3. B` ai tâp .

53

,, , , `˘ng moi ´,ng minh ra ´,a 55 sô´ chon d . 5.12. Chu ¯uo. c t`u tâp . tâp . ho. p chu . . , , , ´ ` ´ ´ a hai sô m` ˘ng 9. ho. p sô {1, 2, 3, . . . , 100} chu a hiêu ung ba . cua ch´ , , , . 5.13. Cho n l`a sô´ tu. nhiên. Cho n tâp ho. p con A1 , A2 , . . . , An cua . , ´,i t´ınh châ´t: vo ´,i moi ˘. t pha ˘ng vo ma . i = 1, 2, . . . , n tô`n tai . n + 1 ph´ep , , , ´ ´ tinh ac anh cua Ai qua c´ ac ph´ep tinh ¯ôi môt . tiên sao ,cho c´ . tiên t`u, ng d . , ` ` ´ ng minh ra ˘ng tôn tai ˘. t không c´ od môt d ¯iêm chung. Chu ¯iêm trên ma . . , , `˘m trong bâ´t cu ´, tâp ˘ng, không na ao, i = 1, 2, . . . , n. pha . ho. p Ai n` , , ´,i canh . 5.14. Trong phâ`n trong cua môt ac d ai 15 d ¯ê`u vo ¯on . tam gi´ . d` , `˘ng phu,o,ng ph´ ´,ng minh vi, ung ta chon ap bâ´t k`y 111 d ¯iêm. Chu . ch´ . ba √ , , ,, `˘ng tô`n tai ´,i d ra on vo a n´ o phu ´ıt nhâ´t 3 d ¯u`ong k´ınh 3, m` ¯iêm . h`ınh tr` , ,, , ˜ chon trong sô´ nh˜ung d ¯iêm d ¯a . o trên.

. 5.15. (Ðê` thi vô d¯ich om . Quô´c gia Bungari, 1977) Trong môt . nh´ ,`, ,`, , nguoi, hai nguoi X v` a Y goi a quen nhau gi´ an tiê´p, nê´u ho. tru. c . l` , ,, ˘. c l` tiê´p quen nhau, hoa a nê´u tô`n tai . môt . dây truyê`n nh˜ung ngu`oi Z1 , Z2 , . . . , Z p sao cho X v` a Z1 quen nhau, Z1 v` a Z2 quen nhau,... ,, , ´ gô`m 134 ngu`oi, c` , Z p v` a Y quen nhau. Cho biê´t nh´ om d on gi˜ua ¯o , ,, , ,, ˜ nh´ ˜ biê´t ´ıt nhâ´t c´ môi om nho 8 ngu`oi t`u nh´ om d o hai ngu`oi quen ¯a , ,, `˘ng c´ ´,ng minh ra gi´ an tiê´p. Chu o môt om nho 12 ngu`oi trong nh´ om . nh´ , ,`, , , ˘. p hai ngu`oi trong nh´ ˜ biê´t, m` nguoi d a moi om nho n` ay d ¯a ¯ê`u quen . ca nhau gi´ an tiê´p.

54

,, ,, Chuong 5. Mo rông nguyên l´y Ðirichlê .

, , CHUONG

6

´ ` TÂP BAI . SÔ HOC . NÂNG CAO

, , , 6.1. Ð.inh l´ y co ban cua sô´ hoc .

, ,, , , Trong l´y thuyê´t sô´ hoc o môt ¯inh . l´y co ban: . co so c´ . d , , , , `,a Moi on hon môt o thê phân t´ıch ra t´ıch c´ ac thu . sô´ tu. nhiên l´ . d¯ê`u c´ , , ´ l` sô´ nguyên tô´ v` a phân t´ıch d¯o a duy nhâ´t nê´u ta không d¯ê ´y d¯ê´n thu´ , , `,a sô´. tu. cua c´ ac thu , Cho p1 , p2 , . . . , pn l` a nh˜ung sô´ nguyên tô´ kh´ ac nhau. Trong , ,, , chuong n` ay ch´ ung ta x´et môt ., sô´ kê´t luân . vê` vê` tâp . ho. p M cua , , `˘m trong c´ ac sô´ tu. nhiên m` a n´ o c´ o thê phân t´ıch ra c´ ac th`ua sô´ na , , , , , ´i dang { p1 , p2 , . . . , pn }. Moi o thê biêu di˜ên duo . sô´ x cua M c´ . x = p1α1 p2α2 . . . pαnn

,, od a c´ ac sô´ nguyên không âm. ¯ây α1 , α2 , . . . , αn l`

6.2. V´ı du. , , `, tâp . 6.1. Tu ach bâ´t k`y 2n + 1 sô´. Chu´ng minh . ho. p M chon . môt . c´ , `,a chon ra ˘`ng tô`n tai a t´ıch cua ch´ ung l` a môt . hai sô´ trong tâp . vu . m` . sô´ ,, ch´ınh phuong. , , α1 α2 αn `˘ng môt `,i giai. Nhân Lo a sô´ . x´et ra . sô´ tu . nhiên x = p1 p2 . . . pn l` , , ,, ˜˘n. Ch´ ch´ınh phuong khi v` a chı khi tâ´t ca c´ ac sô´ m˜ ud ung ta ¯ê`u cha

56

,, Chuong 6. B` ai tâp . sô´ hoc . nâng cao

, ,, , , ´, ´,i dang ´,i bô. ˜ chon biêu di˜ên moi ng vo o trên v` a cho tuong u ¯a . sô´ d . vo . ,, , , a c´ ac sô´ du cua c´ ac sô´ m˜ u n-sô´ (α1 , α2 , . . . , αn ), o d ¯ây α1 , α2 , . . . , αn l` , , ´, ˘. c αi = 1 tuong ung α1 , α2 , . . . , αn khi chia cho 2. R˜ o r` ang αi = 0 hoa , , , ´ ´ tu. n sô´ gô`m ´i moi vo a bô. xêp thu . i = 1, 2, . . . , n. Vây . (α, 1 , α2 , . . . , α,n ) l` , ,, , sô´ 0 v` a 1. Theo l´y thuyê´t tô ho. p, tâ´t ca n-bô. nhu vây o sô´ luo. ng . c´ ,, , l` a 2n , c` on c´ ac sô´ ta d o sô´ luo. ng l` a 2n + 1. Nhu vây ¯ang x´et c´ . ´ıt nhâ´t ´˘p xê´p gô`m sô´ 0 v` c´ o 2 sô´ trong ch´ ung c´ o c` ung bô. sa a sô´ 1 giô´ng , ,, β β β α α α 1 ´,i ´ l` nhau. Gia su c´ ac sô´ d a x = p1 1 p2 2 . . . pnn v` a y = p1 p2 2 . . . pn n vo ¯o , ´,c sau c` ˘ng thu ch´ ung c´ o ( α 1 , α 2 , . . . , α n ) = ( β 1 , β 2 , . . . , β n ) . Ða ung c´ o ´ ´ c´ ngh˜ıa l` a αi = βi , i = 1, 2, . . . , n . Do d ac sô m˜ u αi v` a βi c´ o c` ung ¯o , , , ´ ˜ ´ ˘n le nhu nhau voi bât k`y i = 1, 2, . . . , n. Khi d ´ α1 + β 1 , t´ınh cha ¯o ˜ ´ ˘n v` α2 + β 2 , . . . , αn + β n , l` a c´ ac sô cha a theo nhân ¯â`u t´ıch . x´et ban d β1 β2 βn α1 + β 1 α2 + β 2 αn + β n α1 α2 αn ´ ng xy = ( p1 p2 . . . pn )( p1 p2 . . . pn ) = p1 p2 . . . pn d ¯u ,, ´ l` a sô ch´ınh phuong. , , , ˜,a n + 1 sô´ trong tâp . 6.2. Chu´ng minh ra˘`ng giu hop M c´ o thê chon . . , ,, ,, ´ ´ d¯uo. c môt ai sô m` a t´ıch cua ch´ ung l` a môt . v` . sô ch´ınh phuong. , , , ˜ tâp `,i giai. Cho x1 , x2 , . . . , xn l` ´,i môi Lo a nh˜ung sô´ bâ´t k`y cua M. Vo . , , ´ con kh´ ac trông {i1 , i2 , . . . , ik } cua tâp ung ta . ho. p {1, 2, . . . n + 1},, ch´ ˜ên c´ x´et c´ ac xi1 , xi2 , . . . , xik (tâ´t nhiên sô´ n` ay c˜ ung thuôc M). Biê u di ac . , , , , ˜ tâp sô´ n` ay theo dang a môi . chuân v` . ho. p con {i1 , i2 , . . . , ik } cho tuong ,, , , , , ´ ng vo ´i n-bô. (α1 , α2 , . . . , αn ) , o d a c´ ac sô´ du cua u ¯ây α1 , α2 , . . . , αn l` , , ´, , ,, c´ ac sô´ m˜ u tuong u ng α1 , α2 , . . . , αn khi chia cho 2. Nhung sô´ luo. ng , , ˜ cua {1, 2, . . . , n + 1} l` nh˜ung tâp ac rông a 2n+1 − 1, c` on sô´ . con kh´ ,, , , n ´˘p gô`m nh˜ung sô´ 0 v` ac n-bô. sa a 1 l` a 2 . Suy ra tô`n tai luo. ng c´ . nh˜ung , tâp ac trô´ng kh´ ac nhau {i1 , i2 , . . . , ik } v` a { j1 , j2 , . . . , jl } ., ho. p con kh´ , , , ´, , ´˘p cua ´i c` cua {1, 2, . . . , n + 1}, m` a ch´ ung tuong ung vo ung môt . n-bô. sa , , nh˜ung sô´ du. Ðiê`u n` ay c´ o ngh˜ıa l` a nê´u xi1 xi2 . . . xik = p1α1 p2α2 . . . pαnn β β β v` a x j1 x j2 . . . x jk = p1 1 p2 2 . . . pn n th`ı c´ ac sô´ m˜ u αi v` a β i c´ o c` ung t´ınh

J

6.2. V´ı du.

57

, , , , ˜˘n le vo ´i i = 1, 2, . . . , k. Ðiê`u n` cha ay c´ o ngh˜ıa l` a t´ıch cua nh˜ung ,, sô´ xi1 , xi2 , . . . , xik , x j1 , x j2 , . . . , x jk l` a ch´ınh phuong. Nê´u {i1 , i2 , . . . , ik } , ,, ,, ˜d v` a { j1 , j2 , . . . , jl } không c´ o phâ`n tu chung, th`ı b` ai to´ an d ¯a ¯uo. c giai. ,, ,, , ˘. p lai ´ ng Tru`ong ho. p nguo. c lai, ¯u . trong P = xi1 xi2 . . . xik x j1 x j2 . . . x jk la . d , ´ ˜ nhung sô xs , m` a s thuôc {i1 , i2 , . . . , ik } v`a { j1 , j2 , . . . , jl }. Ch´ ung ta , . , , , , ,, ´ ` ˜ loai ac nhung xs nhu vây a nhân ¯uo. c t´ıch cua . tru trong P tât ca c´ . v` . d ,, ´ ng l` v` ai sô´ trong sô´ x1 , x2 , . . . , xn+1 m` a n´ od a ch´ınh phuong. ¯u

J

, , , ˜,a moi `, tâp . 6.3. Chu´ng minh ra˘`ng giu 3.2n + 1 sô´ tu ho. p M c´ o thê . . , , ,, `,a bâc chon a t´ıch cua ch´ ung l` a l˜ uy thu . d¯uo. c 4 sô´ m` . bô´n cua môt . sô´. , `,i giai. V`ı 3.2n + 1 > 2n + 1 gi˜u,a nh˜u,ng sô´ d ˜ chon Lo ai 6.1 ¯a . theo b` , ,, ´ ´ c´ o hai sô m` a t´ıch cua ch´ ung l` a môt ach hai sô´ . sô ch´ınh phuong. T´ , n ´p dung trên ra th`ı c` on lai aa tiê´p 6.1 cho hai sô´ n˜ua m` a . 3.2 − 1 sô´ v` . , , ,, ˘. p lai t´ıch cua ch´ ung l` a sô´ ch´ınh phuong. Ch´ ung ta c´ o thê la qu´ a tr` ınh . , , (3.2n − 1) − (2n + 1) , n` ay ´ıt nhâ´t = 2n + 1 lâ`n. Gia su ch´ ung ta nhân . 2 ,, n , , , ´i xi yi l` ˘. p ( x1 , y1 ), ( x2 , y2 ), . . . , vo d a c´ ac sô´ ch´ınh phuong, ¯uo. c 2 + 1 ca n `˘ng theo c´ ˘. p sô´ i = 1, 2, . . . , 2 + 1. Ch´ u ´y ra ach chon ac ca . trên th`ı c´ , ( x1 , y1 ), ( x2 , y2 ), . . . , ( x2n +1 , y2n +1 ) t`ung d¯ôi môt ac nhau. Ðê´n d ¯ây . kh´ √ √ √ , suy ra c´ ac sô´ x1 y1 , x2 y2 , . . . , x2n +1 y2n +1 l` a nh˜ung sô´ nguyên v` a , , , `˘ng t´ıch cua hai sô´ ´a ´p dung thuôc ¯o . M. Khi d . môt . lâ`n n˜ua 6.1 chı ra ra ,, , 4 ´ trong ch´ n` ao d ung l` a ch´ınh phuong. Nhu vây a ¯o . xi yi x j y j = t ngh˜ıa l` , , , t´ıch cua 4 sô´ t`ung d¯ôi môt ac nhau xi , yi , x j , y j l` a l˜ uy th`ua bâc . kh´ . bô´n , cua môt . sô´. , ˜,ng sô´ tu., . 6.4. Cho p l`a môt a a1 , a2 , . . . , a p+1 l` a nhu . sô´ nguyên tô´ le v` 2 , , , , , nhiên kh´ ac nhau nho hon p. Chu´ng minh ra ˘`ng v´ oi moi . sô´ tu. nhiên , , , , r nho hon p, tô`n tai o thê ba ˘`ng nhau) ai v` a a j , m` a t´ıch cua . hai sô´ (c´ , ´ng l` ch´ ung khi chia cho p c´ o sô´ du d a r. ¯u

J

,, Chuong 6. B` ai tâp . sô´ hoc . nâng cao

58

, , ,, `,i giai. Áp dung môt Lo ab ¯inh . l´y trong l´y thuyê´t sô´ co so: Nê´u a v` . . d ´ ´ ` ´ l` a hai sô nguyên tô c` ung nhau, th`ı tôn tai . môt . sô nguyên x sao cho ax ≡ 1 (mod b). ˜ i = 1, 2, . . . , p + 1 , c´ ´,i môi ac sô´ ai v` a p l` a nguyên tô´ c` ung Vo 2 , nhau, v`ı 1 ≤ ai ≤ p − 1 v` a p l` a sô´ nguyên tô´. Áp dung d l´y v`ua ¯inh . . , ´,i moi ph´ at biêu trên, tô`n tai ac sô´ nguyên b1 , b2 , . . . , b p+1 sao cho vo . . c´ 2 , , , p +1 ´ c sau d ˘ng thu i = 1, 2, . . . , 2 c´ ac d an ¯a ¯ê`u thoa m˜ a i bi ≡ 1

(mod p)

(6.1)

Ch´ ung ta x´et d˜ ay sô´ a1 , a2 , . . . , a p+1 , rb1 , rb2 , . . . , rb p+1 . Ch´ ung ta c´ o 2 2 , tâ´t ca p + 1 sô´ sao cho ´ıt nhâ´t hai sô´ trong ch´ ung khi chia cho p , , c´ o c` ung môt ac sô´ a1 , a2 , . . . , a p+1 kh´ ac nhau . sô´ du. Theo gia thiê´t c´ 2 , , , , ho` an to` an v` a nho hon p. Suy ra sô´ du cua ch´ ung theo môdd un p ´,i p, nên d˜ê d` l` a kh´ ac nhau. V`ı r l` a sô´ nguyên tô´ c` ung nhau vo ang , , , , ` ´ ng minh d ˘ng c´ chu ac sô´ rb1 , rb2 , . . . , rb p+1 cho sô´ du ho` an to` an ¯uo. c ra 2 , , kh´ ac nhau khi chia cho p. Ngh˜ıa l` a chı tô`n tai a j sao . hai chı sô´ i v` , , , cho ai ≡ rb j (mod p). T`u (6.1) ch´ ung ta nhân ¯uo. c ai a j ≡ rb j a j ≡ . d r.1 ≡ r (mod p). , , , , . 6.5. Bô d¯ê` Tue: Cho n l`a sô´ tu. nhiên l´ on hon 1 v` a a l` a sô´ nguyên , ´ tô`n tai tô´ c` ung nhau v´ oi n. Khi d¯o ac sô´ nguyên x v` a y, m` a ch´ ung . c´ , ,, , , thoa m˜ an phuong tr`ınh ax ≡ y (mod n) v` a c´ ac bâ´t phuong tr`ınh √ √ 1 ≤ x ≤ [ n ], 1 ≤ | y | ≤ [ n ].

J

, , ,, `,i giai. Ch´ Lo ung ta x´et tâ´t ca c´ ac sô´ c´ o dang au − v, o d a v chay ¯ây u v` . . √ , , , ´ ´ ´ ´ ´ d ôc l âp nhau trong c´ a c sô 0, 1, 2, . . . , [ n ] . Sô lu o ng tâ t c a c´ a c sô d ¯o ¯. . √ √ . 2 √ 2 , , 2 ´n hon n v`ı ([ n] + 1) > ( n) = n. l` a ([ n] + 1) . Suy ra sô´ n` ay lo ,, , ˘. p ´ ng n sô´ du kh´ Boi c´ od ac nhau theo môd ¯u ¯un n, nên tô`n tai . hai ca , sô´ kh´ ac nhau (u1 , v1 ), (u2 , v2 ) t`u c´ ac sô´ nguyên sao cho 1 ≤ ui ≤

6.2. V´ı du.

59

√ √ [ n], 1 ≤ vi ≤ [ n], i = 1, 2 v`a au1 − v1 ≡ au2 − v2 (mod n). ,, ,, ´,ng minh d D˜ê d` ang chu ac v2 . Thât ¯uo. c v1 kh´ . vây, . nê´u nguo. c lai . th`ı ´ au1 ≡ au2 (mod n),suy ra u1 ≡ u2 (mod n), do a l` a sô nguyên tô´ , , , , , ´i n. Ma ˘. t kh´ ac, u1 , u2 l` a c´ ac sô´ tu. nhiên nho hon n. Nhu c` ung nhau vo , , , ´,c sau c` ˘ng thu vây ung chı c´ o môt ang duy nhâ´t u1 = u2 . Do ¯a . d . kha n˘ , ´,i c´ ˘. p (u1 , v1 ), (u2 , v2 ) tr` ´ ca d ung nhau, tr´ ai vo ach chon trên. Nhu vây ¯o . . , , , ´ u1 v` a u2 c˜ ung phai kh´ ac nhau. C´ o thê gia thiêt u1 > u2 m` a không , , , ,, , , ´ ´ ´ ` ˘. t x = u1 − u2 , y = anh huong d ¯ên kêt qua chung minh. Bây gio ta d ¯a √ √ , ` ´ suy ra 1 ≤ x ≤ [ n], 1 ≤ |y| ≤ [ n] v` v 1 − v 2 . Tu d a ax ≡ y ¯o (mod n). , , , . 6.6. Áp dung d¯ê` Tue: Moi sô´ nguyên tô´ dang 4k + 1 c´ o thê biêu . . . bô , , ,, ˜ du,o´,i dang diên . tông b`ınh phuong cua hai sô´ nguyên.

J

, , `,i giai. Ch´ ´,ng minh kha ˘ng d Lo ung ta chu a sô´ ¯inh . sau: Nê´u p = 4k + 1 l` , , 2 nguyên tô´, th`ı phuong tr`ınh x ≡ −1 (mod p) c´ o nghiêm. . Thât . vây, . V`ı p l` a sô´ nguyên tô´, theo d l´ y Wilson, ( p − 1 ) ! ≡ − 1 ( mod p ). ¯inh . p +1 Ngo` ai ra p − 1 ≡ −1 (mod p), p − 2 ≡ −2 (mod p), . . . , p − 2 ≡ p +1 2 (mod p ). Suy ra,

( p − 1) ( p + 1) 2 2 p−1 2 1 1 ) ) ≡ (−1 )(−2 ) . . . (−( 2 p −1 p−1 2 p−1 2 = (−1) 2 (1.2 . . . ) = (−1)2k (( )!) 2 2 p−1 2 = (( )!) (mod p). 2 , p−1 , ,, a nghiêm Nhu vây . cua phuong tr`ınh trên. V`ı a2 + 1 . a = ( 2 )! l` ´,i p. Bây gi`o, ta a ´p chia hê´t cho p nên a l` a nguyên tô´ c` ung nhau vo , dung bô d a p. Tô`n tai ac sô´ nguyên x v` a y m` a ¯ê` Tue cho hai sô´ a v` . . c´ −1 ≡ ( p − 1)! = 1.2 . . . .

60

,, Chuong 6. B` ai tâp . sô´ hoc . nâng cao

√ √ ´ a2 x 2 ≡ y2 1 ≤ x ≤ [ p], 1 ≤ |y| ≤ [ p] v` a ax ≡ y (mod p). Khi d¯o , , 2 2 2 ´ (mod p), nhu vây ¯o . x ≡ −y (mod p), v`ı a ≡ −1 (mod p). T`u d 2 2 2 2 ´ ˘. t kh´ suy ra x + y chia hêt cho p. Ma ac 1 ≤ x ≤ p, 1 ≤ y ≤ p. Ta , , 2 2 ` ´ ˘ng x = p v` thây ra a y = p không thê xây ra v`ı p l` a sô´ nguyên tô´, suy ra 0 < x2 + y2 < 2p v` a v`ı x2 + y2 chia hê´t cho p, nên x2 + y2 = p.

J

,, , ,, `˘ng moi ´,ng minh ra ´: Tuong tu. , chu Ch´ uy . sô,´ p,, sao cho phuong ,´, tr`ınh x2 ≡ −2 (mod p) c´ o nghiêm, d o thê biêu di˜ên duo i dang ¯ê`u c´ . . , 2 2 ´i x, y l` p = x + 2y vo a c´ ac sô´ nguyên.

. 6.7. (Ðê` thi To´an Olympic quô´c tê´ lâ`n 18 n˘am 1976) Cho hê. p , ,, , phuong tr`ınh v´ oi q = 2p ân a11 x1 + a12 x2 + · · · + a1q xq = 0 ..................... a p1 x1 + a p2 x2 + · · · + a pq xq = 0 , , , Tâ´t ca c´ ac hê. sô´ aij thuôc ˘`ng tô`n tai . ,tâp . hop {−1, 0, ,1}. Chu´ng minh ra . nghiêm a n´ o thoa m˜ an . ( x1 , x2 , . . . , xq ) cua hê, . m` , , a) tâ´t ca x j ( j = 1, 2, . . . , q) l` a nhung sô´ nguyên; b) ´ıt nhâ´t c´ o môt a x j ≤ 0; . j( j = 1, 2, . . . , q) m` , c) v´ oi moi o x j ≤ q; . j( j = 1, 2, . . . , q) ta luôn c´ , `,i giai. X´et bô. ( x1 , x2 , . . . , xq ) gô`m nh˜u,ng sô´ nguyên bâ´t k`y, m` Lo a , , ,, ch´ ung thoa m˜ an | x1 | ≤ p, | x2 | ≤ p, . . . , | xq | ≤ p. Boi v`ı tâ´t ca c´ ac , , , , , , ´i viêc ˘. c 1, vo hê. sô´ cua hê. phuong tr`ınh chı l` a -1,0 hoa thay c´ ac ân . , ,, ˜ phu,o,ng tr`ınh na `˘m x1 , x2 , . . . , xq ch´ ung ta nhân d a tri. cua môi ¯uo. c gi´ . , , ˜ i = 1, 2, . . . , p ch´ ´i môi trong khoang [− pq, pq]. Thât ung ta . vây, . vo c´ o | ai1 x1 + ai2 x2 + · · · + aiq xq | ≤ | x1 | + | x2 | + · · · + | xq | ≤ pq. Suy , , , , ,, , , ´, ra, nê´u thay nh˜ung ân trong tâ´t ca phuong tr`ınh cua hê. tuong u ng

6.2. V´ı du.

61

,, ´,i x1 , x2 , . . . , xq s˜e nhân vo d ¯uo. c bô. p sô´ nguyên (y1 , y2 , . . . , y p ). Tâ´t . , , , , `˘m trong khoang [− pq, pq]. Trong khoang ca nh˜ung sô´ n` ay d ¯ê`u na , , ´˘p xê´p ´ ng 2pq + 1 sô´ nguyên. Suy ra gi˜ua nh˜ung bô. sa n` ay c´ o d ¯u ,, ,, , ˘. t kh´ p phâ`n tu nhu o trên c´ o (2pq + 1) p bô. xê´p kh´ ac nhau. Ma ac ,, , , , ´ ´ ` ˜ sô luo. ng nhung bô. xêp q-phân tu ( x1 , x2 , . . . , xq ) m` a |xj | ≤ p , , q ˜ ` ´ ` ´ ˘ng tu q = 2p suy ra voi j = 1, 2, . . . q, l` a (2p + 1) . Dê thây ra q p (2p + 1) > (2pq + 1) . Theo nguyên l´y Ðirichlê tô`n tai . hai bô. q0 0 0 00 00 00 ´ sô nguyên ( x1 , x2 , . . . , xq ) v` a ( x1 , x2 , . . . , xq ) kh´ ac nhau m` a ch´ ung , , 0 00 ´ ´ thoa m˜ an | x j | ≤ p, | x j | ≤ p voi j = 1, 2, . . . q v` a sau khi thê v` ao ,, ´ ˘. t hê. phuong tr`ınh cho c` ung môt . bô. p sô nguyên (y1 , y2 , . . . , y p ). Ða 0 00 0 00 0 00 x1 = x1 − x1 , x2 = x2 − x2 , . . . , xq = xq − xq . R˜ o r` ang ( x1 , x2 , . . . , xq ) , ,, ´,i moi l` a nghiêm a x j l` a c´ ac sô´ nguyên vo . cua hê. phuong tr`ınh v` . 0 0 0 00 00 00 j = 1, 2, . . . , q. V`ı ( x1 , x2 , . . . , xq ) v` a ( x1 , x2 , . . . , xq ) l` a hai bô. q sô´ ho` an to` an kh´ ac nhau, th`ı ´ıt nhâ´t môt ac sô´ x j kh´ ac không. . trong c´ , ´ ´ Cuôi c` ung voi moi ung ta c´ o | x j | = | x 0j − x 00j | ≤ . j = 1, 2, . . . , q ch´ , , | x 0j | + | x 00j | ≤ p + p = 2p = q. Nhu vây a nghiêm . ( x1 , x2 , . . . , xq ) l` . cua ,, o t´ınh châ´t mong muô´n. hê. phuong tr`ınh c´

J

. 6.8. (Ðê` thi To´an Olympic quô´c tê´ lâ`n 28 n˘am 1987) Cho , ˜,ng sô´ thu.,c v` x1 , x2 , . . . , xn l` a nhu a x12 + x22 + · · · + xn2 = 1. Chu´ng minh , ˜ ´ ˜,ng sô´ nguyên a1 , a2 , . . . , an ra ˘`ng v´ oi môi sô nguyên k, k ≥ 2, tô`n tai . nhu , ` không d¯ô`ng th`oi ba ˘ng không sao cho | ai | ≤ k − 1, i = 1, 2, . . . , n √v` a , , , ( k − 1 ) n thoa m˜ an bâ´t phuong tr`ınh sau | a1 x1 + a2 x2 + · · · + an xn | ≤ kn −1 , , ,, `,i giai. Ch´ ´,c Cosi-Buniakovski-Svars ˘ng thu Lo ung ta su dung bâ´t d ¯a . q q |α1 β 1 + |α2 β 2 + · · · + |αn β n | ≤ α21 + · · · + α2n β21 + · · · + β2n , , `˘ng xây ra ´,i moi ,d sô´ thu. c α1 , α2 , . . . αn , β 1 , β 2 , . . . , β n . Dâ´u ba ¯us ng vo . , khi v` a chı khi tô`n tai . môt . sô´ λ sao cho α1 = λβ 1 , α2 = λβ 2 . . . , αn = λβ n .

,, Chuong 6. B` ai tâp . sô´ hoc . nâng cao

62

, Bây gi`o ch´ ung ta x´et c´ ac sô´ y0 = − k−2 1 , y1 = − k−2 1 + 1, . . . , , ,, , ung l` a k v` a hiêu yk−1 = − k−2 1 + (k − 1) = k−2 1 Sô´ luo. ng cua ch´ . , t`ung , ˘. p trong ch´ ca ung l` a nh˜ung sô´ nguyên, m` a gi´ a tri. tuyêt o ¯ô´i cua n´ . d ˜ ´ ´ ` ˘p xêp n-th` không qu´ a k − 1. Môi bô. sa anh phân β = (b1 , b2 , . . . , bn ), ,, ´ ´,i moi ´ trong y1 , y2 , . . . , yn vo od a môt ao d ¯ây bi l` ¯o . sô n` . i = 1, 2, . . . , n, , , ´, , ´ ´ ˘. t tuong ung voi môt ch´ ung ta d ¯a . sô Sβ = b1 x1 + b2 x2 + · · · + bn xn . , ´ , , ` ´ ˘ng thuc Cosi Tu bât d ¯a Sβ = b1 x1 + b2 x2 + · · · + bn xn q q 2 2 2 ≤ b1 + b2 + · · · + bn x12 + x22 + · · · + xn2 q = b12 + b22 + · · · + bn2 , Nhung |bi | ≤

´,i moi vo . i = 1, 2, . . . , n sao cho q k − 1√ |Sβ | ≤ b12 + b22 + · · · + bn2 ≤ n 2 √ √ ,, ˘. c l` a − k−2 1 n ≤ Sβ ≤ k−2 1 n. Theo phuong ph´ hoa ap n` ay n-bô. β = ,, ´ ´,i moi (b1 , b2 , . . . , bn ), o d¯ây bi l`a môt ac sô y0 , y1 , . . . , yk−1 vo . trong c´ . ,, , , ´, , ´ ´ ˘ i = 1, 2, . . . , n, d ¯uo. c d ¯a. t tuong ung voi môt . sô Sβ = b1 x1 + b2 x2 + √ k −1 √ ,, ,, k −1 · · · + bn xn trong d¯oan ô. d ¯uo. c . ∆ = [− 2 n, 2 , n]. Sô´ luo. ng n-b√ , ´˘p xê´p l` ´,i d sa a kn . Chia ∆ ra kn − 1 d ai kkn−−11 n. T`u ¯oan ¯ô. d` . nho vo nguyên l´y Ðirichlê suy ra tô`n tai a . hai n-bô. β = (b1 , b2 , . . . , bn ) v` , γ = (c1 , c2 , . . . , cn ), m` a nh˜ung sô´ Sβ = b1 x1 + b2 x2 + · · · + bn xn , , ´, `˘m trong ´,i ch´ v` a Sγ = c1 x1 + c2 x2 + · · · + cn xn tuong u ng vo ung na , ˘. t a1 = b1 − c1 ,a2 = b2 − c2 , . . . , an = bn − c` ung môt ¯oan . ,d . nho. Ða , , , ˜ cho. Thât cn . D˜ê kiêm tra d an d ¯uo. c a1 , a2 .., an thoa m˜ ¯iê`u kiên ¯a . ,d . , ´ ´ ´ vây, v o i m oi i = 1, 2, . . . , n sô a = b − c l` a hi êu c ua hai sô n` a o i i i . . . ,, , ´ trong y0 , y1 , . . . , yk−1 nhu d ˜ n´ d oi o trên v` a l` a sô´ nguyên không ¯o ¯a ,, vuo. t qu´ a k − 1. V`ı hai n-bô. trên kh´ ac nhau ho` an to` an th`ı ´ıt nhâ´t môt ac sô´ ai = bi − ci kh´ ac không. Ngo` ai ra, |b1 x1 + b2 x2 + . trong c´ k −1 2

6.2. V´ı du.

63

· · · + bn xn | = | x1 (b1 − c1 ) + x2 (b2 − c2 ) + · · · + xn (bn − cn )| = |Sβ − √ Sγ | ≤ kkn−−11 n.

J

, ˜ bô. gô`m 11 sô´ thu.,c kh´ . 6.9. Chu´ng minh ra˘`ng môi ac nhau trong , , , ,, khoang [1,1000] c´ o thê chon d¯uo. c hai sô´ x v` a y, m` a ch´ ung thoa m˜ an . , , bâ´t d¯a ˘ng thu´c sau √ 0 < x − y < 3 3 xy (6.2) , , `,i giai. Ch´ ˜ cho Lo ung ta x´et c˘ an bâc ac sô´ trong bô. sô´ d ¯a . ba cua c´ √ , 3 ` ˜ cho suy ra 1 ≤ xi ≤ 10, i = x1 , x2 , . . . , x11 T`u d ¯iêu kiên ¯a . d , ,`, `˘ng nhau. ` 1, 2, . . . , 11. Ch´ ung ta chia khoang [1,10] ra muoi phân ba √ √ √ `˘m trong ´, c´ Khi d o ´ıt nhâ´t môt trong hai sô´ 3 x1 , 3 x2 , . . . , 3 x11 na ¯o , . √ √ ´ l` c` ung môt ac sô´ d a 3 xi v` a 3 x j , i 6= j v` a xi > x j , ¯oan ¯o . d . nho. Nê´u c´ ch´ ung ta c´ o √ p 9 < 1. (6.3) 0 < 3 xi − 3 x j ≤ 10 √ √ , , , 3 3 ´i (6.3) ta c´ o 0 < xi − Nhu vây, . 0 < ( xi − 3 x j ) < 1, kê´t ho. p vo √ √ √ √ 3 x j < 1 + 3 3 xi x j ( xi − 3 x j ) < 1 + 3 3 xi x j . , ´,i bô. 10 sô´ thu.,c trong khoang ´ ng vo ´: B` Ch´ uy ai to´ an không c` on d ¯u , 3 3 3 [1,1000]. Phan v´ı du: a i > j ch´ ung . bô. sô´ 1 , 2 , . . . , 10 . nê´u i 6= j v` , 3 2 2 ta c´ o (i − j) ≥ 1, nhu vây a i−j = . i +, ij + j ≥ ,1 + 3ij. Ngh˜ıa l` , 2 2 ´ c xây ra khi i = j + 1. (i − j)(i + ij + j ) ≥ 1 + 3ij. d¯a˘ng thu , ˜,a ch´ . 6.10. Cho 7 sô´ thu.,c bâ´t k`y. Chu´,ng minh ra˘`ng giu ung c´ o thê , ,, chon ˘ng han a y, sao cho . d¯uo. c hai sô´, cha . x v` √ 3 x−y ≤ . 0≤ 1 + xy 3

J

, , `,i giai. C´ ˜ cho k´y hiêu Lo ac sô´ d l` a x1 , x2 , . . . , x7 . Muc d ung ¯´ıch cua ch´ ¯a . . , ,, ,´, ´ ta l` a biêu di˜ên moi sô du o i d ang x = tg α , o d ây α l` a m ôt ¯ i i i . . . sô´ , π π trong khoang (− , ), i = 1, 2, . . . , 7. Ch´ ung ta chia d ay ra ¯oan . n` 2 2

64

,, Chuong 6. B` ai tâp . sô´ hoc . nâng cao

`˘ng nhau, ngh˜ıa l` `˘ng π . D˜ê d` s´ au d od ai ba a ba ang thâ´y ¯oan ¯ô. d` . con c´ 6 `˘ng ´ıt nhâ´t c´ `˘m trong môt ra o hai sô´ trong α1 , α2 , . . . , α7 c` ung na . d¯oan . , ´ ` ´. Nê´u ch´ ´ ´ coni d¯o ung ta k´y hiêu c´ a c sô d o l` a α v` a α , α ≥ α , th` ı t u d o ¯ ¯ i j i j . , π π π am sô´ tg l` a t˘ ang trong khoang (− 2 , 2 ), suy ra 0 ≤ αi − α j ≤ 6 . V`ı h` suy ra √ tg αi − tg α j xi − x j π 3 0 ≤ tg(αi − α j ) = = ≤ tg = . 1 + tg αi tg α j 1 + xi x j 6 3

J

, , , `˘ng c´ ´: Ba Ch´ uy ach giai cua hai b` ai tâp ay ch´ ung ta c´ o thê s´ ang . n` , ,, , , , , ´i c´ tao ai to´ an tuong tu. , m` a vo ach giai b`ınh thu`ong kh´ o . ra,môt . loat . b` ,, ´ m` a giai quyêt d ¯uo. c.

` tâp 6.3. Bai . . 6.11. (Ðê` thi To´an Olympic quô´c tê´ lâ`n 26 n˘am 1985). Cho tâp . , ,´, ´ ´, , , ho. p M gô`m 1985 sô´ tu. nhiên, không c´ o sô´ n` ao c´ o uo c sô lon hon , , , ,, `˘ng t`u, nh˜u,ng phâ`n tu, cua M c´ ´,ng minh ra 26. Chu o thê chon ¯uo. c 4 . d , , , , sô´ t`ung d ac nhau m` a t´ıch cua ch´ ung l` a l˜ uy th`ua bâc ¯ôi môt . kh´ . 4 cua , 9 ´,i tâp ´ ng vo môt ¯u . sô´ nguyên. (kê´,t luân . d . ho. p gô`m 3.2 + 1 = 1537 sô´ , ,´, ´ m` a nh˜ung uo c sô cua ch´ ung không qu´ a 23). , `˘ng c´ ´,ng minh ra . 6.12. Cho bô´n sô´ duo,ng bâ´t k`y. Chu o hai sô´ trong , , , , ˘ng han ´, cha bô´n sô´ d a y, thoa m˜ an bâ´t phuong tr`ınh sau ¯o . x v` √ x−y 0≤ ≤ 2 − 3. 1 + x + y + 2xy , , , `˘ng môt ´,ng minh ra . 6.13. Chu sô´ tu. nhiên c´ o thê biêu di˜ên th` anh . , , , ,, tông b`ınh phuong cua hai sô´ nguyên khi v` a chı khi, trong phân t´ıch , , ´˘c cua n´ ra dang o, c´ ac th`ua sô´ nguyên tô´ dang o ¯ê`u c´ . ch´ınh ta . 4k + 3 d ˜˘n. sô´ m˜ u cha

6.3. B` ai tâp .

65

, , `˘ng t`u, k + 1 sô´, m` ´,ng minh ra a ch´ ung nho hon 2k, luôn . 6.14. Chu , , , ,, , luôn c´ o thê chon d a ty sô´ cua ch´ ung l` a môt l˜ uy th`ua ¯uo. c hai sô´ m` . . , cua 2. , `˘ng t`u, (n − 1)2 + 1 sô´ ´,ng minh ra . 6.15. Cho n l`a môt ., sô´ le. Chu , , ,, nguyên bâ´t k`y c´ o thê chon ung chia . d¯uo. c n sô´ sao cho tông cua ch´ ´ hêt cho n.

66

,, Chuong 6. B` ai tâp . sô´ hoc . nâng cao

, , CHUONG

7

˜ ´ ` TÂP BAI . DÂY SÔ NÂNG CAO

7.1. V´ı du. , . 7.1. Cho d˜ay Fibonaxi u1 = u2 = 1, un = un−1 + un−2 . V´ oi moi . sô´ , ,, , , , ˜ ˜ a nhu ˜ ng sô´ hang ` d¯â`u dây d¯ê´n sô´ hang nguyên duong m, th`ı giu . thu´ . tu m2 − 1 tô`n tai . môt . sô´ hang . chia hê´t cho m. , , , `,i giai. Ch´ Lo ung ta k´y hiêu k l` a phâ`n du cua sô´ k cho m. Ch´ ung ta . , , ˘. p phâ`n du cua d˜ x´et c´ ac ca ay Fibonaxi khi chia cho m:

( u 1 , u 2 ). ( u 2 , u 3 ), ( u 3 , u 4 ), . . . , ( u n , u n +1 ), . . .

(7.1)

`˘ng nhau khi ˘. p ( a1 , b1 ) v` Nê´u ch´ ung ta qui d a ( a2 , b2 ) l` a ba ¯inh . hai ca , , , , ˘. p phâ`n du a1 = b1 v` a a2 = b2 , th`ı sô´ tâ´t ca kha n˘ ang cua c´ ac ca khi chia cho m l` a m2 . V`ı thê´ ch´ ung ta lâ´y m2 + 1 sô´ hang d ¯â`u tiên . , , ˘. p tr` cua d˜ ay (7.1) th`ı trong ch´ ung phai c´ o hai ca ung nhau,( theo , ,, , ˘. p sau la ˘. p lai. ˘. p (uk , uk+1 ) ´ c´ nguyên l´y Ðirichlê ) v` a t`u d ac ca ¯o . Gia su ca , `˘ng ´ ng minh ra ˘. p d ˘. p lai l` a ca ung ta s˜e chu ¯â`u tiên la . trong (7.1). Ch´ , ,, , , `˘ng ca ˘. p n` ˘. p (1, 1). Thât ˘. p ca ay ba a ca . vây, . gia su nguo. c lai, . ngh˜ıa l` , ´i k > 1. Khi d ˘. p lai ´ ch´ d a ( u k , u k + 1 ) , vo ung ta s˜e t`ım ¯o ¯â`u tiên la . l` ,, `˘ng (uk , uk+1 ) ˘. p (ul , ul +1 ), (l > k ), m` d a n´ o ba ¯uo. c trong (7.1) môt . ca , . Nhu vây a uk−1 = uk+1 − uk , do ul +1 = uk+1 v` a . ul −1 = ul +1 − ,ul v` , `˘ng ul = uk , nên phâ`n du cua ul −1 v` a uk−1 khi chia cho m c˜ ung ba , ´ suy ra (uk−1 , uk ) = (ul −1 , ul ), vây nhau, ngh˜ıa l` a ul −1 = uk−1 . T`u d ¯o .

,, ˜ sô´ nâng cao Chuong 7. B` ai tâp . dây ,´, , `˘m trong d˜ ˘. p (uk−1 , uk )na ´ n´ ca ay (7.1) truo c ca (uk , uk+1 ). Ðiê`u d oi ¯o , , , ` ` ´i gia thiê´t ˘ng (uk , uk+1 ) không phai ca ˘p la ˘p lai d lên ra ai vo ¯âu tiên, tr´ ,. , . . ˘. t ra. Ngh˜ıa l` d a k > 1 không thê xay ra, vây ¯a . k = 1. , ,, , ˘. p (1, 1) l` ˘. p d ˘. p lai Nhu vây a ca o ¯â`u tiên la . ca . trong (7.1). Gia su n´ 2 ˘. p lai ˘. c l` la a (ut , ut+1 ) = (1, 1). Ngh˜ıa l` a ut . t lâ`n (1 < t < m + 1). hoa , , v` a ut+1 khi chia cho m cho phâ`n du c` ung l` a 1, vây ung . hiêu . cua ch´ , , , ´ t−1 chia hê´t cho m. Nhung ut+1 − ut = ut−1 . Nhu vây . sô´ hang . thu , cua d˜ ay Fibonaxi s˜e chia hê´t cho m. 68

J

, ,, , , , , , . 7.2. Gia su a v`a x l`a c´ac sô´ tu. nhiên thu. c su. l´ on hon 1 v` a ( x, a − 1) = , , , 1. D˜ ay sô´ vô han ac d¯inh ¯uo. c x´ . {un } d . nhu sau un = ax n − a + 1, n = 1, 2, . . . , , Chu´ng minh ra ˘`ng trong d˜ ay sô´ n´ oi trên chu´a vô han . sô´ d¯ôi môt . nguyên ´ tô c` ung nhau. , , , , , `,i giai. Gia thiê´t phan chu ´,ng trong d˜ Lo ay sô´ chı c´ o h˜uu han . sô´ u i1 , u i2 , . . . , u i k d ung nhau. ¯ôi môt . nguyên tô´ c` ˘. t q = ui1 ui2 . . . uik . X´et (q + 1) sô´ sau a, ax, ax2 , . . . , ax q . Theo Ða nguyên l´y Ðirichlê tô`n tai . hai sô´ nguyên r, s sao cho 0 ≤ r < s ≤ q v` a axr ≡ ax s

(mod q) =⇒ axr − ax s ≡ 0 (mod q) hay

axr (1 − x s−r ) ≡ 0 (mod q) , Theo gia thiê´t ta c´ o ( x, a − 1) = 1, nên suy ra

(7.2)

( axr , ui j ) = 1, ∀ j = 1, 2, . . . , k.

(7.3)

( axr , q) = 1.

(7.4)

, T`u (7.3) suy ra

7.1. V´ı du.

69

, ´,i l ∈ N. a (7.4) c´ o x s−r ≡ 1 (mod q) =⇒ x s−r = lq + 1 vo T`u (7.2) v` X´et sô´ uik+ j = ax s−r − a + 1. Vây . uik+ j = a(lq + 1) − a + 1 = qal + 1.

(7.5)

, T`u (7.5) ta c´ o

(uik+ j , ui j ) = 1, ∀ j = 1, 2, . . . , k. (7.6) , , , `˘ng luôn c´ ´,c (7.6) chu ´,ng to ra Hê. thu o thê bô sung thêm v` ao bô. sô´ , , ˜ thoa m˜ ´i, m` q = ui1 ui2 . . . uik c´ ac sô´ mo a bô. sô´ n` ay vân an d ¯iê`u kiên: . ´ ´ ´ ` bât k`y hai sô n` ao c˜ ung nguyên tô c` ung nhau. Ðiêu n` ay c´ o ngh˜ıa l` a ´ ´ ˜ cho c´ trong d˜ ay {un } d o vô han ung nhau. ¯a ¯ôi môt . sô d . nguyên tô c`

J

, . 7.3. Cho {un } l`a d˜ay c´ac sô´ tu. nhiên t˘ang dâ`n: u1 < u2 < u3 < . . . , , , v` a thoa m˜ an d¯iê`u kiên oi moi a sô´ tu. nhiên. . u1 = 1, un+1 ≤ 2n, v´ . n l` , , , Chu´ng minh ra ˘`ng v´ oi moi sô´ tu. nhiên n tô`n tai ac sô´ hang a uq . . c´ . u p v` , cua d˜ ay sao cho u p − uq = n. , , , , ,´, , , `,i giai. Gia su, n ∈ N l` Lo a sô´ tu. nhiên cho truo c. T`u gia thiê´t suy ra ,, , ˜ sô´ hang môi u1 , u2 , . . . , un+1 không vuo. t qu´ a 2n. X´et tâp . . ho. p 2n sô´ , , ˘. p tu. nhiên sau {1, 2, . . . , 2n}. Ch´ ung ta chia tâp ay ra l` am n ca . ho. p n` , , ´ a không ´ıt ho,n (1, n + 1), (2, n + 2), . . . , (n, 2n). Do tâp ho. p trên chu . ,, , (n + 1) phâ`n tu cua d˜ay {un } d¯a˜ cho (v`ı n´oi riêng u1 , u2 , . . . , un+1 d¯a˜ , thuôc . tâp . ho. p â´y), vây . , theo nguyên l´y Ðirichlê tô`n t,ai . hai sô´ hang . ,, ˘. p ( gia su u p > uq ). kh´ ac nhau u p v` a uq cua d˜ ay thuôc ao môt . v` . ca , , ˜ ` ´ ` ˘. p d ˘ng n, nên ch´ Nhung hiêu ung ta c´ o u p − uq = n. ¯êu ba . sô cua môi ca

J

, . 7.4. Cho {uk }, k = 1, 2, . . . , n l`a d˜ay sô´ tu. nhiên sao cho 1 ≤ u1 ≤ u2 ≤ . . . ≤ un v` a u1 + u2 + · · · + un = 2n

,, ˜ sô´ nâng cao Chuong 7. B` ai tâp . dây

70

, `, d˜ Chu´ng minh ra ˘`ng nê´u n cha ˘˜n v` a un 6= n + 1, th`ı tu ay trên luôn chon . , , ,, ` ´ ´ ba ra d¯uo. c môt ay con m` a tông c´ ac sô hang ay con d¯o ˘ng n. . d˜ . cua d˜ , `,i giai. Ða ˘. t Sk = u1 + u2 + · · · + uk , k = 1, 2, 3, . . . , n. X´et n + 2 sô´ Lo {0, u1 − un , S1 , S2 , . . . , Sn } Theo nguyên l´y Ðirichlê th`ı ´ıt nhâ´t hai sô´ , , , khi chia cho n c´ o c` ung phâ`n du. Vây, o 4 kha n˘ ang sau d ¯ây: . chı c´ 1) (u1 − un ) chia hê´t cho n. Do u1 + u2 + · · · + un ≥ nu1 =⇒ 2n ≥ nu1 =⇒ u1 ≤ 2 , a) Nê´u u1 = 2 th`ı t`u 1 ≤ u1 ≤ u2 ≤ . . . ≤ un v` a u1 + u2 + · · · + ˜ ˘n nên n = 2m vây un = 2n suy ra u1 = u2 = . . . = un = 2. Do n cha . u1 + u2 + · · · + um = 2m = n , ˘. c b) Nê´u u1 < 2 th`ı t`u u1 − un chia hê´t cho n, suy ra un = 1 hoa l` a un = 1 + n ( do u1 nguyên nên u1 = 1 v` a 1 ≤ un ≤ 2n suy ra ,, ´ , ˘. t kh´ ac d ¯uo. c kêt luân . trên). Nhung un 6= n + 1 suy ra un = 1. Ma 1 ≤ u1 ≤ u2 ≤ . . . ≤ un vây . th`ı u2 = u3 = . . . = un−1 = 1. Suy ra u1 + u2 + · · · + un = n, vô l´y. , , ,, , Nhu vây ay, ta chı ra tô`n tai ay con u1 , u2 . trong tru`ong ho. p n` . d˜ n , ´i m = . , . . . , um sao cho u1 + u2 + · · · + um = n vo 2 2) S j − Si , ( j > i ) chia hê´t cho n. , , , ˘ng Ta c´ o S j − Si = ui+1 + ui+2 + · · · + u j . R˜ o r` ang vê´ phai cua d ¯a ´,c trên c´ thu o ´ıt nhâ´t môt m` a uk ≥ 1, ∀k = 1, 2, . . . , n, suy ra . sô´ hang . , ,, , ˘. t kh´ S j − Si ≥ 1. Ma ac c˜ ung hiêu ac phâ`n tu cua ¯u c´ . trên nê´u không d , d˜ ay th`ı bao gi`o ta c˜ ung c´ o S j − Si < u1 + u2 + · · · + un ≤ 2n − 1. ´ ´ cuôi c` Do d ung ta c´ o 1 ≤ S j − Si < u1 + u2 + · · · + un ≤ 2n − 1 ¯o , , ´ ˘. c m` a S j − Si chia hêt cho n. d ay chı xây ra khi S j − Si = n hoa ¯iê`u n` l` a ui+1 + ui+2 + · · · + u j = n. 3) Si chia hê´t cho n.

7.1. V´ı du.

71

Ta c´ o 1 ≤ Si ≤ Sn−1 = 2n − un < 2n m` a Si chia hê´t cho n, suy ˘. c l` ra Si = n hoa a u1 + u2 + · · · + ui chia hê´t cho n. , ´,i k n` 4) Sk v` a u1 − un cho c` ung phâ`n du khi chia cho n, vo ao ´, 1 ≤ k ≤ n − 1. Suy ra Sk − (u1 − un )|n =⇒ (u2 + u3 + · · · + d ¯o uk + un )|n. M` a u2 + u3 + · · · + uk + un ≤ 2n − u1 < 2n. Suy ra u2 + u3 + · · · + uk + un = n. , , ,, `˘ng ay con m` a tông cua ch´ ung ba T´ om lai ¯uo. c d˜ . luôn luôn chon . d n.

J

, , . 7.5. Cho d˜ay sô´ nguyên u1 , u2 , . . . , un v´ oi n ≥ 2. Chu´ng minh ra ˘`ng ´ 1 ≤ k1 < k2 < . . . < k m ≤ n tô`n tai ay con uk1 , uk2 , . . . , ukm trong d¯o . d˜ 2 2 2 sao cho uk1 + uk2 + · · · + ukm chia hê´t cho n. , `,i giai. X´et (n + 1) sô´ Lo 0, u21 , u21 + u22 , u21 + u22 + u23 , . . . , u21 + u22 + · · · + u2n , Chia c´ ac sô´ n` ay cho n, th`ı ch´ ung cho nhiê`u nhâ´t n sô´ du. Theo , , , , ´ l` nguyên l´y Ðirichlê tô`n tai ung sô´ du, gia su d a u21 + ¯o . hai sô´ cho c` u22 + · · · + u2j v` a u21 + u22 + · · · + u2k (0 ≤ j < k ≤ n). C´ o ngh˜ıa 2 2 2 2 2 2 l` a sô´ (u1 + u2 + · · · + u j ) − (u1 + u2 + · · · + uk ) chia hê´t cho n. , ˘. c l` a u j+1 + u j+2 + · · · + uk chia hê´t cho n. D˜ ay con phai t`ım l` a Hoa u j +1 , u j +2 , . . . , u k .

J

. 7.6. Cho d˜ay Fibonaxi 1, 2, 3, 5, 8,. . . .Ða˘. t f (n) = 19852 + , , 1956n2 + 1960 Chu´ng minh ra ˘`ng tô`n tai uk cua d˜ ay . vô han . sô´ hang . Fibonaxi, sao cho f (uk )|1989 , `,i giai. Ða ˘. t h(n) = 4n2 + 33n + 29 =⇒ h(n) = 1989(n2 + n + 1) − Lo , ´ suy ra f (n)|1989 ⇐⇒ h(n)|1989. f (n). T`u d ¯o

72

,, ˜ sô´ nâng cao Chuong 7. B` ai tâp . dây

´ X´et d˜ ay sô´ {vn } sau d ¯ây, trong d ¯o  v0 = −1, v1 = 1 vn+1 = vn + vn+1 , n = 1, 2, . . . ,, N´ oi c´ ach kh´ ac d˜ ay {vn } l` a d˜ ay sinh ra boi d˜ ay Fibonaxi {un } : ,´, ` ˘ng c´ 1, 1, 2, 3, 5, . . . ba ach thêm v` ao truoc d˜ ay n` ay ba sô´ hang . -1, 1, 0. , Goi a phâ`n du trong ph´ep chia vi cho 1989(i = 0, 1, 2, . . .). . ri l` , ˘. p sô´ sau d Nhu vây o 0 ≤ r ≤ 1988. X´et d˜ ay c´ ac ca ¯ây . ta c´

(r0 , r1 ), (r1 , r2 ), (r2 , r3 ), . . . . , ˜ ca ˘. p ri chı nhân ˘. p kh´ V`ı môi a tri, ac ca ac . vây . môt . trong 1989 gi´ . sô´ c´ , 2 2 ´ theo nguyên l´y Ðirichlê th`ı trong 1989 + nhau tô´i d a 1989 . T`u d ¯a l` ¯o , ,, ˘. p d¯â`u tiên ´ıt nhâ´t c´ ˘. p tr` ˘. p â´y l` 1 ca o hai ca ung nhau. Gia su hai ca a (r p , r p+1 ), (r p+q , r p+q+1 ), p, q ∈ N Ðiê`u â´y c´ o ngh˜ıa l` a r p = r p+1 , r p+q = r p+q+1 . Theo c´ ach x´ ac d ¯inh . d˜ ay, ta c´ o v p−1 = v p+1 − v p =⇒ r p−1 = r p+1 − r p ,, , Tuongtu. , ta c´ o v p+q−1 = v p+q+1 − v p+q =⇒ r p+q−1 = r p+q+1 − r p+q , ,, , ´ suy ra r p−1 = r p+q−1 . Tuongtu. , ta c´ T`u d o ¯o r p −2 = r p + q −2 ....... r 2 = r q +2 r 1 = r q +1 r0 = r q , T`u r0 = rq , r1 = rq+1 v` a vn+1 = vn + vn+1 suy ra ri = ri+q , ∀i = 0, 1, 2, . . .. Do vây . r0 = rq = r2q = r3q = . . . = rkq , ∀k ≥ 1, suy

7.1. V´ı du.

73

, ra h(vkq ) = 1989.A + h(−1) = 1989.A. Nhung ∀k = 1, 2, 3 . . . vkq , d a sô´ Fibonaxi suy ra c´ o vô sô´ sô´ hang vkq cua d˜ ay Fibonaxi m` a ¯ê`u l` . f (vkq )|1989.

J

, . 7.7. Cho d˜ay sô´ {un } x´ac d¯inh . nhu sau u1 = 20, u2 = 100, v` a un+1 = 4un + 5un−1 − 1976, n = 1, 2, . . . , , Chu´ng minh ra ˘`ng tô`n tai ay sô´ chia hê´t cho 1996. . ´ıt nhâ´t môt . sô´ cua d˜ , `,i giai. Ða ˘. t un = 1996pn + qn , ∀n = 1, 2, . . . trong d ´ pn , qn l` Lo a c´ ac ¯o sô´ nguyên v` a 0 ≤ qn ≤ 1995. X´et d˜ ay sô´

( q 1 , q 2 ), ( q 2 , q 3 ), . . . , ( q n , q n +1 ), . . . , ´ V`ı d˜ ay n` ay vô han a sô´ c´ ac sô´ qi l` a h˜uu han . , m` . nên tô`n tai . hai sô , , , , tu. nhiên l, m ( gia su m > l) sao cho (ql , ql +1 ) = (qm , qm+1 ) hiêu `˘ng ´,ng minh ra theo ngh˜ıa ql = qm v` a ql +1 = qm+1 . Ch´ ung ta s˜e chu ql −1 = qm−1 . Thât . vây, . 5(um−1 − ul −1 ) = (um+1 − 4um + 1976) − (ul +1 − 4ul + 1976)

= ( u m +1 − u l +1 ) − 4 ( u m − u l ) , Do uk = 1996pk + qk , nên t`u (7.7) c´ o

(7.7)

5(um−1 − ul −1 ) = 1996( pm+1 − pl +1 ) − (qm+1 − ql +1 )−

− 4[1996( pm − pl ) − (qm − ql )] , `˘ng nhau v` Thay nh˜ung gi´ a tri. ba ao ((7.8) ta d ¯i d ¯ê´n 5(um−1 − ul −1 ) = 1996[( pm+1 − pl +1 ) − 4(qm − ql )]

(7.8) (7.9)

, T`u (7.9) suy ra 5(um−1 − ul −1 )|1996, m` a (5, 1996) = 1 =⇒ (um−1 − ul −1 )|1996 =⇒ 1996( pm−1 − pl −1 ) + (qm−1 − ql −1 )|1996

=⇒ (qm−1 − ql −1 )|1996.

(7.10)

74

,, ˜ sô´ nâng cao Chuong 7. B` ai tâp . dây

Do 0 ≤ qm−1 ≤ 1995, 0 ≤ ql −1 ≤ 1995

=⇒ −1995 ≤ qm−1 − ql −1 ≤ 1995

(7.11)

, T`u (7.10) v` a (7.11) suy ra (qm−1 − ql −1 ) = 0 hay qm−1 = ql −1 . , ,, , ´, tiê´p tuc Tuong tu. ch´ ung ta c˜ ung c´ o thê d a cu ¯i d ¯ê´n qm−2 = ql −2 v` . , nhu thê´ d ¯i d¯ê´n q2 = q2 + (m − 1) v` a q1 = q1 + ( m − l ) (7.12) , `˘ng um−l ch´ınh l` ´,ng minh ra Ta chu a sô´ hang ay m` a chia hê´t cho . cua d˜ 1996. Thât ach x´ ac d ay, ta c´ o ¯inh . d˜ . vây . theo c´ 5um−l = um−l +2 − 4um−l +1 + 1976

= 1996pm−l +2 + qm−l +2 − 4(1996pm−l +1 + qm−l +1 ) + 1996 = 1996( pm−l +2 + 4pm+l +1 ) + (qm−l +2 + 4qm−l +1 ) + 1996 , ´,c trên ta c´ ˘ng thu Thay (7.12) v` ao d o ¯a 5um−l = 1996( pm−l +2 − 4pm−l +1 ) + (q2 − 4q1 ) + 1996.

(7.13)

Do u1 = 20 =⇒ u1 = 0.1996 + 20 =⇒ q1 = 20 u2 = 100 =⇒ u2 = 0.1996 + 100 =⇒ q2 = 100 , Vây . t`u (7.13) suy ra 5um−l = 1996( pm−l +2 − 4pm−l +1 ) + 1996. (7.14) , Do pm−l +2 v` a pm−l +1 l` a sô´ nguyên, vây . t`u (7.14) suy ra 5um−l |1996 m` a (5, 1996) = 1 suy ra um−l |1996.

J

, . 7.8. Cho d˜ay sô´ tu. nhiên u1 , u2 , . . . , un+1 sao cho 1 ≤ u1 < u2 < , , . . . < un+1 ≤ 2n . Chu´ng minh ra ˘`ng tô`n tai a j sao . hai sô´ tu. nhiên i v` cho u j chia hê´t cho ui , ( j > i ). , , ,´, ´ , ´, , , ´, `,i giai. K´y hiêu Lo a uo c sô le lon nhâ´t cua ui tuong u ng, ngh˜ıa l` a . vi ,l` , , p ´i vi le v` ´ (i = 1, 2, . . . , n + 1). ui = 2 i .vi , vo a sô´ tu. nhiên pi n` ao d¯o

7.1. V´ı du.

75

´,i moi Do 1 ≤ u1 < u2 < . . . < un+1 ≤ 2n suy ra vo . i = 1, 2,,. . . , n + 1 , ´ ta c´ o vi < 2n . X´et (n + 1) sô le v1 , v2 , . . . , vn+1 . C´ ac sô´ le n` ay d ¯ê`u , , , ,, , , `˘ng n. Vây duong v` a nho hon 2n. nên sô´ luo. ng cua ch´ ung ba . theo ` ´ nguyên l´y Ðirichlê tôn tai a . hai sô i, j sao cho 1 ≤ i < j ≤ n + 1 m` , pj p i ˘. t trên th`ı ui = 2 vi v` vi = v j . Theo c´ ach d a u j = 2 v j . Nhung ¯a pj p i ui < u j suy ra 2 vi < 2 v j , m` a vi = v j . Suy ra 2 pi < 2 p j . Do pi , p j l` a , , pj p i ´ ´ ´ c´ ac sô nguyên duong nên 2 chia hêt cho 2 . Ngh˜ıa l` a u j chia hêt cho ui .

J

, . 7.9. (Ðê` thi To´an Olympic Quô´c tê´ lâ`n thu´ 13) D˜ay sô´ {un }, n = , , n 2, 3, 4, . . . x´ ac d¯inh ˘`ng d˜ ay sô´ n` ay . nhu sau un = 2 − 3. Chu´ng minh ra , ,, , ` ´ ´ ´ chua môt ac phân tu, sao cho bât k`y hai sô n` ao cua tâp . tâp . vô han . c´ . , ´ ho. p n` ay c˜ ung nguyên tô c` ung nhau. , , , ,, `˘ng quy nap. `,i giai. Ta chu ´,ng minh ba ˜ xây du. ng d Lo Gia thiê´t d ¯a ¯uo. c . ,, , k phâ`n tu cua d˜ ay u1 = 2n1 − 3, u2 = 2n2 − 3, . . . , uk = 2nk − 3 , ´,i 1 ≤ i, j ≤ k, o, d ´,i moi m` a vo ¯ây 2 = n1 < . i 6= j, th`ı (u j , ui ) = 1, vo n2 < . . . < n k . , Ch´ ung ta s˜e xây du. ng sô´ uk+1 = 2nk+1 − 3 nguyên tô´ c` ung nhau , `˘ng c´ ´i c´ ˘. t l = u1 .u2 . . . uk . X´et vo ac sô´ u1 , u2 , . . . , uk ba ach sau d ¯ây: Ða , , 0 1 l (l + 1) sô´ 2 , 2 , . . . 2 Khi chia c´ac sô´ n`ay cho l ta d¯uo.,c môt . tâp . ho. p , gô`m l sô´ du. Vây o ´ıt nhâ´t hai sô´ cho ta . , theo nguyên l´y Ðirichlê c´ ,, , , r s ´ l` c` ung phâ`n du. Gia su hai sô´ d a 2 v` a 2 , (s > r ). Bây gi`o chon ¯o . sô´ p , r s s r − s s sao cho pl = 2 − 2 = 2 (2 − 1). Do l l` a sô´ le nên 2 không chia r r − s ˘. t kh´ hê´t cho l . Ma ac (2 , 2 ) = 1 suy ra (2r−s − 1) chia hê´t cho l. , r −s Ðiê`u n` ay ngh˜ıa l` a tô`n tai . sô´ tu. nhiên q sao cho 2 − 1 = ql, suy ra 2r−s+2 − 3 = 4.2r−s − 3 = 4(ql + 1) − 3 = 4ql + 1.

76

,, ˜ sô´ nâng cao Chuong 7. B` ai tâp . dây

˘. t uk+1 = 2r−s+2 − 3 = ql. Do uk+1 = 4ql + 1 v` Ða a l = u1 .u2 . . . uk , suy ra uk+1 > uk . ´,i ´ (uk+1 , ui ) = 1 vo Do uk+1 = 4ql + 1, suy ra (uk+1 , 1) = 1. Do d ¯o moi . i = 1, 2, . . . , k. , , , ,, ˜ xây du. ng d Nhu vây ay con u1 , u2 , . . . , uk thoa m˜ an ¯a ¯uo. c d˜ . khi d , , , , , ´ i u1 , u2 , . . . , u k , ay con mo d ai, th`ı s˜e xây duo. ng d ¯uo. c d˜ ¯iê`u kiên ¯â`u b` . d , uk+1 c˜ ung c´ o t´ınh châ´t â´y. Theo nguyên l´y qui nap ung ta xây du. ng . ch´ , ,, ˜ cho c´ d ay con vô han ay d o t´ınh châ´t: ¯uo. c d˜ ¯a . u1 , u,2 , . . . , un , . . . . cua d˜ ,, , ´ ca ˘. p phâ`n tu n` Bâ´t cu ao cua d˜ ay con â´y c˜ ung nguyên tô´ c` ung nhau.

J

, . 7.10. Cho u1 v`a u2 l`a hai sô´ nguyên duo,ng nguyên tô´ c` ung nhau. , D˜ ay sô´ {un } x´ ac d¯inh oi u1 , u2 l` a hai sô´ hang on khi n = . v´ . d¯â`u tiên, c` 2, 3, . . . ta x´ ac d ¯inh . u n +1 = u n u n −1 + 1 , , Chu´ng minh ra ˘`ng v´ oi moi . i > 1 tô`n tai . j > i sao cho u j chia hê´t cho ui . , , ,´, ´ , `,i giai. Lâ´y i > 1 t` Lo uy ´y, v` a p l` a uo c sô nguyên tô´ cua ui . Xây du. ng , d˜ ay sô´ {vn } nhu sau  0 ≤ vn ≤ p − 1 vn ≡ un (mod p) ˘. p Ta c˜ ung c´ o vn+1 = vn vn−1 + 1 (mod p), ∀n = 2, 3 . . . V`ı c´ ac ca , , (vn , vn−1 ) l`a vô han, a vn chı nhân a tri. suy ra {vn } . m` . h˜uu han . gi´ , , `˘ng ´,ng minh ra ´. Ta câ`n chu phai l` a d˜ ay tuâ`n ho` an t`u môt uc n` ao d ¯o . l´ , , tô`n tai . sô´ nguyên duong k p > 0 sao cho vi+k p = 0. , X´et hai kha n˘ ang sau

(7.15)

7.1. V´ı du.

77

, ,, , ´,i chu k`y T, trong d ´ s > i. ay {vn } tuâ`n ho` an t`u vs vo 1) Gia su d˜ ¯o Ta c´ o v s +1 = v s v s −1 + 1 = v s + T +1 = v s + T v s + T −1 + 1 v s v s −1 = v s + T v s + T −1

(mod p).

(mod p) (7.16)

a) Nê´u vs = vs+T ≡ 0 (mod p), th`ı vs = vs+T = 0, do 0 ≤ vn ≤ , ´ chı viêc p − 1, ∀n. Khi d ¯o . chon . k p = s − i, Suy ra vi+k p = vs = 0, ´ ng. vây ¯u . (7.15) d , b) Nê´u vs = vs+T 6= 0, th`ı t`u (7.16) c´ o vs−1 = vs+T −1 . Suy ra , , ˜ vo ´ ` ` ´,i {vn } tuân ho`an không phai ba˘t d¯âu t`u vs d¯iê`u n`ay mâu thuân , , c´ ach chon ang n` ay không xây ra. . s.Kha n˘ ,, , ´,c (7.15) d ´ ng trong tru`ong ho. p 1). T´ om lai ¯u . hê. thu , ´,c (7.15) hiên nhiên d ´ ng. Ta c´ 2) Nê´u s ≤ i th`ı hê. thu o v i +1 ≡ ¯u , ui+1 (mod p) ≡ ui ui−1 + 1 (mod p) vây . vi ≡ ui (mod p) nhung , , ,´, c nguyên tô´ cua ui suy ra vi ≡ 0 (mod p), nhu vây do p l` a uo . v i +1 ≡ 1 (mod p) hay l` a vi+1 = 1. ,, , , Tuongtu. , do vi+k p = 0 suy ra vi+k p +1 = 1. Nhu vây . suy ra {vn } , ,, , , ´ c l` ´i chu k`y k p tu ´i moi c˜ ung tuâ`n ho` an vo a vo o . l nguyên duong, ta c´ vi+l.k p = 0.

(7.17)

Ngh˜ıa l` a ui+l.k p chia hê´t cho p. , , ,´, , ,, ´,i moi Nhu vây c nguyên tô´ p cua ui ta xây du. ng d ¯uo. c sô´ k p . vo . uo ´ ng. sao cho (7.17) d¯u , , , Goi a bôi ac sô´ k p , theo (7.3) . m l` . sô´ chung nho nhâ´t cua tâ´t ca c´ , , , ´i moi suy ra vi+lm = 0 vo duong. Suy ra ui+lm chia hê´t cho . l nguyên , , , , ´,c l` ´c sô´ nguyên tô´ cua ui . Tu ´,i tâ´t ca c´ ac uo a ui+lm chia hê´t cho ui vo , ,, ´,ng minh. moi od ¯iê`u phai chu . sô´ nguyên duong l. Lâ´y j = i + lm, ta c´

J

78

,, ˜ sô´ nâng cao Chuong 7. B` ai tâp . dây

` tâp 7.2. Bai . , , . 7.11. Nê´u ba sô´ nguyên tô´ thu. c su. lo´,n ho,n 3 lâp anh môt . th` . câ´p , sô´ công ung chia hê´t cho 6. . th`ı công sai cua ch´ ,, , . 7.12. cho f (n) l`a h`am x´ac d¯inh ac sô´ nguyên duong nhu . trên tâp . c´ ˜ ´,i môi sau: nê´u n = a1 a2 . . . ak , th`ı f (n) = ( a1 + a2 + · · · + ak )1998 . Vo , , , sô´ nguyên duong n, lâp ay vô han . d˜ . ui (n), i = 1, 2, . . . nhu sau ui (n) = f ( f (. . . f (n)i lâ`n f | {z } ,, `˘ng vo ´,ng minh ra ´,i moi Chu ay . n nguyên duong, tô`n tai . p sao cho d˜ ui (n), i = p, p + 1, . . . l` a d˜ ay tuâ`n ho` an. ´,ng . 7.13. Cho u1 , u2 , . . . , un l`a d˜ay sô´ t` uy ´y gô`m n sô´ hang. Chu . ,, `˘ng luôn luôn tr´ıch d minh ra ay con sao cho nê´u goi a ¯uo. c môt . d˜ . S l` , ,, , , ´i sô´ nguyên gâ`n nhâ´t tông c´ ac phâ`n tu cua d˜ ay con â´y, th`ı S kh´ ac vo 1 ,, ,, . môt a . luo. ng không vuo. t qu´ n+1 `˘ng nê´u nh˜u,ng sô´ nguyên a v` ´,ng minh ra . 7.14. Chu a m nguyên tô´ , c` ung nhau, th`ı tô`n tai a t´ıch na chia cho m cho . môt . sô´ tu. nhiên n m` , sô´ du 1.

, , CHUONG

8

, ´ THU,C VO ´ ` MÂT SÔ I TÂP . TRU . .

8.1. Tâp u mât . tr` .

, ,, , ,, ´,i kh´ ˘ng thu. c ch´ Trên d ung ta thu`ong quan tâm to ai niêm ¯u`ong tha . , , , , , ˘ng), ta c´ môt (môt o thê hiêu môt ¯oan . khoang . d . tha . khoang trên , , ,`, , , ` , , ´ , ´ ˘ng thu. c l` ˘m gi˜ua hai d d a tâp ac sô thu. c na ¯uong tha ¯iêm . , ho. p, tât ca c´ , , , `˘m trong ˜ cho. Nhu vây d ac d môt khoang c´ o thê na ¯a ¯iêm d ¯ê x´ ¯inh . . hai d . , , `˘m ngo` ˘. c na ´;d ´ng, hoa ai khoang d a xuâ´t ph´ at cua kh´ ai niêm ¯o ¯ây l` ¯o . d , , ,, ,, , ´ mo,.. o ng` anh giai t´ıch, giai t´ıch h` am trong to´ an hoc cao câp. Nhung . , ,, ´,i c´ ´, m` o d a chı quan tâm to ac kh´ ai ¯ây ta không quan tâm d ¯iê`u d ¯o , , , ´ , , , ´ ´ ´ ´ ´ niêm a sô thu. c, k´y ¯ây voi quan d ¯iêm kiên thuc so câp. Voi a, b l` . sau d , , , , ,, ` hiêu a khoang d a goi l` a khoang ¯uo. c t´ınh ca hai d ¯iêm d ¯âu a, b v` . [a,b] l` . , , ,, , , ´ng. Khoang mo d d l` a ( a, b), không lâ´y hai d ¯o ¯uo. c k´y hiêu ¯iêm d ¯â`u . , , ,, ,, ,, ´ng (nua mo ) do viêc a, b. Ngo` ai ra ta c` on x´et khoang nua d ¯o . ta chı , , ˘. c b,v´ı du. nhu [ a, b) hoa ˘c ( a, b]. Môt lâ´y môt trong hai d ¯iêm d ¯â`u a hoa , . ,. ,. , ´ ´ khoang trong tâp a suy tho´ ai nêu n´ o chı l` a môt ¯iêm . , sô thu. c goi . l` . d , ` ´ (tuc l` a khi hai d ung nhau). ¯iêm d ¯âu tr` , , , , Môt a tr` u mât, nê´u moi khoang . tâp . ho. p A cua sô´ thu. c goi . l` . . ,, , ´,a môt không suy tho´ ai d ¯ê`u chu . sô´ phâ`n tu cua A. , , , , , Môt a tâp ho. p sô´ h˜uu ty gô`m c´ ac sô´ c´ o thê biêu . v´ı du. d˜ê thâ´y l` . , , , ,, , di˜ên nhu môt a tâp . thuong cua hai sô´ nguyên v` . ho. p sô´ vô ty gô`m

80

,, , , Chuong 8. Sô´ thu. c vo´i tâp u mât . tr` .

, , , ` , ´,ng sô´ không phai l` u mât nhu a sô´ h˜uu ty d a nh˜ung tâp ¯êu l` . tr` . trong , ´ tâp . sô thu. c. , ,, ,, Ngu`oi ta c` on mo rông kh´ ai niêm tr` u mât cho môt khoang trên . . . . , , , ,, , ,, ˘ng thu. c. Cho ∆ l` ˘ng d a môt ¯u`ong tha ¯u`ong tha . khoang bâ´t k`y trên d , , , , thu. c. Ch´ ung ta goi u. c l` a tr` u mât . ,tâp . ho. p B gô`m nh˜ung sô´ th . trong , , , ´ a nh˜u,ng ´i moi ∆, nê´u vo ai cua ∆ d ¯ê`u chu . khoang con không suy tho´ , ,, , , , phâ`n tu cua B. R˜ o r` ang tâp ac sô´ h˜uu ty tr` u mât . h,o. p c´ . trong moi . , , ´ ´ ´ khoang; v` a tâp h o p c´ a c sô vô t y c˜ u ng c´ o t´ ınh châ t â y. . . , ,, , ´,i moi Nh˜ung k´y hiêu ung: vo a ¯uo. c d` . hay d . sô´ thu. c x, k´y hiêu . [ x ] l` , , , ´n nhâ´t không vuo. t qu´ sô´ nguyên lo a x, { x } l` a sô´ x − [ x ]; [ x ] v` a {x} , , goi a phâ`n nguyên v` a phâ`n thâp cua sô´ x. T`u d o ¯inh . ngh˜ıa c´ . , l` . phân , , , thê thâ´y ngay 0 ≤ x − [ x ] < 1. Ðê hiêu r˜ o hon kh´ ai niêm u mât . tr` . ,, trong phâ`n n` ay ta x´et môt lo at b` a i to´ a n c´ o liên quan v` a c´ a ch s u d ung . , ., . , , ´ ng minh d nguyên l´y Ðirichlê d ¯ê chu ¯inh l´y co ban Kronecker.

8.2. V´ı du.

, , , , . 8.1. Nê´u môt sô´ thu. c l` a tr` u mât a r l` a sô´ thu. c kh´ ac . tâp . ho. p A cua . v` , , ` ´ , không, th`ı tâp oi a chay ung l` a . ho. p gôm tât ca t´ıch sô´ {ra} v´ . trong A c˜ tâp u mât. . tr` . , , ,, , `,i giai. C´ ´,ng Lo o hai tru`ong ho. p xây ra: r > 0 v` a r < 0. V`ı c´ ach chu , ,, , ,, , minh ho` an to` an tuong tu. nên ta chı x´et tru`ong ho. p d a ¯â`u. Cho x v` , x y , ´ < v` y l` a hai sô´ thu. c v` a x < y. Khi d¯o a theo gia thiê´t A l` a tâp . r r ,, x y tr` u mât, . th`ı tô`n tai . phâ`n tu a thuôc . A sao cho r < a < r . Suy ra , ,, , ´,a sô´ dang ´,i a x < ar < y. Nhu vây ra vo . moi . khoang mo ( x, y) chu . thuôc . A.

J

, , , , . 8.2. (Ðinh oi moi ac sô´ c´ o . l´y Kronecker) V´ . sô´ vô ty α, tâp . ho. p tâ´t ca c´

8.2. V´ı du.

81

,, ˜,ng sô´ nguyên bâ´t k`y. a n l` a nhu dang a tâp u mât, . mα + n l` . tr` . o d¯ây m v` , , , `˘ng moi `,i giai. Câ`n phai chu ´,ng minh ra Lo khoang không suy tho´ ai . , , , , ´ a nh˜ung d ∆ chu ung ta chia khoang [0,1] ra ¯iêm dang . ,mα + n. Ch´ , , ˜ th` anh môt sô´ h˜uu han khoang con ∆1 , ∆2 , . . . , ∆k m` ad ai cua môi ¯ô. d` . . , , , , , `˘ng vo ´i moi khoang con nho hon d ai cua ∆. Ch´ u ´y ra sô´ a tô`n tai ¯ô. d` . , . ` ´ ´ ˘ môt sô nguyên n, sao cho sô a + n n a m trong kho ang [0,1]. Suy ra . , , , ´, ´,i moi ´,i môt vo o thê cho tuong u ng vo sô´ dang . sô´ nguyên m c´ . mα + n, ,. , , ,, `˘m trong khoang [0,1]. Nhung d ˘ng [0,1] d m` a n´ o na ¯oan ¯uo. c chia . tha , , , , ra h˜uu han . khoang nho. Suy ra tô`n tai . nh˜ung sô´ nguyên m, m1 , n, n1 , , `˘m trong c` ´,i m 6= m1 v` vo a nh˜ung sô´ mα + n v` a m 1 α + n 1 na ung môt , , , , , , . , ´ khoang c´ khoang nho ∆i . Do d ach gi˜ua hai d ay nho hon d¯ô. ¯o ¯iêm n` , , , d` ai cua ∆. Ch´ ung ta c˜ ung thâ´y không thê xây ra mα + n = m1 α + n1 , , n1 − n , , , , ´ c n` ˘ng thu v`ı t`u d¯a ay suy ra α = m` a lai l` a môt sô´ h˜uu ty, tr´ ai . . m − m 1 , ´,i gia thiê´t. vo , , `˘ng sô´ (m − m1 )α + (n − n1 ) kh´ ˘ng d Nhu vây ung ta kha ac ¯inh . , ra . ch´ , , , , ´c sô´ ´ suy ra môt không v` a c´ o gi´ a tri. tuyêt ¯o ¯ô´i nho hon ∆. T`u d . sô´ uo . d , , `˘m trong khoang ∆. cua n´ o, c˜ ung c´ o dang . mα + n, s˜e na , ,, , , ´ tâp . 8.3. Cho α l`a môt o dang ¯o . sô´ vô ty duong. Khi d . ho. p tâ´t ca sô´ c´ . , , , ´ ´ ˜ mα − n l` a tâp tr` u m ât,v´ o i m v` a n l` a nh u ng sô t u nhiên bâ t k` y . . . . , , , , `,i giai. Kê´t luân ˘ng Lo ai n` ay manh hon d l´y trên v`ı n´ o kha ¯inh . . cua b` . , , , , ´ c˜ d ung tr` u ¯inh ¯ij nh l´y d ¯o . môt . tâp . sô´ thu. c nho hon tâp . mô ta trong d mât. . , , `˘m o,, bên phai sô´ không v` Cho khoang không suy tho´ ai ∆ na a c´ o , , , , , ,, , ´ ng minh c´ d ai e > 0. Không anh huong d o thê gia ¯ô. d` ¯ê´n kê´t qua chu , e , , thiê´t e < 1. Ngo` ai ra ta c´ o thê chon . sô´ tu. nhiên k sao cho k ≥ α . T`u ´,i p v` 8.1 v` a 8.2 suy ra tâp ac sô´ kpα + kq vo a q l` a c´ ac sô´ nguyên bâ´t . c´

J

82

,, , , Chuong 8. Sô´ thu. c vo´i tâp u mât . tr` .

, k`y, l` a tâp u mât. a tô`n tai ac sô´ nguyên p v` a q thoa m˜ an . tr` . Ngh˜ıa l` . c´ 0 < kpα + kq < e. (8.1) , , , `˘ng gi˜ua nh˜ung ca ˘. p sô´ nguyên ( p, q) trong (8.1) Ch´ ung ta s˜e chı ra ra , , , ,, , ˘. p m` c´ o nh˜ung ca a p > 0. Thât . vây, . tru`ong ho. p p = 0 không thê xây , ˜ vo ´,i d ˜ biê´t l` ra v`ı t`u (8.1) suy ra 0 < kq < e, mâu thuân a kq ¯iê`u ta d ¯a , , , , ´ ´ ng minh cho tru`o,ng l` a sô nguyên v` a e < 1. Do vây on phai chu . chı c` ,, , , ´,i s, t l` ho. p p < 0. Trong tru`ong ho. p n` ay sô´ c´ o dang kpsα + kqt vo a . , , ´ ´ ` nh˜ung sô nguyên bât k`y, tao anh tâp u mât. . th` . tr` . T`u (8.1) suy ra tôn ´ tai a t sao cho . sô nguyên s v` 0 < kpsα + kqt < kpα + kq. (8.2) , ´,ng minh lai ´ ng. V`ı thê´ ta chı x´et Nê´u s < 0, th`ı kps > 0 v` a chu ¯u . d , s ≥ 0. T`u (8.1) v` a (8.2) suy ra 0 < kp(1 − s)α + kq − kpt < e. (8.3) , , , ´,c (8.2) không thê xây ra, nên s > 0. Nhu,ng ´,i s = 0 bâ´t d ˘ng thu V`ı vo ¯a ,, , boi v`ı s l` a sô´ nguyên, th`ı t`u s > 0 suy ra s ≥ 1. Tai . v`ı ((8.3) không , , , , , , ´ c cuô´i c` ´i s = 1, t`u bâ´t d ˘ng thu thê xây ra vo ung lai ¯a . suy ra s > 1. , ´ c´ Nhu vây o ngh˜ıa l` a tô`n tai ac sô´ nguyên p ¯o . , kp(1 − s) > 0. Ðiê`u d . c´ , v` a q thoa m˜ an (8.1) v` a p > 0. T`u (8.1) suy ra e kq < e − kpα < e − p α = e(1 − p) ≤ 0 α ,, , Vây ach n` ay (8.1) d − n < e, ¯uo. c d ¯ua vê` dang . kq < 0. Theo c´ . 0 < mα , ,, ,, , , , ´ od a n l` a nh˜ung sô tu. nhiên. Boi v`ı e l` ad ai cua∆, t`u bâ´t ¯ây m v` ¯ô. d` , , ,´, ´ ´,c cuô´i c` ˘ng thu ´ cua mα − n, c˜ d ung suy ra môt c sô n` ao d ung c´ o ¯a ¯o . uo ,, , , , , ` ´ ng minh d ˘m trong ∆. Tuong tu. c˜ dang n` ay, s˜e na ung chu ¯uo. c khi ∆ . ,, ` ´ ˘m o bên tr´ na ai sô không. , , ˜,ng sô´ dang . 8.4. Cho α l`a sô´ vô ty bâ´t k`y. Khi d¯o´ tâp . ho. p nhu . {αn}, , , , v´ oi n l` a sô´ tu. nhiên bâ´t k`y, l` a tâp u mât . tr` . trong khoang (0,1).

J

8.2. V´ı du.

83

, , , `˘ng moi `,i giai. Câ`n phai chu ´,ng minh ra khoang con không suy Lo . , , ´,a sô´ c´ biê´n ( a, b) cua (0,1) chu o dang {αn} vo´,i môt . . sô´ tu. nhiên ´,ng minh cho tru,`o,ng ho.,p khi α l` ´. Ch´ n n` ao d ung ta chu a môt ¯o . sô´ ,, , , , ´i n, m l` duong. Theo b` ai 8.3 nh˜ung sô´ c´ o dang nα − m vo a c´ ac sô´ tu. . , , nhiên, tao anh tâp u mât. a tô`n tai . , th` . tr` . Ngh˜ıa l` . nh˜ung sô´ tu. nhiên , , m v` a n thoa m˜ an 0 ≤ a < nα − m < b < 1. Nhung t`u d ¯inh . ngh˜ıa ,, [αn] suy ra 0 ≤ nα − [nα] < 1. Ch´ ung ta nhân ¯uo. c hiêu . d . hai sô´, , nguyên m v` a [nα] thoa m˜ an −1 < m − [nαm] < 1, d ay chı ¯iê`u n` , , , ` ` ˘ng sô´ ´ m = [nα]. Tu (8.1) chı ra ra xây ra khi m − [nαm] = 0, do d ¯o , , ˜ {nα} = nα − [nα] na`˘m trong khoang ( a, b). Vo´i α > 0, ch´ ung ta d ¯a , ´ chung minh xong. ,, , , ´ −α > 0 v` Tru`ong ho. p α < 0. Khi d a t`u b` ai 8.3 suy ra tô`n tai ¯o . , , , nh˜ung sô´ tu. nhiên m v` a n, thoa m˜ an −1 ≤ −b < n(−α) − m < ,, − a ≤ 0. Nhân c´ac vê´ vo´,i -1 ch´ ung ta nhân ¯uo. c 0 ≤ a < nα − m < . d , `˘m trong b ≤ 1. Nhu vây ang t´ınh ra m = −[nα] v` a {nα} na . d˜ê d` , khoang ( a, b). , , , , . 8.5. Tâp ac sô´ dang oi n l` a sô´ tu. nhiên bâ´t k`y . ho. p tâ´t ca c´ . {log n} v´ l` a tâp u mât . tr` . trong (0,1).

J

, , , , `,i giai. Ch´ ´,ng minh mo, rông Lo ung ta c´ o thê chu hon môt ut: tâp . . ch´ . , , , ´ ´ ´ ho. p c´ ac sô´ dang { n log 2 } v o i n l` a sô t u nhiên bâ t k` y , l` a t âp tr` u m ât . . . . trong (0,1). , ´,ng minh log 2 l` Ðê d muc d¯´ıch n` ay ch´ ung ta chu a môt ¯at . . . sô´ vô , ,`, ,, , , ty. Thât . vây, . trong truong ho. p nguo. c lai . th`ı tô`n tai . hai sô´ tu. nhiên p p , , ´,c cuô´i ˘ng thu p v` a q, m` a log 2 = , ngh˜ıa l` a 2 = 10 q . L˜ uy th`ua d ¯a q ,, ´,i q ch´ c` ung vo ung ta nhân ay tr´ ai d l´y ¯uo. c 2q = 2 p .5 p . Ðiê`u n` ¯inh . . d , , , , , co ban cua sô´ hoc . vê` viêc . phân t´ıch ra th`ua sô´ nguyên tô´. Nhu vây .

84

,, , , Chuong 8. Sô´ thu. c vo´i tâp u mât . tr` .

, , , a môt log 2 thu. c su. l` 8.4 suy ra tâp ac sô´ c´ o dang . sô´ vô ty. Áp dung . . c´ . {n log 2} l`a tr` u mât . trong (0,1). , ´,i moi ˘. t kh´ Ma ac, theo t´ınh châ´t cua logarit, log 2n = n log 2 vo . n = 1, 2, . . . , , . 8.6. Cho m l`a sô´ nguyên, n l`a sô´ nguyên không âm. Tâp . ho. p tâ´t ca m c´ ac sô´ c´ o dang l` a tâp u mât. . . tr` . 2n , , `˘ng `,i giai. Do c´ ´,ng minh ra Lo ac b` ai to´ an trên, ch´ ung ta chı câ`n chu m 3 , ,, , ´,i m, n l` vo a nh˜ung sô´ nguyên duong, l` a tr` u mât tâp . ho. p sô´ dang . . n 2 , , , , ´i moi ˘. p sô´ duong ´ c´ trong khoang (0, +∞). Ðiê`u d o ngh˜ıa l` a vo ¯o . ca , 3m , , ´ ˜ < b. a, b( a < b) tô`n tai nh u ng sô t u nhiên m v` a n th oa m˜ a n a < . . 2n , , , ´ c n` ˘p bâ´t d ˘ng thu Sau khi logarit h´ oa ca ay theo co sô´ 2, ch´ ung ta ¯a ,. ,, , , , , , ´ ´ c tuong d ˘. p bât d ˘ng thu nhân ¯uo. c ca ¯a ¯uong log2 a < m log2 3 − n < . d , , , , , ´ ng minh tuong tu. nhu, 8.5 cho thâ´y log2 3 l` log2 b. C´ ach chu a vô ty. , `˘ng trong d ´,a ´d Khi d ¯o ¯inh ¯oan . l´y Kronecker chı ra ra . (log2 a, log2 b) chu sô´ c´ o dang . {m log2 3 − n}. , , , , . 8.7. Tâp ac sô´ hang cua d˜ ay x´ ac d¯inh ˘`ng công thu´c . ho. p tâ´t ca c´ . . ba , n 1 xn = [log n]+1 , n = 1, 2, 3 . . ., l` a tr` u mât . trong khoang ( 10 , 1). 10

J

J

, , 1 `,i giai. Lâ´y hai sô´ thu.,c a v` Lo a b sao cho ≤ a < b ≤ 1. Câ`n phai 10 , n , `˘ng tô`n tai ´ ´,ng minh ra chu sô t u nhiên n, th oa m˜ an a < [log n]+1 < . . 10 , , , , ´ ´ ´ ˘ b. Sau khi logarit h´ oa bâ´t d a ng th u c trên v o i co sô 10 ch´ ung ta ¯ ,, ´ , , , , , , ´ c tuong d ˘ng thu nhân ¯uo. c bât d ¯a ¯uong 1 + log a < log n − [log n] = . d , , , {log n} < 1 + log b. Nhung khoang mo, vo´,i c´ac d¯â`u m´ ut 1 + log a , , `˘m trong khoang (0, 1). Ðiê`u n` v` a 1 + log b r˜ o r` ang na ay giai th´ıch tai . , , ,, , , ´ ´ ´ ˘ sao tô`n tai m ôt sô t u nhiên n th oa m˜ a n bâ t d a ng th u c sau c` u ng, b o i ¯ . . .

8.2. V´ı du.

85

, v`ı theo 8.5 tâp ac sô´ c´ o dang {log n} l`a tr` u mât . ho. p c´ . . trong (0, 1). , , ` ` ´ ng minh. Nhu vây ¯iêu ta cân chu . th`ı a < xn < b, d

J

, , ˜,ng sô´ l˜ `,a cua 2, m` . 8.8. Chu´ng minh ra˘`ng tô`n tai uy thu a . vô han . nhu , ´ ` khi viê´t theo co sô´ 10 ch´ ung luôn luôn ba ˘t d¯â`u ba ˘ng 8975. , , m ´ `˘ng nh´ `,i giai. Môt ´,i ˘t d Lo om sô´ 8975 khi v` a chı khi vo ¯â`u ba . sô´ 2 ba , , , , ,, ´,c sau d ˘ng thu ´, nh˜ung d¯a môt ao d an ¯o ¯uo. c thoa m˜ . sô´ tu. nhiên n n` , , ,´, n m n 8975.10 ≤ 2 < 8976.10 . Nhu c´ ach giai c´ ac b` ai truoc ch´ ung ta , , , , , , ´ c trên v` ´ c tu,o,ng ˘ng thu ˘ng thu logarit h´ oa bâ´t d a nhân ac bâ´t d ¯a ¯uo. c c´ ¯a . d ,, d a môt ¯uong log 8975 ≤ m log 2 − n < log 8976. Tai . v`ı log 2 l` . sô´ vô , , , , , ´i n, m l` ty, nên nh˜ung sô´ c´ o dang m log 2 − n vo a nh˜ung sô´ tu. nhiên . ˘. p sô´ tao anh môt u mât. ay c´ o ngh˜ıa l` a tô`n tai . th` . tâp . tr` . Ðiê`u n` . vô sô´ ca , , , , ´ c sau c` ˘ng thu tu. nhiên m v` a n thoa m˜ an c´ ac bâ´t d ung, suy ra c˜ ung ¯a , , , , , ´ c truo ´c d ˘ng thu ´. Ch´ ´ ng khi u ´y b` ai to´ an c` on d thoa m˜ an c´ ac bâ´t d ¯o ¯u ¯a , , , ´ , ´ ` `˘ng ˘ng co sô 10 cua logarit v` thay co sô hai ba a sô´ 8975 c´ o thê thay ba , , ´,ng minh d môt ay d` anh cho ban ¯iê`u n` ¯oc. . tô ho. p sô´ bâ´t k`y, chu . d .

J

, , , ˜, sô´ d¯â`u tiên cua . 8.9. V´ oi moi a chu . sô´ tu. nhiên n, k´y hiêu . xn l` , , , sô´ 2n (trong c´ ach viê´t v´ oi co sô´ 10). Chu´ng minh ra ˘`ng d˜ ay sô´ , ˜ tuâ`n ho` x1 , x2 , . . . , xn , . . . không phai l` a dây an. , , , , ,, `,i giai. Gia su, ngu,o.,c lai Lo a tô`n tai a . l` . nh˜ung sô´ nguyên duong k v` d sao cho xk = xk+d = xk+2d = . . . = xk+nd = . . . N´ oi c´ ach kh´ ac , k k + d k + 2d k + nd 2 ,2 ,2 ,...,2 , . . . c´ o c` ung ch˜u sô´ d¯aah u tiên trong c´ ach , , , , , `˘ng moi ´i co sô´ 10. Bâ`y gi`o ch´ ˘ng d viê´t vo ung ta kha ¯inh . ra . sô´ tu. nhiên , , , , log N [ log ´i dang N c´ o thê biêu di˜ên duo N = 10 = 10 N ]+{log N } = . 10[log N ] .10{log N } . V`ı 0 ≤ {log N } < 1, nên 1 ≤ 10{log N } < 10, suy , , , , ´,i phâ`n nguyên cua cua sô´ 10{log N } . ra ch˜u sô´ d ung vo ¯â`u tiên cua N tr` , , , ˘ng Ðê´n d an bâ´t d ¯ây suy ra tô`n tai ¯a . môt . ch˜u sô´ s, 1 ≤ s ≤ 9, thoa m˜

86

,, , , Chuong 8. Sô´ thu. c vo´i tâp u mât . tr` .

k+nd ´,c sau: s ≤ 10{log 2 } < s + 1 vo ´,i n = 0, 1, 2, . . .. Sau khi logarit thu , ´,c trên ch´ ˘ng thu h´ oa d ung ta c´ o log s ≤ {(k + nd) log 2} < log(s + ¯a , ` ´ 1). Ðiêu n` ay ngh˜ıa l` a tât ca c´ ac sô´ c´ o dang {(k + nd) log 2}, n = . , ` ´,ng minh ˘m trong khoang [log s, log(s + 1)). D˜ê d` ang chu 0, 1, 2, . . . na , , `˘ng tâp ´ tr´ ay tr` u mât ai ra ¯oan ¯o . ho. p nh˜ung sô´ n` . trong d . (0,1). Ðiê`u d , ,, , , ´ ` ´i kêt luân vo o d ¯oan ¯iêm . : tôn tai . tâp . con mo cua d . , (0,1) không c´ ,, , , , , ` ´i [log s, log(s + 1)). Nhu vây chung vo a sai. ¯iêu gia su nguo. c lai . d . l` , , , , . 8.10. H˜ay t`ım trong d¯oan ˘ng [0, π ], tâ´t ca sô´ thu. c t thoa m˜ an . tha , ,, , ´ ´ bât phuong tr`ınh cos nt ≥ cos t v´ oi moi . sô tu. nhiên n.

J

, `˘ng nê´u t c´ `,i giai. - Ch´ ´,ng minh ra ˜ nêu Lo ung ta s˜e chu o t´ınh châ´t d ¯a , , , ,, t t , trong b` ai to´ an th`ı ty sô´ l` a môt a . sô´ h˜uu ty. Thât . vây . gia su π l` π , , , ´ t 6= π ; Không mâ´t t´ınh tông qu´ sô´ vô ty. Khi d¯o at ch´ ung ta c´ o thê , , ,, `˘ng 0 ≤ t < π. V`ı t l` gia thiê´t ra a sô´ vô ty duong, theo d l´y ¯inh . 2π , , , ,, Kronecker tô`n tai a n, thoa m˜ an bâ´t phuong . nh˜ung sô´ tu. nhiên m v` t t t , l. Bo,,i v`ı ngo` gia thiê´t ra ai ra c` on c´ o 0 ≤ k ≤ n, 0 ≤ l ≤ n, ˘. t q = k − l v` ´ p v` nên 1 ≤ k − l ≤ n. Ta d a p = [kx ] − [lx ]. Khi d aq ¯a ¯o

, ,, ˜,ng u ´,ng dung Chuong 9. Nhu kh´ ac cua nguyên l´y Ðirichle . , , ´,i c´ ˘. t n` a thoa m˜ l` a nh˜ung sô´ nguyên v` an 1 ≤ q ≤ n. Vo ach da ay (9.2) 1 , , ` d o (9.1). ¯ua vê dang ¯ây chia hai vê´ cho q ta c´ . |qx − p| ≤ n , t`u d 90

J

, , , , ˜ . 9.2. V´ oi moi han q, v´ oi môi q tô`n . sô´ thu. c x tô`n tai . vô . sô´ tu. nhiên , , , tai ung thoa m˜ an bâ´t d¯a ˘ng thu´c. . sô´ nguyên p, sao cho ch´ x − p ≤ 1 . (9.3) q q2 , r , , , ´, `,i giai. Nê´u x l` ´i r l` a sô´ nguyên v` a s l` a Lo a sô´ h˜uu ty, tu c l` a x = vo s , , , ˘. t p = mr ´ ng, v`ı c´ sô´ tu. nhiên, th`ı kê´t luân ai to´ an l` ad o thê d ¯u ¯a . cua b` , , , , , ´i sô´ tu. nhiên bâ´t k`y m. Vo ´i tâ´t ca c´ v` a q = ms vo ach chon a q nhu . p v` , ,, `˘ng không. Nhu, vây vây an, v`ı vê´ tr´ ai luôn luôn ba ¯uo. c thoa m˜ . , (9.3) d . , , , ,, , ,, , , ´ chı c` on phai x´et tru`ong ho. p x l` a sô´ vô ty. Gia su chı c´ o h˜uu han sô . , , , ´i ch´ tu. nhiên q, m` a vo ung tô`n tai an (9.3), k´y . sô´ nguyên p thoa m˜ , ´i bâ´t k`y λ = 1, 2, . . . , l k´y hiêu hiêu ung l` a q1 , q2 , . . . , ql . Vo a sô´ . ch´ . pλ l` , nguyên thoa m˜ an x − pλ ≤ x − p qλ qλ , ,, , ´i moi vo sô´ nguyên p. V`ı sô´ x l` a vô ty, moi a tri. tuyêt ai ¯ô´i o ph´ıa tr´ . gi´ . d ,. , , , , , ´ ´ ´ ´ c trên l` ˘ng thu bât d¯a a duong, sô nh˜ung gi´ a tri. tuy êt a h˜uu ¯ôi n` . d ay l` 1 p , λ han. . V`ı vây . tô`n tai . sô´ tu. nhiên n, sao cho n < x − q , (λ = λ 1, 2, . . . , l ). ,´, , Theo b` ai truo c s˜e tô`n tai a sô´ nguyên p, sao cho . sô´ tu. nhiên q v` , p 1 1 , ´ x − ≤ an v` a 1 ≤ q ≤ n. Nhung khi d (9.1) thoa m˜ ≤ = ¯o q qn qq , 1 , , ´,c ´,i môt ˘ng thu , suy ra q = qλ vo sô´ λ = 1, 2, . . . , l T`u nh˜ung bâ´t d ¯a . 2 q 1 pλ 1 1 1 p , . Nhu vây < v`ı thê´ trên suy ra < x − ≤ x − ≤ . n qλ q nq n nq

, , 9.1. Xâ´p xı môt . sô´ thu. c

91

, ,, , ,, q < 1, d ay không thê d a sô´ tu. nhiên. Ta nhân ¯uo. c ¯iê`u n` ¯uo. c v`ı q l` . d d ¯iê`u vô l´y. , , , . 9.3. V´ oi moi ˘ng thu´c . c > 2 bâ´t d¯a √ 2 − p ≤ 1 . (9.4) q qc , , , , ˜ u han ´ng chı v´ d¯u oi hu ˘. p sô´ nguyên p v` a sô´ tu. nhiên q. . ca , , `,i giai. Ch´ ´,ng minh bâ´t d¯a ´,c sau l` ˘ng thu ´ ng Lo ung ta chu ad ¯u √ 2 − p ≤ √1 . (9.5) q 3 2q2 , 1 , ´,i moi ´,i moi ˘ng vo a vo ¯a . sô´ nguyên p v` . sô´ tu. nhiên q. V`ı q2 ≤ 1, bâ´t d √ , , p 1 , , ´ ´ c (9.5) tât nhiên thoa m˜ thu an, khi 2 − > √ . Nhu vây chı . q 3 2 √ √ p 1 1 , , , , ´ 2− √ ≤ câ`n x´et tru`ong ho. p 2 − ≤ √ . Nhung khi d ¯o q 3 2 3 2 √ 1 p ≤ 2 + √ suy ra q 3 2 √ p (9.6) 0 < < 2 2. q ˘. t kh´ Ma ac √ p √ p 2 2− q . 2+ q 2q − p2 √ p 2− = . = √ (9.7) √ p p q 2 2− 2+ q q q √ , , Nhung sô´ 2q2 − p2 l` a sô´ kh´ ac không, v`ı 2 l` a môt . sô´ vô ty. Ngo` ai √ p , ra n´ o l` a sô´ nguyên nên |2q2 − p2 | ≥ 1 . T`u (9.7) suy ra 2 − ≥ q 1 , v` ´i (9.6) cho ta (9.5). a c` ung vo √ p 2 q 2+ q

J

, ,, ˜,ng u ´,ng dung Chuong 9. Nhu kh´ ac cua nguyên l´y Ðirichle . , , ,, , ´,c (9.4) d ´,i môt ˘ng thu ˘. p sô´ ( p, q) n` ´ ng vo ´. T`u ao d Gia su bâ´t d ¯u ¯o ¯a . ca , , 1 1 ´,c √ ≤ c−2 . V`ı c > 2 bâ´t d ´,c ˘ng thu ˘ng thu (9.4) v` a (9.5) suy ra d ¯a ¯a q 3 2 , , , , ˜ sô´ q nhu, vây ´ ´,i môi ´ ng cho h˜uu han n` ay chı d sô t u nhiên q. Vo chı ¯u . . . , , ´,c ´,i ch´ ˘ng thu c´ o nhiê`u nhâ´t hai sô´ nguyên p, vo ung thoa m˜ an bâ´t d ¯a , , , ˘. p sô´ p v` (9.4). Ngh˜ıa l` a (9.4) chı thoa m˜ an cho h˜uu han a q. . ca 92

J

, , . 9.4. Nê´u D l`a sô´ tu. nhiên bâ´t k`y, không l`a sô´ ch´ınh phuo,ng, th`ı tô`n , , ,, tai ac ca ˘. p sô´ tu. nhiên ( x, y), l` a nghiêm . vô han . c´ . cua bâ´t phuong tr`ınh sau √ | x2 − Dy2 | ≤ 1 + 2 D. (9.8) , , `˘ng tô`n tai `,i giai. Theo b` ˘. p sô´ ( x, y) tu. Lo ai 9.2 ta biê´t ra . vô han . ca nhiên, sao cho x √ − D ≤ 1 . (9.9) y y2 , ´,c sau l` ˘. t kh´ ˘ng thu ´ ng Ma ac bâ´t d ad ¯a ¯u √ √ √ x √ x √ + D = ( − D ) + 2 D ≤ x − D + 2 D. y y y ˜ ca ´,i môi ˘. p ( x, y) ta c´ Suy ra vo o √ x √ + D ≤ 1 + 2 D. y 2 y , , ´,i c´ ˘. p ( x, y), sao cho thoa m˜ Nhu vây, ach chon an (9.9) . vo . bâ´t k`y ca ,, ch´ ung ta nhân . d¯uo. c √ √ | x2 − Dy2 | = | x − y D |.| x + y D | √ √ √ 1 1 1 ≤ ( + 2 Dy) ≤ 2 + 2 D ≤ 1 + 2 D. y y y `˘ng bâ´t phu,o,ng tr`ınh (9.8) c´ ´,ng minh ra ˜ chu Ðiê`u n` ay d o vô han ¯a . , ´ , nghiêm trong t âp h o p sô t u nhiên. . . . .

J

, , 9.1. Xâ´p xı môt . sô´ thu. c

93

, , . 9.5. Nê´u D l`a sô´ tu. nhiên bâ´t k`y, không l`a sô´ ch´ınh phuo,ng, th`ı ,, , phuong tr`ınh x2 − Dy2 = 1 c´ o ´ıt nhâ´t môt oi . nghiêm . nguyên (u, v) v´ v 6= 0. , , , , `,i giai. Ch´ ˘. p ( x, y) sô´ tu. nhiên, m` Lo ung ta x´et tâ´t ca c´ ac ca a n´ o thoa √ √ ˘. p m˜ an −1 − 2 D ≤ x2 − Dy2 ≤ 1 + 2 D. Theo b` ai 9.4 c´ o vô sô´ ca , , , , , , , 2 ´ c trên, Nhung biêu thu ´ c x − Dy2 ˘ng thu sô´ tu. nhiên thoa m˜ an bâ´t d ¯a , , chı c´ o h˜uu han gi´ a tri. v`ı ch´ ung l` a c´ ac sô´ nguyên trong d ¯oan . (−1 − √. √ ,, 2 D, 1 + 2 D ). Ngh˜ıa l` a tô`n tai . môt . sô´ nguyên k, sao cho phuong , tr`ınh x2 − Dy2 = k c´ o vô han y). R˜ o r` ang k 6= 0, . nghiêm . tu. nhiên ( x, √ , , , , ˜ ˜ v`ı nê´u nguo. c lai d ¯ê´n mâu thuân do sô´ D không thê biêu . th`ı dân , x , ,√ ,´, , , ˘. p n` i dang di˜ên duo ay c´ o thê chon . h˜uu ty D = y . Gi˜ua nh˜ung ca . ´ıt , ˘. p kh´ nhâ´t hai ca ac nhau ( x1 , y1 ) v` a ( x2 , y2 ) m` a ch´ ung thoa m˜ an x1 ≡ x2 (mod |k |), y1 ≡ y2 (mod |k |). (9.10) , , , , , , ˘. p sô´ du theo Thât ı câ`n chı ra tâ´t ca c´ ac kha n˘ ang cua c´ ac ca . vây,ch . ,, , , ˘. p kh´ môd o sô´ luo. ng h˜uu han. o ngh˜ıa l` a tô`n tai ac ¯un |k | c´ . C´ . nh˜ung ca , , 2 2 2 nhau nh˜ung sô´ tu. nhiên ( x1 , y1 ) v` a ( x2 , y2 ), m` a x1 − Dy1 = x2 − , , , 2 ´ c (9.10). ˘ng thu Dy2 = k v` a thoa m˜ an da , , ´c ˘ng thu Ch´ ung ta x´et d ¯a √ √ ( x1 − y1 D )( x2 + y2 D ) = ( x1 x2 − y1 y2 D ) + ( x1 y2 − x2 y1 ). (9.11) , T`u (9.10) ch´ ung ta c´ o x1 x2 − y1 y2 D ≡ x12 − Dy21 ≡ k ≡ 0

(mod |k|),

x1 y2 − x2 y1 ≡ x1 y1 − x1 y1 ≡ 0 (mod |k |). ´ tô`n tai Khi d a v, sao cho x1 x2 − y1 y2 D = ku, x1 y2 − ¯o . sô´ nguyên u v` , x2 y1 = kv. V`ı vây o thê viê´t th` anh . (9.11) c´ √ √ √ ( x1 − y1 D )( x2 + y2 D ) = k(u + v D ).

, ,, ˜,ng u ´,ng dung Chuong 9. Nhu kh´ ac cua nguyên l´y Ðirichle . , , ´,c sau c` ´,i nhau trong c´ ˘ng thu Nhân t`ung sô´ hang ung ch´ ung ta ac d¯a . vo ,, nhân ¯uo. c . d 94

k2 = ( x12 − Dy21 )( x22 − Dy22 ) = k2 (u2 − Dv2 ),

, , ,, ˘. p (u, v) l` ´ suy ra u2 − Dv2 = 1, ngh˜ıa l` t`u d a ca a nghiêm ¯o . cua phuong tr`ınh. , , , ´,ng minh v 6= 0. Nê´u gia thiê´t ngu,o.,c lai, Chı c` on phai chu ung . ch´ ´ ta s˜e c´ o x1 x2 = y1 y2 D = |k | v` a x1 y2 = x2 y1 . Khi d ¯o

|k|y2 = |( x1 y2 ) x2 − y1 y22 D | = |y1 x22 − y1 y22 D | = y1 | x22 − Dy22 | = |k|y1 , , , , t`u d o kha n˘ ang khi y1 = y2 . Ðiê`u n` ay không thê ¯ây, v`ı k 6= 0 chı c´ , , xây ra v`ı ( x1 , y1 ) kh´ ac ( x2 , y2 ) , c` on t`u y1 = y2 suy ra x1 = x2 . , ˜,ng sô´ thu.,c v` ´ . 9.6. Cho x1 , x2 , . . . , xn l`a nhu a N l` a sô´ tu. nhiên. Khi d¯o , n ˜ ng sô´ nguyên p1 , p2 , . . . , pn , q sao cho 1 ≤ q ≤ N v` tô`n tai a . nhu xi − pi ≤ 1 , (9.12) q Nq , v´ oi moi . i = 1, 2, . . . , n. , ,, , `,i giai. Ch´ ´,ng minh b` ´,i Lo ung ta chu ai to´ an cho tru`ong ho. p n = 2. Vo ,, ,, , , ´,ng minh ho` ´,n ho,n 2 chu tru`ong ho. p n lo an to` an tuong tu. v` a d` anh , , , , ´ ˜ cho ban a x2 l` a nhung sô thu. c, c` on N l` a sô´ tu. ¯oc. . d . Nhu vây . cho x1 v` , `˘ng tô`n tai ´,ng minh ra nhiên. Ch´ ung ta s˜e chu nh˜ung sô´ nguyên p1 , p2 . , ´,i 1 ≤ q ≤ N 2 v` v` a q vo a thoa m˜ an x1 − p1 < 1 , x2 − p2 < 1 . (9.13) q Nq q Nq , ˘ng v` ˘. t pha Ch´ ung ta cô´ d a x´et h`ınh vuông Q hê. toa d ¯inh ¯ô. trong ma . . , , , ´i c´ vo ac d a (0, 1) nhu h`ınh v˜e. Chia Q ra N 2 ¯ınh (0, 0), (1, 0), (1, 1) v` , , 1 ` ,, `˘ng nhau vo ´,i canh ˘ng c´ ˘ng l` a ba ac d h`ınh vuông nho ba ¯u`ong tha . N ´,i truc ´,i N=7). song song vo ¯ô. (trong h`ınh v˜e ta chia vo . toa . d

J

, , 9.1. Xâ´p xı môt . sô´ thu. c

95

, ˘. p sô´ c´ Bây gi`o ch´ ung ta ch´ u ´y d¯ê´n ca o dang (qx1 − [qx1 ], qx2 − . ,, , [qx2 ]), o d¯ây q nhân a tri. nguyên 0, 1, 2, . . . , N 2 . V`ı 0 ≤ . nh˜ung gi´ , , , ˜ ca ˘. p nhu vây c´ qxi − [qxi ] < 1, i = 1, 2, . . . , N 2 môi o thê coi nhu , , ˜ sô´ `˘ng c´ ˘. p toa ´ môi môt ach d ¯ô. cua d ¯iêm trong h`ınh vuông Q. Ba ¯o . ca . d , , , , ´ ng trong h`ınh vuông Q , sô´ 0, 1, 2, . . . , N 2 tao ¯iêm tuong u . ra môt . d ,, , ,, ´ l` luo. ng c´ ac sô´ d a N 2 + 1. Nhung Q d ¯o ¯uo. c chia ra N 2 h`ınh vuông , nho, suy ra tô`n tai ac nhau q1 , q2 trong d ¯oan . hai sô´ nguyên kh´ . [0, N ], , , , ´, , ´ m` ad ung c´ o c´ ac toa ¯iêm tuong ung voi ch´ ¯ô. (q1 x1 − [q1 x1 ], q1 x2 − . d `˘m trong môt [q1 x2 ]), (q2 x1 − [q2 x1 ], q2 x2 − [q2 x2 ]) v`a c` ung na . h`ınh , , 1 ´i canh ´ c´ vuông nho vo . Ðiê`u d o ngh˜ıa l` a ¯o . N 1 |(q1 x1 − [q1 x1 ]) − (q1 x2 − [q1 x2 ])| ≤ , N 1 |(q2 x1 − [q2 x1 ]) − (q2 x2 − [q2 x2 ])| ≤ . (9.14) N , , , ˘t q = Không mâ´t t´ınh tông qu´ at c´ o thê gia thiê´t q1 > q2 . Nê´u d ¯a , . ´,c ˘ng thu q1 − q2 , p1 = [q1 x1 ] − [q2 x1 ], p2 = [q1 x2 ] − [q2 x2 ] th`ı bâ´t d ¯a (9.14) c´ o dang .

|qx1 − p1 | ≤

1 1 , |qx2 − p2 | ≤ . N N

(9.15)

`˘ng 1 ≤ q ≤ N 2 , v`ı 1 ≤ q2 < p1 ≤ N 2 . Chia hai vê´ Ch´ ung ta thâ´y ra , ,, cua (9.15) cho q ch´ ung ta nhân ¯uo. c (9.13). . d ,´, ´,ng ´,i môt ˜ chu Hai b` ai to´ an duo id t´ınh châ´t m` a ta d ¯ây liên quan to ¯a . , ,, ,, ,, , minh o chuong 2. Ðê mo rông t´ınh châ´t n` ay ch´ ung ta d ao d ¯ua v` ¯inh . . ngh˜ıa. , `˘ng Nê´u a1 , a2 , . . . , a2n+1 l` a 2n + 1 sô´ thu. c (n ≥ 1). Ch´ ung ta n´ oi ra , d˜ ay n` ay c´ o t´ınh châ´t P, nê´u bâ´t k`y 2n sô´ trong ch´ ung c´ o thê chia , , ˜ nh´ l` am hai nh´ om, môi om n sô´, sao cho tông cua c´ ac sô´ trong hai `˘ng nhau. nh´ om ba

J

96

, ,, ˜,ng u ´,ng dung Chuong 9. Nhu kh´ ac cua nguyên l´y Ðirichle .

, ˜,ng sô´ nguyên . 9.7. Chu´ng minh ra˘`ng moi ., bô. 2n + 1 sô´ gô`m nhu ,, duong c´ o t´ınh châ´t P, th`ı tâ´t ca c´ ac sô´ d¯ê`u ba ˘`ng nhau. , , ˜ sô´. `˘ng qui nap `,i giai. Chu ´,ng minh ba ´,n nhâ´t cua dây Lo theo sô´ lo . , , `˘ng 1 th`ı tâ´t ca c´ ´n nhâ´t ba Nê´u sô´ lo ac sô´ c` on lai ung d a 1, nên ¯ê`u l` . c˜ , , ,, , , ´ ` ´ ˜ ´ b` ai to´ an d ¯a giai. Bây gio gia su kêt luân ¯ung voi moi . d . bô. 2n+1 sô´ ,, , , ˜ sô´ không vuo. t qu´ nguyên duong m` a môi a k, k ≥ 2 v` a c´ o t´ınh châ´t P. , , ˜ sô´ Lâ´y p1 , p2 , . . . , p2n+1 l` a bô. sô´ nguyên duong c´ o t´ınh châ´t P v` a môi ,, , không vuo. t qu´ a k + 1. T`u d ai to´ an suy ra p1 , p2 , . . . , p2n+1 ¯iê`u kiên . b` , ˜˘n le. c´ o c` ung t´ınh cha ˜˘n, ch´ a) Nê´u p1 , p2 , . . . , p2n+1 l` a c´ ac sô´ cha ung ta x´et c´ ac sô´ p1 p2 p2n+1 , ,..., . Ch´ ung ta thâ´y ngay ch´ ung c˜ ung c´ o t´ınh châ´t 2 2 2 , , ´,i moi P. Ngo` ai ra moi a k v`ı pi ≤ k + 1 vo . sô´ không vuo. t qu´ . i = p2 p2n+1 p1 ` ˘ng qui nap 1, 2, . . . , 2n + 1. Ba . suy ra 2 = 2 = . . . = 2 .Ngh˜ıa l` a p1 = p2 = . . . = p2n+1 . , `˘ng qui ´,ng ba b) Nê´u p1 , p2 , . . . , p2n+1 l` a c´ ac sô´ le, ch´ ung ta chu nap ay p1 − 1, p2 − 1, . . . , p2n+1 − 1. Ch´ ung c˜ ung c´ o t´ınh châ´t . cho d˜ ,, ,, P v` a không vuo. t qu´ a k, ch´ ung ta nhân ¯uo. c p1 − 1 = p2 − 1 = . . . = . d p2n+1 − 1. Ngh˜ıa l` a p1 = p2 = . . . = p2n+1 .

J

, ˜,ng sô´ thu.,c du,o,ng . 9.8. Chu´ng minh ra˘`ng moi bô. 2n+1 sô´ gô`m nhu . , c´ o t´ınh châ´t P, th`ı tâ´t ca c´ ac sô´ d¯ê`u ba ˘`ng nhau. , , `˘ng nê´u c´ `,i giai. - Hiên nhiên ra Lo ac sô´ a1 , a2 , . . . , a2n+1 c´ o t´ınh châ´t , P v` a q l` a sô´ thu. c bâ´t k`y, th`ı c´ ac sô´ qa1 , qa2 , . . . , qa2n+1 c˜ ung c´ o t´ınh ´ chât P. , , ,, , ´,i t´ınh Bây gi`o cho x1 , x2 , . . . , x2n+1 l` a nh˜ung sô´ thu. c duong vo , , , , , ˘ng d châ´t P. Nê´u tâ´t ca c´ ac sô´ l` a h˜uu ty th`ı d b` ai to´ an ¯iê`u kha ¯inh . cua , , suy ra không kh´ o. Thât a bôi . vây, . k´y hiêu . q l` . sô´ chung nho nhâ´t cua

, , 9.1. Xâ´p xı môt . sô´ thu. c

97

, ˜ sô´ c´ ´ qx1 , qx2 , . . . , qx2n+1 l` mâu ac sô´ trên. Khi d a nh˜ung sô´ nguyên ¯o ,, ,´, ´,ng minh b` duong c´ o t´ınh châ´t P, theo c´ ach chu ai truo c ch´ ung ta c´ o , qx1 = qx2 = . . . = qx2n+1 . V`ı vây on . x1 = x2 = . . . = x2n+1 . Chı c` , , , , , ˜ ´ ng minh tru`ong ho. p c´ o trong dây x1 , x2 , . . . , x2n+1 môt phai chu . sô´ , , , , , vô ty. Nhung theo kê´t qua cua b` ai 9.6 tô`n tai a c´ ac . sô´ tu. nhiên q, v` , , , ´ sô nguyên duong p1 , p2 , . . . , p2n+1 thoa m˜ an

| xi −

pi |< q

1 1 1+ 2n+1

, i = 1, 2, . . . , n.

(9.16)

q ,`, , ´,i q > (2n)2n+1 v` Truong ho. p riêng, tô`n tai a . p1 , p2 , . . . , p2n+1 , q vo , 1 , i = 1, 2, . . . , n. ch´ ung thoa m˜ an |qxi − pi | ≤ 1 q 2n+1 , , ˘. t αi = qxi − pi , i = 1, 2, . . . , 2n + 1. Nhu phâ`n d ˘ng d Ða ¯â`u kha ¯inh . , ` ´ ´ ˘ng qx1 , qx2 , . . . , qx2n+1 c´ ra o t´ınh chât P. Ch´ ung ta s˜e chung minh ` ´ ˘ng p1 , p2 , . . . , p2n+1 c˜ ´, v´ı ra ung c´ o t´ınh chât P. Chon ¯o . 2n sô´ trong d , ´ du. nhu p1 , p2 , . . . , p2n . V`ı qx1 , qx2 , . . . , qx2n c´ o t´ınh chât P, không , , , ´ ´ ´ ˘ng thuc qx1 + qx2 + · · · + qxn = at ch´ ung ta lây d mât t´ınh tông qu´ ¯a ´ ( p1 + α1 ) + ( p2 + α2 ) + · · · + qxn+1 + qxn+2 + · · · + qx2n , khi d ¯o ( pn + αn ) = ( pn+1 + αn+1 ) + ( pn+2 + αn+2 ) + · · · + ( p2n + α2n ). Ch´ ung ta viê´t lai . p1 + p2 + · · · + pn − pn+1 − pn+2 − · · · − p2n =

= α n +1 + α + · · · + α2n − α1 − α2 − · · · − αn . (9.17) , n +2 , ´ ´ ´ ´ ˘ng thuc trên l` Vê bên tr´ ai d a sô nguyên nên sô α = αn+1 + ¯a , αn+2 + · · · + α2n − α1 − α2 − · · · − αn c˜ ung l` a sô´ nguyên. Nhung 1 ´ | α | ≤ | α n +1 | + | α n +2 | + · · · + | αi | ≤ , i = 1, 2, . . . , n. Do d ¯o 1 2n + 1 q 2n |α2n | + |α1 | + |α2 | + · · · + |αn | < < 1, v`ı q > (2n)2n+1 . Khi 1 q 2n+1

, ,, ˜,ng u ´,ng dung Chuong 9. Nhu kh´ ac cua nguyên l´y Ðirichle . , , ´,c tru,o ´,c d ˘ng thu ´ ta c´ ´ α = 0 v` o p1 + p2 + · · · + p n = d a t`u d ¯o ¯a ¯o , , ,, , ˘ng d¯inh pn+1 + pn+2 + · · · + p2n Nhu vây ung ta kha ¯uo. c nh˜ung . d . ch´ , sô´ nguyên p1 , p2 , . . . p2n+1 c´ o t´ınh châ´t P. Ch´ ung ta c´ o thê kê´t luân . ,, d ¯uo. c p1 = p2 = . . . = p2n+1 , ,, ´,ng minh d ˜ chu Thu. c châ´t d ung ta d ¯ê´n d ¯ây ch´ ¯a ¯uo. c : , , Nê´u p1 , p2 , . . . , p2n+1 , q l` a nh˜ung sô´ nguyên thoa m˜ an (9.16) v` a 2n + 1 q > (2n) th`ı p = p1 = p2 = . . . = p2n+1 . Suy ra (9.16) c´ o dang .

98

p | xi − | < q

1 q

1 1+ 2n+1

, i = 1, 2, . . . , 2n + 1

, , `˘ng tô`n tai ˘ng d Trong 9.6 ch´ ung ta kha ra ¯inh . . vô han . sô´, tu. nhiên q , , `˘ng thoa m˜ an (9.16). Không mâ´t t´ınh tông qu´ at ch´ ung ta gia thiê´t ra ,, , 2n + 1 ´ l` nh˜ung sô´ d a q1 < q2 < . . . < qk < . . ., o d . Trong ¯o ¯ây q1 > (2n) ,`, , , ´i moi truong ho. p n` ay vo . k tô`n tai . sô´ nguyên pk sao cho

| xi −

V`ı lim

1 1 1+ qk 2n+1

pk |< qk

1 1 1+ 2n+1

, i = 1, 2, . . . , 2n + 1.

(9.18)

qk

p , = 0 khi k tiê´n to´,i vô c` ung, t`u (9.18) suy ra lim k = qk

, ˜ d˜ ´,i vô c` ´,i moi xi khi k tiê´n to ung, vo ay . i = 1, 2, . . . , 2n + 1. Nhung môi , , , ´i han, sô´ thu. c không c´ o nhiê`u hon môt . gio . suy ra x1 = x2 = . . . = x2n+1 .

J

, ˜ sô´ c´ ˜, sô´, bao gi`o, . 9.9. Chu´ng minh ra˘`ng trong t´am sô´, môi o ba chu , ,, ,, ˜, sô´ v` c˜ ung chon o ba chu a d¯ê khi gh´ep lai . d¯uo. c hai sô´ c´ . ta d¯uo. c môt . sô´ chia hê´t cho 7. , , `,i giai. Khi mang t´ Lo am sô´ chia cho 7 th`ı thê n` ao c˜ ung c´ o hai sô´ c´ o , ,, , , ´ l` c` ung sô´ du (nguyên l´y Ðirichlê ). Gia su hai sô´ d a abc v` a αβγ. Hiên ¯o

9.2. B` ai tâp .

99

, ach chon o abc = 7m + r nhiên abcαβγ = 1000abc + αβγ. T`u c´ . ta c´ , , ´i 0 ≤ r < 7 v` v` a αβγ = 7n + r vo a m, n, r l` a c´ ac sô´ tu. nhiên. V`ı vây . abcαβγ = 1000(7m + r ) + (7n + r ) = 7(1000m + n) + 1001.r = 7(1000m + n + 143.r ). , , . 9.10. Cho a, b, c, d l`a c´ac sô´ nguyên. Chu´ng minh ra˘`ng t´ıch cua c´ac hiêu a c − b chia hê´t cho 12. . b − a, c − a, d − a, d − c, b − d v` , `,i giai. Câ`n chu ´,ng minh t´ıch: P = (b − a)(c − a)(d − a)(d − c)(d − Lo `˘ng môt b)(c − b) chia hê´t cho 12=4.3. Ch´ ung ta biê´t ra . sô´ nguyên , , bâ´t k`y khi chia cho 4 th`ı chı c´ o c´ ac sô´ du 0, 1, 2, 3. ,´, ´ c nêu c´ o hai sô´ khi chia cho 4 m` a Trong bô´n sô´ a, b, c, d cho truo , , c´ o c` ung sô´ du th`ı hiêu ung s˜e chia hê´t cho 4. Nê´u không c´ o . cua ch´ , , hai sô´ n` ao khi chia cho 4 cho c` ung sô´ du th`ı trong bô´n sô´ phai c´ o , , , , ˜˘n v` ˜˘n c˜ hai sô´ cha a hai sô´ le. V`ı hiêu ung nhu hiêu . cua hai sô´ cha . cua , ˜˘n nên P chia hê´t cho 4. hai sô´ le d a sô´ cha ¯ê`u l` ,, ˘. t kh´ Ma ac trong bô´n sô´ a, b, c, d luôn t`ım d ¯uo. c hai sô´ khi chia cho , , ´ hiêu 3 th`ı c´ o c` ung sô´ du (nguyên l´y Ðirichlê ). Do d ung chia ¯o . cua ch´ hê´t cho 3, suy ra P chia hê´t cho 3. T´ om lai . P chia hê´t cho 12=4.3.

J

J

` tâp 9.2. Bai . , , ´ tô`n tai . 9.11. Cho x l`a môt on n l` a môt ¯o . sô´ thu. c, c` . sô´ tu. nhiên. Khi d . , 1 p , nh˜ung sô´ nguyên p v` a q thoa m˜ an 1 ≤ q ≤ n v` a x − ≤ . q ( n + 1) q , , , . 9.12. Cho x1 , x2 , . . . , xm l`a nh˜ung sô´ thu. c v`a n l`a sô´ tu. nhiên. Khi , , ` ˘ng ´ tô`n tai d ¯o . nh˜ung sô´ nguyên p1 , p2 , . . . , pn , q không d¯ô`ng th`oi ba 0, sao cho ta c´ o qµ ≤ n(µ = 1, 2, . . . , m) v` a 1 | q1 x1 + q2 x2 + · · · + q m x m − p | ≤ . ( n + 1) m

100

, ,, ˜,ng u ´,ng dung Chuong 9. Nhu kh´ ac cua nguyên l´y Ðirichle .

`˘ng vo ´,ng minh ra ´,i moi ˘p sô´ nguyên p v` . 9.13. Chu ca a q > 0 ta d ¯ê`u √ . √. √ , 3− 2 p ´,c sau 2 − ≥ ˘ng thu c´ o bâ´t d . ¯a q q2 `˘ng vo ´,ng minh ra ´,i moi ˘. p sô´ nguyên p v` a q > 0 ta d . 9.14. Chu ¯ê`u . ca √ , 1 p ´,c sau 3 − ≥ √ ˘ng thu c´ o bâ´t d . ¯a q 3 3q2 , , ,, . 9.15. Cho m, n v`a s l`a nh˜ung sô´nguyên v`a α l`a nghiêm . cua phuong 2 `˘ng nê´u α l` ´,ng tr`ınh bâc a . hai mx + nx + s = 0, (m 6= 0). Chu minh ra , , p c , ,, ´i môi sô´ vô ty, th`ı tô`n tai an α − ≥ 2 vo . môt . sô´ duong c thoa m˜ . q q ˘. p sô´ nguyên p v` ca a q > 0.

, , CHUONG

10

´ ÐIRICHLÊ NGUYÊN LY CHO DIÊN . T´ICH

, ´ biêu nguyên l´ 10.1. Phat y Ðirichlê cho diên . t´ıch

, ,, , , ˘. t pha ˘ng, Trong chuong n` ay ch´ ung ta x´et nh˜ung tâp . ho. p trên ma , , , nh˜ung ph´ep to´ an trên c´ ac tâp ac ban o . ho. p nê´u c´ . chua quen biê´t c´ , ,, , ´i c´ thê xem o Phu. d ach. Ch´ ung ta quan tâm to ac kh´ ai niêm ¯´ınh cuô´i s´ . sau d ¯ây: , , ˘. n, khi tô`n tai ˘. t pha ˘ng goi Môt l` a bi. cha . tâp . ho. p trong ma . môt . , , . , , ` ´ a to` ´ h`ınh tr` on chu an bô. c´ ac d h o p d o . Khi không tô n t ai ¯iêm cua tâp ¯ . . . , , , ´ môt h` ınh tr` o n n` a o nhu trên th` ı t âp h o p d o g oi l` a t âp h o p không b i. ¯ . . . . . . , ,, , ` ˘. t pha ˘. n. V´ı du. nhu môt ˘. n c` ˘ng l` ac lôi l` a tâp on nua ma a cha ¯a gi´ . d . bi. cha , , , , ˜ ´ ´ ng minh d ˘. n. Dê d` tâp ang chu ac t´ınh chât sau ¯uo. c c´ ., ho. p không bi. cha , , ˘. n cua nh˜ung tâp . ho. p bi. cha , , , , ˘. n l` 1. Ho. p v` a giao cua h˜uu han nh˜ung tâp a môt . . bi. cha . tâp . bi. , ˘. n. Hiêu ˘. n l` ˘. n. cha a môt . cua hai tâp . bi. cha . tâp . bi. cha , , ˘. n l` ˘. n. Môt 2. Môt a môt . tâp . ho. p con cua môt . tâp . bi. cha . tâp . bi. cha . , , , ´ ˘ tâp h o p ch u a m ôt t âp h o p con không b i ch a n th` ı n´ o c˜ u ng không b i. . . . . . . . ˘. n. cha , , , , , ˘. t pha ˘ng, Môt a d¯iêm biên cua tâp ¯iêm P goi . d . l` . ho. p A trong ma

102

,, Chuong 10. Nguyên l´y Ðirichlê cho diên . t´ıch

, , ´,a nh˜u,ng d nê´u moi ınh tr` on tâm tai P c´ o chu A v` a ca ¯iêm thuôc . h` . . , , , , , , nh˜ung d A.Tâp ho. p tâ´t ca c´ ac d ¯iêm không thuôc ¯iêm biên cua A goi . . . , , ,`, l` a biên cua A v` a k´y hiêu a K ( A). V´ı du. biên cua h`ınh tr` on l` ad ¯uong . l` , , ´ ´i c` ˘. c biêt tr` on vo ung tâm v` a b´ an k´ınh. V´ı du. d a t´ınh chât biên cua ¯a . v` , , tâp . ho. p nhu sau: , , , , , 3. Cho tâp an bô. nh˜ung d o toa ¯iêm c´ ¯ô. h˜uu ty . ho. p A gô`m to` . d ´,i toa trong môt ac d ¯ô. c´ ¯ir nh (0, 0), (0, 1), (1, 0), (1, 1). . h`ınh vuông vo . d , `˘ng biên cua A l` D˜ê d` ang thâ´y ra a to` an bô. h`ınh vuông n´ oi trên. , , , ´,i moi ˘. t pha ˘ng d 4. Vo ac d o ¯iêm trong ma ¯ê`u c´ . tâp . ho. p A gô`m c´ , ´ c sau K (K ( A)) ⊂ K ( A). công thu , , , ˘. p tâp ˘. t pha ˘ng d 5. Moi an c´ ac ¯ê`u thoa m˜ . ca . ho. p bâ´t k`y trong ma , ´ c sau công thu K ( A ∪ B) ⊂ K ( A) ∪ K ( B) K ( A ∩ B) ⊂ K ( A) ∩ K ( B) K ( A\ B) ⊂ K ( A) ∪ K ( B) , , , , , , Môt ad ¯iêm P goi ¯iêm trong ¯iêm trong cua tâp . ,d . l` . ho. p A nh˜ung d `˘m tron ˘. t pha ˘ng, khi tô`n tai ma on tâm P m` a n´ o na . h`ınh tr` . trong A. , , , , T`u d ay thâ´y ngay l` a moi ¯inh ¯iêm trong cua tâp ¯ê`u . ngh˜ıa n` . d . ho. p A d , , , , , ´ ng: trong v´ı du. 3 o trên tâp thuôc oc lai không d¯u . A. Ðiê`u ngu . ho. p A , . . không c´ o môt ao. Ta thâ´y ngay ¯iêm trong n` . d , , , 6. Môt d A l` ad a chi khi n´ o không ¯iêm thuôc ¯iêm trong cua A khi v` . . , , , , , l` ad ay giai th´ıch tai ¯iêm biên cua A. Ðiê`u n` ¯iêm trong . sao nh˜ung d , ,`, ` ˘m trên d cua h`ınh tr` on không na on. ¯uong tr` , , , , ˘. c biêt, Ðê loai ac tâp ung ta d ao môt ai niêm ¯a ¯ua v` . tr`u c´ . ho. p d . ch´ . kh´ . , , , , ´p tâp ˘. c chung cho lo ˘. c biêt ˘. t pha ˘ng : Môt d trong ma ¯a ¯a . . ,ho. p không d , . , ` ˘. t, khi biên ˘. n c´ ˘t pha ˘ng goi tâp ac d a bê ma ¯iêm trong ma ., ho. p bi. cha . l` , ,. , ´ cua n´ o không chua d ¯iêm trong (cua biên). , ˘. c d ˘. t trong ma ˘. t pha ˘ng. V´ı du. c´ ac h`ınh tr` on hoa ac d a bê` ma ¯a gi´ ¯ê`u l`

, 10.1. Ph´ at biêu nguyên l´y Ðirichlê cho diên . t´ıch

103

, , `˘ng c´ ˘. t. Ba Nhung trong v´ı du. 3 tâp a bê` ma ach d` ung . ho. p A không l` , ` ` c´ ac d a c´ ac d ung ¯inh ¯inh . ngh˜ıa vê tâp . ngh˜ıa trong phân trên ch´ . ho. p v` , , , ´ ng minh d ta chu ¯uo. c , , ˘. t, nh˜ung tâp 7. Nê´u A v` a B l` a hai bê` ma a . ho. p A ∪ B, A ∩ B v` , , ˘. t trong ma ˘. t pha ˘ng. A\ B c˜ ung l` a nh˜ung bê` ma , ˘. t v` a A không c´ o chung d 8. Nê´u A, B v` a C l` a c´ ac bê` ma ¯iêm trong , ´,i B v` ´,i C, th`ı A không c´ ´,i B ∪ C. vo a vo od ¯iêm trong chung vo , , , , ˘ng, nhiê`u khi Môt ¯inh . l´y co ban trong h`ınh hoc ., trong nh˜ung d . pha ,, , o phô thông ch´ ung ta công nhân ¯ê`: . nhu môt . tiên d , , , , ,, ˘. t A nh˜ung d ˘. t pha ˘ng c´ 9. Moi o thê cho tuong ¯iêm trong ma . bê` ma , ´,ng vo ´,i môt u . sô´ thu. c không âm S( A) sao cho ´,i ∆ l` a) S(∆) = 1, vo a môt o canh l` a 1; . h`ınh vuông c´ . , ˘. t không c´ b) Nê´u A v` a B l` a hai bê` ma od ¯iêm trong chung, th`ı S ( A ∪ B ) = S ( A ) + S ( B ). , , ´, ,, ´,i c´ Ph´ep cho tuong u ng S vo ac t´ınh châ´t trên d ac d ¯uo. c x´ ¯inh . môt . ´ c´ ach duy nhât. , , ˘. t A bâ´t k`y, sô´ S( A) goi ˘. t Cho bê` ma l` a diên t´ıch cua A. Nh˜ung ma . . , ,, ,`, , ˜d d ac truong phô thông l` a h`ınh ch˜u nhât, ac, ¯a ¯uo. c x´et trong c´ . tam gi´ , ´i kh´ h`ınh tr` on,. . . v` a sô´ S( A) theo d ung vo ai niêm ¯inh . ngh˜ıa trên tr` . , , , , , ` ´ diên t´ ıch c ua c´ a c h` ınh n` a y. V o i c´ a ch tr u u tu o ng h´ o a kh´ a i ni êm di ên . . . . , , t´ıch ch´ ung ta d˜ê d` ang khao s´ at t´ınh châ´t vê` diên t´ ıch c ua c´ a c h` ınh. . , , ,, ´,ng minh d T`u a) v` a b) ch´ ung ta c´ o thê d˜ê d` ang chu ¯uo. c: , ˘. t v` 10. Nê´u A v` a B l` a nh˜ung bê` ma a A ⊂ B, th`ı S( A\ B) = S ( A ) − S ( B ). , ˘. t t`ung d 11. Nê´u A1 , A2 , . . . , An l` a c´ ac bê` ma o ¯ôi môt . không c´ , d ¯iêm trong chung, th`ı S( A1 ∪ A2 ∪ . . . ∪ An ) = S( A1 ) + S( A2 ) +

104

,, Chuong 10. Nguyên l´y Ðirichlê cho diên . t´ıch

· · · + S ( A n ). Nguyên l´ y Ðirichlê cho diên a môt ˘. t, c` on . bê` ma . t´ıch: Nê´u A l` A1 , A2 , . . . , An l` a c´ ac bê` ma ˘. t sao cho Ai ⊂ A(i = 1, 2, . . . , n) v` a S( A) < S( A1 ) + S( A2 ) + · · · + S( An ), th`ı ´ıt nhâ´t c´ o hai bê` ma ˘. t , trong sô´ c´ ac bê` ma ˘. t trên c´ o môt . d¯iêm trong chung. , , C˜ ung nhu nguyên l´y d ung ta c˜ ung c´ o thê thâ´y d ¯â`u tiên, ch´ ¯iê`u , , ,, , , , ´ ng minh d n` ay l` a hiên nhiên v` a chu o ¯uo. c. Thât . vây, . Gia su không c´ , , ˘. p n` ˘. t d ˜ cho c´ ´ theo ca ao trong nh˜ung ma od ¯a ¯iêm trong chung. Khi d ¯o , ˘ng d kha o S ( A1 ∪ A2 ∪ . . . ∪ A n ) = S ( A1 ) + S ( A2 ) + · · · + ¯inh . 11. ta c´ ˘. t kh´ S ( A n ) . Ma ac Ai ⊂ A(i = 1, 2, . . . , n) suy ra A1 ∪ A2 ∪ . . . ∪ An ⊂ , , ˘ng ´ c´ o S( A1 ∪ A2 ∪ . . . ∪ An ) ⊂ S( A). Thô´ng nhâ´t c´ ac bâ´t d A, t`u d ¯o ¯a ˜ to ´,c lai ´,i vô l´y. thu ung ta c´ o S( A) < S( A), dân . ch´ , `˘ng nguyên l´y trên bao tr` C´ o thê thâ´y ra um nguyên l´y Ðirichlê , , , cho nh˜ung tâp h˜uu han. Ngo` ai ra nguyên l´y Ðirichlê trên c´ o thê cu. . . , , , ,, ´,p bê` ma ˘. t thông dung thê h´ oa cho nh˜ung lo trong chuong tr`ınh phô . , , thông v` a c´ ac kh´ ai niêm ai, thê t´ıch c˜ ung c´ o c` ung t´ınh châ´t nhu ¯ô. d` . d , , ,, ´ ch´ kh´ ai niêm ung ta c´ o thê ph´ at biêu nguyên ¯o . diên . t´ıch o trên. Do d ,, ´n kh´ l´y Ðirichlê theo c´ ac phuong a ac nhau. , , ˜ ng d¯oan 12.Cho nhu ˘ng ∆1 , ∆2 , . . . , ∆n na ˘`m trong d¯oan a . tha . ∆ v` , , , , , ´ ´ıt nhâ´t c´ tông d ai cua ∆1 , ∆2 , . . . , ∆n l´ on hon d¯ô. d` ai cua ∆. Khi d¯o o ¯ô. d` , , , ˜ ng d¯oan hai trong sô´ nhu ˘ng ∆1 , ∆2 , . . . , ∆n c´ o d¯iêm chung. . tha , ˜,ng d¯a diên 13. Cho nhu P1 , P2 , . . . , Pn na ˘`m trong d¯a diên P v` a tông . . , , , , , , ´ ´ıt nhâ´t c´ thê t´ıch cua P1 , P2 , . . . , Pn l´ on hon thê t´ıch cua P. Khi d o hai ¯o , , ˜ ng d¯a diên trong sô´ nhu o d¯iêm chung. . P1 , P2 , . . . Pn c´ , , ,, ˜ ng cung c1 , c2 , . . . , cn na 14. Cho nhu ˘`m trên d¯u`ong tr` on c v` a tông , ,, , , ´ ´ıt nhâ´t c´ on c. Khi d¯o o d¯ô. d` ai cua c1 , c2 , . . . , cn l´ on hon d¯ô. d` ai d¯u`ong tr` , , ´ ˜ ng cung c1 , c2 , . . . cn c´ hai trong sô nhu od ¯iêm chung.

10.2. V´ı du.

105

, , Tâ´t ca c´ ac ph´ at biêu 12, 13, 14 ch´ ung ta d a nguyên l´y ¯ê`u goi . l` , , , , ´ ng minh d Ðirichlê v` a ban o thê chu ac nguyên l´y n` ay. ¯oc ¯uo. c c´ . d . c´

10.2. V´ı du. , , . 10.1. Cho M l`a môt ac lô`i v´ oi diên a chu vi P. Chu´ng . d¯a gi´ . t´ıch S v` minh ra ˘`ng , , ,, , , , S a) M c´ on v´ oi b´ an k´ınh l´ on hon ; o thê phu d¯uo. c môt . h`ınh tr` P , S b) B´ an k´ınh cua c´ ac h`ınh tr` on na ˘`m trong M không qu´ a . P , , , ˜ canh `,i giai. a) Ch´ Lo ung ta du. ng trên môi d ac M môt ¯a gi´ . . h`ınh ch˜u S , nhât . chiê`u cao h = P , nhu h`ınh 10.1.

M

M O

H

h= H`ınh 10.1:

C S P H`ınh 10.2:

, , , , , Nh˜ung h`ınh ch˜u nhât ay c´ o nh˜ung d ung; n´ oi ¯iêm chung gi˜ua ch´ . n` , ` ´ ˜ ˘m tron chung môt ınh chu nhât na ung ta c´ o . sô h` . không . trong M. Ch´ , , , , , ˜ thê t´ınh to´ an tông diên ac h`ınh chu nhât ay l` a S. Nhu vây . t´ıch cua c´ . n` , , , , , . , ` ˜ phân cua M bi. c´ ac h`ınh chu nhât o diên . phu phai c´ . t´ıch nho hon S. , , , `˘ng tô`n tai Ðiê`u n` ay chı ra ra ¯iêm O cua M không thuôc . môt . d .

106

,, Chuong 10. Nguyên l´y Ðirichlê cho diên . t´ıch

, , , , , ao ca. Nhu vây o ngh˜ıa l` a khoang c´ h`ınh ch˜u nhât ach t`u O d ¯ê´n . n` . c´ , , , S ´n ho,n h = . Ta lâ´y O l` c´ ac canh cua M phai lo a tâm h`ınh tr` on b´ an . P S `˘m tron k´ınh R > , h`ınh tr` on n` ay s˜e na . trong M. P `˘m trong M. Ch´ ung ta x´et b) Cho h`ınh tr` on C tâm O b´ an k´ınh R na , , , , , ´ gi´ c´ ac tam gi´ ac m` a hai d o l` a hai d ac, c` on ¯ınh cua n´ ¯ınh liên tiê´p cua tu , , , , , ´ ba l` d a tâm h`ınh tr` on O. Ðu`ong cao ha. t`u O xuô´ng c´ ac canh ¯ınh thu ., , , , , , ´n hon R. T`u d cua tam gi´ ac n` ay lo ac diên t´ ıch cua ¯ây suy ra tông c´ . , , S `˘ng S v` ch´ ung ba a không nho hon P.R. Suy ra R ≤ (H`ınh 10.2). P

J

, . 10.2. Trong không gian cho 30 vecto, kh´ac không. Chu´ng minh ra˘`ng , , , ˜,a ch´ ´ c´ trong sô´ d¯o o hai vecto m` a g´ oc giu ung nho hon 450 . , , , , `˘ng tâ´t ca `,i giai. C´ Lo o thê gia thiê´t ra , , v´ecto c´ o chung d ¯iêm d¯â`u O. Lâ´y OA A `˘ng 1 trên vecto, thu ´, nhâ´t. ´,i d vo ¯ôd` . ai ba , , , ´i Ch´ ung ta du. ng h`ınh n´ on d ¯ınh O vo 0 ,, , 0 J 45 truc a g´ oc o d a 45 . B` ai ¯ınh l` . OA, m` ,, O ´,ng minh nê´u ch´ ung to´ an s˜e d ¯uo. c chu , `˘ng ´ıt nhâ´t hai trong sô´ 30 ta chı ra ra ,, ,, h`ınh n´ on (d¯uo. c xây duong theo c´ ach , , , , ´ ng vo ´i 30 vecto d ˜ cho) c´ trên u od ¯a ¯iêm trong chung (h`ınh 10.3). H`ınh 10.3: ´,i tâm Ch´ ung ta x´et h`ınh câ`u S vo ˜ lâ`n du.,ng h`ınh n´ ´˘t ma ˘. t câ`u S môt O v` a b´ an k´ınh 1. Môi on ca . h`ınh , , , , `˘ng ´i diên vo a c´ o thê t´ınh to´ an d ung thâ´y ra ¯uo. c. Ta c˜ . t´ıch δ1 m` , , hai h`ınh n´ on c´ o d a chi khi nh˜ung phâ`n ¯iêm trong chung khi v` , , , ` ˘. t câ`u c˜ trên ma ung phai c´ o d ay v` a ¯iêm trong chung. T`u d ¯iêu n`

10.2. V´ı du.

107

, , , nguyên l´y Ðirichlê ch´ ung ta chı câ`n thiê´t kiêm tra tông diên . t´ıch , , , ` ` ` ´n hon diên ˘. t câu lo ˘. t câu (ba ˘ng 4π). cua 30 h`ınh trên ma . t´ıch ma p √ !   2+ 2 π Ch´ ung ta c´ o δ1 = 2π 1 − cos = 2π 1 − . Vây . 8 2 ! p p √ √ 14 2+ 2 2+ 2 ,, ,, , < 30.2π 1 − > 4π tuong d¯uong vo´i 2 2 15  2 1 167 ˘. c l` a < hoa . 2 225 , , . 10.3. Nê´u môt ˘. t A trong ma ˘. t pha ˘ng thoa m˜ an d¯iê`u kiên . bê` ma . , , S( A) > 1. th`ı n´ o luôn luôn chu´a ´ıt nhâ´t hai d¯iêm trong ( x1 , y1 ), ˜,ng sô´ nguyên. ( x2 , y2 ) m`a hiêu a y2 − y1 l` a nhu . x2 − x1 v`

J

, , ˜ d `,i giai. Qua môi ´,i Lo ¯iêm (m, n) vo y , toa d ô nguyên ch´ u ng ta k e c´ a c ¯ . . , , ,, ,, ´,ng v` ˘ng d¯u ˘ng d ad ¯u`ong tha ¯u`ong tha ngang (h`ınh 10.4). ,, ,´, Ch´ ung ta s˜e tao d i ¯uo. c luo . −3 −2 −1 0 1 2 3 x , , , ´i toa nh˜ung d ¯iêm vo . dô., nguyên. ,´, ˘. t pha ˘ng ra c´ Luoi nguyên chia ma ac ˜ ` ˘ng nhau, môi h`ınh h`ınh vuông ba `˘ng vuông c´ o diên a 1. Ch´ u ´y ra . t´ıch l` , H`ınh 10.4: ˜ h`ınh vuông c´ môi o thê xê dich ¯ê´n . d , , ´,i môt tr` ung vo h`ınh vuông kh´ ac, chı c´ o kh´ ac l` a toa d ¯ô. cua h`ınh vuông . . , ´,i chuyên d mo ung l` a c´ ac sô´ nguyên. ¯ê´n c˜ ,´, Ch´ ung ta chon i nguyên l` am gô´c cô´ . ,môt . h`ınh vuông trong luo , d ¯inh . rô`i dich . ,chuyên moi . h`ınh vuông vê` h`ınh vuông gô´c. Nhu vây ., , `˘m trong c´ ac h`ınh vuông kh´ ac nhau d¯ê`u chuyên nh˜ung phâ`n cua A na , , , ´ vê` h`ınh vuông gô´c (h`ınh v˜e). Tông cua diên ac phâ`n d ¯o . t´ıch cua c´

108

,, Chuong 10. Nguyên l´y Ðirichlê cho diên . t´ıch

, `˘ng diên ´,n ho,n 1. Suy ra ´ıt nhâ´t hai trong ba a suy ra lo . t´ıch cua A v` , `˘m trong h`ınh vuông dich sô´ c´ ac phâ`n na od ¯ê´n s˜e c´ ¯iêm trong chung ., d , , ´, ´,i hai ( x0 , y0 ). Trong tâp A ban d ng vo ¯â`u th`ı d ¯iêm ( x0 , y0 ) tuong u . , d ac nhau ( x1 , y1 ) v` a ( x2 , y2 ) m` a x1 − x0 , y1 − y0 , x2 − x0 , y2 − ¯iêm kh´ , ´ y0 l` a c´ ac sô nguyên. Nhu vây a y2 − y1 c˜ ung l` a sô´nguyên. . th`ı x2 − x1 v`

J

, , ˜,ng d¯iêm trong ma . 10.4. Cho A l`a tâp ho. p lô`i v` a bi. cha ˘. n nhu ˘. t . , , , ˜ ng d¯iêm thuôc pha ˘ng, c` on P1 , P2 , P3 , P4 , P5 l` a nhu A. Goi a tâp . Ai l` . , . , , , ´ ` ho. p nhân t u A sau m ôt ph´ e p t inh tiê n c´ a c d iê m theo vecto P P ( i = ¯ 1 i . . . , , ` 1, 2, 3, 4, 5). Chu´ng minh ra ˘ng ´ıt nhâ´t c´ o hai tâp ac . ho. p trong sô´ c´ , Ai (i = 1, 2, 3, 4, 5) c´ o d¯iêm chung. , `,i giai. Ch´ Lo ung ta chia ra hai , Q Qi ,`, , ˘. c truong ho. p A c´ o thê c´ o hoa , , c´ o thê không c´ od ¯iêm trong. Q00 , ,`, , 1. Truong ho. p A c´ od ¯iêm , 0 trong. K´y hiêu a tâp . A l` . ho. p, ,, , Pi P1 m` a n´ o nhân ¯uo. c t`u A, sau . d , khi t´ ac d ph´ep vi. tu. tâm ¯ông . H`ınh 10.5: , ´ ´ P1 v` a hê. sô vi. tu. 2. Khi d ¯o ´,c sau d ´ ng Ai ⊂ A0 (h`ınh 10.5). công thu ¯u , , Thât a Qi l` a anh cua Q qua ph´ep tinh . tiê´n theo . vây, . nê´u Q ∈ A v` , −−→ vecto P1 Pi (xem h`ınh v˜e), th`ı P1 Pi Qi Q l` a h`ınh b`ınh h` anh. K´y hiêu . , , ,, 00 00 ` Q l` a trung d iê m c ua P Q . R˜ o r` a ng Q ∈ A, v` ı Q v` a P l` a phâ n tu ¯ 1 i i , , , ˘. t kh´ cua A v` a A lô`i. Ma ac P1 Qi = 2P1 Q00 v` a v`ı A0 l` a anh cua A qua , , , 0 `˘m trong tâp ph´ep vi. tu. tâm P1 v` a hê. sô´ 2, nên d ¯iêm Qi na . ho. p A . , ˜ tâp trong c´ ˜ tâp `˘m trong A0 . Ðô`ng th`o,i môi Nhu vây ac tâp . môi . Ai na .

10.2. V´ı du.

109

, ´,i A v` ´ cua Ai d a suy ra S( Ai ) = S( A). Do d ¯ô`ng dang ¯o . vo S( A1 ) + S( A2 ) + S( A3 ) + S( A4 ) + S( A5 ) = 5S( A). , , 0 ô`ng dang vo ´,i A vo ´,i hê. sô´ 2. Suy ra Nhung tâp ¯ . ho. p A d .

(10.1)

S( A0 ) = 4S( A).

(10.2) , , ˘. t kh´ ac A c´ od a t`u (10.1) v` a Ma ¯iêm trong, v`ı thê´ S( A) > 0. Ngh˜ıa l` 0 (10.2) suy ra S( A1 ) + S( A2 ) + S( A3 ) + S( A4 ) + S( A5 ) > S( A ). , ,, ,, , , Nhu vây ai to´ an d ¯uo. c suy ra t`u nguyên l´y . trong tru`ong ho. p 1 b` Ðirichlê cho diên . t´ıch. , ,`, ,, , `˘m trên môt 2. Truong ho. p A không c´ od ¯iêm trong, A s˜e na ¯u`ong . ,d , , ,, `˘m trên môt ˘ng, do ˘ng. Thât tha ¯iêm na ¯u`ong tha . vây, . nê´u ´ıt nhâ´t ba d . d , , , , ´ a to` ´i d¯ınh l` t´ınh châ´t lô`i A chu an bô. h`ınh tam gi´ ac vo a c´ ac d ¯iêm , , ,, ˘. n trên môt n` ay, suy ra A c´ od ¯iêm trong. Nhung tâp ¯u`ong . lô`i,, bi. cha . d , , ,, ˘ng ch´ınh l` ˘ng trên du`ong tha ˘ng n` tha ad ay. Phâ`n c` on lai ¯oan . tha . l´y ,, , , , , , , , , luân tu. nhu phâ`n trên. Nhung trong tru`ong ho. p trên d ¯u`ong ., tuong , , ˘ng chı câ`n 3 d tha a nguyên l´y Ðirichlê vê` d ai. ¯iêm v` ¯ô. d` , , , . 10.5. (Ðinh a tâp ˘. t pha ˘ng . l´y Minkovski) Cho A l` . ho. p d¯iêm trong ma , c´ o t´ınh châ´t lô`i, bi. cha ˘. n v` a d¯ô´i xu´ng qua gô´c toa a S( A) > 4. Khi . d¯ô. v` , , , , , ˜ ng d¯iêm trong kh´ ´ A chu´a nhu d¯o ac d¯iêm gô´c v´ oi toa . d¯ô. nguyên.

J

, 1 , `,i giai. T´ Lo ac dung lên A ph´ep vi. tu. tâm l` a gô´c toa a hê. sô´ . ¯ô. v` . . d , ,, ,´2, ´,i A v` Nhu vây ung ta nhân a c´ o k´ıch thuo c ¯uo. c A0 , d ¯ô`ng dang . ch´ . d . vo , ,, 1 , , `˘ng nua k´ıch thuo ´c cua A. Suy ra S( A0 ) = S( A) > 1. Theo b` ai ba 4 , , 0 ´ a ´ıt nhâ´t hai d 10.13 suy ra A chu ac nhau ( x1 , y1 ) v` a ( x2 , y2 ), ¯iêm kh´ , ´,ng qua a y2 − y1 l` a nguyên. V`ı tâp m` a c´ ac hiêu ¯ô´i xu . x2 − x1 v` . h,o. p A d , , 0 ung vây. Ðiê`u do `˘ng d gô´c toa ¯iêm ¯ô. nên tâp ¯´ giai th´ıch ra . d . ho. p A c˜ . , 0 0 ung lô`i. Do do (− x1 , −y1 ) thuôc ¯´ trung d ¯iêm . A . V`ı A lô`i nên A c˜

110

,, Chuong 10. Nguyên l´y Ðirichlê cho diên . t´ıch

, , 0 a c´ o toa cua c´ ac d ut (− x1 , −y1 ) v` a ( x2 , y2 ) c˜ ung thuôc ¯ô. ¯iêm m´ . d . A v` , x2 − x1 y2 − y1 , , , ´i 2 th`ı d ( , ). Nhân toa ay vo ¯ô. n` ¯uo. c toa ¯ô. cua môt . d . d . 2 ,2 , , ,, d ¯iêm thuôc ¯uo. c . A. Nhu vây . ( x2 − x1 , y2 − y1 ) thuôc ., A. Ðiêm, ta t`ım d kh´ ac gô´c toa a c´ o toa a l` ad ¯ô. v` ¯ô. nguyên v` ¯iêm trong cua tâp . d . d . A.

J

. 10.6. Nê´u A l`a môt ˘. t, . bê` ma (10.3) , , ˜,ng bê` ma l` a nhu ˘. t v` a thoa m˜ an Ai ⊂ A(i = 1, 2, . . . , n), c` on k l` a sô´ tu. nhiên m` a A1 , A2 , . . . , A n .

k.S( A) < S( A1 ) + S( A2 ) + · · · + S( An ). (10.4) , ˜,ng bê` ma th`ı ´ıt nhâ´t k + 1 trong sô´ nhu ˘. t trên c´ o môt . d¯iêm trong chung. , ,`, , `˘ng qui nap,Tru `,i giai. Ch´ ´,ng minh ba ´ Lo ung ta chu ong ho. p k = 1 d ¯o . , ,, , , , , ´ ng minh o trên. Bây gi`o gia su b` ˜ chu l` a nguyên L´y Ðirichlê d ai to´ an ¯a , , ´ ng minh n´ ˜d ´ ng cho k, ch´ ´ ng cho k + 1. d ung ta phai chu o c˜ ung d ¯a ¯u ¯u , , ´c ˘ng thu a c´ o bâ´t d Ta cô´ d ¯inh ¯a . môt . k v`

( k + 1) S ( A ) < S ( A1 ) + S ( A2 ) + · · · + S ( A n ). (10.5) , , , , `˘ng vo ´,i (10.5) tô`n tai môt d ad Ch´ ung ta s˜e chı ra ra ¯iêm l` ¯iêm trong cua . . , , k + 2 tâp . ho. p cua (10.3). Do Aµ ⊂ A(µ = 1, 2, . . . , n), nên S( Aµ ) ≤ , , ´ suy ra S( A1 ) + S( A2 ) + · · · + S( An ) ≤ nS( A). T`u bâ´t S( A), t`u d ¯o , ´,c sau c` ˘ng thu d ung v` a (10.5) ch´ ung ta c´ o (k + 1)S( A) < nS( A) suy ¯a ra k + 1 < n. V`ı vây . n ≥ k + 2. (10.6) , ,, , ´ ´,ng ´ Tô`n tai d iê m chung cho ıt nhâ t k + 2 t âp h o p (10.3) d u o c u ¯ ¯ . . . . ch , , ´˘t `˘ng qui nap minh ba . theo n. T`u (10.4) suy ra (10.5), vây . ta phai ba , , d ¯â`u t`u k + 2. Nhu vây . viê´t lai . ( k + 1 ) S ( A ) < S ( A 1 ) + S ( A 2 ) + · · · + S ( A k +2 ).

(10.7)

10.2. V´ı du.

111

, , , , ´,ng minh tô`n tai Câ`n phai chu a n´ o l` ad ¯iêm trong cua A1 , A2 , . d¯iêm m` . . . , Ak+2 . Lâ´y A0µ = Aµ \ Ak+2 (µ = 1, 2, . . . , k + 1)

(10.8)

A00µ

(10.9)

= Aµ ∩ Ak+2 (µ = 1, 2, . . . , k + 1)

v` a A 0 = A \ A k +2

(10.10)

A00 = Ak+2 .

(10.11) , R˜ o r` ang A0µ ⊂ A0 v` a A0µ ⊂ A00 (µ = 1, 2, . . . , k + 1) . V`ı c´ o tâ´t ca k + 1 , , 0 ´,c trên suy ra tâp am thu . ho. p Aµ , t`u bao h`

(k + 1)S( A0 ) ≥ S( A10 ) + S( A20 ) + · · · + S( A0k+1 ). , ,, Nê´u lâ´y (10.7) tr`u d ung ta nhân ¯i (10.12) ch´ ¯uo. c . d

(10.12)

(k + 1)S( A00 ) < S( Ak+2 ) + S( A100 ) + S( A200 ) + · · · + S( A00k+1 ). (10.13) 0 00 0 00 V`ı S( A) − S( A ) = S( A ) v` a S( Aµ ) − S( Aµ ) = S( Aµ ) do (10.8), (10.11) nên t`u (10.13),(10.9) v` a (10.11) suy ra kS( Ak+2 ) < S( A1 ∩ Ak+2 ) + S( A2 ∩ Ak+2 ) + · · · + S( Ak+1 ∩ Ak+2 ). (10.14) , , ´ ´ ng cho k theo qui nap T`u (10.14) v` a gia thiêt d ¯u . suy ra A1 ∩ , Ak+2 , A2 ∩ Ak+2 , . . . , Ak+1 ∩ Ak+2 c´ o d ay ¯iêm trong chung, d ¯iê`u n` , , , c´ o ngh˜ıa l` a tâp od ¯iêm trong chung. Nhu vây . ho. p A1 , A2 , . . . , Ak+2 c´ . , , , , ´ ´i n = k + 2 t`u (10.5) suy ra ´ıt nhât k + 2 tâp vo o . ho. p t`u (10.3) c´ , d ¯iêm trong chung. , , , ´,i môt Bây gi`o ch´ ung ta gia thiê´t vo ra ., n ≥ k + 2 t`u (10.5) suy , , , ´ıt nhâ´t k + 2 tâp od a s˜e phai kê´t ¯iêm trong chung v` . ho. p t`u (10.3) c´ , `˘ng t`u luân . ra

(k + 1)S( A) < S( A1 ) + S( A2 ) + · · · + S( An ) + S( An+1 ). (10.15)

,, Chuong 10. Nguyên l´y Ðirichlê cho diên . t´ıch

112

, , ˜ A1 , A2 , . . . , An+1 c´ suy ra c´ o ´ıt nhâ´t k + 2 tâp od ¯iêm . ho. p trong dây ˘. t trong chung. Thât ung ta d ¯a . vây, . ch´ A0µ = Aµ \ An+1 , A00µ

= A µ ∩ A n +1 ,

(µ = 1, 2, . . . , n). (µ = 1, 2, . . . , n)

(10.16) (10.17)

v` a A 0 = A \ A n +1

(10.18)

A00 = An+1 .

(10.19)

V`ı A0µ ∪ A0µ = A0µ , A0µ ∩ A00µ = ∅(µ = 1, 2, . . . , n) v` a A0 ∪ A00 = A, A0 ∩ A00 = ∅ nên S( A0µ ) + S( A00µ ) = S( Aµ ), (µ = 1, 2, . . . , n)

(10.20)

v` a S( A0 ) + S( A00 ) = S( A),. (10.21) , , ´ ´ ng minh môt ´ ˘ ´ ng Ch´ ung ta s˜e chu trong c´ a c bâ t d a ng th u c sau l` a d ¯ ¯u .

(k + 1)S( A0 ) < S( A10 ) + S( A20 ) + · · · + S( A0n )

(10.22)

˘. c l` a hoa kS( A00 ) < S( A100 ) + S( A200 ) + · · · + S( A00n ). (10.23) ,, ,, , Thât ung ta s˜e c´ o ( k + 1) S ( A 0 ) ≥ . vây, . trong tru`ong ho. p nguo. c lai . ch´ S( A10 ) + S( A20 ) + · · · + S( A0n ) v` a kS( A00 ) ≥ S( A100 ) + S( A200 ) + · · · + ,, S( A00n ) Công a do (10.20), (10.21) ch´ ung ta nhân . hai vê´ lai . v` . d¯uo. c S( A0 ) + kS( A) ≥ S( A1 ) + S( A2 ) + · · · + S( An ). (10.24) , ´,i S( A00 ) v` Công a t`u (10.19), (10.21) ch´ ung ta nhân . hai vê´ (10.24) vo . ,, d u o c ¯ . ( k + 1) S ( A ) ≥ S ( A1 ) + S ( A2 ) + · · · + S ( A n ) , , ´,c ´,i (10.15). Do d ˘ng thu ´ trong hai bâ´t d Nhung d¯iê`u n` ay tr´ ai vo ¯o ¯a , ´ ng. (10.22) v` a (10.23) phai c´ o ´ıt nhâ´t môt ai d ¯u . c´

10.2. V´ı du.

113

, ,, , ´,i n t`u, (10.22) ´ ng. Theo gia thiê´t qui nap Gia su (10.22) d ¯u ¯ô´i vo . d , , ˜ A0 , A0 , . . . , A0 c´ suy ra ´ıt nhâ´t k + 2 tâp ¯iêm trong . ho. p trong dây n od 2 1 , ˜ `˘ng kê´t luân ´ ng cho dây chung. T`u (10.16) suy ra ra ung d ¯u . trên c˜ (10.3). , ,, , , ´,i k suy ra k + 1 ´ ng. T`u gia thiê´t qui nap Gia su (10.23) d ¯u ¯ô´i vo . d , , , 00 00 00 o diêm chung v` ´,i (10.17) chı tâp a c` ung vo ¯ . ho. p trong A1 , A2 , . ,. . , An c´ , , , , `˘ng tô`n tai ra ra môt d a n´ o l` ad ¯iêm m` ¯iêm trong cua k + 1 tâp . . . ho. p t`u , , (10.3) v` a ca cua An+1 . , , , Nhu vây ay A1 , A2 , . t`u, (10.15) suy ra k + 2 tâp . ho. p trong d˜ , ´i n + 1. T`u, ´ ng vo . . . , An+1 c´ od ¯iêm trong chung. Suy ra kê´t luân ¯u . d ,, , ´,i moi ´,i (10.5) tô`n phuong ph´ ap qui nap . bây ,gi`o lai . suy, ra, vo . n, vo , , tai a n´ o l` ad ¯iêm m` ¯iêm trong cua ´ıt nhâ´t k + 2 tâp . môt . d . ho. p trong (10.3). ,, ´,ng minh d ´. Trong chu Ch´ u y l´y d` ung phuong ph´ ap qui nap ¯inh . . , , , , , , ´c k sang k + 1 ta lai theo k. Nhung d ung phuong ph´ ap qui ¯i t`u buo . d` , , , , , ´ ng minh qui nap ´c n sang n + 1 ta ´i chu nap . theo n. Vo . theo n t`u buo ,´, ,´, ´ v` lai ung buoc k truoc d a n. ¯o . d`

J

. 10.7. Trong môt o canh l` a . h`ınh vuông c´ . , , , ,, 1 chu´a môt on, Tông d¯ô. d` ai cua . sô´ d¯u`ong tr` , ch´ ung l` a 10. Chu´ng minh ra ˘`ng tô`n tai . môt . , ,, ˜,ng d¯u`ong tha ˘ng, m` a n´ o ca ˘´t ´ıt nhâ´t 4 trong nhu ,, d¯u`ong tr` on n` ay. , `,i giai. (H`ınh 10.6) Ch´ Lo ung ta chon . môt . canh h`ınh vuông rô`i chiê´u vuông g´ oc c´ ac . ,, ´. d on xuô´ng canh d ¯u`ong tr` ¯o .

H`ınh 10.6:

114

,, Chuong 10. Nguyên l´y Ðirichlê cho diên . t´ıch

, ,, `˘ng h`ınh chiê´u cua môt D˜ê thâ´y ra on b´ an k´ınh R l` a môt . d¯u`ong tr` . , ˘ ˜ d od ai 2R. V`ı vây h`ınh vuông d ¯oan ¯ô. d` ¯a chon . thang c´ . trên canh . . s˜e , , 10 , , ´i tông d ˘ng chiê´u xuô´ng vo c´ o nh˜ung d a . ¯oan ¯ôd` . tha . ai l` π 10 , > 3 = 3.1, theo nguyên l´y Ðirichlê suy ra c´o môt Nhung . π , , , , ˘ng d ´ thuôc ˜ d ao d¯o ad ¯iêm M n` ¯iêm trong chung cua 4 d ¯oan ¯a . AB l` . tha , , , , , ´i AB ˘ng d ´d chiê´u xuô´ng. Khi d oc vo ¯o ¯u`ong tha ¯i qua d ¯iêm M vuông g´ ,`, ´ ˘t 4 d s˜e ca on. ¯uong tr`

J

,, , ,, , . 10.8. Môt on v´ oi b´ an k´ınh 1 d¯uo. c son sô´ cung na ˘`m trên d ¯u`ong tr` , . , , , , xanh. Tông d¯ô. d` ai cua c´ ac cung mâ`u xanh l´ on hon π. Chu´ng minh , ,, , ˜ ,`, on tô`n tai ra ˘`ng v´ oi môi d¯uong k´ınh d cua d¯u`ong tr` . dây cung song song , , , , , v´ oi d, m` a hai d¯â`u cua dây cung d¯uo. c son xanh. , `,i giai. (H`ınh 10.7) Không mâ´t Lo C , , A t´ınh tông qu´ at ch´ ung ta gia thiê´t , , c´ ac cung bôi son không c´ o d ¯iêm d ,`, chung. Cho d l` ad ¯uong k´ınh bâ´t k`y , , cua d¯u`ong tr` on k v` a c l` a môt . trong ,, ,, d0 ` c´ ac cung d ¯uo. c bôi mâu. Trên d ¯u`ong 0 ` ˘ng c´ tr` on k ta tao ach . ra cung c ba , , , ´ ´ ´ ng c qua d lây d ¯ôi xu ¯u`ong k´ınh d0 k A0 , ´i d. vuông g´ oc vo C0 H`ınh 10.7: D˜ê thâ´y cung c0 cho c` ung d ai ¯ô. d` , ,, ´,i cung c. Ta l` ´,i moi vo am nhu vây on lai ¯uo. c . vo . cung c` . th`ı trên k ,nhân . d , ´,n ho,n 2π. Nhu,ng d môt o tông d ai lo ai cua k l` a 2π, ¯ô. d` ¯ô. d` . hê. cung c´ , nhu vây theo nguyên l´y Ðirichlê c´ o ´ıt nhâ´t hai cung trong hê. n` ay c´ o , . d¯iêm chung.

10.2. V´ı du.

115

, ,, , , , ˜ gia thiê´t o trên nh˜ung cung dang ung ta d c t`ung d Nhung ch´ ¯a ¯ôi . , 0 ` ´ c˜ ´ ng cho c´ ´ không c´ od ung d ac cung c . Khi d ¯iêm chung, d ¯iêu d ¯o ¯u ¯o , , , , 0 ` ´ ´ ng ´i cung c2 , d tôn tai od ¯iêm chung A vo ¯ô i x u . môt . cung bôi son c1 c´ , , 0 ´i cung c2 qua d . R˜ ´i d c´ vo o r` ang dây cung d a song song vo o ¯i qua A v` ,, ,, ´ ´ ` ` t´ınh chât mong muôn v`ı môt on d ¯âu o c1 , c` ¯âu kia o c2 . . d , . 10.9. Trong h`ınh tr`on b´an k´ınh 1 ngu`o,i ta tô so,n môt . sô´ dây cung. , , , ´ ` ˘t nhiê`u nhâ´t k dây cung, th`ı Chu´ng minh ra ˘ng nê´u moi d¯u`ong k´ınh ca , , , . , , , ˜ tô son nho hon πk. tông d¯ô. d` ai cua tâ´t ca c´ ac dây cung d¯a , `,i giai. Lâ´y c l` ˜ cho v` Lo a môt on C d aγ ¯a . dây cung bâ´t k`y trong h`ınh tr` , 0 ´,ng l` a cung nho trong hai cung do c tao a cung d ¯ô´i xu . ra. K´y hiêu . γ l` , , ,, ´˘t dây ´,i γ qua tâm cua h`ınh tr` vo on. R˜ o r` ang môt ¯u`ong k´ınh cua C ca . d , `˘m trong γ v` cung c khi v` a chı khi c´ o hai d ut na a γ0 . ¯â`u m´ , ,, , , , , `˘ng tông d ´n ho,n hoa ˘. c ˜ ke lo Gia su ra ai nh˜ung dây cung d ¯ô. d` ¯a , , , , ` ´ ´n hon πk. ˘ng πk. Khi d ´ tông d ba ai tât ca c´ ac cung dang γ lo ¯o ¯ô. d` . , , , 0 ´ ng nhu vây C˜ ung d cho tông d ai c´ ac cung dang γ . Suy ra tông ¯u ¯ô. d` . . , , ´,n ho,n 2πk. V`ı d ˘. c γ l` d ai cua c´ ac cung dang γ0 hoa a lo ai cua ¯ô. d` ¯ô. d` . ,, ,, , d on C l` a 2π, t`u nguyên l´y Ðirichlê mo rông suy ra tô`n tai ¯u`ong tr` . . ´ıt , ` ´ ´ ˘m trên ´ıt nhât k + 1 cung c´ nhât môt a n´ o na o dang ¯iêm thuôc . d . C, m` . , ,, , , , 0 ´˘t ´ ` ` ˘. c γ . Nêu tu d γ hoa ay ta ke d o ca ¯iêm chung n` ¯uong k´ınh cua C,th`ı n´ , ´i d¯iê`u kiên ´ıt nhâ´t k + 1 dây cung d ˜ cho, d ay tr´ ai vo ¯a ¯iê`u n` ¯â`u . trong d , , ˘ng d ´ ng. b` ai. Vây ai to´ an l` ad ¯iê`u kha ¯inh ¯u . cua b` . d , , , , . 10.10. Môt tâp ho. p M l` a ho. p cua môt sô´ d¯oan tha ˘ng na ˘`m trong . . . . , , , , , ˜ a hai d¯iêm bâ´t k`y cua M khoang [0, 1]. Biê´t ra ˘`ng khoang c´ ach giu , , , ˜,ng d¯oan không ba ˘`ng 0,1. Chu´ng minh ra ˘`ng tông d¯ô. d` ai cua nhu . tao . ,, nên M không vuo. t qu´ a

J

J

a) 0, 55; b) 0, 5.

116

,, Chuong 10. Nguyên l´y Ðirichlê cho diên . t´ıch

, , , , , , `˘ng phan chu `,i giai. a) Chu ´,ng minh ba ´,ng. Gia su, tông S cua d Lo ¯ô. , , , , ´n hon 0,55. Ch´ ˘ng tao d` ai nh˜ung d ung ta x´et tâp ¯oan . tha . nên M lo . N , , , , `, , ˜ ` ´i 0,1. T`u, ˘ng c´ ˘ng cua M vo nhân ach công môi d tha ¯uo. c tu M ba ¯oan . d . . , , d cua b` ai to´ an suy ra M v` a N không c´ od ai ¯iê`u kiên ¯iêm chung. Ngo` . , , , , , ˜ ` ´ ´n hon 0,55. Dê thây hai tâp ˘ng S nên lo ra tông d ai cua N ba ¯ô. d` . ho. p , `˘m trong d M v` a N na V`ı 2.S > 2.0, 55 = 1, 1, t`u nguyên ¯oan . [0;1,1]. , ˜ to ´,i vô l´y. l´y Ðirichlê suy ra M v` a N c´ od ay dân ¯iêm chung, d ¯iê`u n` , ,, , , , ´, b) Gia su d ai cua tâ´t ca c´ acd ¯ô. d` ¯oan . trong  M lon hon 0,5. Ch´ ung 1 1 2 9 ta chia d anh 10 phâ`n 0, , , ,..., , 1 . K´y ¯oan . [0,1] th` 10  10 10  10 , i i+1 `˘m trong d¯oan hiêu a phâ`n cua M na , , i = 0, 1, . . . , 9 . Mi l` . 10 10 , , `˘ng c´ ˘ng tao ra Mi . Ba v` a Si l` a tông d ai c´ ac d a ch tinh ¯ô. d` ¯oan .  . tha  , 2 9 1 , tiê´n th´ıch ho. p ch´ ung ta chuyên moi , ,..., ,1 ¯oan . d . 10 10 10   , , 1 ´,i 0, ´,i i = 2, 3, 4 . . . , 9. V`ı to . K´y hiêu Mi0 l` a anh cua Mi vo . 10 , S = S1 + S2 + · · · + S10 > 0, 5 = 5.0, 1, t`u nguyên l´y Ðirichlê , ,, , 0 0 o diêm mo rông suy ra c´ o ´ıt nhâ´t 6 tâp ¯ . . ho. p trong M0 , M1 , . . . , M9 c´ 1 ´ trong 0, chung. Ðiê`u n` ay c´ o ngh˜ıa l` a môt ao d l` a kê´t ¯o . sô´ n` 10 , , , , , , , ´, qua cua 6 d ac nhau x1 , x2 , . . . , x6 cua M tr`u d ng ¯iêm kh´ ¯i tuong u , k k k , 2 6 , 1 ´ trong nh˜ung sô´ dang , , . . . , , o d¯ây k i l` a môt ao d ¯o . . sô´ n` 10 10 10 `˘ng ´ıt nhâ´t c´ 0, 1, 2, . . . , 9, i = 1, 2, . . . 6. D˜ê d` ang thâ´y ra o hai trong c´ ac , ´ ´ sô k1 , k2 , . . . , k6 l` a liên tiêp (´ ap dung nguyên l´y Ðirichlê). V´ı du. nhu . k1 k2 1 = x2 − , nên x2 − x1 = (k2 − k1 ) = , k2 = k1 + 1. V`ı x1 − 10 10 10 ˜ d d ay d ¯iê`u n` ¯ân ¯ê´n vô l´y.

J

10.3. B` ai tâp .

117

` tâp 10.3. Bai . , ˘. t . 10.11. Cho n l`a môt on A l` a môt . sô´ tu. nhiên bâ´t k`y, c` . , bê` ma , ˘. t pha ˘ng, m` ´ tô`n tai trong ma a s( A) > n. Khi d ¯o ¯iêm trong . n+1 d , kh´ ac nhau ( xi , yi ) (i = 1, 2, . . . , n + 1) cua A, m` a hiêu a . xi − x j v` , yi − y j (i, j = 1, 2, . . . , n + 1) l` a nh˜ung sô´ nguyên. , ˘. t A trong ma ˘. t pha ˘ng v` . 10.12. Nê´u vo´,i môt bê` ma a môt . . sô´ nguyên , ,, duong k thoa m˜ an S( A) < k, th`ı tô`n tai cho . tiê´n sao . môt . ph´ep tinh , , , , ´ ` ´ ´ ´,i anh cua A qua ph´ep tinh ay chua nhiêu nhât k − 1 d ¯iêm vo . tiên n` toa . d¯ô. nguyên. , ´,ng qua gô´c toa ˘. t . 10.13. Cho A l`a tâp ho. p lô`i, d ¯ô´i xu ¯ô. trong ma . . d , , , ´ ´ ´ a ´ıt nhât 2n + 1 ˘ng, n l` ´ A chu pha a sô tu. nhiên v` a S( A) > 4n. Khi d ¯o , , ´i toa d ac nhau vo ¯iêm trong kh´ ¯ô. nguyên. . d , ´,ng qua gô´c toa ˘. n, d . 10.14. Nê´u A l`a tâp d a ¯ô´i xu ¯ô. v` . ho. p lô`i, bi. cha , ,. , ` ´ a ´ıt nhâ´t 307 d ˘ng 615, th`ı A chu c´ o diên a khoang c´ ach ¯iêm m` . t´ıch ba , , , ` ´n hon hoa ˘. p trong ch´ ˘. c ba ˘ng 1. gi˜ua moi ung lo . ca , , , , , ˘. n nh˜ung d¯iêm cua ma ˘. t pha ˘ng. . 10.15. Cho F l`a tâp . ho. p lô`,i, bi. cha , , , , Ch´ ung ta goi cua tâp ho. p F l` a khoang c´ ach nho nhâ´t ¯ô. h˜uu hiêu . d . . , , ,, , ,, , ´,ng ˘ng song song o hai ph´ıa cua tâp gi˜ua hai d ¯u`ong tha . ho. p F. Chu , , `˘ng nê´u trong môt ´,i d ˘. n vo minh ra o ¯ô. h˜uu hiêu . tâp . ho,. p lô`i, bi. cha . d c´ , , , , , , ´ a môt ´n hon chu m` a tông d¯ô. h˜uu hiêu ung lo . sô´ tâp . ho. p con . cua ch´ , , , , ´ ˘ng, m` ˘t ´ıt nhâ´t k + 1 tâp kd, th`ı tô`n tai a n´ o ca oi ¯u`ong tha . d . ho. p con n´ trên.

118

,, Chuong 10. Nguyên l´y Ðirichlê cho diên . t´ıch

, , CHUONG

11

, , ´ TOAN HOC . TÔ HO .P

11.1. V´ı du. , . 11.1. (Ðê` thi To´an Olympic Quô´c tê´,1972) Cho tâp . ho. p gô`m 10 sô´ , , ˜, sô´. Chu´,ng minh ra ´ c´ c´ o hai chu ˘`ng tâp o ´ıt nhâ´t hai tâp . ho. p d¯o . ho. p con , ,, , ˜ ng phâ`n tu trong ch´ không giao nhau, m` a tông nhu ung ba ˘`ng nhau. , , , `,i giai. Nê´u c´ Lo o hai tâp giao nhau m` a tông trong ch´ ung . ho. p con , , ,, , ` ˘ng nhau th`ı ch´ ´ ba ung ta c´ o thê bo nh˜ung phâ`n tu chung d ¯i. Khi d ¯o , ,, , ˜ c` on lai a tông c´ ac phâ`n tu cua ch´ ung vân . hai tâp . không giao nhau v` ` ˘ng nhau. ba , , ˜ cua Ch´ ung ta t´ınh c´ o bao nhiêu tâp môt . ho. p con không rông . tâp . , ,, ,`, ,, , , , , ` ´ ˜ ´ ho. p c´ o muoi phân tu. Sô luo. ng nhung tâp . ho. p con chı chua 1 phâ`n ,, , , , , , 1 . Sô´ luong nh˜ ´,a 2 phâ`n tu, ˘. c l` tu c´ o 10 hoa a C10 ung tâp . . ho. p con chu , , , 2 . Sô´ lu,o,ng nh˜ 3 ,. . . Suy ra ´,a 3 phâ`n tu, l` l` a C10 a C10 ung tâp ho. p con chu . . , , , , tông sô´ luo. ng c´ ac tâp a . ho. p con l` 1 2 3 10 C10 + C10 + C10 + · · · + C10 = (1 + 1)10 = 1023 , , ´,n ho,n 99. Vây ˜ cho không lo Ðiê`u kiên ai ra l` a 10 sô´ d ¯a . tông cua . b` ,, , , ˜ tâp c´ ac sô´ trong môi a 99.10 = 990, nhu vây . ho. p con không vuo. t qu´ . , ,, , sô´ luo. ng nh˜ung tông kh´ ac nhau nhiê`u nhâ´t l` a 990. Theo nguyên l´y , , , `m 10 sô´ s˜e c´ Ðirichlê trong sô´ 1023 tâp ho. p con cua tâp o ´ıt . . ho. p gô , , ,, `˘ng nhau. nhâ´t hai tâp a tông c´ ac phâ`n tu trong ch´ ung phai ba . m`

J

, , ,, Chuong 11. To´ an hoc . tô ho. p

120

˜ b` `, 1 d¯ê´n 15, môi ´nh sô´ tu . 11.2. Cho 15 b`ai to´an tra˘´c nghiêm, d¯a ai . , , , , ´ng hoa chı c´ o hai kha n˘ ang tra l`oi : d¯u ˘. c sai. C´ o 1600 th´ı sinh tham gia , , , , ` ´ng 2 b` thi, nhung không c´ o ai tra l`oi d¯u ai liên nhau. Chu´ng minh ra ˘`ng , , , c´ o ´ıt nhâ´t hai th´ı sinh tra l`oi to` an bô. 15 b` ai hêt . nhu nhau. , , ,, ˜ th´ı sinh tu,o,ng u `,i giai. B` ´,ng vo ´,i môt Lo ai l` am cua môi ay 15 phâ`n tu, . d˜ , ˜ phâ`n tu,, hoa ˘. c l` ˘. c l` ´ ng) hoa môi a Ð (d a S (sai). Theo gia thiê´t không ¯u ,, ,, c´ o d˜ ay n` ao c´ o 2 phâ`n tu kê` nhau l` a Ð, Ð, nên sô´ tô´i d ac phâ`n tu ¯a c´ SS trong d˜ ay l` a 8. ˜ 0 ≤ k ≤ 8 th`ı sô´ c´ ´,i môi ´ ng k phâ`n Ta nhân ac d˜ ay c´ od ¯u . thâ´y vo ,, ,, , , ,, , 15−k +1 16−k ˜ môt tu Ð l` a Ck = Ck (boi v`ı môi ay nhu thê´ tuong d ¯uong . d˜ , , , ´,i c´ ˘. t k tâ´m b`ıa gi˜ua 15 − k quyên s´ vo ach d ach: c´ o tâ´t ca 16 − k vi. tr´ı ¯a , , , , ˘. t b`ıa, kê ca hai d ´). d ach d¯o ¯ê d ¯a ¯â`u cua chô`ng s´ Vây a sô´ c´ ac d˜ ay c´ o t´ınh châ´t nêu trên l` a . l` 8

∑ Ck16−k = 1 + 15 + 91 + 286 + 495 + 462 + 210 + 36 + 1

k =0

= 1597 < 1600 , , , , `˘ng phai c´ ´,ng to ra Ðiê`u n` ay chu o ´ıt nhâ´t 2 th´ı sinh c´ o c´ ac câu tra l`oi , hêt . nhu nhau. , ,, . 11.3. Trong môt h`ınh 9 canh d¯ê`u c´ o môt d¯ınh d¯uo. c tô mâ`u tra ˘´ng . . . , ,, , c` on c´ ac d¯ınh kh´ ac d¯uo. c tô d¯en. Chu´ng minh ra ˘`ng tô`n tai hai tam gi´ ac , , . , ˜ ` phân biêt an d¯a ˘ng (diên ˘ng nhau) m` a c´ ac d¯ınh cua môi tam . to` . t´ıch ba ,, ` gi´ ac d¯uo. c tô c` ung môt . mâu. , , , ,, ,, `,i giai. T`u, 9 d Lo ung c´ o5d ¯ınh d ¯uo. c tô mâ`u th`ı ´ıt nhâ´t c˜ ¯ınh d ¯uo. c tô , ,, ´˘ng. N˘ ´˘ng tao ´ l` c` ung môt a mâ`u tra am mâ`u tra anh ¯o . mâ`u. Gia su d . th` , 5.4.3 `˘ng ´˘ng. Ðê ´y ra C35 = = 10 tam gi´ac phân biêt ung môt . c` . mâ`u tra 2.3

11.1. V´ı du.

121

, 2kπ (k = 0, 1, . . . , 8) xung quanh tâm O cua 9 , , , , ,, h`ınh 9 canh d ac d ¯ê`u không anh huong d ¯ê´n tâp ¯ınh cua h`ınh 9 . . M c´ ˜ tam gi´ canh n` ay. V`ı thê´ sau khi quay môi ac trong 10 tam gi´ ac trên . 2kπ ,, môt oc (0 ≤ k ≤ 8) xung quanh O ta d¯uo. c 10.9 = 90 tam gi´ac . g´ 9 , , , , c´ od ung không thê phân biêt ¯ınh trong tâp . ho. p M. Tâ´t ca ch´ . v`ı sô´ , 2.8.7 , `˘ng C5 = tam gi´ ac kh´ ac nhau c´ od = ¯ınh trong tâp . ho. p M ba 3 2.3 78 < 90. , ,, ´˘ng ∆1 v` Nhu vây ac tra a ∆2 sau môt ai ¯uo. c hai tam gi´ . ta t`ım d . v` , , ´i c` ph´ep quay tr` ung vo ung môt ac ∆. Luu ´y l` a hai tam gi´ ac ∆1 . tam gi´ , , v` a ∆2 l` a phân biêt. ac nhu nhau không thê c´ o . Thât . thê´, hai tam gi´ ,, , d ac. ¯uo. c sau khi quay t`u môt . tam gi´ ˜ tam gi´ ´,i tam gi´ V`ı môi ac ∆1 v` a ∆2 s˜e tr` ung vo ac ∆ sau khi quay, , , ,, ` ` ˘ng môt nên tu tam gi´ ac ∆1 ba o thê nhân ¯uo. c tam . ph´ep quay ta c´ . d , ˘ng han gi´ ac ∆2 (cha ac ph´ep quay ∆1 → ∆, ∆ → ∆2 ). Vây a ∆2 . , c´ . ∆1 v` l` a hai tam gi´ ac phai t`ım. ph´ep quay môt oc . g´

J

, , , . 11.4. Trên ma˘. t pha˘ng c´o n h`ınh. Gia su, Si1 ...ik , l`a diên . t´ıch ,phâ`n , , giao cua c´ ac h`ınh thu´ i1 , . . . , ik , c` on S l` a diên ˘. t pha ˘ng bi. . t´,ıch phâ`n ma , ,, , , , ´ ´ ´. K´y hiêu phu boi c´ ac h`ınh d¯o Mk l` a tông tât ca c´ ac Si1 ...ik , tuc l` a tông . , , , , , ˜ cho. Chu´ng diên ac h`ınh c´ o thê c´ o l` a giao cua k h`ınh d¯a . t´ıch cua tâ´t ca c´ minh ra ˘`ng a) S = M1 − M2 + M3 − . . . − (−1)n+1 Mn , b) S ≥ M1 − M2 + M3 − . . . − (−1)n+1 Mn v´ oi m cha ˘˜n v` a , , S ≤ M1 − M2 + M3 − . . . − (−1)n+1 Mn v´ oi m le. , ,, , ,, n a sô´ c´ `,i giai. a) K´y hiêu Lo ach chon . Ck l` . k phâ`n tu t`u n phâ`n tu, ta ´,c Niuto,n ( x + y)n = ∑nk=0 Ckn x k yn−k . K´y hiêu c´ o nhi. thu a diên . Dm l` .

, , ,, Chuong 11. To´ an hoc . tô ho. p , , , ,, ˘ng bi. phu boi d ˘. t pha ˘ng d ˘. t pha ´ ng m h`ınh. Phâ`n ma ´ t´ıch phâ`n ma ¯u ¯o , , , ,, , ˜ ` ´. gôm nh˜ung manh, môi manh bi. phu boi m h`ınh x´ ac d ao d ¯inh ¯o . n` , ,, , ,, m ˜ ` Khi t´ınh Mk diên . , t´ıch môi manh nhu ,vây, . s˜e d¯uo. c t´ınh Ck lân, boi v`ı , , , m ´ t`u m h`ınh c´ o thê tao ¯o . d¯uo. c Ck giao cua k h`ınh. Do d 122

Mk = Ckk Dk + Ckk+1 Dk+1 + · · · + Ckn Dn Suy ra M1 − M2 + M3 − · · · + (−1)n+1 = C11 D1 + (C12 − C22 ) D2 + · · · + (C1n − C2n + C3n − · · · ) Dn = D1 + D2 + · · · + Dn ,, m = (−1 + ´,c Niuto,n C1m − C2m + 3m − · · · − (−1)m Cm boi v`ı do nhi. thu m ) + 1 = (1 − 1) m + 1 = 1 C1m − C2m + C3m − · · · − (−1)m Cm , `˘ng S = D1 + D2 + · · · + Dn . Cuô´i c` ung ch´ ung ta luu ´y ra , b) T`u phâ`n a) ta c´ o S − ( M1 − M2 + · · · + (−1)m+1 Mm ) =

= (−1)m+2 Mm+1 + (−1)m+3 Mm+2 + · · · + (−1)n+1 Mn n

=

∑ ((−1)m+2 Cmi +1 + · · · + (−1)n+1 Cni ) Di

i =1 , , ,´, ´ ´,ng minh ´ ta chı câ`n phai chu (Ta qui uo c nêu k > i th`ı Cki = 0. Do d ¯o `˘ng: ra m + n + 1 C i ≥ 0 vo i i i ´,i i ≤ n. Cm n +1 − Cm+2 + Cm+3 − . . . − (−1) , , , ´,c ( x + y)i = ( x + y)i−1 ( x + y) suy ra d ´,c ˘ng thu ˘ng thu T`u d ¯a ¯a −1 i −1 Cji = Cji− 1 + Cj .

´ Do d ¯o i i i m + n +1 i Cm Cn = +1 − Cm+2 + Cm+3 − . . . − (−1) i −1 i −1 i −1 i −1 = Cm + Cm +1 − Cm+2 + Cm+3 − . . . i −1 +(−1)m+n+1 Cni−−11 + (−1)m+n+1 Cni−1 = Cm ± Cni−1 , , , i − 1 `˘ng Cn = 0 vo ´i i ≤ n. Cuô´i c` ung chı câ`n luu ´y ra

J

11.1. V´ı du.

123

, ´o diên . 11.5. Môt ai a ˘`ng 1 c´ o 5 miê´ng v´ a, m` a diên . c´ . , t´ıch ba . t´ıch cua ,, , , ˜ miê´ng v´ môi a không nho hon 0,5. Chu´ng minh ra ˘`ng luôn t`ım d¯uo. c hai , , , miê´ng v´ a c´ o diên ung không nho hon 0,2. . t´ıch phâ`n chung cua ch´ , , , , `˘ng M, diên `,i giai. Gia su, diên ´ o ba Lo ai a ac . t´ıch c´ . t´ıch phâ`n giao cua c´ , , ` ´ i1 , . . . , ik ba ˘ng Si1 ...ik , c` miê´ng v´ a thu on Mk = ∑ Si1 ...ik . T`u b` ai trên suy , ,, ˘ng ra M − M1 + M2 − M3 + M4 − M5 ≥ 0, boi v`ı M ≥ S. C´ ac bâ´t d ¯a , ,, , , ´,c tu,o,ng tu., c´ ´o, m` thu o thê d ai a a c` on cho ¯uo. c viê´t không chı cho ca c´ , , , ´i c´ ´o vo t`ung miê´ng v´ a. Nê´u ta coi miê´ng v´ a S1 nhu môt ai a ac miê´ng . c´ ,, v´ a S12 , S13 , S14 , S15 th`ı ta d ¯uo. c S1 − ∑ S1i + ∑ S1ij − ∑ S1ijk + S12345 ≥ , , ,, ´,c nhu, vây ˘ng thu 0. Công c´ ac bâ´t d a, ta d ¯a ¯uo. c . . cho tâ´t ca 5 miê´ng v´ ,, M1 − 2M2 + 3M3 − 4M4 + 5M5 ≥ 0 (hang tu Si1 ...ik c´ o trong c´ ac . , , , , ´ ´ ´ ´ c cho c´ ˘ng thu bât d ac miêng v´ a i1 . . . ik , nên trong tông tât ca c´ ac ¯a , , , , ´ ´ ´ ´ c n´ ´ c 3( M − M1 + ˘ng thu ˘ng thu bât d o c´ o hê. sô k). Công c´ ac bât d ¯a ¯a . M2 − M3 + M4 − M5 ) ≥ 0 v` a M1 − 2M2 + 3M3 − 4M4 + 5M5 ≥ 0, , ,, ˘ng ´ bâ´t d ta d v` ao d¯o ¯uo. c 3M − 2M1 + M2 − M4 + 2M5 ≥ 0. Công ¯a . , ,, , ´,c M4 − 2M5 ≥ 0, (d ac Si1 i2 i3 i4 thu o trong tâ´t ca c´ ¯uo. c suy ra t`u S12345 c´ , , , ´ c l` ´,c l` tu a M4 ≥ 5M5 ≥ 2M5 ), ta d a ¯uo. c 3M − 2M1 + M2 ≥ 0, tu , ,, , , , ´ M2 ≥ 2M1 − 3M ≥ 5 − 3 = 2. Boi v`ı t`u 5 miêng v´ a c´ o thê tao ¯uo. c . d , , ´ ˘. p, nên diên ˘. p d ´ không nho 10 ca ac ca ¯o . t´ıch giao cua môt . trong sô c´ , M2 hon ≥ 0, 2. 10 , ,, , , , ˜ d¯iêm cua . 11.6. (Ðê` thi To´an vô d¯ich CHLB Ðu´c, 1979) Gia su môi . , ,, , ,, ma ˘. t pha ˘ng d¯uo. c tô ba ˘`ng 1 trong n mâ`u (n l` a sô´ tu. nhiên cho truo´c). , ,, , , ˜, nhât Chu´ng minh ra ˘`ng tô`n tai oi c´ ac d¯ınh d¯uo. c tô c` ung . môt . h`ınh chu . v´ môt . mâ`u.

J

, , `,i giai. Ta thiê´t lâp ˘. t pha ˘ng môt Lo ac vuông ¯ô. Ðê` c´ . trong ,ma . hê. toa. d , , ´i d ´i 1 ≤ i ≤ n + 1 v` g´ oc tu`y ´y v` a ch´ u ´y to a ¯iêm nguyên (i, j) vo

, , ,, Chuong 11. To´ an hoc . tô ho. p , , ˜ ´, tu., n + 1 d¯iêm trên môi 1 ≤ j ≤ nn+1 + 1. Viêc . tô mâ`u kê theo ,thu ˜ d ´ c´ d` ong c´ o nn+1 c´ ach kh´ ac nhau (môi o n c´ ach tô). Do d o ¯iêm c´ ¯o ,, n + 1 ´ ´ ` ´ ´ıt nhât l` a 2 d` ong trong sô n + 1 d`ong d¯uo. c tô mâu giông nhau , , ´˘c tô o,, d ´ ´, nhâ´t, d (ngh˜ıa l` a hai d` ong giông nhau vê` mâ`u sa ¯iêm thu ¯iêm , , , , ,, , ´, hai,. . . ,d ´, n + 1 cha ˘ng han thu t´ınh t`u tr´ ai qua phai). Gia su ¯iêm thu . , , ˜ ´,i môi hai d` ong â´y l` a d` ong nh˜ung d o toa. d a m; ngh˜ıa l` a vo ¯iêm c´ ¯ô. k v` , ,, `u. Boi v`ı i ∈ {1, 2, . . . , n} c´ ac d a (i, m) c´ o c` ung môt ¯iêm (i, k ) v` . mâ , , , chı c´ o n mâ`u nên trong sô´ n + 1 d ong c´ ac d o tung ¯iêm trên d` ¯iêm c´ , , , ´ ´ ` ˘t c´ ˘ng han, ´ l` d o ´ıt nhât hai d o c` ung mâu, cha d a2d ¯ô. k a ¯iêm c´ ¯o ¯iêm . , , ´,i c´ ( a, k) v`a (b, k). Khi d¯o´ h`ınh ch˜u nhât ac d ¯ınh ( a, k ), (b, k ), (b, m) . vo , v` a ( a, m) c´ o4d ung mâ`u. ¯ınh c` , , , . 11.7. Cho 1000 d¯iêm trên ma˘. t pha˘ng M1 , M2 , . . . , M1000 . Chu´ng ,, , ,, on b´ an k´ınh 1 n` ao ta d¯ê`u t`ım d¯uo. c môt minh ra ˘`ng trên bâ´t cu´ d¯u`ong tr` . , , , , `, S d¯ê´n c´ d¯iêm S sao cho tông khoang c´ ach tu ac d¯iêm M1 , M2 , . . . , M1000 , , không nho hon 1000. 124

J

, , , , , ,, `,i giai. Gia su, S1 v` on a S2 l` a hai d Lo ¯iêm tu`y ´y d ¯ô´i tâm cua d ¯u`ong tr` T b´ an k´ınh 1. Ta c´ o S1 S2 = 2, vây . S1 M1 + S2 M2 ≥ S1 S2 = 2; S1 M2 + S2 M2 ≥ 2;. . . ; S1 M1000 + S2 M1000 ≥ 2. V`ı thê´ (S1 M1 + S2 M1 ) + (S1 M2 + S2 M1 ) + · · · + (S1 M1000 + S2 M1000 ) = (S1 M1 + S1 M2 + · · · + S1 M1000 ) + (S2 M1 + S2 M2 + · · · + S2 M1000 ) ≥ 2000. , , ´˘c cha ´˘n thoa m˜ ´ ´ıt nhâ´t môt Do d¯o trong hai d a S2 cha an ¯iêm S1 v` . , , , , ´ ` b` ai ra l` a tông khoang c´ ach tu n´ od ac d ¯ên c´ ¯iêm M1 , M2 , . . . , M1000 , , không nho hon 1000. , ˜,ng . 11.8. Cho 37 d¯iêm phân biêt không gian c´ o toa. d¯o. l` a nhu . trong , , , sô´ nguyên sao cho không c´ o ba d¯iêm n` ao tha ˘ng h` ang. Chu´ng minh , , , , ra ˘`ng ta c´ o thê chon a toa. d¯ô. giao d¯iêm c´ ac trung tuyê´n cua ¯iêm m` . 3d tam gi´ ac tao anh d¯ê`u l` a sô´ nguyên. . th`

J

11.1. V´ı du.

125

, , , , ˜ d `,i giai. Vo ´,i môi Lo o toa. d a nh˜ung sô´ nguyên (d ¯iêm ¯iêm x; y; z c´ ¯ô. l` , , ´, , , , ¯ y; ¯ z¯ l` nguyên) ta cho tuong u ng c´ ac sô´ x; a nh˜ung sô´ du cua ph´ep chia c´ ac sô´ x; y; z cho 3. V`ı x¯ nhân a 3 gi´ a tri. nên ´ıt nhâ´t . không qu´ , , 13 trong sô´ 37 d o c` ung gi´ a tri. (nê´u không, thê´ th`ı sô´ c´ ac d ¯iêm c´ ¯iêm , , ´,n ho,n 12.3=36). Tu,o,ng tu.,, c´ không lo o không ´ıt hon 5 trong 13 d¯iêm , ´ c´ d o gi´ a tri. y¯ nhu nhau. ¯o , , , Thê´ th`ı giao d¯iêm c´ ac trung tuyê´n cua tam gi´ ac c´ od ¯ınh ( x1 , y1 , z1 ), ( x2 , y2 , z2 ) v`a ( x3 , y3 , z3 ) c´o toa. d¯ô. x1 + x2 + x3 y1 + y2 + y3 z1 + z2 + z3 ; y0 = ; z0 = 3 3 3 Nê´u x¯1 = x¯2 = x¯3 v` a y¯1 = y¯2 = y¯3 th`ı c´ ac sô´ x0 v` a y0 l` a nguyên c` on , , ´ sô z0 chı nguyên khi v` a chı khi z1 + z2 + z3 ≡ 0( mod 3). Trong , , ,`, , `˘ng nhau v` truong ho. p b` ai ra ta chon ra 5 d a tâ´t ca sô´ x¯ ba a tâ´t ¯iêm m` . , , , , , `˘ng nhau. Nê´u trong c´ ca sô´ y¯ ba ac d ay ta t`ım d a ¯iêm n` ¯uo. c ba d ¯iêm m` , , ´ ´ ¯ ´ c´ ac sô z nhân ac gi´ a tri. 0; 1; 2 th`ı voi c´ ac d o ¯iêm d ¯o ta c´ . c´ x0 =

z1 + z2 + z3 ≡ z¯1 + z¯2 + z¯3 ≡ 0 + 1 + 2 ≡ 0( mod 3) , , , ´,i 5 d ˜ chon Nê´u không c´ o nh˜ung d ay th`ı sô´ z¯ vo ¯iêm n` ¯iêm d ¯a . s˜e nhân . , ,, ¯ ´ không qu´ a 2 gi´ a tri, do d o ta t` ım d u o c 3 d iê m m` a gi´ a tr i z nh ân c` u ng ¯ ¯ . ¯ . . . , , ´ ´ ´ ´ môt gi´ a tr i m` a sô z u ng v o i n´ o l` a sô nguyên. 0 . . , , , . 11.9. Trên ma˘. t pha˘ng cho 7 d¯u`o,ng tha˘ng trong d¯o´ không c´o hai , ,, , ,, d¯u`ong tha ˘ng n` ao song song. Chu´ng minh ra ˘`ng ta t`ım d¯uo. c hai trong , , , ,, ˜,a ch´ 7 d¯u`ong tha ˘ng n´ oi trên m` a g´ oc giu ung nho hon 260 .

J

, , , ,, `,i giai. Ta h˜ ˘ng d ˜ cho sao cho ch´ Lo ay tinh ung ¯u`ong tha ¯a . , tiê´n tâ´t ca 7 d , , ,`, , ´ ˘ c` ung d oc ¯i qua môt ¯iêm cô d ¯inh ¯uo. c 7 d ¯uong thang chia g´ . O. Ta s˜e d . d , , ` ` ´ d â y d ınh O th` a nh 14 phâ n. V` ı thê m ôt trong c´ a c g´ o c t ao th` a nh nh o ¯ ¯ . . 0 , 5 , 360 ˜ hon = 250 < 260 (nê´u tâ´t ca c´ac g´oc d¯ê`u ba`˘ng nhau th`ı môi 14 7

, , ,, Chuong 11. To´ an hoc . tô ho. p , , ,, ´˘t nhau ´,n ho,n 260 ). Nhu,ng g´ ˘ng ca g´ oc không lo oc gi˜ua hai d ¯u`ong tha , , , , `˘ng g´ ˘ng ban d ba oc gi˜ua hai d ¯u`ong tha ¯â`u. 126

J

´ nguyên bâ´t . 11.10. Cho A l`a môt ac sô´ nguyên v` a x l` a môt . tâp . c´ . sô , , k`y. Tâp a tinh ac d ¯inh . tiê´n cua A, x´ . . ho. p x + A = { x + a| a ∈ A} goi . l` ,, ,, , ` ` bo i x. Hay n´ oi c´ ach kh´ ac tâp ac phân tu cua A công . ho. p x + A gôm c´ . , , , ` ´ ˜ ´ ˘ng môt thêm voi c` ung môt sô x. Ch´ ung ta n´ oi ra . tâp . , ho. p nhung sô´ , . nguyên l` a k-nho nhâ´t, nê´u tô`n tai a ch´ ung . tiê´n cua A, m` . k tâp . tinh , , , , ´ ` ˜ tung d o d¯iêm chung. V´ı du: a k-nho ¯ôi không c´ . moi . tâp . sô huu han . l` , ,, ´,i moi nhâ´t vo ung c´ o tâp a k-nho nhâ´t . sô´ nguyên duong k. C˜ . vô han . l` , , ´,ng minh nhu d˜ ay k + 1, (k + 1)2 , . . . , (k + 1)n , . . . Nhung ch´ ung ta chu kê´t luân . sau: , , , , ˜,ng sô´ nguyên không thê biêu ´ tâp Cho n l` a sô´ tu. nhiên. Khi d¯o ho. p nhu . , , , ˜ nhu, ho.,p cua n tâp diên . ho. p (n + 1)-nho nhâ´t. , , ,, ,, , , `,i giai. Ch´ Lo ung ta gia su nguo. c lai: tô`n tai nh˜ung tâp ho. p (n + 1). . . , ˜ sô´ nguyên thuôc nho nhâ´t A1 , A2 , . . . , An sao cho môi ôt trong . m , , . , , ´,i nh˜ung tâp ay. Theo d ¯inh . ngh˜ıa cua tâp . ho. p n` . (n + 1)-nho nhâ´t vo moi . i = 1, 2, . . . , n tô`n tai . n + 1 sô´ nguyên ai,1 , ai,2 , . . . , ai,n+1 sao cho , , ´i 1 ≤ r ≤ n + 1, 1 ≤ s ≤ n + 1 v` vo a r 6= s nh˜ung ph´ep tinh . tiê´n , , air + Ai v` a ais + Ai không c´ o d N l` a tâp ho. p ¯iêm chung. K´y hiêu . . , , ´˘p thu ´, tu., ( j1 , j2 , . . . , jn ) o, d tâ´t ca n-bô. sa ¯ây j1 , j2 , . . . , jn d ¯ôc . lâp . nhân . , , ˜ ´i môi n-bô. a = ( j1 , j2 , . . . , jn ) t`u N c´ ac gi´ a tri. 1, 2, . . . , n, n + 1. Vo ,, , , ´, , ´ ´i môt cho tuong ung vo sô nguyên x a = a1j1 + a2j2 + · · · + anjn . Boi . , , , , , ˜ gia thiê´t tâp v`ı ch´ ung ta d¯a a ho. p cua c´ ac . ho. p nh˜ung sô´ nguyên l` , ˜ ´ ´ trong d˜ tâp ôc ao d¯o ay . ho. p A1 , A2 , . . . , An , môi sô x a thu . môt . tâp . n` , ,, , , n ´ ` tâp ai ra sô phân tu cua N l` a (n + 1) (d a t´ınh tô ¯ây l` . ho. p trên. Ngo` ,, , , ho. p).T`u nguyên l´y Ðirichle mo rông ch´ ung ta c´ o ´ıt nhâ´t môt . . trong ( n + 1) n , , , ´ a nh˜ung sô´ dang ´i ´ıt nhâ´t c´ ac tâp x a vo . A1 , A2 , . . . , An chu . n

11.2. B` ai tâp .

127

, bô. a t`u N.

, , Không mâ´t t´ınh tông qu´ at ch´ ung ta gia thiê´t A1 c´ o t´ınh châ´t , , ,, trên. Ch´ ung ta x´et tâ´t ca c´ ac phâ`n tu α cua N m` a sô´ xα thuôc . A1 . n ,, ( n + 1) Ch´ ung gô`m ´ıt nhâ´t phâ`n tu. Nê´u α = ( j1 , j2 , . . . , jn ) l` a môt . n ,, , , , , ´ ng vo ´i (n − 1)-bô. phâ`n tu trong ch´ ung, th`ı ch´ ung ta cho tuong u , 0 ´˘p α = ( j2 , j3 , . . . , jn ). V`ı 1 ≤ jµ ≤ n + 1 vo ´i moi sa . µ = 2, 3,, . . . , n, , , , 0 ´i α c´ theo t´ınh châ´t tô ho. p vo o nhiê`u nhâ´t (n + 1)n−1 kha n˘ ang. n , ( n + 1) , ´,c ˘ng thu T`u bâ´t d > (n + 1)n−1 suy ra ´ıt nhâ´t c´o hai n-bô. ¯a n α = ( j1 , j2 , . . . , jn ), β = (k1 , k2 , . . . , k n ) ho` an to` an kh´ ac nhau, m` a , , ´, , ´i c` xα ∈ A1 , x β ∈ A1 , tuong ung vo ung môt . (n − 1)-bô. . , , ´˘t nga ´˘n gon Ch´ ung ta c´ o thê t´ om ta ac . nhu sau: tô`n tai . hai bô. kh´ nhau α = ( j1 , j2 , . . . , jn ) v` a β = (k1 , k2 , . . . , k n ) thuôc a xα ∈ . N, m` , A1 , x β ∈ A1 , j1 6= k1 v` a j2 = k2 , j3 = k3 , . . . , jn = k n . Nhung, xα = , a1j1 + a2j2 + · · · + anjn x β = a1k1 + a2k2 + · · · + ankn nhung v`ı jµ = k µ ˜ µ = 2, 3, . . . n ch´ ´,i môi ˘. c l` vo ung ta c´ o xα − x β = a1j1 − a1k1 hoa a ´ sô´ x = a1k1 + xα = a1j1 + x β thuôc a1k1 + xα = a1j1 + x β . Khi d ao ¯o . v` ,, , tâp a a1j1 + A1 . Suy ra tâp tinh tiê´n A1 x´ ac d boi ¯inh . . . ho. p a1k1 + A1 v` . , , ´˘t ´,i gia thiê´t khi ba ´ tr´ c´ ac sô´ a1k1 v` a a1j1 c´ od ai vo ¯iêm chung, d ¯iê`u d ¯o , ´ ng minh. d ¯â`u chu

J

` tâp 11.2. Bai . ˘. t ba d . 11.11. a) Trong h`ınh vuông diên ac diên ¯a ¯a gi´ . t´ıch 6 d . t´ıch , , , ` ´ ´ ng minh ra ˘ng trong sô d ´ luôn t`ım d 3. Chu ac m` a diên ¯o ¯uoc hai d ¯a gi´ . , , , . ` t´ıch phân chung cua ch´ ung không nho hon 1. ´,ng ˘. t 9 d b) Trong h`ınh vuông diên ac diên ¯a ¯a gi´ . t´ıch 5 d . t´ıch 1. Chu ,, `˘ng trong sô´ d ´ luôn t`ım d minh ra ac m` a diên ¯o ¯uo. c hai d¯a gi´ . t´ıch phâ`n

128

, , ,, Chuong 11. To´ an hoc . tô ho. p

, , , 1 chung cua ch´ ung không nho hon . 9 , , , ,, ´ . 11.12. S´au d¯iêm d¯uo. c sa˘p xê´p trên ma˘. t pha˘ng sao cho ba d¯iêm , , bâ´t k`y l` ad ac m` a c´ ac canh c´ od ai kh´ ac nhau. ¯ınh cua môt ¯ô. d` . tam, gi´ . , , ` ´ ´ ng minh ra ˘ng canh Chu ac tam gi´ ac d ¯ô`ng . , nho nhât cua môt . trong c´ , , ´n nhâ´t cua môt th`oi l` a canh lo ac kh´ ac. . . tam gi´ , , . 11.13. Cho P1 , P2 , . . . , P7 l`a bây d¯iêm trong không gian, trong d¯o´ , , ˜ d ˘ng. Tô mâ`u môi không c´ o bô´n d ao d ¯iêm n` ¯ô`ng pha ¯oan . Pi Pj (i < j) , , , `˘ng c´ ´ ng minh ra ´i môt ˘. c d vo o hai tam ¯o hoa ¯en. Chu . trong hai mâ`u d , ´ ˘c không c´ gi´ ac d o chung canh. ¯o n s a . , `˘ng nhau, . 11.14. C´o hai d¯˜ıa d¯ê`u d¯uo.,c chia th`anh 1998 h`ınh quat . ba ˜ d `˘ng môt v` a trên môi ach bâ´t k`y (ba ¯˜ıa tô môt . c´ . mâ`u) 200 h`ınh quat. . ,, , ˜ ˘. t chô`ng lên nhau v` ˜ C´ ac d a quay môt d ıa theo nh u ng g´ oc ¯˜ıa d ¯uo. c d ¯a ¯ . 0 , 360 `˘ng tô`n tai ´,ng minh ra l` a bôi o không . cua 1998 . Chu . ´ıt nhâ´t 94 vi. tr´ı c´ ,, , qu´ a 20 h`ınh quat ung nhau. ¯uo. c son tr` . d , , , , . 11.15. Trên ma˘. t pha˘ng cho n d¯u`o,ng tha˘ng t`ung d¯ôi không song , , ,, `˘ng g´ ´,ng minh ra ´,i nhau. Chu ˘ng n` ´ song vo oc gi˜ua hai d ao d¯o ¯u`ong tha 1800 , , ´n hon ´ không lo . trong sô´ d ¯o n

, , CHUONG

12

´ ` TÂP H`INH HOC KHAC ´ MÔT . . SÔ BAI .

12.1. V´ı du.

, , ˜, nhât . 12.1. Trong h`ınh chu ˘. t 6 d¯iêm. Chu´ng minh ra ˘`ng trong . , 3x4 d¯a , , , , , ˜ a ch´ ´ luôn t`ım d sô´ d¯o o khoang c´ ach giu ung không l´ on ¯uo. c hai d¯iêm c´ √ , hon 5. , `,i giai. Chia h`ınh ch˜u, nhât Lo am . ra l` , 5 h`ınh nhu h`ınh 12.1. Trong môt . ´ s˜e c´ trong sô´ c´ ac h`ınh d o ´ıt nhâ´t ¯o , , , 2 d a khoang c´ ach gi˜ua hai ¯iêm, v` √ , ´,n ho,n 5. ´ s˜e không lo d ¯iêm d ¯o H`ınh 12.1: , , , . 12.2. Trên ma˘. t pha˘ng c´o 25 d¯iêm, biê´t ra˘`ng trong 3 d¯iêm bâ´t k`y , , , , ´ luôn c´ trong sô´ d o 2 d¯iêm c´ ach nhau nho hon 1. Chu´ng minh ra ˘`ng ¯o , , , ˜ cho. tô`n tai on b´ an k´ınh 1 chu´a không ´ıt hon 13 d ¯iêm d¯a . h`ınh tr`

J

, , , , , , `,i giai. Gia su, A l` ˜ cho. Nê´u tâ´t ca c´ a môt ac d on lai Lo ¯iêm d ¯a ¯iêm c` . d ., `˘m trong h`ınh tr` na on S1 tâm A b´ an kinh 1 th`ı ta không câ`n phai , , , `˘m ´,ng minh g`ı thêm. Gia su, c´ ˜ cho na chu o môt ¯iêm B trong sô´ d ¯a . d ,`, , ´ c l` ngo` ai d on S1 tu a AB > 1. X´et h`ınh tr` on S2 tâm B b´ an k´ınh ¯uong tr` , , ´ C l` ˜ cho bâ´t k`y luôn 1. Trong sô´ c´ ac d ad ¯iêm A, B, C trong d ¯o ¯iêm d ¯a , , , , , , ˜, ´ không thê l` c´ o hai d ach nhau nho hon 1, hon nua d a2d ¯iêm c´ ¯o ¯iêm

130

,, Chuong 12. Môt ai tâp ac . sô´ b` . h`ınh hoc . kh´

, , ´,a tâ´t ca c´ ´ c´ ˜ cho, A v` a B. Do d ac h`ınh tr` on S1 v` a S2 chu ac d ¯o ¯iêm d ¯a , , , , ´ c l` ´ a không ´ıt hon 13 d ´ chu ˜ cho. tu a môt on d ¯o ¯iêm d ¯a . trong hai h`ınh tr`

J

, , tha ˘ng c´ od ai . 12.3. Bên trong d¯u`o,ng tr`on b´an k´ınh n d¯a˘. t 4n d¯oan ¯ô. d` . , , , , , , ` ˘ng song song hoa ˘. c vuông 1. Chu´ng minh ra ˘ng c´ o thê ke môt . d¯u`ong tha , , ,´, , ,`, ´ ˜ cho. g´ oc v´ oi d ˘ng l cho truoc v` a ca ˘t ´ıt nhâ´t 2 d¯oan ˘ng d¯a ¯uong tha . tha , , , , ,, `,i giai. Gia su, l1 l` ´,i l. K´y hiêu ˘ng bâ´t k`y vuông g´ oc vo Lo ad ¯u`ong tha , , . , , ,`, ´ ˘ ˘ d ai c´ ac h`ınh chiê´u cua d¯oan th a ng th u i lên c´ a c d u o ng th a ng l ¯ô. d` ¯ . , , ,, , , ´, ˜ `˘ng ˘ng ba v` a l1 l` a ai v` a bi tuong ung. Boi v`ı d ai cua môi d ¯ô. d` ¯oan . tha ´ ( a1 + · · · + a4n ) + (b1 + · · · + b4n ) ≥ 4n. 1, nên ai + bi ≥ 1. Do d ¯o , , ,, Không mâ´t t´ınh tông qu´ at gia su ( a1 + · · · + a4n ) ≥ (b1 + · · · + b4n ). , , ,, ˘ng d ´ a1 + · · · + a4n ≥ 2n. Tâ´t ca c´ ˜ cho d Khi d ac d tha ¯o ¯oan ¯a ¯ê`u d ¯uo. c . , ,, `˘m trong ˘ng c´ chiê´u xuô´ng d od ai 2n, boi v`ı ch´ ung d¯ê`u na ¯oan ¯ô. d` . tha , , ,, , ˘ng d on b´ an k´ınh n. Nê´u nhu c´ ac h`ınh chiê´u cua c´ ac d tha ¯u`ong tr` ¯oan . , , , ,, ˘ng l không c´ ˘ng ˜ cho lên d d od o bâ´t d ¯a ¯u`ong tha ¯iêm chung, th`ı s˜e c´ ¯a , , , ´,c a1 + · · · + a4n < 2n. Do d ´ trên l phai c´ thu o môt d¯iêm bi. c´ ac d ¯o ¯iêm . , , ,, ˘ng d ˜ cho chiê´u lên d ´. Ðu`ong cua ´ıt nhâ´t hai trong sô´ c´ ac d tha ¯oan ¯a ¯o . , , ´˘t ´ıt nhâ´t hai d ´,i l tai ˘ng d ´ s˜e ca ˜ cho. vuông g´ oc vo ¯iêm d ¯o ¯oan ¯a . d . tha

J

, , . 12.4. Trên doan tha ˘ng c´ od ai 1 ta tô môt sô´ d¯oan tha ˘ng sao ¯ô. d` . . . , , ,, , ˜,a hai d¯iêm d cho khoang c´ ach giu ˘`ng 0,1. Chu´ng ¯uo. c tô bâ´t k`y không ba , , ,, , , minh ra ˘`ng tông d¯ô. d` ai c´ ac doan ˘ng d¯uo. c tô không l´ on hon 0,5. . tha , , , `,i giai. Chia d ˘ng ra l` ˘ng c´ ˘. t Lo am 10 d od ai 0,1, d ¯oan ¯oan ¯ô. d` ¯a . tha . tha , , ˘ng nhu vây. ch´ ung theo môt côt v` a chiê´u ch´ ung xuô´ng môt ¯oan . d . tha . , . . , ,, , , , ` ´ ˘ Boi v`ı khoang c´ ach gi˜ua hai d iê m d u o c tô bâ t k` y không b a ng 0,1, ¯ ¯ . , , , , ,, ˘ng canh nên c´ ac d iê m d u o c tô c ua c´ a c d o an th a nhau không thê c` ung ¯ ¯ . ¯ . . , , , ´ không c´ chiê´u xuô´ng 1 d od ao c´ o thê l` a h`ınh chiê´u ¯iêm. Do d ¯o ¯iêm n`

12.1. V´ı du.

131

, , , , , ,, , ˘ng. Suy ra tông cua c´ ac d tha ¯iêm d ¯uo. c tô cua nhiê`u hon 5 d ¯oan . , , ,, ´,n ho,n ˘ng d d ai c´ ac h`ınh chiê´u cua c´ ac d ¯ô. d` ¯oan ¯uo. c tô không lo . tha 5.0, 1 = 0, 5.

J

, , ,, ˘ng l na ˘`m trong ma ˘. t . 12.5. Chu´ng minh ra˘`ng nê´u môt d¯u`ong tha . , , , , ´, pha ˘ng cua tam gi´ ac ABC v` a không d¯i qua d¯ınh n` ao cua tam gi´ ac d¯o , ´ ˜ cho. th`ı n´ o ca ˘t không qu´ a hai canh cua tam gi´ ac d¯a . , , , ,, `,i giai. K´y hiêu ˘. t pha ˘ng do l chia ma ˘. t pha ˘ng Lo α v` a α l` a hai nua ma . , , ,, ˜ d `˘m trong môt ˘. t cua tam gi´ ac ABC. Môi a C na nua ma ¯ınh A, B v` . , ,, ˘ng trên. Theo nguyên l´y Ðirichlê ´ıt nhâ´t môt ˘. t pha trong hai nua ma , , , . , , , ´ a hai d ˘ng trên, cha ˘ng han ac ABC, pha ¯ınh cua tam gi´ . nhu α, chu , , , , ,`, ´ ˘ng han ˘t d ˘ng ˘ng l không ca ´d tha cha a B. Khi d¯o ¯oan ¯uong tha . . nhu A v` , ´˘t môt AB, ngh˜ıa l` a n´ o không ca ac ABC. . trong ba canh . cua tam gi´

J

, , ,, , ` ˜,ng d¯iêm trong ma ˘ng môt . 12.6. Nhu ˘. t pha ˘ng d¯uo. c son ba . trong ba , , , , mâ`u. Chu´ng minh ra ˘`ng luôn t`ım d¯uo. c hai d¯iêm c` ung mâ`u c´ ach nhau ´ng ba d¯u ˘`ng 1. , , , , ,, `˘ng c´ `,i giai. Gia su, hai d Lo ach nhau 1 d ac ¯iêm bâ´t k`y c´ ¯ê`u d ¯uo. c tô ba , ` ` ´ ˘ mâ`u kh´ ac nhau. X´et tam gi´ ac d ê u ABC c´ o c anh b a ng 1. Tâ t c a c´ ac ¯ . , , , ,, , ,, `˘ng c´ d od ac mâ`u kh´ ac nhau. Gia su d ¯ınh cua n´ ¯uo. c tô ba ¯iêm A1 d ¯ô´i , , ,, , , , , ´ ng vo ´i A qua d ˘ng BC. Boi v`ı A1 B = A1 C = 1, nên d xu ¯u`ong tha ¯iêm , ,, `u ´,c l` ´,i mâ`u cua B v` A1 c´ o mâ`u kh´ ac vo a C, tu a n´ od ung môt ¯uo. c tô c` . mâ √ , , , , `˘ng nê´u AA1 = 3, ´i d ´ thu. c châ´t d ˜ chı ra ra vo ac lâp ¯iêm A. C´ ¯o ¯a . luân . d , , , `˘m ´ tâ´t ca c´ th`ı c´ ac d a A1 tô c` ung môt ac d ¯iêm A v` ¯o ¯iêm na .√ mâ`u. Do d ,`, trên d on tâm A b´ an k´ınh 3 c´ o c` ung môt mâ`u. R˜ o r` ang trên ¯uong tr` , ,. ,`, ,, , ´ luôn t`ım d d on d o khoang c´ ach gi˜ua ch´ ung ¯uong tr` ¯o ¯uo. c hai d ¯iêm c´ , , , , , ˜ vây `˘ng 1. Ta d¯uo. c mâu thuân, ba ung mâ`u ¯uo. c hai d ¯iêm c` . luôn t`ım d , , `˘ng 1. c´ o khoang c´ ach gi˜ua ch´ ung ba

J

132

,, Chuong 12. Môt ai tâp ac . sô´ b` . h`ınh hoc . kh´

, , , . 12.7. Cho 11 d¯iêm kh´ac nhau trong h`ınh câ`u thê t´ıch V. Chu´ng , , , , minh ra ˘`ng qua tâm cua h`ınh câ`u c´ o thê du. ng hai ma ˘. t pha ˘ng sao cho , V , a phâ`n trong ch´ ung ca ˘´t h`ınh câ`u th` anh môt oi thê t´ıch , m` . "miê´ng" v´ 6, , , , ˜ cho. cua n´ o không chu´a trong phâ`n trong bâ´t cu´ môt ao d¯a . d¯iêm n` , , `˘ng môt `,i giai. Chia h`ınh câ`u ra hai b´ ˘. t pha ˘ng d Lo an câ`u ba ma ¯i qua . , , , , ` ´ a trong phâ`n ˜ cho. Môt tâm v` a hai d ac d b´ an câu chu ¯iêm t`u c´ ¯iêm d ¯a , , . ,, , trong nhiê`u nhâ´t l` a4d ac d on lai. Chia nua h`ınh câ`u ¯iêm t`u c´ ¯iêm c` . , , ˜ ma `˘ng hai ma ˘. t pha ˘ng, m` ˘. t pha ˘ng d ba a môi a ¯i qua tâm h`ınh câ`u v` , , ,, , ` on lai. anh 3 hai d ¯iêm trong 4 d¯iêm c` ., Nhu vây . nua h`ınh câu chia, th` , , ´ ´ ´ a môt "miêng" không chu ao bên trong. ´ıt nhât thê t´ıch cua ¯iêm n` . d , , 1 ´,n ho,n thê t´ıch cua b´ môt an h`ınh câ`u. . miê´ng lo 3 , . 12.8. Cho khô´i d¯a diên o 9 d¯ınh A1 , A2 , . . . , A9 .K´y hiêu . lô`i P1 c´ . ,, ,, ´n ˜,ng d¯a diên P2 , P3 , .., P9 l` a nhu d u o c t ao th` a nh b o i c´ a c ph´ e p t inh tiê ¯ . . . . −−−→ , , , ´, , −−−→ tuong ung theo c´ ac vecto A1 A2 , . . . , A1 A9 . Chu´ng minh ra ˘`ng ´ıt nhâ´t , 2 trong sô´ 9 d¯a diên o ´ıt nhâ´t môt . trên c´ . d¯iêm chung. , , , , `,i giai. X´et h`ınh d Lo a anh cua P1 qua ph´ep vi. tu. tâm A1 ¯a diên . P l` `˘ng 9 d `˘m trong P. Thât ´,ng minh ra v` a hê. sô´ 2. Ta s˜e chu ¯a diên ¯ê`u na . d . , , , ∗ ∗ ´ ng a c´ ac d P. Ch´ ung ta s˜e chu vây, , A , . . . , A9 l` ¯ınh cua d ¯a diên . cho A ,1 2 , ., , ` ` ´ ng minh d ˘ng han ˘m trong P. Ðê chu ´ ch´ minh, cha ung ¯iêu d ¯o . nhu P2 na , , −−−→ , ´ ´i ph´ep tinh ta ch´ u ´y to tiên A1 A2 chuyên c´ ac d ¯iêm A1 , A2 , . . . , A9 . , , , ,, , 0 ∗ 0 0 ´i c´ a trung d to ac d ¯ây Ai l` ¯iêm cua d ¯oan ¯iêm A2 , A2 , A3 , . . . , A9 , o d . ∗ ∗ A2 A i ). , , , `˘m trong P ba `˘ng Tông thê t´ıch cua c´ ac d ¯a diên . P1 , P2 , . . . , P9, na , , , ,, 9V, o d a thê t´ıch cua d on thê t´ıch cua d ¯ây V l` ¯a diên ¯a diên . P1 , c` . P , `˘ng 8V. Suy ra ´ıt nhâ´t c´ ba o hai d od ¯a diên ¯iêm chung (nguyên l´y . c´ , Ðirichlê cho thê t´ıch.

J

J

12.1. V´ı du.

133

, , . 12.9. Trong h`ınh câ`u d¯u`o,ng k´ınh 3 d¯a˘. t môt sô´ h`ınh câ`u m` a tông . , , ,, ˜,ng h`ınh câ`u n` d¯u`ong k´ınh cua ch´ ung ba ˘`ng 25 (nhu ay c´ o thê giao , , , , nhau). Chu´ng minh ra ˘`ng v´ oi moi ˘. t pha ˘ng tô`n tai ˘. t pha ˘ng . ma . môt . ma , song song v´ oi n´ o v` a ca ˘´t ´ıt nhâ´t 9 h`ınh câ`u con. , , ,, `,i giai. Ch´ ˘ng Lo ung ta x´et h`ınh chiê´u c´ ac h`ınh câ`u lên d ¯u`ong tha , ´,i ma ´,n chiê´u v` ˘. t pha ˘ng d ˜ cho. H`ınh câ`u lo vuông g´ oc vo ao môt ¯a . d¯o, an . , ` ´ ˘ng c´ ˘ng od ai 3, c` on c´ ac h`ınh câu con chiêu v` ao c´ ac d tha ¯ô d` ¯oan . tha , , , . , ,, , ` ˘ng n` ˘. t pha ao nhu d¯ê b` ai ra, nho c´ o tông 25. Gia su không c´ o môt . ma , , , , ´˘t 8 ´ ˘. t pha ˘ng d ˘. t pha ˘ng song song voi ma ˜ cho chı ca ngh˜ıa l` a moi ¯a . , ma , , , ˜ d ˘ng d ´ môi h`ınh câ`u nho. khi d tha ai 3 chı thuôc ¯o ¯iêm trong d ¯oan ¯ô. d` , ,. , . , ` ´ ´ ` ˜ ˘ nhiêu nhât l` a8d ¯oan . thang h`ınh chiêu cua nhung h`ınh câu nho. Suy , , ,, , ra tông cua ch´ ung không qu´ a 24. Nhân ¯uo. c su. vô l´y. . d

J

,, , uc L . 12.10. Trong h`ınh vuông canh 1 d¯on vi. c´ o môt . . d¯u`ong gâ´p kh´ , ´ , , , , ` ` không tu. ca ˘ v´ oi d¯ô. d` ai l´ on hon 1000. Chu´ng minh ra ˘ng tôn tai . môt . , ,`, ,`, , , ´ hon d¯uong tha ˘ng m song song v´ oi canh h`ınh vuông v` a ca ˘t d¯uong L tai . . , 500 d¯iêm. , , , , ,, ´˘t thu `,i giai. Gia su, li l` ´, i cua d Lo ad ai ma uc, ai v` a bi ¯ô. d` ¯u`ong gâ´p kh´ , , ´ ´ l` ad ai h`ınh chiêu cua n´ o lên c´ ac canh cua h`ınh vuông. Khi d ¯ô. d` ¯o . li ≤ ai + bi . Suy ra 1000 = l1 + · · · + ln ≤ ( a1 + · · · + an ) + (b1 + · · · + bn ) ´,c l` ˘. c a1 + · · · + an ≥ 500 hoa ˘. c l` Tu a hoa a b1 + · · · + bn ≥ 500. Nê´u , , , ´˘t lên môt tông d ai h`ınh chiê´u cua c´ ac ma ai 1 không nho ¯ô. d` ¯ô. d` . canh . d , , , ˘ng phai c´ hon 500, th`ı theo nguyên l´y Ðirichlê cho d ai d tha o ¯ô. d` ¯oan . , , , , ´˘t gâ´p kh´ ´ c l` uc, tu a d ac ma ¯iêm chung cho hon 500 h`ınh chiê´u cua c´ , , , ,`, , , ´˘t d ´ s˜e ca d oc ke t`u d uc tai ¯uong vuông g´ ¯iêm chung d ¯o ¯u`ong gâ´p kh´ . ´ıt , nhâ´t 500 d ¯iêm.

J

134

` tâp 12.2. Bai .

,, Chuong 12. Môt ai tâp ac . sô´ b` . h`ınh hoc . kh´

, , , ˘. t câ`u v` a c´ o diên . 12.11. Cho A l`a tâp . . ho. p nh˜ung d¯iêm trên môt . ma ,, , , , , ` ` ´t ´ ´ ´n hon nua diên ˘ ˘ ´ t´ıch lo t´ ıch m a t câ u. Ch u ng minh r a ng A ch u a ıt nhâ . . , , , ,`, ˘. p d môt a n´ o l` a2d ¯iêm m` ¯aah u d¯uong k´ınh cua qua câ`u. . ca , ,, , ,, . 12.12. Ta goi a h`ınh tao ac d ¯u`ong ch´eo cua . h`ınh ch˜u thâp . l` . boi c´ `˘ng trong h`ınh tr` ´,ng minh ra h`ınh vuông canh 1. Chu on b´ an k´ınh 100 . , , ,, , , ´˘t nhau. ´ ˜ ˜ ˘. t d chı c´ o thê d ac chu thâp ¯a ¯uo. c môt . sô huu han . c´ . không ca , , , , , . 12.13. Trên ma˘. t pha˘ng cho d¯iêm O. Hoi c´o thê d¯a˘. t trên ma˘. t pha˘ng , , , ´˘t 5 h`ınh tr` on không phu d at t`u O ca ¯iêm O sao cho moi . tia xuâ´t ph´ , ,, , , ´˘t o d không ´ıt hon hai h`ınh tr` on d o ngh˜ıa l` a ¯uo. c hay không ? (ca ¯ây c´ , c´ od ¯iêm chung). , , , , . 12.14. Nh˜ung d¯iêm trong ma˘. t pha˘ng d¯uo.,c so,n ba`˘ng môt . trong , , , , ` ´ ng minh ra ˘ng luôn t`ım d hai mâ`u. Chu ung mâ`u c´ ach ¯uo. c hai d ¯iêm c` ` ˘ng 1. ´ ng ba nhau d ¯u

, , CHUONG

13

´ ` MÔT . . SÔ ÐÊ THI VÔ ÐICH

, , ,´, Nh˜ung b` ai to´ an thi vô d ac nuo c l` a c´ ac b` ai to´ an d ¯ich ¯iên h`ınh cho . c´ , , , `i hoi hoc viêc tr´ı thông minh, s´ ang tao d od ¯ê giai. N´ ¯o . , vân . dung . . . sinh , ,, , , , , ´ ´ ´ ´ ´ ˜ ˘m d ˘c v` ´o. phai na ac kiên thuc co ban vung cha a hiêu thâu d ¯uo. c c´ ¯a , , , Ðê giai c´ ac b` ai to´ an thi vô d cao t´ınh phân t´ıch ¯ich . phai vân . dung . , , , ´ ´ ` ´ v` a tông ho. p c´ ac kiên thuc to´ an hoc. ai to´ an thi vô d ¯ich . . Rât nhiêu b` , ,, ,´, ,, ` c´ ac nuoc c´ o môt nguyên l´y Ðirichlê, c´ ac chuong . phân giai su ,dung . ,´, ´ ˘ ˜ truoc ta d ai. Ðê thuân ¯a ga. p môt ¯oj c muô´n tham . , sô b` . tiên . cho ban . d , , khao d ay tôi ch´ep lai a thô´ng kê ra d ac d ¯â`y d ¯u vê` chu d ¯ê` n` ¯ây c´ ¯ê` . v` , , ,´, ach [4] v` a môt sô´ tuyên thi vô d ac nuoc chu yê´u trong cuô´n s´ ¯ich . c´ . , , , , tâp ac. Nhung v`ı t` ai liêu tham khao c´ o han, phâ`n suu tâ`m cua tôi . kh´ . . , , ´˘c cha ´˘n chu,a d cha ac ban a cho ´y kiê´n. ¯â`y d ¯u mong c´ ¯oc . d . bô sung v` , , , , ˜ b` ´c v` Môi ai trong chuong n` ay k`em theo tên nuo a n˘ am k`y thi vô d ¯ich . ,´, ´ nuoc d o . ¯ , `, 52 sô´ nguyên bâ´t k`y luôn . 13.1. (Anh, 1966) Chu´ng minh ra˘`ng tu , , , ,, c´ o thê chon a tông hoa ˘. c hiêu ung chia hê´t cho . d¯uo. c ra hai sô´ m` . cua ch´ 100. , , , ,, `,i giai. Tâ´t ca c´ Lo ac sô´ du trong ph´ep chia cho 100, d anh ¯uo. c chia th` , , t`ung nh´ om nhu sau: {0}, {1; 99}, {2; 98}, . . . , {49, 51}, {50}. V`ı c´ o , , tâ´t ca 51 nh´ om, m` a lai o 52 sô´, nên theo nguyên l´y Ðirichlê gi˜ua . c´

136

,, Chuong 13. Môt ¯ê` thi vô d ¯ich . . sô´ d

, , , ch´ ung phai c´ o hai sô´ m` a c´ ac sô´ du trong ph´ep chia cho 100 roi v` ao , , ` ´ ´ ` ´ ´ ˘ng môt om. Hai sô n` ay l` a hai sô cân t`ım v`ı nêu sô du cua ch´ ung ba . nh´ , , , ´ ´ ´ nhau th`ı hiêu ch´ ung chia hêt cho 100, c` on nêu sô du cua ch´ ung . cua , , ´ kh´ ac nhau th`ı tông cua ch´ ung chia hêt cho 100.

J

, , , `, tâp . 13.2. (Anh, 1970) Chu´ng minh ra˘`ng tu . ho. p tu`y ´y gô`m n sô´ tu. ,, , , ˜ nhiên luôn t´ ach ra d¯uo. c môt tâp ho. p con (kh´ ac rông) chu´a c´ ac sô´ m` a . . , , ´ tông cua ch´ ung chia hêt cho n. , , , , , `,i giai. Gia su, vo ´,a c´ ´i môt ´ chu Lo tâp ho. p n` ao d ac sô´ a1 , a2 , . . . , an m` a ¯o , . . , , ˘ng d ´ không c´ không thoa m˜ an kha ai to´ an. Khi d o sô´ n` ao ¯inh ¯o . cua b` trong c´ ac sô´ S1 = a 1 , S2 = a 1 + a 2 , . . . , S n = a 1 + a 2 + · · · + a n , chia hê´t cho n. V`ı sô´ c´ ac sô´ du kh´ ac không trong ph´ep chia cho ,, n l` a n − 1, nên theo nguyên l´y Ðirichlê t`ım d a Sj ¯uo. c hai sô´ Si v` , (1 ≤ i < j ≤ n) c´o c` ung sô´ du. Suy ra hiêu a . S,j − Si = ai−1 + · · · + , j ,, , ˜ ´i gia su n´ ˘ng oi trên v` a kha ay mâu thuân vo chia hê´t cho n, d ¯iê`u n` , ,, , ´ ng minh. d ai to´ an d ¯inh ¯uo. c chu . cua b` , ,, , ˜, . 13.3. (Anh, 1976) Gia su trong tâp u han . ho. p hu . X chon . ra 50 tâp . ,, , , , , ˜ tâp `n ´ ho. p con A1 , . . . , A50 sao cho môi h o p con ch u a ho n m ôt n u a phâ . ., . ,, , , , , , tu cua tâp ˘`ng c´ o thê t`ım d¯uo. c tâp . X. Chu´ng minh ra . ho. p con B ⊂ X ,, ,, , , chu´a không nhiê`u hon 5 phâ`n tu v` a c´ o ´ıt nhâ´t môt . phâ`n tu chung cho , c´ ac tâp . ho. p A1 , A2 , . . . A50 .

J

, , , ,, , , ˜ tâp `˘ng n. Môi `,i giai. Gia su, sô´ c´ Lo ac phâ`n tu cua tâp . X ba . ho. p con , n ,, , ´,a không ´ıt ho,n d phâ`n tu, c´ o ngh˜ıa l` a ¯uo. c chon . A1 , . . . , A50 chu 2 , , ,, , n ,, tông sô´ c´ ac phâ`n tu cua tâ´t ca c´ ac tâp ay vuo. t qu´ a 50. = 25.n. . n` 2 ,, , ` Theo nguyên l´y Ðirichlê tô`n tai m ôt phâ n t u c ua X thu ôc . . . không ´ıt

137 ,, , , ´,ng minh vo ´,i gi´ ˜ chon. hon 26 tâp a tri. bâ´t k`y ¯a . con d . Tuong tu. ta chu , , , k . . d¯uo. c c´ 2 , , ´ ` ´ Chu´ng minh ra ˘`ng v´ oi moi sô k ∈ Z tô n t ai c´ a c sô i, j ∈ { 1, 2, . . . , n } . . (không nhâ´t thiê´t kh´ ac nhau) sao cho sô´ ai + a j − k chia hê´t cho m. , `,i giai. X´et 2n sô´ a1 , a2 , . . . , an , k − a1 , k − a2 , . . . , k − an . V`ı 2n > m, Lo , ´ c´ nên c´ o 2 trong c´ ac sô´ d o c` ung sô´ du trong ph´ep chia cho m. Theo ¯o , d ai to´ an, c´ ac sô´ a1 , a2 , . . . , an , c´ o sô´ du kh´ ac nhau trong ¯iê`u kiên . b` , ph´ep chia cho m, nên c´ ac sô´ k − a1 , k − a2 , . . . , k − an c˜ ung c´ o sô´ du , , , ` ˘. p sô´ c´ ˘ng nhau chı c´ ´ ca kh´ ac nhau. Do d¯o o sô´ du ba o thê l` a hai sô´ c´ o , , ´i i, j n` ´. Khi d ´ hiêu dang ai v` a k − a j vo ao d ung ai + a j − k ¯o ¯o . . cua ch´ chia hê´t cho m.

J

, , . 13.5. (Ba Lan, 1977) Chu´ng minh ra˘`ng v´ oi moi gi´ a tri. a, b ∈ R v` a , , . , e > 0, tô`n tai ac sô´ k, m ∈ Z v` a n ∈ N thoa m˜ an c´ ac bâ´t d¯a ˘ng thu´c . c´

,, Chuong 13. Môt ¯ê` thi vô d ¯ich . . sô´ d

138

|na − k| < e v`a |nb − m| < e. , 1 ˜ ca `,i giai. Cho sô´ nguyên N > v` ´,i môi ˘. p x, y ∈ [0; 1] ta thay Lo a vo e , `˘ng ca ´,i hai ˘. p u, v thoa m˜ ´ nê´u vo thê´ ba an u = [ Nx ], v = [ Ny], khi d ¯o , , ´, ´,i môt ˘. p duy nhâ´t (u, v) th`ı ˘. p ( x1 , y1 ), ( x2 , y2 ) tuong u ng vo ca . ca 1 1 | x1 − x2 | = (u + { Nx1 }) − (v + { Nx2 }) = N N 1 1 = |{ Nx1 } − { Nx2 }| < j) t`u tâp ay tuong u ung môt ¯o . ho. p n` . ca , ´i k´y hiêu vo . n = i − j, k = [ia] − [ ja], m = [ib] − [ jb] , ,, ´,c câ`n chu ´,ng minh : ˘ng thu ta nhân ac bâ´t d ¯a . d¯uo. c c´

|na − k| = |(ia − [ia]) − ( ja − [ ja])| = |{ia} − { ja}| < e, |nb − m| = |(ib − [ib]) − ( jb − [ jb])| = |{ib} − { jb}| < e.

J

, . 13.6. (Bı, 1977) Trong h`ınh tr`on c´o b´an k´ınh n ∈ N c´o 4n d¯oan , ,. , , , ` ` ` ´ tha ˘ng d¯êu c´ o d¯ô. d` ai ba ˘ng 1. Chu´ng minh ra ˘ng nêu c´ o môt ˘ng . d¯u`ong tha , ,´, ,, ,`, cho truoc th`ı t`ım d¯uo. c môt d¯uong tha ˘ng kh´ ac, hoa ˘. c song song, hoa ˘. c , . , , ,`, ´ ´ vuông g´ oc v´ oi d ˘ng n` ay v` a ca ˘t ´ıt nhât hai d¯oan ˘ng n´ oi trên. ¯uong tha . tha , , , , ˜ môt `˘ng tông d `,i giai. Ðê ´y ra Lo ai hai h`ınh chiê´u cua môi ¯ô. d` ¯oan . d . , , , ,`, ,`, , 0 ´ ˘ng lên d ˘ ˘ tha u o ng th a ng l v` a d u o ng th a ng l , vuông g´ o c v o i n´ o , không ¯ ¯

139 , , ´,i d¯oan b´e hon 1. Thât od ai l` a 1 song song vo ¯ô. d` . vây, . nê´u vecto a c´ . , , , , ´ ˘ng n` ´, c` tha ao d on c´ ac vecto x v` a y l` a c´ ac h`ınh chiêu cua vecto a lên ¯o , ,`, , 0 ˘ng l v` d a l th`ı a = x + y, suy ra | x | + |y| ≥ | a| = 1. Nhung ¯uong tha , , , , `˘ng | x | v` ˘ng ba ´ tông cua d ai c´ ac h`ınh chiê´u cua d a |y| do d¯o ¯ô. d` ¯oan . tha , , , ˜ d ch´ ung c˜ ung không b´e hon 1. Dân ai h`ınh chiê´u cua tâ´t ¯ê´n tông d ¯ô. d` , , , , ,, , ´,i vây ˘ng không b´e hon 4n. Bo ˘ng l ca c´ ac d t`u hai d ¯oan ¯u`ong tha . , tha . , , , ,, ,, ˘ng, tông d v` a l 0 c´ o thê chon d d ai cua h`ınh chiê´u ¯uo. c môt ¯u`ong tha ¯ô. d` . . , , ˘ng trên n´ c´ ac d¯oan tha o không b´e hon 2n (nguyên l´y Ðirichlê). V`ı tâ´t . , , ,, ´ ´ , ˘ng d ˘p xêp trong h`ınh tr` ca c´ ac d on b´ an k´ınh n, nên ho. p ¯oan ¯uo. c sa . tha , , ,, , ˘ng bâ´t k`y c´ c´ ac h`ınh chiê´u cua ch´ ung trên d od ai b´e hon ¯u`ong tha ¯ô. d` , , ,, ,, ,, ˘ng d 2n. Suy ra trên d t`ım d ao ¯u`ong tha ¯uo. c chon ¯uo. c 1 d ¯iêm thuôc . v` , . , ´ ´ ˘ng (Nguyên l´y Ðirichlê cho d¯oan h`ınh chiêu cua ´ıt nhât hai d a ¯oan . th , ,. , , ,`, , ,`, ´ ˘ng d ˘ng ˘ng). Ðuong tha ay vuông g´ oc voi d tha ¯i qua d ¯iêm n` ¯uong tha , , ,, ,`, ´ ´ ˘ ˘ ˘ ´ d ay. V`ı d¯uong thang n` ay ¯uo. c chon, ¯oan . s˜e cat ıt nhât hai d . thang n` , , , , ,`, ´ ´ ˘ ˘ ˘ hoa. c vuông g´ oc voi, hoa. c song song voi d o thoa ¯uong thang l, th`ı n´ m˜ an d¯iê`u kiên ai to´ an. . b` , . 13.7. (Bungari (d¯ê` thi chon a . d¯ôi . tuyê,n),1973) Cho a1 , a2 , . . . , an l` , , ´ ˜ nhung sô nguyên kh´ ac nhau trong khoang [100,200], m` a ch´ ung thoa , ˜,a nhu ˜,ng sô´ n` m˜ an a1 + a2 + · · · + an ≥ 11100. Chu´ng minh ra ˘`ng giu ay ,, , ´ ´ ´ ´ ´ ˜ c´ o ´ıt nhât môt a viêt n´ o o dang o ´ıt nhât hai chu sô . sô, m` . thâp . phân c´ ´ giông nhau. , , `,i giai. Ch´ Lo ung ta lâp ach c´ ac sô´ trong khoang [100,200], m` a . danh s´ ,, , ch´ ung viê´t o hê. thâp o hai ch˜u sô´ tr` ung nhau.: 100, . phân ´ıt nhâ´t c´ 101, 110, 111, 112, 113, 114, 115, 116, 117, 118, 119, 121, 122, 131, 133, 141, 144, 151, 155, 161, 166, 171, 177, 181, 188, 191, , , , , , ˘. t kh´ ac tông tâ´t ca 199, 200. Tông cua tâ´t ca c´ ac sô´ trên l` a 4050. Ma , , c´ ac sô´ nguyên trong khoang [100,200] l` a 15150. Nê´u trong nh˜ung ˜ cho a1 , a2 , . . . , an không c´ sô´ d o sô´ n` ao trong danh s´ ach trên, th`ı ¯a

J

140

,, Chuong 13. Môt ¯ê` thi vô d ¯ich . . sô´ d

a1 + a2 + · · · + an < 15150 − 4050 = 11100, d ay vô l´y. Ngh˜ıa ¯iê`u n` ,, , ,, ´ ´ ´ ´ l` a trong c´ ac sô a1 , a2 , . . . , an c´ o ´ıt nhât môt o ´ıt . sô viêt o co sô´ mu`oi c´ , ´ ´ nhât hai ch˜u sô tr` ung nhau.

J

, . 13.8. (Bungari,1973) Trong môt o 20000 cuô´n s´ ach, . thu viên . c´ ,, ´ , ˜ gi´ ˜ ng gi´ ch´ ung d¯uo. c xêp v` ao nhu a s´ ach sao cho môi a s´ ach c´ o ´ıt nhâ´t , , môt ˘. c nhiê`u nhâ´t 199 cuô´n s´ ach. Chu´ng minh ra ˘`ng ´ıt nhâ´t . quyên hoa , , c´ o hai gi´ a s´ ach c´ o c` ung sô´ luo. ng s´ ach. , , , , `,i giai. Gia su, ngu,o.,c lai. Lo o n gi´ a s´ ach v` a ch´ ung . Nê´u trong thu viên . c´ ,, , ´ ´ ` ´nh sô tu 1 d d o r` ang l` a n ≤ 199. Ch´ ung ta k´y hiêu ¯uo. c d¯a ¯ên n, th`ı r˜ . ai ,, , ´ ´ ´ ˘. t lên gi´ l` a sô cuôn s´ ach d a s´ ach thu i, ch´ ung ta s˜e c´ o 1 ≤ ai ≤ ¯uo. c d ¯a ,, ,, , ` ´ ´ tu d 199. khi d ung ta nhân ¯o ¯iêu kiên ¯uo. c . ai 6= a j voi i 6= j, ch´ . d 20000 = a1 + a2 + · · · + an ≤ 1 + 2 + · · · + 199 = 19900 < 20000 ´,a c` d ay vô l´y. Suy ra c´ o ´ıt nhâ´t hai gi´ a s´ ach, trên ch´ ung chu ung ¯iê`u n` ´ sô s´ ach. , , . 13.9. (Bungari, 1983) T`ım h`ınh vuông c´o k´ıch thuo´,c b´e nhâ´t, d¯ê , ´ c´ trong h`ınh vuông d¯o o thê sa ˘´p xê´p 5 h`ınh tr` on b´ an k´ınh ba ˘`ng 1, sao , cho không c´ o 3 h`ınh tr` on n` ao trong ch´ ung c´ o d¯iêm trong chung.

J

, , , A B `,i giai. Gia su, h`ınh vuông ABCD Lo A1 B 1 ´,a 5 h`ınh c´ o tâm O v` a canh a, chu . O ´˘t nhau v` tr` on không ca a d¯ê`u c´ o D1 C1 `˘ng 1, khi d ´ c´ b´ an k´ınh ba ac tâm ¯o , C D `˘m trong h`ınh vuông cua ch´ ung na `˘ng A1 B1 C1 D1 c´ o tâm O v` a canh ba H`ınh 13.1: . ,, a − 2 (o d¯ây A1 B1 //AB ). , , , , ,, , ˘ng nô´i t`u c´ C´ ac d ac trung d ac canh ¯u`ong tha ¯iêm cua c´ ¯ô´i diên . d . cua ´ th` h`ınh vuông A1 B1 C1 D1 chia h`ınh vuông d anh bô´n h`ınh vuông ¯o

141 , ,, nho, o môt ung ´ıt nhâ´t c˜ ung c´ o hai trong sô´c´ ac tâm (Nguyên . trong ch´ , , ˘. t không ´ khoang c´ l´y Ðirichlê). Khi d ach gi˜ua hai tâm n` ay môt ¯o . ma , , , , , ´n hon d ˘. t kh´ lo ac không b´e hon 2. Do ¯u`ong ch´eo h`ınh vuông b´e, ma vây o . c´ A1 B1 √ a − 2√ 2 ≤ OA1 = 2= 2 √ 2 2 suy ra a ≥ 2 2 + 2. √ , Cuô´i c` ung, nê´u a = 2 2 + 2 v` a tâm cua c´ ac h`ınh tr` on l` a c´ ac , , , ,, ´ ` d ac d nêu trên d ¯iêm O, A1 , B1 , C1 , D1 th`ı tât ca c´ ¯iêu kiên ¯uo. c thoa . √ , , m˜ an. Nhu vây cua h`ınh vuông câ`n t`ım l` a 2 2 + 2. . canh . , , ,´, . 13.10. (Nam Tu, 1977) Cho truoc 20 sô´ tu. nhiên a1 < a2 < . . . < ,, , ˜,a c´ a20 không vuo. t qu´ a 70. Chu´ng minh ra ˘`ng giu ac hiêu . a j − ak ( j > k ) ,, ` ´ luôn t`ım d¯uo. c ´ıt nhât 4 hiêu ˘ng nhau. . ba , , , , , , , `,i giai. Gia su, kha ˘ng d ´ gi˜ua 19 sô´ tu. Lo ai to´ an l` a sai. Khi d¯o ¯inh . cua b` `˘ng nhau. nhiên a20 − a19 , a19 − a18 , . . . , a2 − a1 không c´ o 4 sô´ n` ao ba , ˜ sô´ 1, 2, 3, 4, 5, 6 c´ ˘. t không qu´ ´ gi˜ua ch´ Do d ung môi o ma a 3 lâ`n. ¯o , , , ´n hon 6 (nê´u không th`ı sô´ c´ ´ phai lo ac Suy ra c´ o môt ¯o . trong 19 sô´ d , , , , ´ ` ´ ´ sô không lon hon 6 s˜e nhiêu hon 18), c´ o 3 trong 18 sô c` on lai phai . , , , , ´,n ho,n 5, 3 trong 15 sô´ c` ´n ho,n 4,. . . Do d ´ tông cua lo on lai ¯o . phai lo ch´ ung ( a20 − a19 ) + ( a19 − a18 ) + · · · + ( a2 − a1 ) ≥ 7 + 3(6 + 5 + 4 + , ˜ `˘ng a20 − a1 ≤ 70 − 1 = 69. Mâu thuân 3 + 2 + 1) = 70 không thê ba , , ,, , ´ ng minh kha ˘ng d ˜ chu nhân ai to´ an. ¯uo. c d ¯a ¯inh . cua b` . d , ,, , . 13.11. (Nam Tu, 1981) Tâp ac sô´ {1, 2, . . . , 100} d ¯uo. c chia . ho. p c´ ,, , , , l` am 7 tâp ˘`ng ´ıt nhâ´t o môt ac tâp . ho. p con. Chu´ng minh ra . trong c´ . ho. p ,, con â´y t`ım d¯uo. c hoa ˘. c 4 sô´ a, b, c, d sao cho a + b = c + d, hoa ˘. c ba sô´ e, f , g sao cho e + f = 2g. , , , `˘ng c´ `,i giai. Ðê ´y ra ´,a không Lo o ´ıt nhâ´t môt . trong 7 tâp . ho.,p con chu ,, ,, , , , ´ıt hon 15 sô´ (trong tru`ong ho. p nguo. c lai, ac tâp . tâ´t ca c´ . ho. p con

J

J

142

,, Chuong 13. Môt ¯ê` thi vô d ¯ich . . sô´ d

, , ˜ ca ´,a không nhiê`u ho,n 7.14 = 98 sô´). Môi ˘. p sô´ a > b cua tâp chu . ho. p ,, , , ´, ,, ´,i hiêu ˘. t tuong u ´ ta nhân con n` ay d ng vo ¯uo. c d ¯a ¯o ¯uo. c . a − b. Khi d . d , , 2 = 15 14 = 105 hiêu, trong sô´ do không ´ıt hon C15 o c´ ac hiêu ¯´ phai c´ . . 2 , , `˘ng nhau (v`ı c´ ba ac hiêu nhân d a 99 gi´ a tri. kh´ ac nhau ¯uo. c không qu´ , , ,, . , . ´,c ´i hai ca ˘. p sô´ a > b, c > d ta c´ ˘ng thu 1, 2,. . . ,99). Gia su vo o d ¯a ´ a + d = b + c. Nê´u a = d a − b = c − d( a 6= c, b 6= d). Khi d ¯o , , , , ´ c kh´ ˘. c b = c, d ˘ng thu ˘. c (hoa ac không thê xây ra), th`ı c + b = 2a (hoa ¯a a + d = 2b). , , , . 13.12. (Nam tu,1977) Chu´ng minh ra˘`ng diên ınh vuông . t´ıch cua h` , ,, , , bâ´t k`y na ˘`m trong tam gi´ ac, không l´ on hon nua diên ac . t´ıch cua tam gi´ ´. d¯o , `,i giai. (H`ınh 13.2) Ta chu ´,ng Lo A , , , ˘ng d minh kha tông qu´ at hon : ¯inh . , M diên anh bâ´t . t´ıch cua h`ınh b`ınh h` L , `˘m o, trong tam gi´ M1 F k`y KLMN na ac L1 E ,, N , , ´n hon nua diên ABC không lo . t´ıch K , C B ´. cua tam gi´ ac d ¯o N1 K1 , , ,, ˜ môt `˘ng môi ˘ng Ðê ´y ra ¯u`ong tha . d , H`ınh 13.2: ´˘t hai canh KL v` a MN ca cua tam . , ,, , , gi´ ac ABC (c´ o thê o d¯ınh tam gi´ ac), ngh˜ıa l` a c´ o 2 trong 4 giao d ¯iêm , , ,, `˘m trên 1 canh ˘ng han, ˘ng na theo nguyên l´y Ðirichlê. Cha ¯u`ong tha . . d ´˘t BC lâ`n lu,o.,t o,, K1 v` KL v` a MN ca a N1 . Trên canh AC v` a BC . , AB, , , ´,i KL chon ac d a F sao cho L1 , M1 l` a giao d ¯iêm D, E v` ¯iêm cua DE vo . c´ , v` a MN thoa m˜ an: , K1 L1 = KL; L1 M1 //K1 N1 d ¯ô`ng th`oi BF//BD ,, `˘ng ´ h`ınh b`ınh h` Khi d anh KLMN v` a K1 L1 M1 N1 c´ od ¯o ¯u`ong cao ba nhau d on c´ ac h`ınh b`ınh h` anh BDEF v` a K1 L1 M1 N1 ¯ê´n d ¯as y chung, c`

J

143 ,, , , ´y DE không b´e hon d ´y L1 M1 v` a L1 c´ od a hai d ¯a ¯a ¯u`ong cao ha. t`u D v` , ,, ` ´ ˘ng nhau. Boi vây xuông BF ba o: SKLMN = SK1 L1 M1 N1 ≤ SBDEF . Gia . c´ ,, ´ EC = (1 − x ) AC, c´ su AE = x.AC, khi d ac tam gi´ ac ABC, ADE ¯o , ` ´i nhau nên ta c´ v` a EFC d o: ¯ông dang . vo SBDEF = S ABC − S ADE − SEFC

= S ABC − x2 S ABC − (1 − x )2 S ABC 1 = 2x (1 − x )S ABC ≤ S ABC . 2 V`ı x (1 − x ) ≤

1 , 1 ´i x lâ´y gi´ vo a tri. bâ´t k`y, nên SKLMN ≤ S ABC . 4 2

J

, , , ˜ . 13.13. (Nam tu, 1972) Ðô´i v´ oi môi gi´ a tri. n ∈ N, h˜ ay t`ım sô´ k l´ on , ,, , nhâ´t (k ∈ N ) thoa m˜ an t´ınh châ´t sau: Trong tâp ho. p gô`m n phâ`n tu . , , , c´ o thê chon ac nhau, sao cho hai tâp . ra k tâp . ho. p con kh´ . ho. p con bâ´t ˜ k`y d¯ê`u c´ o tâp ac rông. . giao kh´ , , , ,, , `,i giai. Cô´ d Lo a chı ¯inh . phâ`n tu ai cua tâp . ho. p X = { a1 , a2 , . . . , an } v` , , , , `˘ng sô´ ´,a phâ`n tu, a1 .Sô´ c´ x´et c´ ac tâp chu ac tâp . ho. p con . ho. p nhu vây . ba , , , `˘ng 2n−1 . Suy ra c´ ac tâp a ba . ho. p con cua tâp . , ho. p { a2 , . . . , an }, ngh˜ıa l` , , , ,, , ˘. t kh´ ˜ chon d k ≥ 2n−1 . Ma ac gia su d con cua X. Ta ¯a ¯uo. c hon 2n−1 tâp . . , , ,, ,, ˘. p d chia tâ´t ca c´ ac tâp con cua X th` anh 2n−1 ca boi 1 tâp ¯uo. c tao . . . con , , cua X v` a phâ`n b` u cua n´ o. Theo nguyên l´y Ðirichlê c´ o ´ıt nhâ´t 2 tâp . ˘ ˜ chon con d t ao th` a nh m ôt c a p, suy ra ch´ u ng không giao nhau. V ây ¯a . . . . . k = 2n −1 .

J

, 1 . 13.14. (M˜y, 1983) Trên truc o d¯ô. d` ai (n ∈ . sô´ lâ´y môt . khoang c´ n , n+1 , , , ` ` N ). Chu´ng minh ra ˘ng khoang n` ay c´ o chu´a nhiêu hon phân sô´ 2 , p ´ p, q ∈ Z, 1 ≤ q ≤ n. tô´i gian dang , trong d ¯o . q

,, Chuong 13. Môt ¯ê` thi vô d ¯ich . . sô´ d

144

, , , 1 `˘ng trong môt `,i giai. Gia thiê´t ra ´,a ´ c´ Lo ao d od ai , chu ¯o ¯ô. d` . khoang n` n , p , , n+1 ´ ´ ` ´ phân sô tôi gian , voi q ∈ {1; 2; . . . ; n}. Ta s˜e nhiêu hon 2 q , , , ˜ `˘ng gi˜ua c´ ´ ng minh ra ac mâu sô´ cua c´ ac phân sô´ n` ay luôn t`ım chu ,, ˜ ˜ ˜ d a mâu sô´ n` ay chia hê´t cho mâu sô´ kia. Thât ¯uo. c hai mâu sô´ m` . vây . , , , , , r + . Sô´ ˜ ˜ ´ ´i dang ´ ta biêu diên c´ ac mâu sô´ duo 2 .s v o i s l` a sô l e, r ∈ Z .   , , `˘ng n + 1 ( ngh˜ıa c´ ac sô´ le kh´ ac nhau gi˜ua c´ ac sô´ 1, 2, 3, . . . , n ba 2 ,, , ˜ sô´ d ˜ sô´ l` a ´ıt hon sô´ c´ ac mâu ¯ang x´et), suy ra luôn t`ım d ¯uo. c hai mâu ˜ ´ c´ q = 2r .s v` a q1 = 2r1 s1 m` a s = s1 v` a r = r1 . Khi d o môt ¯o . mâu , , ˜ sô´ kia, hay q1 = kq. Nhu vây sô´ chia hê´t cho mâu ac phân sô´ . gi˜ua c´ m l ,, ´,i kq ≤ n. Khi d ´ chon ac nhau dang v` a , vo ¯uo. c hai sô´ kh´ ¯o . d . q kq m − l < 1, q kq n , , 1 `˘m trong khoang c´ ´ km − l = 0, v`ı trong v`ı ca hai na od ai . Do d ¯ô. d` ¯o n ,`, ,, , truong ho. p nguo. c lai . th`ı m − l = |km − l | ≥ 1 ≥ 1 , q kq kq kq n ´ km = l v` v` a do d a ¯o nhau.

l km ,, = , ngh˜ıa l` a hai phân sô´ d ung ¯uo. c chon . tr` kq kq

J

, . 13.15. (Viêt ˘`ng tô`n tai ac sô´ c´ o . nam, 1976) Chu´ng, minh ra . ,vô sô´ c´ n ˜ ˜ ´ ´ c´ o a trong c´ ac biêu diên thâp dang ¯o . phân cua môi sô d . 5 (n ∈ N ), m` , , , ´ ´ ˜ sô 0 d¯u´ng liên tiêp. không ´ıt hon 1976 chu , `˘ng vo `,i giai. Ta chu ´,ng minh ra ´,i moi Lo ac sô´ . k ∈ N tô`n tai . vô sô´ c´ , m k m ∈ N thoa m˜ an d ¯iê`u kiên . 5 = 1 (mod 2 ).

145 k , ,, ac sô´ 50 , 51 , . . . , 52 luôn t`ım d Thât a ¯uo. c hai sô´ 5 p v` . vây, . gi˜ua c´ , , q k ´ ´ hiêu 5 ( p > q) c´ o c` ung sô du trong ph´ep chia cho 2 . Khi d ¯o . cua p q q p − q k p ´ ´ ch´ ung 5 − 5 = 5 (5 − 1) chia hêt cho 2 nghi˜a l`a sô 5 −q − 1 , r ( p−q) − 1 (r ∈ N ) dê`u chia hê´t cho 2k . v` a tâ´t ca c´ ac sô´ c´ o dang ¯ . 5 , , ˜ gi´ ´i môi Nhu vây a tri. m = r ( p − q), r ∈ N, ta c´ o 5m ≡ 1 . vo , , ung (mod 2k ), t`u d¯o´ 5m+k ≡ 5k (mod 10k ) ngh˜ıa l`a k ch˜u sô´ tân . , c` , m+k , , , , , , k ´i k ch˜u sô´ tân cua 5 tr` ung vo ung cua 5 . Gia su sô´ k thoa m˜ an . c` , , , , k 1976 ´ a không nhiê`u hon k − 1976 ch˜u sô´. Do d ´ gi˜ua k 2 > 10 chu ¯o , , , , m + k ch˜u sô´ tân ung cua sô´ 5 chı c´ o k − 1976 ch˜u sô´ kh´ ac không, c` on . c` , `˘ng 0. 1976 ch˜u sô´ c` on lai . (liên tiê´p nhau) ba , ,, ˘ng c´ o n2 + 1( n ∈ N ) . 13.16. (Tiêp kha ˘´c, 1979) Trên môt d¯u`ong tha . . , , , ˜,a ch´ d¯oan ˘ng. Chu´ng minh ra ˘`ng, hoa ˘. c l` a giu ung c´ o thê chon . tha ., n + 1 ,, ´ d¯oan ˘t nhau, hoa ˘. c l` a t`ım d¯uo. c n + 1 d¯oan ˘ng n` ao . d¯ôi,môt . không ca . tha ´ c´ d¯o o d¯iêm chung.

J

, , ,´, ,, `˘ng môt `,i giai. Ta d ˘ng khi n´ Lo ng bên tr´ ai cho d oi ra ¯inh ¯u`ong tha . huo , ,. , , ` ´ ` ˘ ˘ d ai d ac, nêu d ut bên tr´ ai cua ¯oan ¯oan ¯âu m´ . na,m o bên tr´ . thang kh´ , ,, , ` ´ ` ´ ˘ ˘ d thang thu nhât nam o bên tr´ ai d¯âu m´ ut bên tr´ ai cua d ¯oan ¯oan . ,. , ,, ,, ˜ d ` ´ ´ ´, hai. Môi ˘ng thu ˘ ˘ ´ tha o an th a ng d u o c d a nh sô b a ng m ôt sô tu o ng ¯ . ¯ . ¯ . , ,, , , `˘ng c´ ´,ng t`u, c´ ´, nhâ´t d ´c thu ˘ng u ac sô´ 1, 2, . . . , n ba ach sau: o buo ¯oan . tha ,, , , ´, ˜ bu,o ´,c tiê´p theo ´ môi o tân ung bên tr´ ai cho tuong u ng sô´ 1. Sau d ¯o . c` , , , ˘ng chua d ´nh sô´, d ta lai ung ¯oan ¯a ¯oan tân c` . chon . trong sô´ nh˜ung d . tha , . ,. , , , , ´ ng, kh´ ´i c´ ˘. t cho n´ bên tr´ ai v` ad o sô´ tuong u ac vo ac sô´ cua nh˜ung d ¯a ¯oan . , , , , , , ´ ´ ´ ´ ´i n´ ´ ˘ng giao vo ˜ ´ tha o (d a d u o c d a nh sô ). Nê u d ê n bu o c n` a o d â y, ta ch on ¯ ¯ . ¯ ¯ ¯ . , ,, , , , ´ ´, , , ´ ´ ˘ d u o c d o an th a ng, nhu ng d ô i v o i n´ o không ch on d u o c sô th u t u , th` ı ¯ ¯ . ¯ . ¯ . . , . , `˘m bên tr´ ´i n d ˘ng na d ay c´ o ngh˜ıa l` a n´ o giao vo ai n´ o v` a c´ o ¯iê`u n` ¯oan . tha , , , , , nh˜ung sô´ kh´ ac nhau. Trong tru`ong ho. p n` ay d ut tr´ ai cua d ¯â`u m´ ¯oan . , , ,, , , ,, ´ ´ ´ ˘ng d ˘ u o c ch on thu ôc v` a o n + 1 d o an th a ng. Nê u o bu tha o c â y, d o an ¯ . ¯ . ¯ . . .

146

,, Chuong 13. Môt ¯ê` thi vô d ¯ich . . sô´ d

, ,, ˘ng cuô´i c` ´nh sô´ th`ı theo nguyên l´y Ðirichlê ´ıt nhâ´t ung d tha ¯uo. c d ¯a , , , , ´ ng vo ´i nhiê`u ho,n n d ˘ng, m` ˘ng môt a c´ ac d¯oan ¯oan . trong n sô´ u . tha . tha ´, tu., th´ıch ho.,p, không giao nhau. ´,i c´ ´ vo d ac sô´ thu ¯o , . 13.17. (Rumani, 1978) C´ac h`am sô´ f , g, h : N → N thoa m˜an ba d¯iê`u kiên . sau: , , a) H` am h(n) không nhân a tri. n` ao tai . gi´ . nhiê`u hon môt . d¯iêm n ∈ N. , b) Tâp a tri. h` am sô´ g(n) l` a N. . ho. p gi´

J

c) f (n) ≡ g(n) − h(n) + 1, n ∈ N. , , ´ng. Chu´ng minh ra ˘`ng d¯ô`ng nhâ´t thu´c f (n) ≡ 1, n ∈ N, l` a d¯u , `,i giai. Ta chu ´,ng minh d ´,c g(n) ≡ h(n) (n ∈ N ). T`u, Lo ¯ô`ng nhâ´t thu ˜ d¯ê´n f (n) ≡ g(n) − h(n) + 1 ≡ 1, n ∈ N. ´ v` d ad ¯o ¯iê`u kiên . b) s˜e dân , ´i bâ´t k`y n ∈ N c´ Vo o h(n) = g(n) + 1 − f (n) ≤ g(n) , , ,, `˘ng, d ´,c ´,i gi´ ˘ng thu ´n ∈ N d v`ı f (n) ≥ 1. Gia su ra a tri. n` ao d ¯ô´i vo ¯o ¯a ´ ng, khi d¯o ´ h(n) < g(n) = k. Theo d g(n) = h(n) không d ¯u ¯iê`u kiên . , ´ b) ta t`ım c´ ac sô n1 , n2 , . . . , nk−1 ∈ N, d ¯ê sao cho g(ni ) = i khi i = ,, ˜ ´ ´ 1, . . . , k − 1. Boi vây . môi sô trong k sô h(n1 ), h(n2 ), . . . , h(nk−1 ), h(n) , ´ theo nguyên l´y Ðirichlê thuôc ao tâ. p ho. p {1, 2, . . . , k − 1}, do d ¯o . v` , ` ´ ´ nhiêu hon môt h` am sô h(n) nhân a tri. n` ao d ay tr´ ai ¯o ¯iê`u n` . ,gi´ . lâ`n, d , , , , ` ´ ng minh. ´i d¯iêu kiên ˘ng d ˜d vo ¯inh ¯a ¯uo. c chu . d . a).Kha , , , ˜,u Uo´,c), 1979) Chu´,ng minh ra . 13.18. (M˜y (Nu ˘`ng c´ ac d¯ınh cua n-gi´ ac ,´, ` ´ ´ d¯êu c´ o diên ac cho truoc, . t´ıch b´e, nhât (n > 3), nôi . tiêp trong môt . n-gi´ ,´, , tr` ung v´ oi trung d¯iêm c´ ac canh cua n-gi´ ac cho truoc. .

J

, , , `,i giai. (H`ınh 13.3) Gia su, n-gi´ Lo ac d o diên ¯ê`u B1 , . . . , Bn c´ . t´ıch SB nôi ac d ¯ê`u A1 , . . . , An diên . tiê´p trong n-gi´ . t´ıch S A .

147 ˜ môt ´ nê´u ch´ Khi d ung không tr` ung nhau th`ı trên môi A i A i +1 ¯o . canh . , , , , ´i i = 1, 2, . . . , n ( An+1 = A1 ) c´ vo o môt ac d ¯ınh Bi d¯uo. c x´ ¯inh . . d ,`, ,, , ´ trên canh ây. Thât l´y . . vây, . trong truo, ng ho. p nguo. c lai . theo nguyên , , ´ ´ a hai d ˘ng han Ðirichlê ´ıt nhât c´ o môt cha ¯iêm B1 . canh, ., . A1 A2 , chu , , ,, ,, ´ (d ´ d v` a B2 d ac d¯inh, gia su A1 B2 > A1 B1 ), khi d ¯o ¯ê x´ ¯o ¯iêm Bn (o . , , , ´, `˘m trên canh ∆A1 A2 An ) c´ o thê na A2 A3 v` a A1 An tuong u ng (v`ı n > . , ,`, 3, d a A2 An l` ad a không l` a canh cua n-gi´ ac ¯oan ¯uong ch´eo, m` . A1 A3 v` . , ` ´ ˘ng A1 , . . . , An ), ngh˜ıa l` a B1 = A1 v` a B2 = A2 . Ta chung minh ra A1 B1 = A2 B2 = . . . = An Bn `˘ng nhau, v`ı Thât ac B1 A2 B2 v` a tam gi´ ac B2 A3 B3 ba . vây . tam gi´ 0n −2 \ \ B\ 1 A2 B2 = B2 A3 B3 = B1 B2 B3 = 180 n 0 \ \ A\ 2 B1 B2 = 180 − B1 A2 B2 − A2 B2 B1

\ \ = 1800 − B\ 1 B2 B3 − A2 B2 B1 = A2 B2 B3 ,, ´,ng minh tu,o,ng v` a B1 B2 = B2 B3 . Boi vây A2 B2 = A3 B3 . Ta c˜ ung chu . , , ,, ´,c c` ˘ng thu tu. c´ ac d on lai. ¯a . Ðai . luo. ng SB = S A − SB1 A2 B2 − SB2 A3 B3 − . . . − SBn A1 B1 = S A − nSB1 A2 B2 , , ´,n nhâ´t. Gia nhân a tri. nho nhâ´t, khi diên ac B1 A2 B2 lo . gi´ . t´ıch tam gi´ ,, ,, ´d su A1 A2 = a, A1 B1 = x, khi d ¯o ¯ai . luo. ng 1 1 \ B1 A2 .A2 B2 sin B\ 1 A2 B2 = ( a − x ) x. sin B1 A2 B2 2 2 1 a2 a = ( − ( x − )2 ) sin B\ 1 AB2 2 4 2 a ´,n nhâ´t khi x = , ngh˜ıa l` nhân a tri. lo a A1 B1 = B1 A2 . . gi´ 2 , , 2 . 13.19. (CHLB Ðu´c, 1978) Môt . bô. gô`m n con tem choi (n>2) mang , ˜ loai c´ ac nh˜ an hiêu an hiêu o n con tem. Hoi . "1","2","3",. . . ,"n". Môi . nh˜ . c´ SB1 A2 B2 =

J

148

,, Chuong 13. Môt ¯ê` thi vô d ¯ich . . sô´ d

, , , c´ o thê xê´p tâ´t ca c´ ac con tem â´y th` anh môt ay thoa m˜ an c´ ac d¯iê`u kiên . d˜ . , , ˜ a môt sau không : v´ oi moi an . x ∈ {1, 2, . . . , n} giu . con tem mang nh˜ , ´ ` a d¯u ´ng x hiêu a con tem mang nh˜ an hiêu o vu . "x" v` . "x" tiêp theo luôn c´ con tem mang nh˜ an hiêu ac "x" ? . kh´ , ˜ d `˘m gi˜u,a `,i giai. Trong môi Lo ¯oan . na B2 A3 A2 ´,i nh˜ hai con tem liên tiê´p vo an "n" B3 B1 , d o n con tem. C´ o n-1 d ¯ê`u c´ ¯oan . nhu A4 A1 ,, , ´ sô´ tem o gi˜ua con tem vây ¯o . do d B4 d an hiêu a con ¯â`u tiên mang nh˜ . "n" v` A5 ´,i c` tem cuô´i c` ung vo ung nh˜ an hiêu . , , ´i hai con tem v`ua n´ ´ công d vo oi l` a ¯o . 2 (n − 1)n + n, ngh˜ıa l`a n con tem. H`ınh 13.3: Vây a con tem ¯â`u tiên v` . con tem d , cuô´i c` ung trong d˜ ay d ung mang nh˜ an hiêu ¯ê`u phai c` . "n". , , ´i nh˜ Trong sô´ n con tem vo an "n-1" phai c´ o (theo nguyên l´y `˘m trong ´ıt nhâ´t môt Ðirichlê) ´ıt nhâ´t hai con tem c` ung na ¯oan . d . , ,, , , ˜ ´ trong sô´ n-1 d o an kê trên. Nhu ng khi d o o gi u a hai con tem n` a y ¯ . ¯ , , ˜ vo ´i t´ınh châ´t ´ mâu thuân chı c´ o nhiê`u nhâ´t l` a n-2 con tem; d ¯iê`u d ¯o , cua d˜ ay.

J

, , . 13.20. (CHLB Nga 1972) Ch´ın d¯u`o,ng tha˘ng c` ung c´ o t´ınh châ´t l` a , , , , , ˜ ´ môi d¯u`ong tha ˘ng chia h`ınh vuông th` anh hai tu´ gi´ ac c´ o ty sô diên . t´ıch , 2 , , , ´ c` ba ˘`ng . Chu´ng minh ra ˘`ng c´ o ´ıt nhâ´t ba d¯u`ong tha ˘ng trong sô´ d¯o ung 3 , d¯i qua môt . d¯iêm. , , , ,, ´˘t c´ `,i giai. (H`ınh 13.4) C´ ˘ng d ˜ cho không thê ca Lo ac d ac ¯u`ong tha ¯a , , ,, canh kê` nhau cua h`ınh vuông ABCD, boi v`ı nê´u thê´ không thê tao . . , ,, ,, , ,`, ´ ra d u o c hai t u gi´ a c, m` a l` a tam gi´ a c v` a ng˜ u gi´ a c. Gi a s u m ôt d u o ng ¯ . . ¯

149 , , ´˘t c´ ˘ng ca ac canh BC v` a AD tai ac d¯iêm M v` a N. C´ ac h`ınh thang tha . . c´ , ,`, ` ˘ng nhau do d ´ ty sô´ diên ABMN v` a CDN M c´ o c´ ac d ¯uong cao ba ¯o . , , ,`, , ` ´ ´ c l` ˘ng ty sô c´ t´ıch cua ch´ ung ba ac d a MN chia ¯uong trung b`ınh, tu , , , , 2 ˘ng nô´i trung d d ac canh AB v` a CD theo ty sô´ . ¯oan ¯iêm cua c´ . tha . 3, , , , ,, Tông sô´ c´ ac d¯iêm chia c´ ac d ¯u`ong trung b`ınh cua h`ınh vuông theo ty , , ,, 2 ,, ˘ng d ˜ cho l` sô´ l` a 4 (H`ınh v˜e). Boi sô´ d a 9 v` ad ¯u`ong tha ¯a ¯ê`u phai d ¯i 3 , , qua môt sô´ 4 d oi trên, nên c´ o môt ¯iêm n´ . trong . d¯iêm thuôc . ´ıt nhâ´t 3 , ,`, ˘ng. d ¯uong tha M

B

C

D

A N

H`ınh 13.4:

150

,, Chuong 13. Môt ¯ê` thi vô d ¯ich . . sô´ d

, , CHUONG

14

, , ` BAI TÂP . TU. GIAI

, ,, `˘ng 15 chu ´,a 11.000 d . 14.1. Môt o canh ba ¯iêm. . h`ınh lâp . phuong c´ . `˘ng c´ `˘ng 1 chu ´,ng minh ra ´,a ´ıt nhâ´t 6 Chu o môt an k´ınh ba ., h`ınh câ`u b´ , , ˜ cho. d ¯iêm trong sô´ nh˜ung d¯iêm d ¯a , , , ´ . 14.2. Cho F = { a1 , a2 , . . . , al } l`a tâp . ho. p h˜uu han . nh˜ung sô ,, , ,, nguyên duong v` a x1 , x2 , . . . , xn , . . . . . . l` a d˜ ay vô han, . moi . phâ`n tu cua , ` ´i môt ˘m trong F, ngh˜ıa l` ´ trong a1 , a2 , . . . , al n´ o na a tr` ung vo ao d ¯o . sô´ n` , , , , ` ´ ng minh ra ´i moi ˘ng vo . Chu duong bâ´t k`y tô`n tai môt . sô´ nguyên . sô´ , , ,. ,, , phâ`n tu liên tiê´p cua d˜ ay, m` a t´ıch cua ch´ ung l` a l˜ uy th`ua cua môt . sô´ ´. nguyên n` ao d ¯o , , . 14.3. Cho 5 d¯iêm P1 , P2 , P3 , P4 , P5 trong phâ`n trong cua môt . h`ınh , , , , vuông c´ o canh 1 d¯on vi. a khoang c´ ach gi˜ua hai d ¯iêm . K´y hiêu . . dij l` , `˘ng ´ıt nhâ´t c´ ´,ng minh ra Pi v` a Pj . Chu o môt trong sô´ c´ ac khoang c´ ach . √ , , , 2 , . gi˜ua c´ ac d ¯iêm nho hon 2 , ,, ,, , . 14.4. Trong môt c´ o canh 50 d ¯on vi. , ngu`oi ta ke d ¯u`ong . h`ınh vuông . , , gâ´p kh´ uc sao cho moi h`ınh vuông c´ o khoang c´ ach . d¯iêm trên canh . , , , , , , , `˘ng d ´ ng minh ra nho hon 1 d uc. Chu ai cua cua ¯ê´n d ¯u`ong gâ´p kh´ ¯ô. d` ,, ´,n ho,n 1248. d uc lo ¯u`ong gâ´p kh´ ´,i diên . 14.5. Cho A l`a môt ac lô`i vo a chu vi P, c` on r ¯a gi´ . d . t´ıch S v`

,, , , Chuong 14. B` ai tâp . tu. giai , , , ,, , ˘. t l` a sô´ thu. c duong. K´y hiêu a tâp ac d ¯iêm trong ma . ,M l` ., ho. p tâ´t ca c´ , , , ˜ ` ´i môi d ˘ng, sao cho vo pha ¯iêm R cua M tôn tai ¯iêm Q thuôc . môt . d . A , , , , ` ˘. c ba ˘ng r. H˜ m` a khoang c´ ach gi˜ua R v` a Q nho hon hoa ay t`ım diên . , t´ıch cua h`ınh M. 152

`˘ng 70, ngu,`o,i ta n´em ba h`ınh . 14.6. Trong h`ınh vuông c´o canh ba . , ´,i k´ıch thu,o ´,c 20x10, 25x15 v` ch˜u nhât a 30x30 v` a ba h`ınh tr` on b´ an . vo , , , ` ´ ng minh ra ˘ng trong h`ınh vuông c´ k´ınh 5. Chu o thê dich ., chuyên môt . , h`ınh tr` on b´ an k´ınh 5 sao cho n´ o không c´ o nh˜ung d ¯iêm trong chung , ´i 5 h`ınh d ˜ cho. vo ¯a , ,, uc A1 A2 . . . An v` a . 14.7. Trong ma˘. t pha˘ng cho môt ¯u`ong gâ´p kh´ . d , ,`, h`ınh tr` on b´ an k´ınh r, tâm h`ınh tr` on chuyên d trên d ¯ông ¯uong gâ´p . , ,`, kh´ uc n` ay. Cho L l` ad ai cua d uc, c` on F l` a h`ınh sinh ¯ô. d` ¯uong gâ´p kh´ , , ,, , ,`, ´,ng ra boi su. chuyên d cua h`ınh tr` on trên d uc. Chu ¯ông ¯uong gâ´p kh´ . , ´,c S( F ) ≤ 2Lr + πr2 . ˘ng thu minh bâ´t d ¯a , , , , , `˘m ˘ng na . 14.8. Tâp a ho. p môt ¯oan . ho. p ,M l` . sô´ h˜uu han ., nh˜ung d . tha , `˘ng δ trong (0,1). Khoang c´ ach gi˜ua môt ¯iêm trong M không ba . sô´ d , , ,, , , `˘ng tông d ´ ng minh ra ˜ ,od ai cua ¯ây 0 ≤ δ ≤ 1 . Chu ¯"ô. d` ¯oan . # nhung d 1 , [ ] + 1 ,, , ˘ng m` ´nh tha a ch´ ung tao a δ δ v` a su. d ¯a . nên M, không vuo. t qu´ 2 gi´ a n` ay l` a ch´ınh x´ ac. , , . 14.9. Trên d¯u`o,ng tr`on b´an k´ınh 1 cho n d¯iêm P1 , P2 , . . . , Pn v`a tâp . , ,, , , ´ ´ ˜ ˘ ho. p A tao b o i m ôt sô h u u h an cung không c a t nhau, m` a tô ng d ô d` ¯ . . . . ai , , ` ´ ng minh ra ˘ng cua ch´ ung l` a l ( A). Chu , 2kπ , (k = 1, 2, . . . , n − 1), th`ı c´ o thê quay tâp a) Nê´u l ( A) > . n , , , ,`, , , ´ a ´ıt nhâ´t k + 1 d ho. p A trên d on sao cho anh cua n´ o chu ¯uong tr` ¯iêm ˜ P1 , P2 , . . . , Pn . trong dây

153 , 2kπ b) Nê´u l ( A) < , (k = 1, 2, . . . , n − 1), th`ı c´ o thê quay tâp . n , , ,`, , , ´ a nhiê`u nhâ´t k − 1 ho. p A trên d on sao cho anh cua n´ o chu ¯uong tr` , ˜ P1 , P2 , . . . , Pn . d ¯iêm trong dây , , , ´,i diên ˘. t câ`u vo . 14.10. Cho tâp ¯iêm trên môt . ho. p A, nh˜ung d . ma . t´ıch ,, , , , , ` ` ´ ng minh ra ´ a ´ıt nhâ´t ´n hon nua diên ˘t câu. Chu ˘ng A chu lo a . t´ıch cua m , . , , hai d ¯â`u môt ¯u`ong k´ınh cua h`ınh câ`u. . d , , . 14.11. K´y hiêu a nh˜ung tâp a ch´ ung c´ o . N1 ,, N2 , . ,. . , Nn l` . ho. p, m` ,, ,, , , , , , ´i nguyên d d ac d boi n ¯uo. c sau khi nh˜ung d ¯ınh cua môt ¯uo. c t´ ¯ông . luo . , ˘. t v` ph´ep tinh a môt a k l` a sô´ tu. nhiên sao cho 1 ≤ k ≤ . tiê´n, A l` . bê` ma , ` ´ ng minh ra ˘ng n − 1. Chu , , , k , a) Nê´u S( A) > th`ı c´ o thê tinh tiê´n tâp ho. p A sao cho anh cua . . r , , , , ´,a ´ıt nhâ´t k + 1 d n´ o chu a nh˜ung d ¯iêm trong l` ¯ınh cua N1 , N2 , . . . , Nn . , k , b) Nê´u S( A) < th`ı c´ o thê tinh tiê´n tâp . . ho. p A sao cho r , , , , , , ´,a nhiê`u nhâ´t k − 1 d anh cua n´ o chu a nh˜ung d ¯iêm trong l` ¯ınh cua N1 , N2 , . . . , Nn . , ,, , . 14.12. Trong h`ınh vuông vo´,i canh 1 d¯on vi. du. ng d uc ¯u`ong gâ´p kh´ . , ,`, ,`, , ´ ´ ˘ ˘ sao cho moi h`ınh vuông cat d ¯uong thang song song voi canh ¯uong . d . , , , ,, ` ´ ´ ˘ gâp kh´ uc không qu´ a môt ai cua d ¯iêm. Chung minh rang d ¯ô. d` ¯u`ong . d , , gâ´p kh´ uc nho hon 2. , , , , ˘. t pha ˘ng cho 6 d ˘ng nô´i . 14.13. Trong môt ma tha ¯iêm. Nh˜ung d ¯oan . . , , ,, , `˘ng vo ´,ng minh ra ´,i ˘. p d ˘. c xanh. Chu c´ ac ca ¯iêm d ¯uo. c son mâ`u d ¯o hoa , c´ ach tao ac c´ o c´ ac canh . h`ınh nhu, vây . tô`n tai . ´ıt nhâ´t hai h`ınh tam gi´ . ` ` c` ung mâu. (c´ o thê hai tam gi´ ac kh´ ac mâu nhau).

154

,, , , Chuong 14. B` ai tâp . tu. giai

, , CHUONG

15

, , , ` ` GO ´ LOI GIAI VA .IY

, ,, , , ` go. i y 15.1. L`oi giai va ´ chuong 1

, , . 1.11. L`o,i giai: Nê´u tâ´t ca c´ac ông quan d¯ê`u quen nhau th`ı viêc . , ,`, , ` ´ ´ xêp b` an bôn nguoi nhu d o g`ı kh´ o kh˘ an. Gia c´ o ông A ¯ê ra không c´ , , ` v` a ông B không quen nhau. Tu 2n − 2 ông quan c` on lai ung nhu A . c˜ ,, v` a B c´ o ´ıt nhâ´t n ngu`oi quen. V`ı n + n = 2n = (2n − 2) + 2, th`ı tô`n , ,, ˘. t ´ ngu`oi xê´p d tai a D, m` a ho. quen A c˜ ung nhu B. Khi d ¯o ¯a . hai, ông C v` , ´ ´ ´ ˜ c´ o thê xêp A d a giua ho. l` aCd ¯ôi diên ¯ôi diên . B v` . D. ˜ d . 1.12. Go.,i ´y: Chia môt h`ınh vuông th` anh 48 d ¯oan ¯oan . canh . . môi . , ,, , ` ˜ 20m, khoang c´ ach giua 2 d a 0,6m, hai d ai ¯oan ¯oan ¯âu d` . l` . o hai d , 5,9m. (Vây =1000m=1km). Canh thu . 48.20m+47.0,6m+2.5,9m . , hai chia ra l` am 95 d ach hai d¯oan a 0,52m, hai d ¯oan, ¯oan . khoa ng c´ . l` . ` d â u d` a i 0,56m ( v ây 95.10m+94.0,52m+20,56m= 1000m=1km). ¯ . , , , 2 Nhu vây c´ o 48.95=4560 manh c´ o diên a sô´ cây chı c´ o . . t´ıch 200m m` , 4500, nên c` on ´ıt nhâ´t 60 ô nhu vây o cây n` ao. . không c´ , ´nh dâ´u t`u . 1.13. Go.,i ´y: Ta nh´om c´ac ng˘an c´o c` ung sô´ s´ ach v` ad ¯a , ´,a sô´ s´ 0,1,. . . ,9 (c´ ac ng˘ an chu ach ´ıt hon 10) v` a môt an c´ o 10 cuô´n. . ng˘ , ,, , ´ a c` Gia su không c´ o ba ng˘ an chu ung sô´ s´ ach, th`ı trong c´ ac nh´ om ta , ´nh dâ´u c´ d o ´ıt hon ba ng˘ an c´ o c` ung sô´ s´ ach (nhiê`u nhâ´t l` a 2), vây ¯a .

156

, ,, , `,i giai v` Chuong 15. Lo a go. i ´y

´,i 10 nh´ ´,i môt th`ı vo om trên 10.2=20 ng˘ an s´ ach công an 10 cuô´n . vo . ng˘ , ˜ d n˜ua không cho ta sô´ ng˘ an s´ ach l` a 25, dân ¯ê´n vô l´y. , , , . 1.14. Go.,i ´y: Ta lâ´y sô´ cua ôtô chia cho 10 th`ı d¯uo.,c sô´ du 0, 1, 2, 3, , 4, 5, 6, 7, 8, 9. V`ı c´ o 11 sô´ m` a 10 sô´ du th`ı theo nguyên l´y Ðirichlê , , , c´ o hai sô´ c´ o c` ung sô´ du. Nhu vây . hiêu . cua hai sô´ chia hê´t cho 10 , ngh˜ıa l` a n´ o c´ o c` ung môt ung. . ch˜u sô´ tân . c` , , . 1.15. L`o,i giai: Goi ac tram a A, B, C, D, c´ ac h`ınh . c´ . chuyên tiê´p l` , ,`, , , ,`, ´ ˜ ´ tr` on voi d a c´ ac d ac tram l` a v` ung bao phu ¯uong k´ınh l` ¯uong nôi giua c´ . , , cua môt trung tâm ph´ at s´ ong. Ch´ ung ta c´ o 4 h`ınh tr` on nhu vây a . , v` , . , , , `˘ng môt d `˘m trong tu ´ ng minh ra ´ gi´ phai chu ac cua 4 ¯iêm M bâ´t k`y na , ,, . , , , tram on. Thât ¯ê`u d ¯uo. c phu boi ´ıt nhâ´t môt ¯iêm M . d . h`ınh tr` . vây, . ,nê´u d ,, `˘m trong h`ınh tr` ˘. c oc AMB phai l` a t` u hoa na on d ¯u`ong k´ınh AB th`ı g´ , 0 `˘ng 90 . Ta nô´i M vo ´i c´ ´,i ba ac tram oc d ¯ô´i vo . A, B, C, D, tao . ra bô´n g´ , , , ´, gi´ c´ ac canh tu ac. Tông cua bô´n g´ oc n` ay l` a 3600 . Nhu vây o . . ´ıt nhâ´t c´ 0 ´˘m l` `˘ng 90 . Vây ˘. c c` ung la a ba on môt oc t` u hoa . g´ . M thuôc . môt . h`ınh tr` , , , , ´ gi´ ´i môt m` a tai cua tu ac duo oc t` u. . M nh`ın canh . . g´

, ,, , , ` go. i y 15.2. L`oi giai va ´ chuong 2 ,, . 2.11. Go.,i ´y: Áp dung Phuong ph´ ap b` ai 2.2. . ,, . 2.12. Go.,i ´y: Áp dung phuong ph´ ap b` ai 2.4 .

. 2.13. Go.,i ´y: Ða˘. t bk = a1 + a2 + · · · + ak , k = 1, 2, . . . , 41 v`a a´p dung b` ai tâp . . trên. , ´,i . 2.14. Go.,i ´y: Nhu b`ai 2.8, nê´u M l`a sô´ nguyên tô´ c` ung nhau vo , , , 10, th`ı t`u su. chia hê´t cua l − k = 111 . . . 11.10k cho M, suy ra sô´

, ,, , `,i giai v` 15.2. Lo a go. i ´y chuong 2

157

111 . . . 11} chia hê´t cho M. | {z ˜, sô´ 1) (k-l chu ,, . 2.15. Go.,i ´y: D` ung phuong ph´ ap b` ai 2.8. H˜ ay x´et d˜ ay N, NN, NNN, . . . , |NNN {z. . . N} . ˜, sô´ ) (1968 chu , . 2.16. L`o,i giai: Ta viê´t 1998 sô´ 1997, 19971997, . . . , ˜ , 1997 .{z . . 1997} . V` a x´et c´ ac sô´ du trong ph´ep chia môi |

sô´ 1997 la ˘. p 1998 lâ`n ´ cho 1998. R˜ ˜ viê´t môt o r` ang không môt ao trong c´ ac sô´ d ¯o ¯a . sô´ d . sô´ n` , , ˜˘n, m` ˜ viê´t l` chia hê´t cho 1998 (v`ı 1998 l` a sô´ cha a c´ ac sô´ d a le), nhu ¯a , , ´,n ho,n vây, ac sô´ du cua ph´ep chia d ac không, v`ı sô´ c´ ac sô´ lo ¯ê`u kh´ . c´ , , ,, sô´ c´ ac sô´ du (c´ o 1998 sô´ m` a sô´ c´ ac sô´ du l` a 1997), nên t`ım d ¯uo. c , , , ´ c´ hai sô´ c´ o sô´ du nhu nhau, hiêu o dang câ`n t`ım v` a ¯o . cua hai sô´ d . chia hê´t cho 1998.

, . 2.17. L`o,i giai: Ta x´et 1997 sô´ dang 1998, 19981998,. . . . Sô´ cuô´i . , , c` ung trong c´ ac sô´ n` ay tao anh t`u 1997 nh´ om trong bô´n ch˜u sô´ 1, . th` ˘. c môt ´ l` 9, 9, 8. Hoa ac sô´ n` ay chia hê´t cho 1997 (vây, a sô´ ¯o . trong c´ . d , ,, , , ˘. c t`ım d phai t`ım), hoa ¯uoc hai sô´ du nhu nhau trong ph´ep chia cho , . 4m a chia ´ hiêu 1997. Khi d ung c´ o dang ¯o . cua ch´ . 19981998 . . . 1998.10 v` ,, , ´ hê´t cho1997. V`ı 104m v` a 1997 nguyên tô´ c` ung nhau nên nhân tu thu nhâ´t, ngh˜ıa l` a sô´ 19981998. . . 1998, chia hê´t cho 1997. , , . 2.18. L`o,i giai: Trong n + 1 sô´ m, m2 , . . . , mn+1 t`ım d¯uo.,c hai sô´ , , , ´ hiêu c´ o sô´ du nhu nhau trong ph´ep chia cho n. Khi d cua ch´ ung ¯o . , ,, l t t l − t ´ ˘. c m (m − 1) = a.n. V`ı chia hêt cho n. Gia su m − m = a.n hoa t l − (m, n) = 1 nên (m , n) = 1, ngh˜ıa l`a, m t − 1 chia hê´t cho n. Vây . , l − t ´ m − 1 l` a sô phai t`ım.

158

, ,, , `,i giai v` Chuong 15. Lo a go. i ´y

, ,, , , ` go. i y 15.3. L`oi giai va ´ chuong 3

, , , . 3.11. L`o,i giai: Ta x´et 104 c´ac lu˜y th`ua kh´ac nhau cua 3: 3, 32 , 33 4 , , ˜ lu˜y th`u,a d ˜ ´ cho 104 . Môi , . . . , 310 v` a c´ ac sô´ du cua ph´ep chia môi ¯o , , sô´ cho môt ac không khi chia cho 104 ; sô´ c´ ac sô´ du kh´ ac . sô´ du kh´ , , , 4 4 ´, t`ım d không l` a 10 − 1; sô´ c´ ac sô´ l` a 10 . Do d ac ¯o ¯uo. c hai lu˜y th`ua kh´ , , m n 4 nhau 3 v` a 3 c´ o sô´ du nhu nhau trong ph´ep chia cho 10 , ngh˜ıa l` a m n 4 n m − n 4 n 4 ˘. c 3 (3 3 − 3 = 10 .l hoa − 1) = 10 .l. V`ı 3 v`a 10 nguyên tô´ , m − n c` ung nhau nên 3 − 1 chia hê´t cho 104 hoa˘. c 3m−n − 1 = 104 .k. T`u , , , , ´ suy ra 3m−n = 104 .k + 1. Nhu vây, d o thê. ¯o . tra l`oi: c´ , . 3.12. Go.,i ´y: L´y luân ai 3.10. . nhu b` ,, ´p dung . 3.13. Go.,i ´y: Ch´ u ´y u3 = 1986, a phuong ph´ ap b` ai 3.4. . , `˘ng vo ´,ng minh ra ´,i moi . 3.14. Go.,i ´y: L´y luân . nhu trong 3.4. Chu . sô´ , tu. nhiên m ´ıt nhâ´t môt ac sô´ x1 , x2 , . . . , chia hê´t cho m. Sau . trong c´ ˘. t m = 38 v` ´ da a m = 43. d ¯o ,, , , , , . 3.15. Go.,i ´y: Tuo,ng tu. nhu 3.8 ta x´et bô. xê´p s-phâ`n tu nh˜ung sô´ , , ,, `˘ng tô`n tai nh˜u,ng chı sô´ i v` ´,ng. D˜ê thâ´y ra du tuong u a j sao cho , s ˜ ` ´ 1 ≤ i < j ≤ k v` a môi tông trên d ¯êu chia hêt cho k.

, ,, , , ` go. i y 15.4. L`oi giai va ´ chuong 4 , 1 ,, ˜ d ˜ cho v˜e d . 4.11. Go.,i ´y: Quanh môi on b´ an k´ınh . ¯iêm d ¯a ¯u`ong tr` 15 , . 4.12. Go.,i ´y: B`ai to´an suy ra t`u b`ai 4.9. , , ,, , , , . 4.13. L`o,i giai: Gia su tô`n tai d¯iêm X t`u c´ac d¯iêm d¯a˜ cho m`a n´o nô´i , , , , ´, ˘ng XX1 ´ c´ d i6d ac d ¯uo. c vo ¯iêm X1 , X2 , X3 , X4 , X5 , X6 . Khi d ¯o ¯oan . tha

, ,, , `,i giai v` 15.4. Lo a go. i ´y chuong 4

159

,, , , trong hai c´ ach sau d o c´ ach v` a XX2 d ¯ây (thu. c ra c´ ¯uo. c du. ng theo môt . , , , , , , ´ ´ ba nhung l` thu a hê. qua cua (b) khi ta d ¯ôi chı sô) , (a) Nê´u X l` ad a A2 , th`ı XX1 < X1 X2 v` a ¯iêm gâ`n nhâ´t d ¯ê´n X1 v` , ´n XX2 < X1 X2 , ngh˜ıa l` a trong tam gi´ ac XX1 X2 c´ o canh X1 X2 l` a lo . nhâ´t. , , (b) Nê´u X1 gâ`n d¯iêm X nhâ´t, c` on X gâ`n d ¯iêm X2 nhâ´t, th`ı ´,n nhâ´t trong XX1 < XX2 < X1 X2 ch´ ung ta c˜ ung c´ o canh X1 X2 l` a lo . tam gi´ ac XX1 X2 ,, , 0 ´,ng minh ho` Suy ra X\ an to` an tuong tu. , ch´ ung ta 1 XX2 > 60 . Chu , , 0 \ \ \ \ \ ´n hon 60 , d c˜ ung c´ o X2 XX3 , X3 XX4 , X4 XX5 , X5 XX6 , X6 XX1 lo ¯iê`u , ,, , , , `˘ng 3600 . Nhu vây, n` ay không thê d¯uo. c v`ı tông c´ ac g´ oc n` ay phai ba . , , , , , , ˜ ´ ` ´ ´i 5 d môi d ¯iêm chı nôi d ¯uo. c nhiêu nhât vo ¯iêm thôi. , `˘ng nhau (c´ . 4.14. L`o,i giai: Ta chia h`ınh tr`on th`anh 6 h`ınh quat o . ba , ˜ ´ tai d on). Khi d o qu´ a ¯ınh tai ¯o . , tâm h`ınh tr` . môi môt . h`ınh ,quat, . không c´ , ,, , , môt ao (boi v`ı khoang c´ ach gi˜ua hai d ¯iêm roi v` ¯iêm bâ´t k`y trong môt . d , . , , ˜ ´ ´ h`ınh quat không l o n ho n 1). Nê u t ai m ôi h` ınh qu at c´ o m ôt d iê m th`ı ¯ ., . . . , ,, , , , ta c´ o thê t`ım d a g´ oc gi˜ua c´ ac b´ an k´ınh vecto cua ¯uo. c hai d ¯iêm m` , , ´,n ho,n 600 v` ´ khoang c´ ch´ ung không lo a do d ach gi˜ua ch´ ung không ¯o , , , , ´n hon 1. Do vây, lo o thê chon a5d ¯iêm. . c´ . không qu´ , ˜ h`ınh vuông ba `˘ng môt ´,n ho,n gio ´,i . 4.15. L`o,i giai: Ta thay môi . h`ınh lo , , ,, 1 ,, ,, han ach biên cua h`ınh vuông môt ¯u`ong c´ ¯u`ong . boi môt . d . khoang 2 (d , 1 , ˘ng d n` ay gô`m bô´n d a bô´n cung tr` on c´ o b´ an k´ınh ). ¯on vi. v` ¯oan . tha 2 π , ´ ,, ˜ môt ` ˘ ˜ Môi h` ınh nhu thê c´ o di ên t´ ıch b a ng 3 + , c` o n 120 h` ınh d a d uo. c ¯ ¯ . . 4 , π "viê`n ra" s˜e phu môt a 120.(3 + ) = 360 + 30π. . diên . t´ıch không qu´ 4 , , , ` ˘ ˜ Ta bao vây biên cua h`ınh ch˜u nhât d a cho b a ng m ôt o chiê`u . ¯ . dai c´

160

, ,, , `,i giai v` Chuong 15. Lo a go. i ´y

, , , 1 `˘ng 44. Nhu, vây, rông . Diên t´ıch cua dai ba diên t´ıch tông công . . . . . 2, , , , , `˘ng 360 + 30π + 44 = cua dai v` a tâ´t ca c´ ac h`ınh d ¯uo. c viê`n ra ba , , , ´ c l` 404 + 30π < 404 + 94, 5 < 500, tu a b´e hon diên . t´ıch cua ,h`ınh , , ´, trong h`ınh ch˜u nhât ch˜u nhât od 20.25 = 500). Do d ¯o ¯iêm O . (S = . c´ , ,, , ,, ˜d không bi. phu boi dai v` a c´ ac h`ınh vuông d a ¯a ¯uo. c viê`n ra. Ngh˜ıa l` , , , d ach bien cua h`ınh ch˜u nhât a c´ ach moi ¯iêm O c´ . v` . h`ınh vuông môt . , 1 1 , , ´n hon . H`ınh tr` khoang lo on b´ an k´ınh c´ o tâm tai a h`ınh tr` on . O l` 2 2 câ`n t`ım.

, ,, , , ` go. i y 15.5. L`oi giai va ´ chuong 5

, , ´,i . 5.11. L`o,i giai: X´et môt o giao vo . tâp . A bâ´t k`y t`u 1978 tâp. . V`ı n´ ,, , ´ıt hon 50 1977 tâp on lai, . c` . v`ı vây . tô`n tai . phâ`n tu a ∈ A, thuôc . không ,, , , , ˜ môt tâp ay (thât t`u 40 phâ`n tu cua tâp . ho. p n` . vây . nê´u môi . . A thuôc . , , , ` ´ ` không nhiêu hon 49 tâp, o không nhiêu hon 40.49 = 1960 . th`ı tât ca c´ ,, ` ´ ng). Vây tâp ac A, d ay không d ac tâp ¯iêu n` ¯u . kh´ . phâ`n tu a thuôc . c´ . , , , ´ ´ ho. p A, A1 , . . . , A50 . Ta s˜e chung minh n´ o s˜e thuôc . tâp . bât k`y B t`u , 1978 tâp. o hai tâp ao t`u c´ ac tâp . Thât . vây . không c´ . n` . A, A1 , . . . , ,A50 ,, , ´ ` ´ ac voi a (v`ı hai tâp o l` a c´ o phân tu chung kh´ . bât k`y giao nhau chı c´ , ,, ,, , ˜ ` ´ ´ môt phâ n t u chung). Gi a s u a ∈ B. Khi d o t âp B c´ o v o i m ôi t âp ¯ . . . ,, , ` ´ A, A1 , . . . , A50 môt phâ n t u chung kh´ a c v o i a, suy ra t âp B c´ o không . . , ,, ,, , , ` `n tu, a thuôc ´ıt hon 51 phâ`n tu, d iê u n` a y không thê d u o c. Suy ra phâ ¯ ¯ . . , , tâ´t ca c´ ac tâp h o p. . . , , . 5.12. L`o,i giai: Môt o c´ ac phâ`n du {0, 1, 2, . . . , 8}. . sô´ chia cho 9 th`ı c´ ,, Theo nguyên l´y Dirichlê mo rông, trong 55 sô´ chon o . . ra th`ı ´ıt nhâ´t c´ , , , môt om 7 sô´ khi chia cho 9 cho c` ung phâ`n du (nê´u nguo. c lai . nh´ . th`ı , ,, , c´ ac nh´ om chı c´ o 6 phâ`n tu vây a ta lâ´y ra nh˜ung 55 sô´). . 6.9=54 m` ´ l` Ch´ ung ta k´y hiêu ac sô´ d a a1 , a2 , a3 , . . . , a7 . V`ı ai+1 ≡ ai (mod 9), ¯o . c´

, ,, , `,i giai v` 15.6. Lo a go. i ´y chuong 6

161

, `˘ng ai+1 − ´,ng minh ra nên ai+1 − ai ∈ {9, 18, . . .}. Ch´ ung ta phai chu , ,, , , , , ´i môt ´i moi ´. Gia su nguo. c lai, a i = 9 vo ao d ¯o . i n` . vo . i, ai+1 − ,ai ≥ 18, ,, ` ` d ay ngh˜ıa l` a a7 − a1 ≥ 6.8 = 108. Ðiêu n` ay không thê d ¯iêu n` ¯uo. c v`ı ,, , , , a7 − a1 < 100. Nhu vây gi˜ua hai phâ`n tu cua a1 , a2 , a3 , . . . , a7 c´ o hai . , , ` ´ ˘ng 9. sô m` a hiêu ung phai ba . cua ch´ ´,ng minh nhu, b` . 5.13. Go.,i ´y: C´ach chu ai 5.10. , , , ,´, ` . 5.14. L`o,i giai: (Ban y h`ınh ) Ch´ ung ta xây du. ng luo i gôm . tu. v˜e lâ´√ , , ,, 3 , ˜ h`ınh luc nh˜ung h`ınh luc ac c´ o canh . Môi gi´ ac c´ o thê phu boi . gi´ . . √ 2 , ,, ,, ,, ´,i b´ on vo an k´ınh 3. C´ o thê t´ınh to´ an d d ¯uo. c sô´ luo. ng luc ¯u`ong tr` . , ´,i tam gi´ ˜ cho l` gi´ ac m` a ch´ ung c´ od ac d a 1+2+ ¯iêm chung vo ¯ê`u d ¯a 3 + · · · + 10 = 55. V`ı 111 = 55 × 2 + 1, nên c´ o môt ac trong . luc . gi´ , , ,´, , , ´ a ´ıt nhâ´t 3 d ˜ chon. luoi trên chu ¯iêm trong sô´ 111 d ¯iêm d ¯a . Nhu vây . ,, ´ ˜ d on bao luc gi´ a c n` a y c´ o t´ ınh châ t d a nêu. ¯u`ong tr` ¯ . , ,, , , ´, . 5.15. L`o,i giai: K´y hiêu a sô´ luo. ng ngu`oi lo n nhâ´t . A1 , A2 , . . . , Ak l` , , , , ´ hai ngu`oi n` m` a bâ´t cu ao c˜ ung không quen nhau gi´ an tiê´p. T`u d¯iê`u ˜ ngu,`o,i c` kiên ai ra k ≤ 7. Môi on lai om quen gi´ an tiê´p ´ıt . b` . trong nh´ , , , , , , , nhâ´t môt ung . ngu`oi trong A1 , A2 , . . . , Ak (tru`ong ho. p nguo. c lai . ch´ ,`, ,`, , ` ta c´ o nhiêu hon k nguoi, m` a hai nguoi không quen nhau gi´ an tiê´p). , , Môt o ´ıt nhâ´t 20 ngu`oi quen gi´ an tiê´p, v`ı . trong sô´ A1 , A2 , . . . , Ak c´ , , , , ,, , , ´ ng nhu vây, nê´u không d ung ta s˜e nhân ¯u ¯uo. c tông sô´ luo. ng ngu`oi . ch´ . d , , `˘ng tâ´t ca ˘ng d¯inh nhiê`u nhâ´t l` a 7.19 = 133 < 134. C` on lai ra . . kha , ,, ,, nh˜ung ngu`oi quen gi´ an tiê´p qua c` ung môt a quen gi´ an tiê´p. . ngu`oi l`

, ,, , , ` go. i y´ chuong 6 15.6. L`oi giai va

, , . 6.11. Go.,i ´y: Ðây l`a tru`o,ng ho.,p riêng cua b`ai 6.1 v`a b`ai 6.2.

162

, ,, , `,i giai v` Chuong 15. Lo a go. i ´y

1 . 6.12. Go.,i ´y: Sô´ d¯a˜ cho l`a x1 , x2 , x3 , x4 v`a yi = 1 + , i = 1, 2, 3, 4. xi , , `˘ng hai sô´ n` ´,ng minh ra Chı câ`n chu ao d an ¯â´y trong y1 , y2 , y3 , y4 thoa m˜ √ yi − y j , , , , ≤ 2 − 3. Phâ`n c`on lai 0≤ . tuong tu. nhu 2.6. 1 + 2yi y j , ,, ` ´,ng minh d . 6.13. L`o,i giai: Vo´,i d¯inh ˘ng nê´u môt ¯uo. c ra . l´y Fecma chu . , ,, 2 2 ´ ´ ´ sô nguyên tô p c´ o dang 4k + 3 chia hêt cho tông a + b , o d ¯ây . , , ´ ´ ´ ˜ ` a, b l` a nhung sô nguyên, th`ı p chia hêt cho tung sô a v` a b. H˜ ay , , 2 2 2 2 2 2 ´ ˘ d` ung c´ ac d¯ang thuc sau 2 = 1 + 1 , k ( a + b ) = (ka) + (kb)2 , ( a21 + b12 )( a22 + b22 ) = ( a1 a2 + b1 b2 )2 + ( a1 b2 − a2 b1 )2 . , , ˜ sô´ vo ´,ng môi ´,i c˘ . 6.14. L`o,i giai: Cho tu,o,ng u an bâc . hai cua ch´ınh , , ˜ sô´ tu,o,ng u ´,ng vo ´,i c˘ n´ o. Nhu vây khi phân t´ıch ra môi an bâc hai cua . . , , , , , sô´ le. M` a sô´ tâ´t ca c´ ac sô´ le nho hon 2k l` a k. Vây nguyên l´y . theo , , ´ ty sô´ Ðirichlê c´ o hai sô´ trong k + 1 sô´ c´ o c` ung phâ`n c˘ an sô´ le, do d¯o , , , cua n´ o s˜e l` a l˜ uy th`ua cua 2. , , . 6.15. L`o,i giai: Nê´u gi˜ua c´ac sô´ d¯a˜ cho c´o n sô´ m`a khi chia cho n , , , , ch´ ung cho nh˜ung phâ`n du kh´ ac nhau. Tông cua ch´ ung s˜e chia hê´t , , , , , , ´p dung cho n, v`ı n l` a sô´ le. Trong tru`ong ho. p nguo. c lai, nguyên l´y . a . Ðirichlê.

, ,, , , ` go. i y´ chuong 7 15.7. L`oi giai va

, , , ´, `˘ng moi . 7.11. L`o,i giai: Ch´ ung ta thâ´y ra n . sô´ nguyên tô´ thu. c su. lo , , , ´n hon ˘. c 6n + 5. V`ı ba sô´ nguyên tô´ lo hon 3 d o dang 6n + 1 hoa ¯ê`u c´ . , 3 lâp anh môt nên theo nguyên l´y Ðirichlê phai c´ o ´ıt . th` . câ´p sô´ công, . , ´ c l` ´ chia hê´t cho 6. Goi nhâ´t hai sô´ c` ung dang, tu a hiêu a ¯o . . hai, sô´ d . d l` , ˘. c l` ˘. c l` công sai cua câ´p sô´ công, th`ı hiêu a d, hoa a . . cua hai sô´ â´y hoa

, ,, , `,i giai v` 15.7. Lo a go. i ´y chuong 7

163

, , ˘. c l` ˘. c 2d|6. Ch´ a d|6 hoa u ´y công sai d l` a hiêu 2d. Nhu thê´ hoa . cua hai ˜˘n. Nhu, thê´ v`ı d|3 v` ´,n ho,n 3, nên n´ sô´ nguyên tô´ lo o l` a sô´ cha a d|2 suy ra d|6. , 10m . 7.12. L`o,i giai: Ta luôn c´o lim = ∞. N´oi c´ach kh´ac tô`n tai . m→+∞ 1998 m 10 ,, sô´ nguyên duong m0 sao cho ∀m ≥ m0 th`ı > 91998 . X´et môt . 1998 , ,, , , ´ ´ ´ sô nguyên duong n bât k`y v` a n c´ o k0 ch˜u sô n = a1 a2 . . . ak0 . Tru`ong ,, , ho. p n` ay ta c˜ ung chon ¯uo. c . d 10 N > (9k0 )1998 v` a N > m0

(15.1) , , (cu. thê c´ o thê lâ´y N = max{[1998lg(9k0 )] + 1, m0 + 1}) ´,ng minh - Ch´ ung ta s˜e chu ´,i moi ui (n) < 10 N vo (15.2) . i = 1, 2, . . . `˘ng qui nap. ´,ng minh ba Thât . vây, . ta s˜e chu . , ´i i = 1 ta c´ 1) Vo o u1 (n) = (k1 + · · · + k0 )1998 ≤ (9k0 )1998 < 10 N ´ ng khi i = 1. (do (15.1)). Vây ¯u . (15.2) d , ,, ´,i i = k, ngh˜ıa l` ´ ng vo 2. Gia su (15.2) d a uk (n) < 10 N . Ta c´ o ¯u , N ´ ´ uk+1 (n) = f (uk (n)). Theo gia thiêt qui nap ¯o . th`ı uk (n) < 10 . do d , , ´ ´ c l` uk (n) c´ o không qu´ a N ch˜u sô, tu a uk (n) c´ o dang . , ´i p ≤ N uk (n) = a1 a2 . . . a p vo Theo d ¯inh . ngh˜ıa th`ı uk+1 (n) = ( a1 + a2 + · · · + a p )1998 ≤ (9N )1998 . , Do N > m0 vây . t`u (1) suy ra 10 N > (9N )1998 . , T`u (15.3) v` a (15.4) suy ra uk+1 (n) < 10 N .

(15.3)

(15.4)

, ,, , `,i giai v` Chuong 15. Lo a go. i ´y

164

´,i i = k + 1. Theo nguyên l´y qui nap ´ ng vo Vây ¯u . (15.2) d . th`ı (15.2) , ´i moi ´ ng vo d ¯u . i = 1, 2, . . .. ,, ˘. n - D˜ ay vô han ac sô´ nguyên duong {ui (n)}, i = 1, 2, . . . bi. cha . c´ , , ,, N boi 10 nên theo nguyên l´y Ðirichlê phai tô`n tai . hai chı sô´ p < q sao cho u p (n) = uq (n) =⇒ u p+k (n) = uq+k (n),

∀k

´,i chu k`y N´ oi c´ ach kh´ ac d˜ ay ui (n), i = p, p + 1, .. l` a d˜ ay tuâ`n ho` an vo u p ( n ), u p +1 ( n ), . . . , u p + q +1 ( n ). , . 7.13. L`o,i giai: Cho a l`a môt a phâ`n . sô´ tu`y ´y, th`ı { a},= a − [ a] goi . l` , , ,, , le cua sô´ a, o d a phâ`n nguyên cua sô´ a. Xây du. ng d˜ ay ¯ây k´y hiêu . [ a] l` , , ´i nhu sau mo v1 = { u1 } v2 = { u1 + u2 } .... v n = { u1 + u2 + · · · + u n } ´,i moi R˜ o r` ang vo o 0 ≤ vk < 1. . k = 1, 2, . . . , n ta c´ , , Chia [0, 1) ra l` am n + 1 tâp . ho. p nhu sau 1 1 2 n ∆0 = [0, ); ∆1 = [ , ); . . . ; ∆ n = [ , 1) n+1 n+1 n+1 n+1 , , , Chı c´ o c´ ac kha n˘ ang sau xây ra ˘. c tô`n tai 1. Hoa a v k ∈ ∆ 0 ∪ ∆ n +1 : . k m` ´,c l` - Nê´u vk ∈ ∆0 , tu a 0 ≤ { u1 + u2 + · · · + u k } < 1 n+1 1 ⇔ [S] ≤ S < [S] + . n+1

1 ˘. t . Ða n+1

Sk = u1 + u2 + · · · + u k : ⇒ 0 ≤ { S } <

(15.5)

, ,, , `,i giai v` 15.7. Lo a go. i ´y chuong 7

165 , , Vây a sô´ nguyên gâ`n S nhâ´t, nên t`u (15.5) suy ra d ¯iê`u phai . [S] l` ´,ng minh. chu n ´,c l` -Nê´u vk ∈ ∆n tu a ≤ {u1 + u2 + · · · + uk } < 1 hay l`a n+1 n n ≤ {S} < 1 ⇔ ≤ S − [S] < 1 hay l`a n+1 n+1 n + [S] ≤ S < [S] + 1. (15.6) n+1 , T`u (15.6) suy ra [S] + 1 l` a sô´ nguyên gâ`n S nhâ´t v` a kh´ ac n´ o môt . n 1 ,, luo. ng ≤ [S] + 1 − ([S] + )= . n+1 n+1 ˘. c l` 2. Hoa a c´ ac sô´ v1 , v2 , . . . , vn 6∈ ∆0 ∪ ∆n , vây . v1 , v2 , . . . , v n ∈ n −1 ∪i=1 ∆i . Theo nguyên l´y Ðirichlê tô`n tai ung môt . vk , vl , k > l, thuôc . c` . , ´ tâp h o p ∆ n` a o d o , 1 ≤ j ≤ n − 1. ¯ j . . ´ c´ Ðiê`u d o ngh˜ıa l` a ¯o j j+1 ≤ { u1 + u2 + · · · + u l } < n+1 n+1 j j+1 ≤ { u1 + u2 + · · · + u k } < n+1 n+1 ˘. t Sk = u1 + u2 + · · · + uk v` Ða a Sl = u1 + u2 + · · · + ul ta c´ o j+1 j j+1 j ≤ Sk − [ Sk ] < ⇒ [ Sk ] + ≤ Sk < [ Sk ] + n+1 n+1 n+1 n+1 j j+1 j j+1 ≤ Sl − [ Sl ] < ⇒ [ Sl ] + ≤ Sl < [ Sl ] + n+1 n+1 n+1 n+1 1 1 [ Sk ] − [ Sl ] − ≤ Sk − Sl ≤ [ Sk ] − [ Sl ] + n+1 n+1 1 1 [ Sk ] − [ Sl ] − ≤ u l +1 + u l +2 + · · · + u l ≤ [ S k ] − [ S l ] + n + 1 n + 1 , Nhu vây ay con ul +1 + ul +2 + · · · + ul c´ o sô´ nguyên gâ`n nhâ´t l` a . d˜ 1 , [Sk ] − [Sl ] vo´i d¯ô. lêch a . . không qu´ n+1 , ,, , T´ om lai, o tô`n tai ay con thoa . trong moi . tru`ong ho. p ta d¯ê`u c´ . d˜ m˜ an yêu câ`u d ¯ê` ra.

166

, ,, , `,i giai v` Chuong 15. Lo a go. i ´y

, ˜ sô´ p1 = 1a = a, p2 = 2a, p3 = 3a,. . . , . 7.14. L`o,i giai: X´et dây , , , ´, pm−1 = (m − 1) a. K´y hiêu q1 , q2 , . . . , qm−1 l` a c´ ac sô´ du tuong u ng . , ˜ ` cua dây trên chia cho m. V`ı theo d ai to´ an a v` a m l` a nguyên ¯iêu kiên . b` , , ´ ´ ´ ` tô c` ung nhau, th`ı tât ca c´ ac sô du trên d ac không. Nê´u môt ¯êu kh´ . , , , , , ` ´ ´ ´ ˘ng 1 th`ı b` ˜d trong c´ ac sô du ba ai to´ an d ac sô´ ¯a ¯uo. c giai. Nêu tât ca c´ , ` , ˜ sô´ trong m − 1 sô´ du q1 , q2 , . . . , qm − 1 ´ môi du d¯êu kh´ ac 1. Khi d ¯o `˘ng môt ba l´y Ðirichlê . trong m − 2 sô´ 2, 3, . . . , m − 1. Theo nguyên , , ` ´ ´ ˘ng nhau, cha ˘ng han c´ o ´ıt nhât hai sô trong q1 , q2 , . . . , qm−1 ba . nhu , ´ suy ra sô´ ( j − i ) a = p j − pi chia hê´t cho m, qi = q j , i < j. T`u d ¯o , , , , d ay không thê xây ra. Su. vô l´y n` ay do gia thiê´t không môt ¯iê`u n` . sô´ , `˘ng 1, do d ´ suy ra ´ıt nhâ´t môt du trong q1 , q2 , . . . , qm−1 ba ac ¯o . trong c´ , , ` ´ ˘ng 1. sô du phai ba

, ,, , , ` go. i y 15.8. L`oi giai va ´ chuong 8

, , ´,ng minh a + b = c. T`u, ung ta s˜e chu . 8.11. L`o,i giai: Truo´,c tiên ch´ , [ an] 1 ´,c 0 ≤ an − [ an] < 1 suy ra | ˘ng thu bâ´t d − a| < vo´,i moi ¯a . n≥1 n n , [ an] ´,c tu,o,ng tu., ˘ng thu = a. Ch´ ung ta c˜ ung c´ o bâ´t d sao cho limn→∞ ¯a n , , [cn] [ an] [bn] ´,c ´,i han ˘ng thu + = cho b v` a c. Ch´ ung ta lâ´y gio ¯a . cua d n n n ,, v` a nhân ¯uo. c a + b = c. . d ,, ˘. t a = [ a] + α, b = [b] + β o d ´ Ða ¯ây 0 ≤ α < 1, 0 ≤ β < 1. Khi d ¯o [ an] + [bn] = [([ a] + α)n] + [([b] + β)n] = [ a]n + [b]n + [αn] + [ βn], [( a + b)n] = [([ a] + [b] + α + β)n] = [ a]n + [b]n + [(α + β)n]. , , , `˘ng nê´u a ´,ng minh ra ˘. t lai Nhu vây ai to´ an c´ o thê d ¯a . b` . nhu sau: Chu , , , , ´,c ˘ng thu v` a b l` a nh˜ung sô´ trong khoang [0, 1) m` a ch´ ung thoa m˜ an d ¯a

[ an] + [bn] = [( a + b)n] , `˘ng 0. ´,i moi ´ ba vo ¯o . sô´ tu. nhiên n, th`ı ´ıt nhâ´t môt . trong hai sô´ d

(15.7)

, ,, , `,i giai v` 15.8. Lo a go. i ´y chuong 8 167 , ,, , ´,ng minh tô`n tai a b 6= 0. Ch´ ung ta s˜e chu Gia su a 6= 0 v` . sô´, tu. , ´,c (15.7) không d ˘ng thu ´ ng. Không mâ´t t´ınh tông nhiên n sao cho d ¯a ¯u , , `˘ng a + b < 1, v`ı nê´u ngu,o.,c lai qu´ at ch´ ung ta c´ o thê gia thiê´t ra . th`ı , ´i n = 1. ´ ng thâm (15.7) s˜e không d ¯u . ch´ı vo , , , ,`, , ´ c´ a b l` a nh˜ung sô´ h˜uu ty. Khi d o a) Ch´ ung ta x´et truong ho. p a v` ¯o , , , A B ,´, , , i dang thê biêu di˜ên ch´ ung duo a N l` a nh˜ung ¯ây A, B v` . a = N, N, o d , , sô´ nguyên v` a thoa m˜ an 0 < A < N, 0 < B < N. D˜ê d` ang kiêm tra , `˘ng (15.7) không thoa m˜ ´,i n = N − 1. Thât ra an vo . vây, .

[ a( N − 1)] = [ aN − a] = [ A − a] = A + [− a] = A − 1, [b( N − 1)] = [bN − b] = [ B − b] = B + [−b] = B − 1, [( a + b)( N − 1)] = [ A + B − ( a + b)] = A + B − [ a + b] = A+B−1 , Nhu vây . [ a( N − 1)] + [b( N − 1)] 6= [( a + b)( N − 1)]. , ,, , trong c´ ac sô´ a v` a b l` a sô´ vô ty. b) Ch´ ung ta x´et tru`ong ho. p môt . , , `˘ng c´ Ch´ ung ta s˜e chı ra ra o môt a . sô´ tu. nhiên n, m`

{ an} + {bn} ≥ 1. (15.8) , , ´,c (15.8) s˜e suy ra không thê c´ ˘ng thu ´d Khi d¯o o (15.7). ¯a , ,, , , , a môt b` ai 8.1 tâp Gia su a l` . sô´ vô ty, theo . ho. p nh˜ung sô´ { an}, n = , , ´,n 1, 2, . . . l` a tr` u mât a sô´ tu. nhiên lo . trong khoang (0, 1). K´y hiêu . k l` , ´ tô`n tai nhâ´t m` a a + kb < 1. Khi d a ¯o . sô´ tu. nhiên n m` a + kb < { an} < 1. , , Nê´u {bn} ≥ b, th`ı t`u d ¯inh . ngh˜ıa cua k suy ra

(15.9)

{ an} + {bn} > ( a + kb) + b = a + (k + 1)b ≥ 1 , , ´,c (15.8) d ˘ng thu ´ ng. hay n´ oi c´ ach kh´ ac tô`n tai ad ¯a ¯u . sô´ tu. nhiên n m`

, ,, , `,i giai v` Chuong 15. Lo a go. i ´y , , ´,i viêc ˘ng Nê´u {bn} < b, th`ı vo ao c´ ac vê´ cua bâ´t d ¯a . thêm 1 − b v` ,, ´,c 0 ≤ {bn} < b, ch´ thu ung ta nhân ¯uo. c . d

168

1 − b ≤ b(n − 1) − [bn] + 1 < 1 ´ c´ Ðiê`u d o ngh˜ıa l` a [b(n − 1)] = [bn] − 1 v` a {b(n − 1)} ≥ 1 − b. ¯o ,, , `, Ngo` ai ra, tuong tu. tu (15.9) suy ra kb < a(n − 1) − [ an] < 1 − a < 1 , ´ v`ı thê´ [ a(n − 1)] = [ an] v` a { a(n − 1)} > kb. Nhung khi d ¯o

{ a(n − 1)} + {b(n − 1)} > kb + (1 − b) = 1 + (k − 1)b ≥ 1, ,, , , ´ ng cho sô´ tu. nhiên n − 1. v`ı k ≥ 1. Trong tru`ong ho. p n` ay (15.8) d ¯u , , ˜ d Nhu vây ac sô´ a v` a b l` a sô´ vô ty. ¯ê´n vô l´y khi môt . dân . trong c´ , , , `˘ng {n} chı c´ . 8.12. Go.,i ´y: Ch´ u ´y ra o thê nhân a tri. . . h˜uu han . gi´ ,, , , . 8.13. Go.,i ´y: L´y luân . tuong tu. nhu 8.4. 2 aa . 8.14. Go.,i ´y: X´et c´ac sô´ c´o dang ´p dung 8.4. . n = 2k v` .

, ,, , , ` go. i y´ chuong 9 15.9. L`oi giai va . 9.11. Go.,i ´y: X´et c´ac sô´ kx − [kx ](k = 0, 1, 2, . . . , n) v`a sô´ 1, chia ,, ´p dung d on lai phuong ph´ ap d ¯oan ¯oan . [0,1] ra n + 1 phâ`n, phâ`n c` . a . . n` ay. , ˜ c´ . 9.12. Go.,i ´y: Vo´,i môi ach chon . nh˜ung sô´ q1 , q2 , . . . qn tô`n tai . sô´ , , ´i n´ nguyên p, vo o 0 ≤ q1 x1 + q2 x2 + · · · + qm xm − p < 1. Bây gi`oi m oan con ba `˘ng nhau. chia d ¯oan ¯ . . [0,1] ra (n + 1) d

, ,, , `,i giai v` 15.10. Lo a go. i ´y chuong 10

169 √ √ √ p , . 9.13. Go.,i ´y: H˜ay x´et hai tru`o,ng ho.,p 2 − > 3 − 2 v`a q √ √ √ p 2− ≤ 3− 2 q , , . 9.14. Go.,i ´y: Tuo,ng tu. b`ai trên. , , . 9.15. Go.,i ´y: Gia thiê´t vo´,i moi a q thoa m˜ an . p v` 2 2 |mp + npq + sq | ≥ 1.

, ,, , , ` go. i y´ chuong 10 15.10. L`oi giai va

, , , ´,ng . 10.11. L`o,i giai: Ðây l`a su. tông qu´at h´oa b`ai 10.3. C´ach chu ,, , ,´, ´, minh ho` an to` an tuong tu. . Ch´ ung ta c˜ ung tao i voi c´ ac h`ınh . ra luo ´ ´ ´ lây môt vuông c´ o diên vuông l` am gôc rô`i tinh ¯o . . t´ıch 1. Sau d ., h`ınh , , ´ ` ´ tiên c´ ac h`ınh vuông c´ o chua c´ ac manh cua A vê h`ınh vuông gô´c. , , , , ´,n ho,n n. Nhu vây . tiê´n cua A s˜e lo . tông diên . t´ıch nh˜ung phâ`n tinh ,, Theo nguyên l´y Ðirichlê mo rông cho diên o ´ıt nhâ´t . t´ıch suy ra c´ ,. , , ´i h`ınh vuông gô´c ˜ tinh n + 1 trong sô´ nh˜ung phâ`n cua A d¯a . tiê´n to , , , , , ac h`ınh vuông nho ban d c´ od ¯iêm chung ( x0 , y0 ). Nh˜ung d ¯iêm t`u c´ ¯â`u , tinh ay c´ o toa. d a ( xi , yi ) (i = 1, 2, . . . , n + 1), m` a ¯ê´n d ¯iêm n` ¯ô. l` . tiê´n d , ´ ˜ xi − x j v` a yi − y j (i, j = 1, 2, . . . , n + 1) l` a nhung sô nguyên. , ´,a bê` ma ˘. t A . 10.12. L`o,i giai: Ch´ ung ta x´et h`ınh vuông V, m` a n´ o chu , ´i d ˘. t B = V \ A. Ch´ v` a c´ o canh vo ai n nguyên. Ða ung ta thâ´y ngay ¯ô. d` . , 2 2 ´ S( B) > n − k. Bây gi`o a ´p dung S(V ) = n v` a S( A) < k, do d b` ai ¯o . , , 2 ´ a ´ıt nhâ´t n − k + 1 10.11. V`ı thê´ B c´ o thê tinh o chu . tiê´n sao cho n´ , , , , ´i toa. d d A1 , V1 , B1 l` a c´ ac anh cua ph´ep tinh ¯iêm vo ¯ô. nguyên. K´y hiêu . . , , , ´, tiê´n trên cua A, V, B tuong u ng. R˜ o r` ang B1 = V1 \ A1 . Ch´ ung ta s˜e , `˘ng A1 chu ´,ng minh ra ´,a nhiê`u nhâ´t k − 1 d ´,i toa. d chu ¯iêm vo ¯ô. nguyên.

, ,, , `,i giai v` Chuong 15. Lo a go. i ´y , ,, , ,, ´,i toa. d Thât ung ta c´ ok d ¯iêm vo ¯ô. nguyên . vây, . Gia su nguo. c lai . ch´ , , , 2 ´ ´ a ´ıt nhât n − k + 1 d¯iêm trong A1 . V`ı theo c´ ach du. ng trên B1 chu ´,i toa. d ´ trong vo a B1 ⊂ V1 , A1 ⊂ V1 , A1 ∩ B1 = ∅, Khi d ¯ô. nguyên v` ¯o , , , 2 ´ ` ´ a ´ıt nhât n + 1 d ´i toa. d h`ınh vuông V1 chu ay ¯iêm vo ¯ô. nguyên, d ¯iêu n` , , ˜ ´ không thê xây ra, dân d ¯ên vô l´y. 170

, , , ´, . 10.13. L`o,i giai: C˜ ung nhu b` ai 10.5 t´ ac dung lên A ph´ep vi. tu. vo i . 1 , , , , 0 ´i A . tâm tai a hê. sô´ , ch´ ung ta nhân ¯ô. v` ¯uo. c tâp . gô´c ta. d . d . ho. p mo 2 , , V`ı S( A) > 4n nên S( A0 ) > n. Theo nguyên l´y Ðirichlê mo rông . , , 0 chu ´ ´ ´ cho diên t´ ıch A a ıt nhâ t n + 1 d iê m kh´ a c nhau ( x , y ) ( i = ¯ i i . , 1, 2, . . . , n + 1) m` a xi − x j v` a yi − y j l` a nh˜ung sô´ nguyên. , , , ´,a tâ´t ca c´ Ch´ ung ta x´et d ac lô`i nho nhâ´t chu ac d ¯a gi´ ¯iêm trên. D˜ê , , , ˜ d `˘ng môi d` ang thâ´y ra ac l` a môt trong c´ ac d ¯ınh cua d ¯a gi´ ¯iêm ( xi , yi ). . , , , Nh˜ung d ac không thuôc ao m` a hai d¯â`u l` a ¯ınh cua d ¯a gi´ ¯oan . trong d . n` , , , , ` ´ ` ˘m trong d nh˜ung d ac. V`ı thê trong nh˜ung d ¯iêm na ¯a gi´ ¯iêm ( xi , yi ) tôn , , , ` ´ ˘m trong d ˘ng nôi hai d tai a không na tha ¯iêm m` ¯oan ¯iêm trong . môt . d . , , ,, , , ˜ chon. ´ l` tâp Gia su d a ( x1 , y1 ). C˜ ung theo b` ai ¯iêm d ¯a ¯iêm d ¯o . ho. p d . , , , ´ ´i toa. d 10.5 cho ta n d ac nhau v` a kh´ ac d ¯iêm kh´ ¯iêm gôc toa. d ¯ô, ¯ô. . vo ` ` ˘m trong A. V`ı nguyên ( xi − x1 , yi − y1 ), i = 2, 3, . . . , n + 1, d ¯êu na , , ´ ´ ng nên c´ t´ınh d ac d ¯ô i x u ¯iêm ( x1 − xi , y1 − yi ), i = 2, 3, . . . , n + 1, c˜ ung thuôc . A. , ´,ng minh nh˜u,ng d Ch´ ung ta s˜e chu ac d ach trên l` a ¯iêm x´ ¯inh . theo c´ , ,, , , , kh´ ac nhau t`ung d ung ta c´ o ¯ôi mô. t. Thât . vây, . gia su nguo. c lai . ch´

( x i − x1 , y i − y1 ) = ( x1 − x j , y1 − y j ) , 1 ´,i hai chı sô´ n` ´ i, j m` vo ao d a i, j = 2, 3, . . . , n + 1. V`ı vây ¯o . x1 = 2 ( x i + 1 x j ) v` a y1 = (yi + y j ). Nê´u i = j th`ı ( x1 , y1 ) = ( x j , y j ), vô l´y v`ı 2

, ,, , `,i giai v` 15.11. Lo a go. i ´y chuong 11

171

, , , ˘ng nô´i hai 1 6= j. Nê´u i 6= j, th`ı ( x1 , y1 ) l` a trung d tha ¯iêm cua d ¯oan . , , ,, d a ( x j , y j ), d ay không thê d ach chon. ¯iêm ( xi , yi ) v` ¯iê`u n` ¯uo. c theo c´ . , , , , `˘ng A chu ´ ng minh trên chı ra ra ´ a ´ıt nhâ´t 2n d Chu ac nhau ¯iêm kh´ , , v` a kh´ ac gô´c toa. d o toa. d ung ¯ô, ¯ô. nguyên. Nhung d ¯iêm gô´c toa. d ¯ô. c˜ . c´ , `˘m trong tâp na a c´ o toa. d ai to´ an. ¯ô. nguyên, suy ra kê´t luân . A v` . cua b` ´ dung . 10.14. Go.,i ´y: Ap b` ai tâp . . 10.13. ´,ng minh tu,o,ng tu., 10.9. . 10.15. Go.,i ´y: C´ach chu

, ,, , , ` go. i y´ chuong 11 15.11. L`oi giai va , ´,c . 11.11. a) T`u b`ai 11.3 suy ra 6 = 9 − (S12 + S23 + S13 ) + S123 , tu ´ môt l` a S12 + S23 + S13 = 3 + S123 ≥ 3. Do d ac sô´ S12 , S23 , S13 ¯o . trong c´ , , không nho hon 1. ,, , , , ´,c l` b) L`oi giai: T`u b` ai 11.3 suy ra 5 ≥ 9 − M2 , tu a M2 ≥ 4. Boi v`ı , , 8 ,, , ˘. p, diên t`u 9 d ac c´ o thê tao t´ıch phâ`n chung cua ¯a gi´ ¯uo. c 9. = 36 ca . d . 2 , , M2 1 ´ ˘. p d ´ không nho hon ≥ . môt ac ca ¯o . sô c´ 36 9 , , ˜ ˜ cho. Trong môi . 11.12. L`o,i giai: Goi a6d ¯iêm d ¯a . M1 , M2 , . . . , M6 l` , , , `˘ng mâ`u d ´n nhâ´t ba tam gi´ ac Mi M j Mk ta tô canh a ¯o. Kê´t qua s˜e l` . , lo , , , ˘ng Mr Ms l` môt ad on c´ ac canh ac s˜e không d ¯oan ¯o c` ¯o. Chı câ`n . d . tha . kh´ , , , `˘ng tô`n tai ´ ng minh ra ˜ cho v` chu ac c´ od a c´ ac d a ¯ınh l` ¯iêm d ¯a . , môt . tam gi´ , ,, , , ´n nhâ´t cua tam gi´ ba canh d lo ac nhu ¯uo. c tô mâ`u d ¯o. Thât . . vây, ., canh . , ,, , thê´ d a canh ac kh´ ac v`ı n´ od ¯ô`ng th`oi l` ¯uo. c tô mâ`u . nho nhâ´t cua tam gi´ , , , , , ˜ ´,i c´ ˘ng nô´i n´ ˜ cho xuâ´t ph´ o vo ac d d d at 5 d ¯o. T`u môi ¯iêm d ¯a ¯oan ¯iêm . tha , , ,, ˘. c ´ıt nhâ´t c´ ˘ng n` c` on lai. o 3 trong nh˜ung d ay d ¯oan ¯uo. c tô . , Vây . hoa . tha , ,, ˜ không d ˘. c ´ıt nhâ´t c´ mâ`u d o3d ¯o, hoa ¯oan ¯uo. c tô mâ`u d ¯o. . vân

, ,, , `,i giai v` Chuong 15. Lo a go. i ´y , , , , ,, ˘ng han Nê´u t`u d at 3 d ¯iêm M1 xuâ´t ph´ ¯oan ¯uo. c tô mâ`u d ¯o (cha . d . ´ ´,n M1 M2 , M1 M3 , M1 M4 ) th`ı trong ∆M2 M3 M4 ´ıt nhât c´ o môt canh (lo . . , , ,, ,, ´ l` nhâ´t) d a d¯oan ¯uo. c tô mâ`u d ¯o, gia su d¯o . M, 2 M3 . Thê´ th`ı trong , , , ∆M1 M2 M3 tâ´t ca c´ ac canh d ¯ê`u d ¯uo. c tô mâ`u d¯o. . , , ,, Nê´u t`u d at ´ıt nhâ´t 3 d ¯iêm M1 xuâ´t ph´ ¯oan . không d¯uo. c tô mâ`u , , ˘ng han d ac ¯o (cha ¯oan . 3d . M1 M2 , M1 M3 , M1 M4 ) th`ı ta x´et 3 tam gi´ ˜ tam gi´ M1 M2 M3 , M1 M2 M4 , M1 M3 M4 . Trong môi ac n` ay ´ıt nhâ´t c´ o , , ,, , , ´a d môt d không chu ¯uo. c tô mâ`u d ¯o, nhung canh ¯ınh M1 . Vây . canh . . , ,. ,, , ` ´ 3 d o an M M , M M , M M d u o c tô mâ u d o, t u c l` a 3 c anh c ua ¯ . ¯ 2 3 2 4 3 4 ¯ . . , ` ` ∆M2 M3 M4 d ê u mâ u d o. ¯ ¯ 172

, , , . 11.13. L`o,i giai: Truo´,c hê´t ta nhân x´et c´ o ´ıt nhâ´t môt tam gi´ ac d ¯o n . . , , ,, , , , ´ ´˘c (B` ˘c, cha ˘ng han sa ai 5.4). Gia su P1 P2 P3 l` a tam gi´ ac d d ¯on sa ¯o. Ðê . , , ´ ˘c không riêng P1 ta c` on lai 6d a ta lai c´ o môt tam gi´ ac d ¯iêm v` ¯on sa . . . , , ´ ´ a canh c´ o P1 l` ad ai 5.4). Nêu tam gi´ ac n` ay không chu P2 P3 th`ı ¯ınh (B` . , , ,, ´ , , 0 0 ´ ´ tu P1 sao cho P1 P2 P3 c˜ ta d o môt ung ¯uo. c kêt qua, thê th`ı c´ ¯iêm thu . d , ,, , , ` ,, 0 0 l` a tam gi´ ac d a P3 ta d a P3 sao cho ¯o. Tuong tu. d ¯ê riên P2 v` ¯uo. c P2 v` , 0 0 P1 P2 P3 v` a P1 P2 P3 l` a tam gi´ ac d ¯o. , , ,´, ´ Truoc hêt ta x´et kha n˘ angl` a3d a P30 không phân biêt. ¯iêm P10 , P20 v` . ,`, , , ´ Trong truong ho. p n` ay môt trong ch´ u ng t ao th` a nh v o i P P P m ôt 1 2 3 ., . ,. , , ´ diên. h`ınh tu on lai 3 d¯iêm. Nê´u c´ ac canh ¯iêm . C` . , gi˜ua môt . trong ba d , . , , , , ´ diên ´ v` c` on lai d a2d ad ac ¯o ¯ınh cua h`ınh tu ¯o ta s˜e d ¯uo. c hai tam gi´ . l` , ,. , , , ´i 2 canh ´i v` d r`oi nhau, môt ac c´ o môt a môt ¯o vo ¯ınh mo . . tam gi´ . d . canh . , , , , ˜ ´ ´ diên ´ cua tu v` a tam gi´ a c kia trên t u di ên. Nê u không, m ôi d iê m c´ o ´ıt ¯ . , . , , , , , ´ diên. ´ diên nhâ´t canh tu o ´ıt nhâ´t môt ¯o thuôc ¯ınh cua tu . , d . Nhu thê´ c´ . d . , . , ` ` ´ ´i ca 3 d ˘ ˘ nô´i vo ınh c` o n l ai b a ng c anh mâ u d en. Nê u c a p n` a o trong 3 ¯ ¯ . . . , `˘ng canh nô´i ba o môt tam gi´ ac d a môt ¯en th`ı ta c´ ¯en v` ¯o. Nê´u không . d . d , ,. , ,, ta d ac d od ¯uo. c hai tam gi´ ¯o c´ ¯ınh r`oi nhau.

, ,, , `,i giai v` 15.12. Lo a go. i ´y chuong 12 173 , , ,, , `˘ng ba d Vây ta c´ o thê gia su ra a P30 l` a phân biêt. ¯iêm P10 , P20 v` . . Nê´u, , , , , , , `˘ng canh 2d a P20 , d d ac d ¯iêm, P10 v` ¯uo. c nô´i ba ¯o th`ı ta d ¯uo. c hai tam gi´ ¯o . , 0 0 0 0 0 ´ ´ tu., P1 P2 P3 v` a P1 P2 P3 . Nêu không th`ı P1 P2 P3 v` a P1 P2 P3 cho theo thu , môt ac d a môt ac d ¯o v` ¯en. . tam gi´ . tam gi´ , , , , ´, hai cua . 11.14. L`o,i giai: Lâ´y 1998 d¯˜ıa d¯uo.,c tô giô´ng nhu d¯˜ıa thu , ´, nhâ´t sao cho ch´ ˘. t chô`ng tâ´t ca ch´ ch´ ung ta v` a d¯a ung lên d ung ¯˜ıa thu , , , ˜ h`ınh quat ´ trên môi c´ o tâ´t ca c´ ac vi. tr´ı c´ o thê (nhu khi quay). Khi d ¯o . , , , , , , 2 ´ ´ nhâ´t c´ ´ cua d o 200 h`ınh quat, d u o c tô, t u c l` a c´ o tâ t c a 200 ¯˜ıa thu ¯ . . , , , ,, , ´, ˘. p h`ınh quat ca ung nhau. Gia su c´ o n vi. tr´ı cua d ¯˜ıa thu . d¯uo. c tô tr` ,, , ˘. p h`ınh quat ´ ung nhau. Khi d hai c´ o không ´ıt hon 21 ca ¯uo. c tô tr` ¯o . d , , , , ´ sô´ c´ ac h`ınh quat ung nhau không nho hon 21n. Do d ¯uo. c tô tr` ¯o . d ,, , 2 ´ c l` 21n ≤ 200 , tu a n ≤ 1904, 8. Boi v`ı n l` a sô´ nguyên, nên n ≤ 1904. ˘. p h`ınh Suy ra c´ o ´ıt nhâ´t 1998 − 1904 = 94 vi. tr´ı c´ o không qu´ a 20 ca ,, quat ung nhau. ¯uo. c tô tr` . d , ´,ng minh ho` . 11.15. Go.,i ´y: Chu an to` an nhu b` ai 11.2.

, ,, , , ` go. i y´ chuong 12 15.12. L`oi giai va

, , , ´,ng qua . 12.11. L`o,i giai: Lâ´y A∗ l`a nh˜ung d¯iêm trên ma˘. t câ`u d¯ô´i xu , , , , , , , , , tâm cua qua câ`u t`u nh˜ung d ho. p A. Theo gia thiê´t tông ¯iêm cua tâp . , ´,n ho,n diên ˘. t câ`u. Theo nguyên l´y diên a A∗ lo . t´ıch cua A v` ., t´ıch ma , ´ v` Ðirichlê cho diên t´ıch ch´ ung c´ od d a ¯iêm chung. Vây ¯iêm chung d ¯o . . , , , , ,`, ´ ` ´ ˘. p d d o l` a ca ¯iêm d ¯ôi xung cua n´ ¯iêm d ¯êu thuôc ¯uong k´ınh . A tao . ra d , ` qua câu. , ˜ h`ınh ch˜u, thâp . 12.12. L`o,i giai: Vo´,i môi on c´ o tâm tai . ta x´et h`ınh tr` . 1 , , ` ` ´ ng minh ra ˘ng √ . Ta chu ˘ng nê´u hai tâm ch˜u thâp a b´ an k´ınh ba . v` 2 2

, ,, , `,i giai v` Chuong 15. Lo a go. i ´y , ´˘t nhau. Khoang ´˘t nhau th`ı hai ch˜u, thâp ´ ca c˜ ung s˜e ca h`ınh tr` on d¯o . , , `˘ng nhau v` ´˘t nhau không lo ´,n c´ ach gi˜ua hai tâm cua h`ınh tr` on ba a ca , , , , , ´ khoang c´ hon hai lâ`n b´ an k´ınh cua ch´ ung, do d ach gi˜ua tâm cua ¯o 1 , , , ´, ´,i ch´ ´,n ho,n √ . X´et h`ınh ch˜u, c´ ac ch˜u thâp ng vo ung không lo . tuong u 2 , , ,, , ´, nhâ´t v` nhât ac d ac c´ anh cua h`ınh ch˜u thâp thu a tâm cua ¯inh . boi c´ . x´ . , , , ´, hai. S˜e c´ ´, hai d ch˜u thâp o môt anh cua ch˜u thâp ¯i qua h`ınh . thu . c´ . thu , ,, , , , ´ ´ ´ nhât, boi v`ı d ˘t ch˜u thâp ´, do d ´ s˜e ca ch˜u nhât ai cua c´ anh ¯o ¯o ¯ô. d` . d . thu 1 1 , , , , , `˘ng √ , c` ´n hon √ . ba on d ai d ¯ô. d` ¯u`ong ch´eo h`ınh ch˜u nhât . không lo 2 2 , , ,, ˘. t d R˜ o r` ang trong h`ınh tr` on b´ an k´ınh 100 chı c´ o thê d ¯a ¯uo. c môt . 1 , ´ ´ ˘t nhau. sô h˜uu han ac h`ınh tr` on b´ an k´ınh √ không ca . c´ 2 2 174

, , , ,, , . 12.13. L`o,i giai: C´o thê. Gia su O l`a tâm cua ng˜ u gi´ ac d ¯ê`u ABCDE. ´ c´ Khi d ac h`ınh tr` on nôi ac g´ oc AOC, BOD, COE, DOA, ¯o . tiê´p trong c´ ˜ nêu. EOB c´ o t´ınh châ´t d ¯a , , `˘ng 1. Tâ´t ca 3 . 12.14. L`o,i giai: X´et môt tam gi´ ac d o canh ba ¯ê`u c´ . . , , , ,, , `˘ng c´ ´ phai d o không thê d ac mâ`u kh´ ac nhau, do d ¯ınh cua n´ ¯uo. c tô ba ¯o , , , `˘ng 1. c´ o hai d ung mâ`u, v` a khoang c´ ach gi˜u ch´ ung ba ¯iêm c`

PHU. LUC .

A

, ` ANH ´ TÂP H O XA . . . P VA

,, , , ` ´ A.1. Tâp h o p v a To an t u trên tâp . . . ho. p.

, , ,, `n tu, cua tâp ´ l` Khi ch´ ung ta coi môt ao d a môt ¯ô´i tuo. ng a n` ¯o . . d . phâ ,, , , ` ho. p A, th`ı k´y hiêu l` a a ∈ A, c` o n khi a không l` a phâ n t u c ua A th` ı k´ y . hiêu a a 6∈ A. . l` , , , , Môt tâp ho. p A goi l` a tâp ho. p con cua tâp ho. p B, khi moi . . . . . phâ`n . ,, , ,, , , , ˘. c tu cua tâp a phâ`n tu cua tâp a A ⊂ B hoa . ho. p A l` . ho. p B, K´y hiêu . l` , , ˘. c l` B ⊃ A. Ngh˜ıa l` a t`u a ∈ A suy ra a ∈ B hoa a t`u a 6∈ B suy ra a 6∈ A. ,, , ˜ ao, k´y hiêu a ∅. Moi Tâp o môt ¯ô´i . l` . d . phâ`n tu n` . ho. p rông không c´ , , , ˜ x 6∈ ∅ . Tâp ˜ `˘m trong tâp tuo. ng x d ¯ê`u không na . ho. p rông . ho. p rông , , `˘m trong moi ´i moi d¯ê`u na a ∅ ⊂ A, vo . tâp . ho. p, ngh˜ıa l` . A. ,, , Hai tâp a B tr` ung nhau khi ch´ ung c´ o c` ung c´ ac phâ`n tu . ho. p A v` , , ,, ,, , nhu nhau, k´y hiêu a chı khi t`u ¯uong , A = B khi v` . A = B. Tuong d , ˘. c A ⊂ B v` a ∈ A suy ra a ∈ B v` a t`u a ∈ B suy ra a ∈ A hoa a B ⊂ A. , , , , , Cho A1 , A2 , . . . , An l` a môt tâp ho. p. Ho. p cua . sô´ h˜uu han . nh˜ung . , ,, , , , nh˜ung tâp a môt ac phâ`n tu m` a n´ o . ho. p trên l` . tâp . ho. p gô`m tâ´t ca c´ , , , thuôc ao môt ac tâp c´ ac tâp . v` . trong c´ . ho. p A1 , A2 , . . . , An . Ho. p cua . , , ho. p A1 , A2 , . . . , An , k´y hiêu l` a A ∪ A ∪ . . . ∪ A . C u thê , a ∈ A ∪ n 2 1 1 . . , A2 ∪ . . . ∪ An ngh˜ıa l` a tô`n tai . môt . chı sô´ i trong {1, 2, . . . , n} sao cho 175

176

,, , ´ xa. Chuong A. Tâp a Anh . ho. p v`

a ∈ Ai . , , , Cho A1 , A2 , . . . , An l` a môt . sô´ h˜uu han . nh˜ung tâp . ho. p. Giao , ,, , , , cua nh˜ung tâp a môt ac phâ`n tu m` a . ho. p trên l` . tâp . ho. p gô`m c´ , , , `˘m trong moi n´ o na ac tâp . tâp . ho. p A1 , A2 , . . . , An . Giao c,ua c´ . ho. p A1 , A2 , . . . , An k´y hiêu a A1 ∩ A2 ∩ . . . ∩ An . Cu. thê, a ∈ A1 ∩ A2 ∩ . l` , , ´i moi . . . ∩ An ngh˜ıa l` a vo o a ∈ Ai . . chı sô´ i trong {1, 2, . . . , n} ta c´ , , , Hiêu tâp a B l` a môt . ho. p A v` . tâp . ho. p, k´y hiêu . A\ B, gô`m . cua hai , , , nh˜ung phâ`n tu a c´ o t´ınh châ´t a ∈ A v` a a 6∈ B.

, , ´ hoc ` Bai ` toan ´ tô ho. p A.2. Qui nap . toan . va , , ´,i nguyên l´y Ðirichlê c` Trong tâp ung vo on môt . ho. p sô´ tu. nhiên, c` ,. , , ,, , nguyên l´y qui nap an hoc ung hay d Dang ¯uo. c su dung. ¯on gian . to´ . c˜ . . d , ,, , cua phuong ph´ ap qui nap an hoc . to´ . nhu sau: , , ˜ ng d¯iê`u kha Cho môt ay nhu ˘ng d¯inh . d˜ . K1 , K2 , . . . , K n , . . .

(A.1)

c´ o t´ınh châ´t sau: , ´ng. a)K1 l` a kha ˘ng d¯inh . d¯u , , ˜ ´ , b) V´ oi môi sô tu. nhiên n,nê´u Kn l` a kha ˘ng d¯inh . d¯us ng th`ı suy ra , ´ng. Kn+1 c˜ ung l` a kha ˘ng d¯inh . d¯u , , ´ tâ´t ca c´ ´ng. Khi d¯o ac kha ˘ng d¯inh ay (A.1) d¯ê`u d¯u . trong d˜ , , ´ ng, sau d ´a ´p dung ´: d Ch´ uy b) ¯iê`u kiên ¯am bao cho K1 d ¯u ¯o . a) d . ´ ´ ´ ng, tiêp tuc ´p dung ´ ng v` cho K2 c˜ ung d ung d a tiêp tuc ¯u ¯u .,a . b) cho K3 c˜ . , , ´ ´i moi ´ vo qu´ a tr`ınh d ung ta chung minh môt ¯o ¯inh . . n. Ðê minh hoa . ch´ . d , , ` ´ ´ ´p dung l´y vê tô ho. p m` a rât hay a trong cuôn s´ ach n` ay: . ,, ,, , ˜, , , n ´ng 2 tâp Moi oi n phâ`n tu c´ o d¯u . tâp . ho. p huu han . ,v´ . ho. p con tao . boi ,, c´ ac phâ`n tu kh´ ac nhau cua n´ o.

, ´ xa. trên tâp A.3. Anh . ho. p

177

, `˘ng Kn . ´,i moi ˘ng d Vo ung ta k´y hiêu trên ba ¯iê`u kha ¯inh . . sô´ n ch´ . d , , ,, , ´i n = 1 kha ˘ng d Nhu vây ung ta s˜e nhân ay (A.1). Vo ¯uo. c d˜ ¯inh . . ch´ . d ,, , , ` ´ ng, v`ı moi ´ ng hai tâp ´ l` d o môt o d¯u a ¯u ¯o . tâp . ho. p c´ . phân tu c´ . ho. p con, d ,, ˜ ` ´ tâp a ch´ınh tâp o t´ınh chât ¯os . Vây . rông v` . ( môt . phân tu ) d . (A.1) c´ a). , , ´,ng minh b) d ´ ng. Ngh˜ıa l` Bây gi`o ch´ ung ta phai chu a cho n l` a ¯u , , , , ´ c l` ˘ng d ´ ng tu môt a kha a moi o ¯inh ¯u . Kn d . sô´ tu. nhiên bâ´t k`y v` . tâp . ho. p c´ , ,, , , n `˘ng Kn+1 ´ ng minh ra ´ ng 2 tâp n phâ`n tu c´ od ¯u . ho. p con. Câ`n phai chu ,, , l` ad ung ta x´et S l` a tâp o n + 1 phâ`n tu v` a s l` a môt ¯us ng. Ch´ . ho. p c´ . , ,, , , , , ´ phâ`n tu cua n´ o. Ch´ ung ta chia nh˜ung tâp h o p con c ua S l` a m hai l o p: . , , . , ´,a s. Lo ´,p a1 gô`m tâ´t ca c´ ´,p a2 gô`m Lo ac tâp h o p con c ua S không chu ., . , , , ´,a s. Nh˜u,ng tâp ´,p a1 tâ´t ca c´ ac tâp . ho. p, con cua S chu . ho. p con thuôc . lo , , , , , l` a tâp a n´ o nhân . , ho. p con cua môt . tâp . ,ho. p n phâ`n tu S1 , m` . t`u S sau , ,, ,, , khi bo d ¯i phâ`n tu s. Theo gia thiê´t qui nap . ,sô´ luo. ng tâp . ho. p con cua , , ´, , ´,p a2 l` ˘. t kh´ ´ ng l` n´ od a 2 n . Ma ac c´ ac tâp a tuong u ng 1-1 ¯u . ho. p con cua lo , , , , , ´,a s ´ i vo ´i c´ vo ac tâp . ho. p con cua S1 . Thât . vây, . moi . ,tâp . ho. p con Z chu , , , ´, , , `˘ng c´ ´,i tâp cua S ta cho tuong u ng vo ach t`u Z . ho. p con Z1 cua S1, ba , ,, ,, , ´,p a2 ta bo d a sô´ luo. ng tâp ¯i. phâ`n tu s. Ngh˜ıa l` . ho. p con cua S trong lo , , , `˘ng 2n . Do d ´,i sô´ lu,o.,ng tâp ´ tâ´t ca tâp tr` ung vo ho. p con cua S1 v` a ba ¯o . . , , , , ´ ˜ ho. p con cua S l` a 2n + 2n = 2n+1 . Nhu vây ch´ u ng ta d a ch u ng minh, ¯ . , ` ´,ng minh ´,i moi vo n, t u K d us ng suy ra K d us ng. Ch´ ung ta chu ¯ ¯ n n +1 . , , ,, ˘ng d¯inh ´ ng. d o t´ınh châ´t b). Suy ra tâ´t ca c´ ac kha ¯uo. c (A.1) c´ ¯ê`u d ¯u . d

J

, ´ A.3. Anh xa. trên tâp . ho. p

,, , , , , ´,n ho,n sô´ T`u nh˜ung kh´ ai niêm sô´ phâ`n tu cua môt tâp ho. p n` ay lo . . . ,, , ,, , , ´nh xa. phâ`n tu cua môt tâp ho. p kia, ngu`oi ta xây du. ng kh´ ai niêm . . . a , , , ´ d a ch´ınh x´ ac h´ oa trong khi chung minh. ¯ê giai th´ıch v`

178

,, , ´ xa. Chuong A. Tâp a Anh . ho. p v`

, , `˘ng cho môt ´nh xa. f : A → B t`u tâp Ch´ ung ta n´ oi ra ao . ho. p A v` . a ,, , , ´, , , ` ´i môt tâp . ho. p B, khi moi . phân tu a thuôc . A cho tuo,ng ung vo . phâ`n ,, ,, ´,i phâ`n ´nh xa. f vo tu f ( a) thuôc B. Phâ`n tu f ( a) goi l` a gi´ a tri. cua a . . , , , ,, , , ´nh xa. sô´ tu. nhiên v` ao sô´ tu a cua A. C´ o thê chı ra nhiê`u v´ı du. nhu a , , , , , ˜˘n, vo ´ ng vo ´i moi ´i 2n,. . . . cha . n cho tuong u , ,`, , ´nh xa: ´nh xa. Nguoi ta c` on d ¯ua ra môt . sô´ t´ınh châ´t cua a . Môt . a , , ´nh, nê´u t`u f ( a1 ) = f ( a2 )( a1 , a2 ∈ A) suy ra f : A → B goi ad ¯on a . l` , ,, , ´nh khi moi a1 = a2 . N´ oi c´ ach kh´ ac f l` ad ac nhau cua ¯o n a . phâ`n, tu kh´ , , , ´, ´,i nh˜u,ng phâ`n tu, kh´ A cho tuong u ng vo ac nhau cua B. ,, ´,i moi ´nh xa. f : A → B goi ´nh, nê´u vo Môt a l` a to` an a phâ`n tu . . . , , ,, cua b cua B tô`n tai o nhiê`u v´ı . phâ`n tu a thuôc . A sao cho f ( a) = b., C´ , , ´nh xa. loai du. vê` a n` ay, nhu ph´ep chiê´u lên truc toa. d d ¯ô. cua moi ¯iêm . . . , ˘. t pha ˘ng. trong ma , , ´nh xa. f : A → B goi ´nh, khi n´ Môt a song a od ad ¯ô`ng th`oi l` ¯o n . a . l` ,, ´,i moi ´nh v` ´nh. Nê´u f : A → B l` ´nh, th`ı vo a a to` an a a song a phâ`n tu b . , ,, cua B tô`n tai . duy nhâ´t phâ`n tu a thuôc . A sao cho f ( a) = b. ˘. t g(b) = a khi d ´nh f : A → B. Nê´u ch´ ˜ Cho song a ung ta d ¯a ¯a ,, ´nh xa. g : B → A goi ´nh aa c´ o f ( a) = b, ch´ ung ta nhân ¯uo. c a . d . l` ,, , , − 1 − 1 xa. nguo. c cua f v` a k´y hiêu a f = g. Nhu vây a . l` . f ( f (b)) = b v` , − 1 ´i moi f ( f ( a)) = a vo a b thuôc . a thuôc . A v` . B. , ´nh xa. f : A → B v` ´nh xa. Cho hai a a g : B → C, c´ o thê tao . ra a , `˘ng công thu ´ c h( a) = f ( g( a)). K´y hiêu h : A → C ba . h = g ◦ f.

, ` TAI LIÊU . THAM KHAO

, , , [1] Tuyên tâp am tap an hoc a Tuôi tre, NXBGD, 1997. . 30 n˘ . ch´ı To´ . v` [2] 255 b` ai to´ an sô´ hoc anh, N. N. . chon . loc, . V.D. Thuy, . T. C. Th` ,, Ðam, So GD H` a tây, 1993 . , , , [3] 10000 B` ai to´ an so câ´p - D˜ ay sô´ v` a gi´ oi han, . Phan Huy Khai, NXB H` a nôi, . 1997. ,, [4] C´ ac d¯ê` thi vô d¯ich an c´ ac nuo´c, Xv. Cônhiagin, G.A.Tônôian, . to´ , If. Sarugin, NXB GD 1996. , , [5] C´ ac b` ai to´ an h`ınh hoc ˘ng, V.V. Praxolov, Tâp . pha . II, NXB Hai ph` ong 1997. , [6] Zadatri Vsesaiunu´c matematitreskii olimpiada, N.B. Vasilev, A.A. Egorov, Maskova- Nauka 1988. [7] Matematichskii akvarium V. A. Uphnarovskii, Kishinev " shtiinsa" 1987.(Tiê´ng Nga). [8] Problem-Solving Through Problems, Loren C. Larson, SpringVerlag, 1983. [9] Princil na Dirichle, Ivan Prodanov, "Narodna Prosveta", Sophia 1988 (Tiê´ng Bungari). 179

180

Muc . luc .

, [10] Zadachi da idvunklasna Rabota po matematika, R.Rusev, K.Bankov, Sv. Slavchev, "Narodna Prosveta", Sophia 1986 (Tiê´ng Bungari). [11] Sbornhik ot Zadachi za Matematicheski Olympiadi, St. Budurov, V. Phlorov, "Narodna Prosveta", Sophia 1966 (Tiê´ng Bungari).

NÔI . DUNG

`,i n´ Lo oi d ¯â`u . . . . . . . . . . . . . . . . . . . . . . . . . . . . . . . . . . . . . . . . . . . . . . . . . . . ,, Chuong 1. Nguyên l´ y Ðirichlê v` a v´ı du. . . . . . . . . . . . . . . . . . . . . . .

5

1.1. Nguyên l´y Ðirichlê . . . . . . . . . . . . . . . . . . . . . . . . . . . . . . . . . . . . .

5

1.2. V´ı du. . ..................................................

6

3

1.3. B` ai tâp. . ............................................... ,, Chuong 2. Sô´ hoc . .......................................... , 2.1. Ph´ep chia sô´ tu. nhiên . . . . . . . . . . . . . . . . . . . . . . . . . . . . . . . . .

13

2.2. V´ıdu. . . . . . . . . . . . . . . . . . . . . . . . . . . . . . . . . . . . . . . . . . . . . . . . . . .

14

2.3. B` ai tâp. . ............................................... ,, Chuong 3. D˜ ay sô´ . . . . . . . . . . . . . . . . . . . . . . . . . . . . . . . . . . . . . . . . . .

20 23

3.1. Nguyên l´y Ðirichlê cho d˜ ay sô´ vô han . ..................

23

3.2. V´ı du. . . . . . . . . . . . . . . . . . . . . . . . . . . . . . . . . . . . . . . . . . . . . . . . . .

23

3.3. B` ai tâp. . ............................................... ,, Chuong 4. H`ınh hoc . ........................................

31 33

4.1. V´ı du. . . . . . . . . . . . . . . . . . . . . . . . . . . . . . . . . . . . . . . . . . . . . . . . . .

33

4.2. B` ai tâp. . ............................................... ,, ,, Chuong 5. Mo rông nguyên l´ y Ðirichlê. . . . . . . . . . . . . . . . . . . . . . ,, 5.1. Nguyên l´y Ðirichlê mo rông . ...........................

41 43

5.2. V´ı du. . . . . . . . . . . . . . . . . . . . . . . . . . . . . . . . . . . . . . . . . . . . . . . . . .

44

181

11

13

43

182

NÔI . DUNG

5.3. B` ai tâp. . ............................................... ,, ai tâp Chuong 6. B` . sô´ hoc . nâng cao . . . . . . . . . . . . . . . . . . . . . . . . , , , 6.1. Ðinh . l´y co ban cua sô´ hoc . .............................

52 55

6.2. V´ı du. . . . . . . . . . . . . . . . . . . . . . . . . . . . . . . . . . . . . . . . . . . . . . . . . .

55

6.3. B` ai tâp. . ............................................... ,, ˜ Chuong 7. B` ai tâp . dây sô´ nâng cao . . . . . . . . . . . . . . . . . . . . . . . .

64 67

7.1. V´ı du. . . . . . . . . . . . . . . . . . . . . . . . . . . . . . . . . . . . . . . . . . . . . . . . . .

67

7.2. B` ai tâp. . ............................................... ,, , ´, Chuong 8. Sô´ thu. c vo i tâp u mât . tr` . .........................

78 79

8.1. Tâp u mât . tr` . ...........................................

79

8.2. V´ı du. . . . . . . . . . . . . . . . . . . . . . . . . . . . . . . . . . . . . . . . . . . . . . . . . .

80

8.3. B` ai tâp. . ............................................... , ,, ˜,ng u ´,ng dung kh´ ac cua nguyên l´ y Ðirichle . . Chuong 9. Nhu . , , 9.1. Xâ´p xı môt . sô´ thu. c . . . . . . . . . . . . . . . . . . . . . . . . . . . . . . . . . . . .

87

55

89 89

9.2. B` ai tâp. . . . . . . . . . . . . . . . . . . . . . . . . . . . . . . . . . . . . . . . . . . . . . . . . 99 ,, Chuong 10. Nguyên l´ y Ðirichlê cho diên . t´ıch . . . . . . . . . . . . . 101 , 10.1. Ph´ at biêu nguyên l´y Ðirichlê cho diên . t´ıch . . . . . . . . . . 101 10.2. V´ı du. . . . . . . . . . . . . . . . . . . . . . . . . . . . . . . . . . . . . . . . . . . . . . . .

105

10.3. B` ai tâp . ............................................. , , ,, Chuong 11. To´ an hoc . tô ho. p . . . . . . . . . . . . . . . . . . . . . . . . . . . . . .

117 119

11.1. V´ı du. . . . . . . . . . . . . . . . . . . . . . . . . . . . . . . . . . . . . . . . . . . . . . . .

119

11.2. B` ai tâp . ............................................. ,, Chuong 12. Môt ai tâp ac . . . . . . . . . . . . . . . . . sô´ b` . h`ınh hoc . kh´

127 129

12.1. V´ı du. . . . . . . . . . . . . . . . . . . . . . . . . . . . . . . . . . . . . . . . . . . . . . . .

129

Muc . luc .

183

12.2. B` ai tâp . ............................................. ,, Chuong 13. Môt ¯ê` thi vô d¯ich . ........................ . sô´ d , ,, , ai tâp Chuong 14. B` . tu. giai . . . . . . . . . . . . . . . . . . . . . . . . . . . . . . . . , ,, , `,i giai v` ´ .............................. Chuong 15. Lo a go. i y , , , , , 15.1. L`oi giai v` a go. i ´y chuong 1. . . . . . . . . . . . . . . . . . . . . . . . . . . , ,, , , 15.2. L`oi giai v` a go. i ´y chuong 2. . . . . . . . . . . . . . . . . . . . . . . . . . . ,, , , , 15.3. L`oi giai v` a go. i ´y chuong 3. . . . . . . . . . . . . . . . . . . . . . . . . . . ,, , , , 15.4. L`oi giai v` a go. i ´y chuong 4. . . . . . . . . . . . . . . . . . . . . . . . . . . ,, , , , 15.5. L`oi giai v` a go. i ´y chuong 5. . . . . . . . . . . . . . . . . . . . . . . . . . . ,, , , , 15.6. L`oi giai v` a go. i ´y chuong 6. . . . . . . . . . . . . . . . . . . . . . . . . . . ,, , , , 15.7. L`oi giai v` a go. i ´y chuong 7. . . . . . . . . . . . . . . . . . . . . . . . . . . ,, , , , 15.8. L`oi giai v` a go. i ´y chuong 8. . . . . . . . . . . . . . . . . . . . . . . . . . . ,, , , , 15.9. L`oi giai v` a go. i ´y chuong 9. . . . . . . . . . . . . . . . . . . . . . . . . . . ,, , , , 15.10. L`oi giai v` a go. i ´y chuong 10 . . . . . . . . . . . . . . . . . . . . . . . . ,, , , , 15.11. L`oi giai v` a go. i ´y chuong 11 . . . . . . . . . . . . . . . . . . . . . . . . ,, , , , 15.12. L`oi giai v` a go. i ´y chuong 12 . . . . . . . . . . . . . . . . . . . . . . . . , ´ nh xa. . . . . . . . . . . . . . . . . . . . . . . . . . . . . Phu. luc aA . A. Tâp . ho. p v` ,, , , A.1. Tâp a To´ an tu trên tâp . ho. p v` . ho. p. . . . . . . . . . . . . . . . . . . . . , , A.2. Qui nap an hoc a B` ai to´ an tô ho. p . . . . . . . . . . . . . . . . . . to´ . v` , ´ xa. trên tâp A.3. Anh . ho. p . . . . . . . . . . . . . . . . . . . . . . . . . . . . . . . . .

134

Muc . luc . .................................................

135 151 155 155 156 158 158 160 161 162 166 168 169 171 173 175 175 176 177 180

, ˜ HU ˜,U ÐIÊN NGUYÊN

, , ´,NG DUNG ´ P ÐIRICHLÊ VA `U PHUONG PHA . , , , c

Ebook 1.0 cua cuô´n s´ ach nguyên gô´c t`u ban in, c´ ac ban . tham , , , ´ ` khao, cho ´y kiên sai s´ ot v` a loi khuyên t´ ai ban. Moi . liên hê. , , , ˜ ˜ T´ ac gia: Nguyên Huu Ðiên Ðiên . thoai: . 0989061951 Email: [email protected] Web: http://nhdien.wordpress.com

, Chiu ach nhiêm . tr´ . xuâ´t ban: ,, , Biên tâp a sua ban in: . v` , Tr`ınh bâ`y v` a chê´ ban: Tr`ınh bâ`y b`ıa:

, ˘ NG HAI TÔ ÐA ˜ PHU ´ ÐÔ , , ˜ ` LINH HUU ÐIÊN, PHU , , HUONG LAN

, ´ BAN KHOA HOC VA ` XUÂT ` KY ˜ THUÂT NHA . . , ` ` 70 TRÂN HUNG ÐAO . - HA NÔI . 6.6T7.3 290 - 4-98 KHKT − 98 , , In 1000 ban khô 14, 5 × 20, 5 tai an Viêt . Công ty In Công Ðo` . Nam , 191 Son Tây - Ðô´ng Ða - H` a Nôi. . Giâ´y ph´ep XB sô´: 290-22/4/98. , , In xong v` a nôp ang 1 n˘ am 1999. . luu chiêu th´

View more...

Comments

Copyright ©2017 KUPDF Inc.
SUPPORT KUPDF